NBME 28 A.pdf

FeliciaAlexander4 785 views 52 slides Jul 30, 2023
Slide 1
Slide 1 of 200
Slide 1
1
Slide 2
2
Slide 3
3
Slide 4
4
Slide 5
5
Slide 6
6
Slide 7
7
Slide 8
8
Slide 9
9
Slide 10
10
Slide 11
11
Slide 12
12
Slide 13
13
Slide 14
14
Slide 15
15
Slide 16
16
Slide 17
17
Slide 18
18
Slide 19
19
Slide 20
20
Slide 21
21
Slide 22
22
Slide 23
23
Slide 24
24
Slide 25
25
Slide 26
26
Slide 27
27
Slide 28
28
Slide 29
29
Slide 30
30
Slide 31
31
Slide 32
32
Slide 33
33
Slide 34
34
Slide 35
35
Slide 36
36
Slide 37
37
Slide 38
38
Slide 39
39
Slide 40
40
Slide 41
41
Slide 42
42
Slide 43
43
Slide 44
44
Slide 45
45
Slide 46
46
Slide 47
47
Slide 48
48
Slide 49
49
Slide 50
50
Slide 51
51
Slide 52
52
Slide 53
53
Slide 54
54
Slide 55
55
Slide 56
56
Slide 57
57
Slide 58
58
Slide 59
59
Slide 60
60
Slide 61
61
Slide 62
62
Slide 63
63
Slide 64
64
Slide 65
65
Slide 66
66
Slide 67
67
Slide 68
68
Slide 69
69
Slide 70
70
Slide 71
71
Slide 72
72
Slide 73
73
Slide 74
74
Slide 75
75
Slide 76
76
Slide 77
77
Slide 78
78
Slide 79
79
Slide 80
80
Slide 81
81
Slide 82
82
Slide 83
83
Slide 84
84
Slide 85
85
Slide 86
86
Slide 87
87
Slide 88
88
Slide 89
89
Slide 90
90
Slide 91
91
Slide 92
92
Slide 93
93
Slide 94
94
Slide 95
95
Slide 96
96
Slide 97
97
Slide 98
98
Slide 99
99
Slide 100
100
Slide 101
101
Slide 102
102
Slide 103
103
Slide 104
104
Slide 105
105
Slide 106
106
Slide 107
107
Slide 108
108
Slide 109
109
Slide 110
110
Slide 111
111
Slide 112
112
Slide 113
113
Slide 114
114
Slide 115
115
Slide 116
116
Slide 117
117
Slide 118
118
Slide 119
119
Slide 120
120
Slide 121
121
Slide 122
122
Slide 123
123
Slide 124
124
Slide 125
125
Slide 126
126
Slide 127
127
Slide 128
128
Slide 129
129
Slide 130
130
Slide 131
131
Slide 132
132
Slide 133
133
Slide 134
134
Slide 135
135
Slide 136
136
Slide 137
137
Slide 138
138
Slide 139
139
Slide 140
140
Slide 141
141
Slide 142
142
Slide 143
143
Slide 144
144
Slide 145
145
Slide 146
146
Slide 147
147
Slide 148
148
Slide 149
149
Slide 150
150
Slide 151
151
Slide 152
152
Slide 153
153
Slide 154
154
Slide 155
155
Slide 156
156
Slide 157
157
Slide 158
158
Slide 159
159
Slide 160
160
Slide 161
161
Slide 162
162
Slide 163
163
Slide 164
164
Slide 165
165
Slide 166
166
Slide 167
167
Slide 168
168
Slide 169
169
Slide 170
170
Slide 171
171
Slide 172
172
Slide 173
173
Slide 174
174
Slide 175
175
Slide 176
176
Slide 177
177
Slide 178
178
Slide 179
179
Slide 180
180
Slide 181
181
Slide 182
182
Slide 183
183
Slide 184
184
Slide 185
185
Slide 186
186
Slide 187
187
Slide 188
188
Slide 189
189
Slide 190
190
Slide 191
191
Slide 192
192
Slide 193
193
Slide 194
194
Slide 195
195
Slide 196
196
Slide 197
197
Slide 198
198
Slide 199
199
Slide 200
200

About This Presentation

NMBE 28. all questions. USMLE. USMLE Step 2


Slide Content

ExamSection1:Item1of50 NationalBoardofMedicalExaminers^
ComprehensiveBasicScienceSelf-Assessment
Y
1.A25-year-oldmanisbroughttotheemergencydepartmentbecauseofsevereabdominalpain,nausea
:andvomitingfor1hour.Thepainoriginatesintheleftflankandradiatestohisgroin.Hispulseis100/min,respirationsare18/minpandbloodpressureis
150/100mmHg.Physicalexaminationshowstendernessoftheleftflankandtheleftlowerquadrantoftheabdomen.Bowelsoundsaremildlyhypoactive.Testofthestoolforoccultbloodisnegative.Whichofthefollowingbestexplainsthesefindings?
A)Colonneoplasm
B)Diverticulitis
C)Epididymitis
D)Renalinfarction
E)Torsionofthetestis
F)Ureteralcalculus
CorrectAnswer:F.
Ureteralcalculustypicallypresentswithcolickyunilateralflankpainradiatingtothegroin
:andwithgrossormicroscopichematuria.Painmaybesignificantenoughtotriggernausea,asinthiscase.Thecommontypesofurinarytractcalculiarecalciumoxalateor
phosphate,ammoniummagnesiumphosphate
:uricacid
:andcystine.Onurinalysis,redbloodcellswithoutcastsarecommon.Feverdysuria
;andpyuriawouldnotbeexpectedunlesstherewasaconcomitantinfection.Treatmentforureteralcalculusis
symptomaticwithpaincontrolandnausearelief.Mostureteralcalculipassspontaneouslyafteraperiodofobservationforpatientswithwell-controledpainandnosignsofsepsisorinfection.Stoneremovalbyshockwaveithotripsyorendoscopicremovalisan
optionforpatientsrequiringemergencytherapy.Itisalsoanoptionforpatientswithpersistentobstruction,uncontrolledsymptoms,orfailureofstoneprogression.Ingeneral,stonessmallerthan5mmwillpasswithoutoperativeassistance.Obstructingstonesmay
requiretemporaryplacementofaureteralstenttopreventhydronephrosisandrenalparenchymalinjury.
IncorrectAnswers:A.B.CaD,andE
Colonneoplasm(ChoiceA)wouldbeunlikelyinanotherwisehealthyyoungpatientwithnofamilyhistoryofpolyposissyndromesandacute,severe,flankpain.Itwouldtypicallypresentwithinsidiousweightloss,anemia,constipationorbloodperrectum.In
addition,testforstoolforoccultbloodisnegative,makingthisdiagnosisunlikely.
Diverticulitis(ChoiceB)canpresentwithleftlowerquadrantabdominalpainandtendernessbutwouldbelessabruptinpresentationandtypicallypresentwithfever,diarrhea,andhyperactivebowelsounds.Itwouldbeunlikelytocauseflankpair
Epididymitis(ChoiceC)isacommoncauseofpainfulscrotalswellingandreferstoacuteinfectionandinflammationoftheepididymis.Inyoungermales,thisiscommonlysecondarytosexuallytransmittedinfectionssuchasChlamydiatrachomatisorNeisseria
gonorrhoeae.Inoldermales,Escherichiacoliismorecommon.
Renaiinfarction(ChoiceD)cancauseflankpain,nausea,andvomiting,andcanbeduetothromboembolicdisease,renalarterydissection,orahypercoagulablestate.However,itisrareandureteralcalculusismarecommonandlikelyinthispatient.
Torsionofthetestis(ChoiceE)occurswhenthetesticletwistsonthespermaticcordresultinginsubsequentlossoftesticularbloodsupply.Patientstypicallypresentwithacute,severetesticularpain,swelling,anderythema.Onphysicalexamination,thetesticle
typicallydemonstratesanabnormallie(eg,transverse),extremetendernesstopalpation,absentcremastericreflex,andpainthatdoesnotimprovewithelevationofthescrotum(asitdoesinepididymitis).
EducationalObjective:Ureteralcalculustypicallypresentswithcolicky,unilateralflankpainradiatingtothegroin,alongwithgrossormicroscopichematuria.
00
<t= A
Next ScoreReport LabValues Calculator Help Pause

ExamSection1:Item2of50 NationalBoardofMedicalExaminers^
ComprehensiveBasicScienceSelf-Assessment
Y
2.WhichofthefollowingtypesofsensoryinformationiscompromisedbylesionsofthestructureatsiteXinthephotographshown?
A}Consciousproprioception
B)Painsensation
C)Two-pointdiscrimination
D)Unconsciousproprioception
E)Vibrationsense
CorrectAnswer:D.
Theanteriorlobeofthecerebellum(labeledX.picturedincross-sectionasanarborizedbrainareaposteriortothebrainstemandanteriortotheprimaryfissureofthecerebellum)mediatesunconsciousproprioception.Theanteriorlobeofthecerebellumreceives
informationfromthespinocerebellartractaboutproprioception,orbodyposition,thatisgatheredfrommusclestretchandtensionreceptorsontheipsilatera!sideofthebody.Thisproprioceptiveinformationistransmittedoutsideofconsciousawareness.Thedeep
cerebellarnucleiusethisproprioceptiveinformationtocontrolmotorlearning,movementcoursechanges,andbalance.Damagetotheanteriorlobeofthecerebellum,whichcommonlyoccursinchronicalcoholism,mayleadtobroad-basedgaitataxia.
IncorrectAnswers:A.B
:C.andE.
Consciousproprioception(ChoiceA),two-pointdiscrimination(ChoiceC),andvibrationsense(ChoiceE)aremediatedbythedorsalcolumn-medialemniscuspathway,whichrelaysthissensoryinformationupthespinalcordtothethalamusandterminatesinthe
primarysensorycortexintheparietallobe.Thecortexisahigh-orderbrainareainvolvedinseveralconsciousbrainfunctions,whichreflectsthispathway'smediationoftheconscious(ratherthanunconscious)awarenessofproprioception.
Painsensation(ChoiceB)ismediatedbythespinothalamicpathway.Thespinothalamicpathwaytransmitsinformationaboutpain,temperature,andcrudetouchupthespina!cordtothethalamus,terminatingintheprimarysensorycortex.
EducationalObjective:Theanteriorlobeofthecerebellummediatesunconsciousproprioception,whereasconsciousproprioceptioniscontrolledbythedorsalcolumn-mediallemniscuspathway.Lesionsoftheanteriorlobeofthecerebellumcanresultinbroad-
basedgaitataxia.
000
Previous Next ScoreReport LabValues Calculator Help Pause

ExamSection1:Item3of50 NationalBoardofMedicalExaminers^
ComprehensiveBasicScienceSelf-Assessment
Y
3.Movingtheforearmagainstresistancefrompalm-downtopalm-up(supination)positionrequirestheuseofwhichofthefollowingmuscles?
A)Bicepsbrachii
B)Brachialis
C)Triceps
D)Flexorcarpiradialis
E)Pronatorteres
CorrectAnswer:A.
Thebicepsbrachiimusclehastwomainactions,flexionoftheelbowjointandsupinationoftheforearm.Thebicepsbrachiicontainstwoproximalheads,withtheshortheadattachingtothecoracoidprocessofthescapulaandthelongheadenteringtheshoulder
jointandattachingtothesupraglenoidtubercle.Thedistalbicepstendoninsertsonthebicipitaltuberosityoftheproximalradius.Becauseofitsorientationcrossingtheelbowjoint,contractionofthismusclecauseselbowflexion.Itseccentricinsertiononthe
proximalradiusallowsforittowindaroundtheradiusduringpronationandunwindwhencontractedfromaroundtheproximalradiusduringsupination.
IncorrectAnswers:B,C.DandE
Thebrachialismuscle(ChoiceB)originatesontheanteriorsurfaceofthehumerusandcrossestheelbowinsertingonthetuberosityoftheulna.Itdoesnotwraparoundtheulnaandtheulnadoesnotrotate.Becauseofthis,itdoesnotcontributetosupinationor
pronation.
Thetricepsmuscle(ChoiceC)servestoextendtheelbowjoint.Proximally,itoriginatesfromtheinfraglenoidtubercleofthescapula(longhead},justproximaltotheradialgroove(lateralhead),andjustdistaltotheradialgroove(medialhead).Distally
:itinsertson
theolecranonprocessoftheulna.Contractionofthismuscleextendstheelbowanddoesnotcontributetorotation.
Mexorcarpiradialis(ChoiceD)originatesonthemedialepicondyleofthehumerusandinsertsonthesecondandthirdmetacarpalbones.Thisallowsforflexionofthewrist.
Pronatorteres(ChoiceE)isamuscleoftheproximalforearmthatextendsfromthemedialsupracondylarridgeofthehumerusandinsertsonthelateralaspectoftheradius.Contractionalongthisaxiswillpromotepronation
:notsupination.
EducationalObjective:Thebicepsbrachiimusclehastwomainfunctions.Itservestosupinatetheforearmthroughitswindingmechanismaroundtheproximalradiusandbicipitaltuberosity.Itflexestheelbowasthemuscleandtendoncrosstheelbowjoint
anteriorly.
00 & 00
Previous Next ScoreReport LabValues Calculator Help Pause

ExamSection1:Item4of50 NationalBoardofMedicalExaminers^
ComprehensiveBasicScienceSelf-Assessment
Y
4.A35-year-oldmanisadmittedtothehospitabecauseofa5-dayhistoryoffeveranddyspneaHeunderwentabonemarrowtransplantationSmonthsago;theprocedurewascomplicatedbyseveregraft-versus-hostdisease.Histemperatureis33°C
(100.4T)andrespirationsare30/min.Scatteredcracklesareheardonauscultationofthechest.Achestx-rayshowspatchyinfiltratesAtransbronchialbiopsyspecimenshowsfindingsconsistentwithcytomegalovirusinfection.Intravenousadministrationof
ganciclovirisbegun.Thisdruginterfereswiththefunctionofwhichofthefollowingenzymes?
A)DNApolymerase
B)Jntegrase
C)Reversetranscriptase
D)RNApolymerase
E)Thymidinekinase
CorrectAnswer:A.
Cytomegalovirus(CMV),alsoknownashumanherpesvirus5(HHV-5)
;isanopportunisticinfectioncommonlyoccurringinimmunocompromisedpatientswithsolid-organorallogeneicbonemarrowtransplantationsevereulcerativecolitis,orHIV/AIDSinfection.It
canbetransmittedthroughmultiplemodes,includingsexuaicontacturine,respiratorydroplets
:andtoafetusviatheplacenta.Itcancauseavarietyofpresentations,includingmononucleosisinimmunocompetentpatients,andretinitis,esophagitis,andpneumonia
inimmunocompromisedpatients.TreatmentforallhumanherpesvirusinfectionsinvolvesdrugsthatinhibitviralDNApolymerase,classicallybyguanosineanalogssuchasacyclovirvalacyclovir,andfamciclovir.Priortoexertingtheirantiviraeffects,mos
'
guanosineanalogsmustbephosphorytatedbytheviralenzymethymidinekinase.TheyarethenabletoinhibittheviralDNApolymerasebyterminatingthenascentDNAchainduringreplication.Thesedrugsareeffectiveagainstherpessimplexvirusandvaricella
zostervirus,weaklyeffectiveagainstEpstein-Barrvirus,andnoteffectiveagainstCMV,whichdoesnothavethenecessarythymidinekinaseneededforphosphorylation.However,itdoeshavethenecessaryphosphorylatingenzymetoactivateganciclovir,another
guanosineanalogthatinhibitsDNApolymerase,andthusthisisaneffectiveanti-viraltreatmentforCMV
IncorrectAnswers:B.C,D,andE.
Integraseinhibitors(ChoiceB)preventintegrationofproviralDNAintothehostgenome.Theyareacomponentofthehighlyactiveanti-retroviraitherapyutilizedinthetreatmentofHIV.
Reversetranscriptase(ChoiceC)transcribesDNAfromviralmRNAforincorporationintothehostcellgenome.Thisenzymeisinhibitedbybothnucleosidereversetranscriptaseinhibitors,suchasabacaviranddidanosine.andnon-nucleosidereversetranscriptase
inhibitors,suchasefavirenzandnevirapine.ThesemedicationsareusedtotreatHIVinfections.
RNApolymerase(ChoiceD)istheenzymethatfacilitatestheconversionofDNAtoRNA.Itisthetargetofnucleotideandnucleosidepolymeraseinhibitors,whichareutilizedinthetreatmentofHepatitisCinfection.
Thymidinekinase(ChoiceE)isaphosphorylatingenzymerequiredforactivationoftheguanosineanalogsacyclovirvalacyclovir.andfamciclovir.Developmentofamutationintheviralthymidinekinaseenzymepreventsdrugphosphorylationandconfers
resistancetothesemedications.CMVdoesnotcontainthethymidinekinaseenzyme.
EducationalObjective:GanciclovirisaguanosineanalogthatinhibitsDNApolymeraseandisusedtotreatCMV.Likeotherguanosineanalogsactivationofthismedicationrequiresphosphorylationwithintheinfectedcell
© © ©
Previous Next ScoreReport LabValues Calculator Help Pause

ExamSection1:Item5of50 NationalBoardofMetlicaiExaminers^
ComprehensiveBasicScienceSelf-Assessment
y
0
200-T3
rSE
E
CL100-
<t>
c/>
0
25271
Cycle(day)
M=menstruation
5.Thegraphshowschangesinserumestradiolconcentrationduringanormalmenstrualcycle.Whichofthefollowingovariancellsisprimarilyresponsibleforthearomatizationofandrostenedionetoestradiolatthetimeindicatedbythearrow?
A)Granulosa
B)Luteal
G)Stromal
D)Thecaexterna
E)Thecainterna
CorrectAnswerA.
Thefirsthalfofthemenstrualcycle
;thefollicularphase
:whichvariesinlength,beginswithmenses.Duringmenses,follicle-stimulatinghormone(FSH)andluteinizinghormone(LH)concentrationsincreaseandstimulatethedevelopingfollicle.Androstenedione
isconvertedtoestroneandestradiolviaaromataseinthegranulosacellsofthefollicle.Theestrogenthensecretedfromthegranulosacellisresponsibleforfolliclegrowthandendometrialproliferation.Asestrogenrises,asurgeoccurs,whichinturnstimulatesa
surgeinLHthatcausesovulation.Immediatelyfollowingovulation,thelutealphasebeginsasthecorpusluteumforms.Thecorpusluteumsecretesprogesteronetomaintaintheendometriallining.Howeverifnoimplantationoccurs,thecorpusluteumdegrades
tothecorpusalbicans,andestrogenandprogesteronelevelsdecrease,causingmenstruationandminorincreasesinFSHandLH.
IncorrectAnswersB,C,D,andE.
Lutealcells(ChoiceB)arepresentinthecorpusluteumandarederivedfromthegranulosacellsofthepre-ovuatoryfollicle.Theysecreteprogesteroneandestrogen.Howevertheydonotdevelopuntilafterovulation(14dayspriortomenstruation).
Stromalcells(ChoiceC)aretheconnectivetissueandsupportingcellsoftheovary.Theydonotsecreteestradiol.
Thecaexternacells(ChoiceD)arethecellsthatformtheouterlayerofadevelopingfollicle.Thethecaexternaisprimarilylooseconnectivetissueandthereforethecellsaregenerallyfibroblasts,macrophages,andsmoothmuscle;thesecellsdonotsecrete
hormones.
Thecainternacells(ChoiceE)arecellsofthefolliclethatareresponsibleforgeneratingandrostenedionefromcholesterol,afterwhichtheandrostenedioneistransportedtothegranulosacelltobeconvertedtoestradiol.
EducationalObjective:Granu-osacellsinthedevelopingfollicleareresponsibleforconvertingandrogensreceivedfromthethecainternaceilsintoestradiolviaaromatase.
e
Previous Next ScoreReport LabValues Calculator Help Pause

ExamSection1:Item6of50 NationalBoardofMedicalExaminers^
ComprehensiveBasicScienceSelf-Assessment
Y
6.A14-year-oldgirlisbroughttothephysicianbyherparentsbecauseofa1-monthhistoryofarashthatappearswithsunexposure.Herparentstellthephysicianthatshehasbeeneatinglittlefood.Physicalexaminationshowsapruriticrashontheexposed
areasofthebody.Herserumtryptophanconcentrationisdecreased.Urinestudiesshowincreasedexcretionofaminoacids,predominantlyalanine
:isoleucine
:leucine,phenylalanine,tryptophan,andvaline.Productionofwhichofthefollowingvitaminsis
mostlikelyimpairedinthispatient?
A)Niacin
B)VitaminB
1
(thiamine)
C)VitaminB
2
(riboflavin)
D}VitaminB
5
(pantothenicacid)
E)VitaminC
CorrectAnswer:A.
Hartnupdiseaseisanautosomalrecessivedisorderinvolvingadefectinakidneyandintestinalneutralaminoacidtransporterprotein.Thisdefectleadstoaminoaciduriaandadecreasedabsorptionofneutralaminoacidsfromthegastrointestinaltractresultingin
deficienciesofneutralaminoacids.Neutralaminoacidsincludetryptophan,phenylalanine,glycine,alanine,vafine
:isoleucine,eucine
:methionine,andproline.Tryptophanisconvertedtoniacin,soadeficiencyintryptophancanresultinniacindeficiency.Niacin
deficiencyischaracterizedbyrash,glossitis,diarrhea,andneuropsychologicaldisturbancessuchasdementiaandhallucinations.
IncorrectAnswers:B,C,D,andE.
VitaminB-|(thiamine)(ChoiceB)isacofactorforseveralenzymesinglucosemetabolismandadenosinetriphosphateproduction,includingpyruvatedehydrogenaseanda-ketoglutaratedehydrogenase.DeficiencyischaracterizedbyWernickeencephalopathy,a
triadofconfusion,ophthalmoplegia,andataxia.Wernickeencephalopathyistheoreticallyreversiblewithadministrationofhigh-dosethiamine;ifuntreated,itcanprogresstoKorsakoffsyndromewhichischaracterizedbydementia,confabulation,hallucinations,and
psychosis.
VitaminB
2
(riboflavin)(ChoiceC)deficiencyischaracterizedbyinflammationandcrackingofskinaroundthelips,mouth,andtongue.ItisnotassociatedwithaminoaciduriaorHartnupdisease
VitaminB
5
(pantothenicacid)(ChoiceD)deficiencyischaracterizedbydermatitis,enteritis,alopecia,andadrenalinsufficiency,itisnotassociatedwithaminoaciduriaorHaitnupdisease
VitaminC(ChoiceE)isfoundinfruitsandvegetablesandisnecessaryfarcollagensynthesis,ironabsorption,immunefunctionandconversionofdopaminetonorepinephrine.Deficiencycausesscurvy,whichischaracterizedbyswollengums,bruisingandpoor
woundhealing,petechiae,perifollicularandsubperiostealhemorrhages,andshort,fragile,curlyhair.
EducationalObjective:Hartnupdiseaseisanautosomalrecessivedisorderinvolvingadefectinakidneyandintestinalneutralaminoacidtransporter,leadingtodeficienciesinneutralaminoacidssuchastryptophan.Tryptophanisconvertedtoniacin,soa
deficiencyintryptophancanresultinniacindeficiency.Niacindeficiencyischaracterizedbyrash,glossitis,diarrhea,andneuropsychologicaldisturbancessuchasdementiaandhallucinations.
It
Previous Next ScoreReport LabValues Calculator Help Pause

ExamSectiont:Item7of50 NationalBoardofMedicalExaminers^
ComprehensiveBasicScienceSelf-Assessment
Y
7.Aminoglycosideantibioticsareusedfortheirsynergisticactionagainstbacteria,incombinationwithotheragents.Theseantibioticsdemonstrateinvitrosynergyforseveralbacterialspecieswhencombinedwithwhichofthefollowingclassesofantibiotics?
A)Fluoroquinolones
B)Macrolides
C)Penicillins
D)Rifamycins
E)Tetracyclines
CorrectAnswer:C.
Aminoglycosidesincludegentamicin,neomycin,amikacin,tobramycin
:andstreptomycin.Theirbactericidalfunctioncomesfromtheinhibitionofthe3GSsubunitofthebacterialribosome,whichprecludesproteinsynthesisbycausingmisreadingofmRNA.
Aminoglycosidesworksynergisticallywithpenicillins,meaningthatthecombinedeffectofthetwoclassesisstrongerthantheeffectofeitherclassalone.Thepenicillingroupofantibioticsincludespenicillinase-sensitivepenicillins(penicillinG
;ampicillin,amoxicillin)
penicillinase-resistantpenicillins(oxacillin,nafcHlin,dicEoxaciflin),andanti-pseudomonalpenicillins(ticarcillin,piperacillin).Theseallinhibitpeptidoglycancross-Einkingofthebacterialwall.Aminoglycosidesalsodemonstratesynergisticactivitywithmonobactams
suchasaztreonam,whichalsotargetpeptidoglycancross-linkingfunction.Howevertheydorequireoxygenfortheiruptakeinfothebacterialcellsoareineffectiveagainstanaerobes.Bacteriamayalsodevelopresistance1othisclassofantibioticsduetothe
inactivationofthedrugbybacterialtransferaseenzymes,whichslightlymodifytheaminoglycosidestructurethroughacetylationorphosphorylation.Aminoglycosideusagemaybecomplicatedbynephrotoxicity,ototoxicity,orneuromusculardamage.Theyshould
notbeusedduringpregnancyastheyarealsoateratogen.
IncorrectAnswers:A.B
(D.andE.
Fluoroquinolones{ChoiceA)inciudeciprofloxacin,levofloxacin,andmoxifloxacin.TheyinhibitprokaryoticDNAgyrase(alsoknownastopoisomerase).Potentialsideeffectsincludevasculardamage,cartilagedamage,tendonitis,ortendonrupture.Theydonetwork
synergisticallywithaminoglycosides.
Macrolides(ChoiceB)includeantibioticssuchasazithromycinanderythromycin.TheseantibioticsalsoinhibitproteinsynthesisbutinsteadbyinhibitingtheSOSsubunitoftheribosome.Useofmacrolideswithaminoglycosideswouldnotprovideadditional
therapeuticbenefitparticularlygiventhattheseclassesofdrugshavesimilarmechanismsofaction.
Rifamycins(ChoiceD)inhibittheRNApolymeraserequiredtotranscribebacterialDMA.Theydonotleadtosynergisticeffectswhenusedwithaminoglycosides.
Tetracyclines(ChoiceE)includetetracycline,doxycycline,andminocycline.Likeaminoglycosides,theseinterferewiththe30Ssubunitoftheribosome.However,ratherthancausingmisreadingofmRNA,theypreventattachmentoftheaminoacyl-tRNA.Theydo
notleadtoasynergisticeffectwhenusedwithaminoglycosides.
EducationalObjective:Aminoglycosidesincludegentamicin,neomycin,amikacin,tobramycin,andstreptomycin.Theyinhibitthe3DSsubunitofthebacterialribosome.Whenusedwithpeptidoglycanlinkingantibioticssuchasthepenicillinclass,theyleadtoa
synergisticeffectandresultinimprovedbacterialkilling.
© © © ©
f*
Previous Next ScoreReport LabValues Calculator Help Pause

ExamSection1:Item8of50 NationalBoardofMedicalExaminers^
ComprehensiveBasicScienceSelf-Assessment
8.A16-year-oldboywithmoderateintellectualdisabilityisbroughttothephysicianforaroutineexamination.Thereisafamilyhistoryofmildandmoderateintellectualdisabilityinhismotherandbrother,respectively.Physicalexaminationshowsalongface
prominentears,andmoderatelyenlargedtesticles.Whichofthefollowingbestdescribesthegeneticmechanismofthispatient'sdisorder?
A)Mutationinamitochondrialgene
B)Presenceofanextrasexchromosome
C)Translocationofaportionofanaufosome
D)TrinucleotiderepeatmutationontheXchromosome
E)Trisomyofanautosome
CorrectAnswer:D.
Thispatientsconstellationoffindings,includingintellectualdisab:rty,anelongatedface
:largeears
:andmacro-orchidismissuggestiveofFragileXsyndrome.Patientsmayalsopresentwithhyperextensiblejointsandahigh-archedpalate.FragileXsyndromeisa
commoncauseofinheritedintellectualdisabilityandisinheritedanX-linkeddominantfashion.ItiscausedbyaCGG-trinucleotiderepeatexpansionwithintheFMR1gene.PatientswithFragileXsyndromeareatincreasedriskofmitralvalveprolapseand
educationaldifficulties.Theyarealsooftendiagnosedwithautism.Aswithalltrinucleotiderepeatexpansions,geneticanticipationisseen,whereinfuturegenerationshaveincreasedseverityand/orearlieronsetofdisease.
IncorrectAnswers:A,B.C,andE.
Mutationinamitochondrialgene(ChoiceA)isobservedinneurologicandmusculardiseasessuchasLeberhereditaryopticneuropathy,maternallyinheriteddiabetesanddeafness.myodonicepilepsywithraggedredfibers,andmitochondria!encephalopathy,
lacticacidosis,andstroke-likeepisodes(MELAS)syndrome.
Presenceofanextrasexchromosome{ChoiceB)sobservedinKlinefeltersyndrome,inwhichpatientshaveanXXYkaryotype.Characteristicfeaturesincludeintellectualdisability,eunuchoidbodyshape,tallstature,elongatedextremities,andhypogonadism.
Translocationofaportionofanautosome(ChoiceC)isobservedinRobertsoniantranslocationsandcausesasmallportionofcasesofDownsyndrome.CharacteristicphysicalfindingsofDownsyndromeincludeintellectualdisability,broad,flat,facialfeatureswith
prominentepicanthalfolds,andasinglepalmarcrease.PatientswithDownsyndromeareatincreasedriskofAlzheimerdementia,acutelymphoblasticandacutemyeloidleukemia,cardiacseptaldefects,duodenalatresia,andHirschsprungdisease.
Trisomyofanautosome(ChoiceE)isobservedinDownsyndrome(trisomy21),Edwardssyndrome(trisomy18),andPatausyndrome(trisomy13).Edwardssyndromepresentswithcharacteristicphysicalfeaturesincludingintellectualdisability,aprominent
occiput,low-setears,micrognathia,clenched,overlappingfingers,andfeetwithaprominentcalcaneusandconvexlyroundedsoles.Patausyndromepresentswithcharacteristicphysicalfeaturesincludingintellectualdisability,deftlipandpalate,
holoprosencephaly,microphthalmia,cutisaplasia,feetwithaprominentcalcaneusandconvexlyroundedsoles,andpolydactyly.
EducationalObjective:FragileXsyndromeisanX-inkeddominantcauseofinheritedintellectualdisabilitythatpresentsduetoaCGG-trinucleotiderepeatexpansionintheFMR1gene,itpresentswithclinicalfeaturesincludingintellectualdisability,anelongated
facewithaprominentjaw.ahigh-archedpalate,largeears,hyperextensiblejoints,andpostpubertalmacro-orchidism.
© © © 0
*
&
Previous Next ScoreReport LabValues Calculator Help Pause

ExamSection1:Item9of50 NationalBoardofMedicalExaminers^
ComprehensiveBasicScienceSelf-Assessment
Y
9.A75-year-oldwomancomestothephysicianbecauseofa3-monthhistoryofanenlarginglesiononherface.Physicalexaminationshowsa1.5-cm,brown-black,mottled,scalylesionwithirregularborders.Microscopicexaminationofabiopsyspecimenof
thelesionshowsatypicalmelanocytesspreadalongthebasilarlayeroftheepidermis.WhichofthefollowingisthemostEikefycauseofthesefindings?
A)Acanthosisnigricans
B)Actinickeratosis
C)Compoundnevus
D)Lentigomaligna
E)Seborrheickeratosis
CorrectAnswer:D.
Malignantmelanomaislikelytobepresentwhenalesiondemonstratesasymmetry,irregularappearingborders,variablecoloration,adiametergreaterthan6mm,andrapidevolutionincharacteristics.Malignantmelanomacouldrapidlyinvadeandmetastasize,
whichcarriesapoorprognosiswhendiagnosedlate.Subtypesincludesuperficialspreading,nodular,lentigomaligna,andacrallentiginousThelentigomalignatypeisclassicallyseeninelderlyindividualsinareasofextensivesundamage,suchastheface.
Lentigomalignatypicallygrowsslowlyandsuperficialalongthedermal-epidermaljunction.Anylesionwithfeaturessuggestiveofmalignantmelanomashouldbesurgicallyexcisedwithnegativemarginsandpathologicallyexaminedforthedepthofderma
*
invasion.
IncorrectAnswers:A.B,C.andE.
Acanthosisnigricans{ChoiceA)ischaracterizedbyhyperpigmented,velvetypatchesseenontheneck,upperback,breasts,andaxillaewhichisamarkerofmetabolicsyndromeanddiabetes.Itisneitherpre-malignantnormalignantandtherearenoatypicalcells
onhistopathologicexamination.
Actinickeratosis(ChoiceB)isapremalignantlesionthatmayprogresstosquamouscellcarcinoma.Clinically,lesionstypicallyappearaslightpink,ill-definedmaculeswithagrittytextureinareasofprolongedsunexposure,suchastheface,ears,anddorsal
hands.
Compoundnevus(ChoiceC)isabenignproliferationofmelanocyteslocatedinboththeepidermisanddermisTheyareverycommon.Compoundnevishouldnotdisplayasymmetry,borderirregularity,ormultiplecolors.Developmentofcompoundandother
benignnevishouldceaseinthefourthtofifthdecade.Neworchangingneviafterthistimeareconcerningformelanoma.
Seborrheickeratosis(ChoiceE)isabenignproliferationoftheepidermis;'esionsexhibitagreasy,adherentappearance.Whileseborrheickeratosesareoftenbrown,thisisduetothekeratinproducedbytheepidermisratherthanmelanin.
EducationalObjective:Lentigomalignaisasubtypeofmalignantmelanomathatmostcommonlymanifestsonthesun-exposedskinofelderlypatients.Itisoftenslowgrowingandtypicallyconfinedfothedermal-epidermaljunction.
© © © ©
f*
Previous Next ScoreReport LabValues Calculator Help Pause

ExamSection1:Item10ol50 NationalBoardofMedicalExaminers^
ComprehensiveBasicScienceSelf-Assessment
Y
10.Aphysicianwishestodeterminetheproportionofnewbornsdeliveredatalocalhospitalwhohadadiagnosisofcongenitalheartdiseasewithinthepastyear.Whichofthefollowingstatisticalmeasurementsbestdescribesthesedata?
A)Attributablerisk
B)Incidence
C)Oddsratio
D)Prevalence
E)Relativerisk
CorrectAnswer:B.
incidenceisanimportantepidemiologicalmeasurethatassessestherateofoccurrenceofnewdiseaseinapopulationat-risk.Incidenceisthenumberofnewcasesexpressedasapercentageofthetotalpopulationatriskoveraspecifiedperiod.Inthiscase
:the
incidencewouldbethenumberofnewbornswithcongenitalheartdiseasedividedbythetotalnumberofnewbornsinthestudysamplewithinthepastyear.Forexample
:if20newbornsarediagnosedwithcongenitalheartdiseaseoutofatotalof1000newborns
deliveredatthelocalhospitalwithinthepastyear,theincidencewouldbe2%.
IncorrectAnswers:A,C,D.andE.
Attributablerisk(ChoiceA)isarepresentationoftheamountofriskthatisassociatedwithaparticularexposure.Formally,itsdefinitionistheincidencerateintheexposedgroupminustheincidencerateinthecontrolgroup.Forexample,ifoverthecourseofa
yearinagroupof100smokerstherearefivemyocardialinfarctions(incidencerateof5%),andinagroupof100non-smokerstherearetwomyocardialinfarctions(incidencerateof2%),theriskattributabletosmokingwouldbe5%minus2%,or3%.
Oddsratio(ChoiceC)isacomparisonoftheoddsofanoutcomeoccurringintheexposedgroupwiththeoddsofthatoutcomeoccurringiinanonexposedcontrolgroup.Usingtheexampleofsmokersandnon-smokersreferencedinChoiceA,theoddsof
myocardialinfarctioninthesmokinggroupis5/95=0.053.Theoddsofmyocardialinfarctioninthenon-smokinggroupis2/93=0.0204.Theoddsratiowouldsimplybetheoddsofmyocardialinfarctioninthesmokinggroupdividedbytheoddsofmyocardial
infarctioninthenon-smokinggroup((5/95)/{2/93))=2.53.
Prevalence(ChoiceD)iscalculatedastheratioofthenumberofpeoplewithadiseasedividedbythetotalnumberofat-riskpersonsinapopulationataparticularpointintime.Thisisalsoknownaspointprevalenceordiseasefrequency.Forexample,ifasurvey
wasconductedofapopulationof1000personsand100oftheseindividualswereidentifiedashavingheartdisease,thepointprevalenceofheartdiseasewouldbe100/1000=0.10
:or10%.
Relativerisk(ChoiceE}comparestheriskofanoutcomeinonegroupwiththeriskofanoutcomeinanothergroupandisoftenusedincohortstudies.Itissimilartoanoddsratioandcanbeconfusedwithanoddsratio.Theoddsratiocanbeusedtoapproximate
relativeriskwhenthediseaseoroutcomestateisrare.Anexamplecalculationofrelativeriskisasfollows.InthedescribedpopulationofsmokersreferencedinChoicesAandC,theriskofmyocardialinfarctioninthenon-smokinggroupis2/100=0.020,or2%.
Theriskofmyocardialinfarctioninthesmokinggroupis5/100=0.05,or5%.Therelativeriskis,therefore,((5/100)/(2/100))=2.50,meaningthattheriskofmyocardialinfarctioninthesmokinggroupis2.5timestheriskinthenon-smokinggroup.Since,inthis
example,myocardialInfarctionisrare,thesmalldiseaseassumptionisvalid,andtheoddsratioapproximatestherelativerisk(2.58-2.50).
EducationalObjective:Incidenceisameasureoftherateofoccurrenceofnewdiseaseinapopulationat-risk.,whichisdistinctfromprevalence,themeasureofthecurrentamountofdiseaseburdeninapopulation.Oddsratioisdistinctfromrelativerisk,althoughif
diseaseburdenisrare,theoddsratiomaybeusedtoapproximaterelativerisk.
© © ©
Previous Next ScoreReport LabValues Calculator Help Pause

ExamSectiont:Item11of50 NationalBoardofMedicalExaminers^
ComprehensiveBasicScienceSelf-Assessment
Y
11A25-year-oldwomancomestothephysicianafterherbloodpressurewasfoundtobe130/105mmHgatahealthfair.Shetakesnomedications.Thereisnofamilyhistoryofhypertension.Herlastmenstrualperiodwas1weekago.Herbloodpressure
todayis130/110mmHg.BilateralabdominalbruitsareheardTreatmentwithanangiotensin-convertingenzyme(ACE)inhibitorwillmostlikelyhavewhichofthefollowingacuteeffectsonthispatient'srenalfunction?
A)Decreasedconcentratingabilitysecondarytorenalangioedema
B)Decreasedglomerularfiltrationratesecondarytodilationofefferentarterioles
C)Decreasedrenalbloodflowsecondarytodilationofafferentarterioles
D}increasedconcentratingabilitysecondarytoachangeinpermeabilityofthecollectingduct
E)Interstitialnephritissecondarytoallergicdrugreaction
CorrectAnswer:B.
Fibromusctilardysplasiaoftherenalarteryisthemostcommoncauseofrenalarterystenosisinyoungerandmiddle-agedwomen.Fibromusculardysplasiaisanon-inflammatoryandnon-atheroscierotrcangiopathyofmedium-sizedarteries(eg,renal,carotid)that
resultsinmultifocalfibrousandmuscularthickeningofthearterialwali.whichcanleadtostenosis.Renalarterystenosisisacauseofsecondaryhypertensionduetoabnormalstimulationofthejuxtaglomerularapparatusfromlowafferentbloodflowleadingto
excessiveproductionofreninandangiotensin.Thereducedafferentbloodflowcanresultinprogressiverenalatrophy.Secondaryhypertensionshouldbeconsideredinnew-onsetortreatment-resistanthypertension,orinyounger,otherwisehealthypatients.ACE
inhibitorsareafirst-linetreatmentforhypertensionastheyblocktheconversionofangiotensin\toangiotensinSI,whichhasdirectvasoconstrictiveeffectsaswellaspromotessaltandwaterretention.ACEinhibitorsmayresultinatransientacutedecreasein
glomerularfiltrationrate(GFR)secondarytodilationofefferentarterioles.Thiseffectismorepronouncedinpatientswithrenalarterystenosis,asthebaselinereducedafferentbloodflowleavesthenephrondependentonefferentarteriolevasoconstriction
(mediatedbyangiotensin11)tomaintainadequatefiltrationpressureacrosstheglomerulus.ACEinhibitorsblockthiseffect.
IncorrectAnswers:A.C,D,andE.
Decreasedconcentratingabilitysecondarytorenalangioedema(ChoiceA)isincorrect.ACEinhibitorsareassociatedwithangioedemaasanadverseeffectduetoincreasedbradykininlevels,whichmayresultinswellingoftheface,lips,tongue,upperairway,and
gastrointestinaltract.
Decreasedrenalbloodflowsecondarytodilationofafferentarterioles(ChoiceC)isincorrectasdilationoftheafferentarterioleswouldresultinincreasedrenalbloodflow.
Increasedconcentratingabilitysecondarytoachangeinpermeabilityofthecollectingduct(ChoiceD)occurswithanti-diuretichormone(ADH)re:easefromthepituitaryorexogenousadministrationofADHanalogs.ADHresultsInincreasedaquaporinexpression
ontheluminalsurfaceofcollectingductcellswhichincreasesthemembranepermeabilitytowater.
Interstitialnephritissecondarytoallergicdrugreaction(ChoiceE)isapossibilitywithmanymedications,butiscommonlyassociatedwithsulfa-baseddiuretics,nonsteroidalanti-inflammatorymedications,antibiotics,protonpumpinhibitors,andrifampin.Patients
classicallypresentwithfever,hematuria,eosinophiluria,rash,andarthralgias.
EducationalObjective:ACEinhibitorsshouldbeusedwithcautioninpatientswithrenalarterystenosis,asreducedefferentarteriolevasoconstrictionmayresultinadecreasedGFR.
© © © oo
Previous Next ScoreReport LabValues Calculator Help Pause

ExamSection1:Item12ol50 NationalBoardofMedicalExaminers^
ComprehensiveBasicScienceSelf-Assessment
Y
12.A16-year-oldboyisbroughttothephysicianbecauseofa3-monthhistoryofshortnessofbreathwhileplayingsports.Hehasnoshortnessofbreathatrest.Hesays,
''WheneverIrunaroundicough,soIdon'twanttobeonthebasketballteamanymore."
Hetakesnomedicationsandhasnoknownallergies.Thereisafamilyhistoryofhypertensionandasthma.Heis165cm(5ft5in)taf!andweighs63kg(150lb);BMIis25kg/m
2Hisrespirationsarel2/min
;andbloodpressureis115/75mmHg.Cardiac
examinationshowsnoabnormalitiesexceptforamidsystolicclickattheapex.Thelungsarecleartoauscultationofthechest.Whichofthefollowingbestexplainsthispatient'ssymptoms?
A)Deconditioning
B)Exercise-inducedasthma
C)Malingering
D)Mitralvalveprolapse
E)Thyroiddisease
CorrectAnswer:B.
Exercise-inducedasthmamostlikelyaccountsforthispatientsexertionaldyspnea.Asthmaischaracterizedbyreversibleobstructionofthebronchisecondarytohyperreactivityandairwayinflammation.Patientspresentwithepisodesofwheezing
:drycough,and
dyspneaoccurringduringorshortlyafterexercise,relievedafterrestortheuseofbronchodilators.Physicalexaminationduringanexacerbationoftenrevealstachycardia,tachypnea,diffusewheezes(orrhonchi)
;andprolongedexpirationrelativetoinspiration.
Decreasedtactilefremitusmaybenotedduetoairtrappingwhichdecreaseslungdensity(leadingtoreducedtransmissionofvibrationsthroughthelungparenchymatothebodywall).Treatmentisusuallywithashortactingbronchodilator(SABA)immediately
beforeexercise
:althoughinpatientswithconcomitantasthmanotrelatedtoexercise
:treatmentisdirectedbytheseverityofunderlyingasthma,ifpatientsdonottolerateSABAs,montelukastisanalternativeoption.
IncorrectAnswers:A.C,D,andE.
Deconditioning(ChoiceA)couldcausedyspnea,orthesubjectiveexperienceofshortnessofbreath,butitshouldnotcauseacough.ItwouldalsobeunlikelyinayoungpersonwithanormalBMIwhohaspreviouslyparticipatedinsportswithoutdifficulty.
Malingering(ChoiceC)isdefinedbyfalsificationofsymptomstoobtainasecondarygain.Thispatient'ssymptomshaveanotherpossibleexplanation,exercise-inducedasthma,andthereisnoclearsecondarygainthathemightobtain.
Mitralvalveprolapse(MVP)(ChoiceD)islesslikelytoexplainthispatient'ssymptomsthanexercise-inducedasthma.Themid-systolicclickofMVPisoften,butnotalways
:followedbyasystolicmurmurofmitralregurgitation(MR)whensymptomaticandcausing
cardiogenicpulmonaryedema.PatientswithcongenitalMVPoftenhavephysicalfindingsincludingscoliosis,pectusexcavatum
:andlowBMI.SymptomsofMVP(ifsympfomatic)aremorelikelytoincludechestpain.palpitations
:andlightheadednessinadditionto
dyspnea.
Thyroiddisease(ChoiceE)isunlikelyinthispatient,asotherassociatedfindingsshouldbepresent.Dyspneacanresultfromhyperthyroidismorthyroidstorm,buttheseconditionsgenerallypresentwithdiaphoresis,weightloss,exophthalmos,andtremor.Thyroid
diseaseisalsolesscommoninmalepatientsofthisagegroup.
EducationalObjective:Exercise-inducedasthmapresentswithdyspnea,cough
:and/orwheezingthatbeginsduringorshortlyafterexercise.ItisusuallytreatedwithSABAsalthoughmontelukastisanalternativetherapy.
Previous Next ScoreReport LabValues Calculator Help Pause

ExamSection1:Item13of50 NationalBoardofMedicalExaminers^
ComprehensiveBasicScienceSelf-Assessment
Y
13.A25-year-oldmancomestothephysicianbecauseofa3-dayhistoryofpainandswellingofhisrightleg.Hehasnohistoryofmajormedicalillnessorrecenttrauma.Examinationoftherightlowerextremityshowsedemaandtenderness.Duplex
uftrasonographyoftherightlowerextremityshowsathrombusextendingintothesuperficialfemoralvein.FurtherstudiesshowproteinCdeficiency.Inactivationofwhichofthefollowingcoagulationfactorsismostlikelyasaresultofthisdeficiencyinthis
patient?
A)FactorsV(proaccelerin)andVIII(antihemophilicfactor}
B)FactorsV(proaccelerin)andIX(plasmathromboplastincomponent)
C)FactorsV(proaccelerin)andXI(plasmathromboplastinantecedent)
D)FactorsVIII(antihemophilicfactor)andIX(plasmathromboplastincomponent)
E)FactorsVIIII(antihemophilicfactor)andXI(plasmathromboplastinantecedent)
F)FactorsIX(plasmathromboplastincomponent)andXI(plasmathromboplastinantecedent)
CorrectAns
1
wer:A.
ProteinCinactivatesfactorsVandVIII;deficiencyofproteinCleadstoahypercoagulablestatepredisposingtothedevelopmentofbothvenousandarterialthrombi.Thecoagulationcascadeconsistsoftheintrinsicextrinsic,andcommonpathways.Theintrinsic
pathwayconsistsofsequentialactivationoffactorsXIl
:XI.IV,andVIII,andactivityofthispathwayismeasuredbythepartialthromboplastintime.TheextrinsicpathwayinvolvestheactivationoffactorVIIandismeasuredbytheprothrombintimeortheINFLBoth
theintrinsicandextrinsicpathwayscantriggerthecommonpathwayviaactivationoffactorXtoXa.ActivatedfactorX(factorXa)hasseveralfunctions,butoneistomodifyfactorVandallowittoformaprothrombinasecompfexwithfactorXa.Thiscomplexhelps
toformafibrinclot.FactorXaisalsonecessaryfortheconversionofprothrombintothrombin.ThrombinbindstothrombomodulinonthesurfaceofendothelialcellsinducingaconformationalchangethatallowsittoactivateproteinC
:whileproteinClocalizestothe
endotheliumbybindingtotheendothelialproteinCreceptor(EPCR).ActivatedproteinCbindstothesurfaceofactivatedplateletsanddegradesfactorsVa.andVilla,whichisthemeansbywhichitexertsitsanticoagulanteffects.
IncorrectAnswers:B.C,D
:E
;andF.
FactorsVandIX(ChoiceB)andFactorsVandXI(ChoiceC)areincorrectasfactorsIXandXIarenotunderinhibitorycontrolbyproteinC.ActivationoffactorIXbyXlaisanimportantstepinthecoagulationcascadeanddeficiencyoffactorIXisassociatedwith
hemophilia.FactorXIactivatesfactorIXintheintrinsicpathway.
FactorsVIIIandIX(ChoiceD).whenactivated,bothtakepartintheintrinsicclottingcascade.WhilefactorVI11isunderinhibitorycontrolbyproteinC,factorIXisnot.
FactorsVIIIandXI(ChoiceE)areincorrectasfactorVIIIisunderinhibitorycontrolbyproteinCbutfactorXIisnot.
FactorsIXandXI(ChoiceF)arebothactivatedirrtheintrinsicdottingcascadeandservetoaccelerateclotting.TheyarenotinfluenceddirectlybyproteinC.
EducationalObjective:ProteinCisanaturalanticoagulantthatisactivatedbythrombinafterthrombinbindstotheendothelialsurface.ActivatedproteinCbindstothesurfacesofactivatedplateletsanddegradesfactorsVaandVilla,therebyexertingnegative
feedbackontheclottingcascade.
© © ©
A oo
Previous Next ScoreReport LabValues Calculator Help Pause

ExamSection1:Item14ol50 NationalBoardofMedicalExaminers^
ComprehensiveBasicScienceSelf-Assessment
Y
14.Whichofthefollowingismostdirectlyresponsibleforconcentratingtestosteroneinthelumenoftheseminiferoustubules?
A)Androgen-bindingprotein
B)Follicle-stimulatinghormone(FSH)
C)FSH/gonadotropin-releasinghormone
D)InhJbrn
E)Luteinizinghormone
CorrectAnswer:A.
Androgen-bindingprotein(ABP)isproducedbytheSertolicellsoftheseminiferoustubulesviatheregulationoffollicle-stimulatinghormone(FSH).TestosteroneisproducedbyLeydigceilsintheinterstitiumadjacenttotheseminiferoustubules
;theproductionof
whichisregulatedbyofluteinizinglormone(LH).Oncereleasedintothelumenoftheseminiferoustubuies
;ABPfacilitatesspermatogenesisbybindingtotestosterone,allowingthisotherwiselipophilichormonetoconcentrateinthelumen.Normal
spermatogenesisrequireshighlocalconcentrationsofluminaltestosterone.
IncorrectAnswers:B,C,D
;andE.
Follicle-stimulatinghormone(FSH)(ChoiceB)isproducedbygonadotropiccellsintheanteriorpituitary.ItplaysanimportantroleinspermatogenesisbystimulatingSertolicellstoproduceABPandbydirectlystimulatingspermdevelopment.Itsroleinthe
concentrationofluminaltestosteronelisindirectandmediatedbyABP
FSH/gonadotropin-releasinghormone(ChoiceC)isproducedbythehypothalamusandstimulatestheproductionandreleaseofFSHandLHfromgonadotropiccellsintheanteriorpituitary.Itsroleintheconcentrationoftestosteroneinthelumenofthe
seminiferoustubuiesisindirectandismediatedbybothFSHandABP.
Inhibin(ChoiceD)isproducedbySertolicellsandexertsnegativefeedbackongonadotropiccellsintheanteriorpituitarytoregulatetheproductionofFSH.
Luteinizinghormone(ChoiceE)isalsoproducedbygonadotropiccellsintheanteriorpituitary.Itisimportantforstimuatingtheproductionoftestosteronebutisnotinvolvedintheprocessofconcentratingtestosteroneinthelumenoftheseminiferoustubules.
EducationalObjective:Androgen-bindingprotein(ABP)isproducedbytheSertolicellsoftheseminiferoustubulesunderthedirectionoffoilic
:
e-stimulatinghormone.Oncereleasedintothelumenoftheseminiferoustubules,ABPfacilitatesspermatogenesisby
bindingtotestosteroneandconcentratingitinthelumenofthetubules.
00#
Previous Next ScoreReport LabValues Calculator Help Pause

ExamSection1:Item15ol50 NationalBoardofMedicalExaminers^
ComprehensiveBasicScienceSelf-Assessment
15.A23-year-oldwomanhashadthelesionsshowninhermouthfor3days.Shehashadfrequentsimilarepisodesoverthepast15years.Thelesionsareexacerbatedbyspicy,salty,andacidicfoodanddrinks.Theylastapproximately1weekandresolve
spontaneously.Visitstothedentistseemtotriggerthedevelopmentofthesores.Whichofthefollowingisthemostlikelydiagnosis?
A)Aphthousulcers
6)Candidiasis
C)Geographictongue
D)Koplikspots
E)Leukoplakia
F)Lichenplanus
G)Psoriasis
CorrectAnswer:A.
Aphthousulcersarepainful,roundtooval,shalloworalulcers.Tneyarethemostcommoncauseofmouthsoresandcanbeidiopathicorrelatedtounderlyingconditionssuchaslupuserythematosus(inwhichcasetheyarenotpainful)orBehgetsyndrome.They
maydemonstratepathergy,whichisthedevelopmentofnewerosionsatthesiteofatrauma,suchasafteradentalprocedure.Whentheselesionsdeveloprecurrentlyadiagnosisofrecurrentaphthousstomatitisismade.Itiscommonlyseeninadolescenceand
youngadulthood,andepisodestypicallydecreasewithincreasingage.Theetiologyismultifactoriat
:butthelesionscanbeexacerbatedbyspicyacidic,orsaltyfoods,asinthiscase.Stresscanalsoleadtoexacerbations.Recurrentaphthousstomatitismaybe
treatedbyoptimizingoralhygiene,avoidingexacerbatingfactors,andtreatingpainwithtopicalanestheticsandcoatingagents.
ncorrectAnswers:B.C.D.E.F.andG.
Oralcandidiasis(ChoiceB)demonstratesthick,whiteplaquesonthetongueorbuccalmucosa,whichcanbescrapedoffwithatongueblade.Itiscommonlyseeninimmunosuppressedindividuals,suchasthosewithpoorlycontrolledHIVinfection,orpatients
usingasteroidinhalerandalteringthenormaloralmicrobiome.
Geographictongue(ChoiceC)isafeatureofpsoriasisandoftenseeninchidren.Thetonguedemonstratesamaze-likepatternofwhite,linearpatches.Ulcersarenotatypicalfeature.
Koplikspots(ChoiceD)arebrightredmaculeswithabluish-whitecenteronthebuccalmucosa,whichareasignofanactivemeaslesinfection.Koplikspotsareaccompaniedbyaprodromalfever,cough,coryza,conjunctivitis,andaconfluentmaculopapular
rashthatstartsattheheadfneckandspreadstothetrunk,excludingthepalmsandsoles.
Leukoplakia(ChoiceE)referstothedevelopmentofwhiteplaquesinthemouth,whichcannotbescrapedoffbyatongueblade,andaretypicallyseenonthetongueorbuccalmucosa.ItmaybeduetoanunderlyingEpstein-Barrvirusinfectionandiscommonin
patientswithHIVinfectionormalignancy.
Orallichenplanus(ChoiceF)ischaracterizedbywhitepatcheswithastellateappearanceonthebuccalandgingivalmucosa.Erosionscanalsooccur,butthewhite,stellatepatcheswillalsobepresent,unlikeinthiscase.
Psoriasis(ChoiceG)mayaffecttheoralmucosaintheformofgeographictongue.However,itismoreclassicallycharacterizedbythick,salmon-coloredplaqueswithsi:very-whitescaleontheextensorextremities.
EducationalObjective:Aphthousulcersarepainful,roundtooval,shalloworalulcers.Theyarethemostcommoncauseofmouthsoresandcanbeexacerbatedbycertainfoods,trauma,oremotionastress.
m
ft"
in
Previous Next SconeReport LabValues Calculator Help Pause

ExamSection1:Item16ol50 NationalBoardofMedicalExaminers^
ComprehensiveBasicScienceSelf-Assessment
y
16.A20-year-oldwomancomestotheemergencydepartment30minutesafterslippingoniceandextendingherhandtobreakherfall.Palpationoftheanatomicsnuff-boxproducespain.Awristx-rayismostlikelytoshowafractureofwhichofthefollowing
carpalbones?
A)Scaphoid
B)Lunate
C)Triquetrum
D)Pisiform
E)Trapezium
F)Trapezoid
G)Capitate
H)Hamate
CorrectAnswer:A.
Fallingontoanoutstretchedhandcanieadtotraumaticinjuriessuchasdistalradiusfractures,elbowdislocations,orfracturesofthecarpalbonesofthewrist.Acommonpatternofinjurywiththismechanismisascaphoidfracture.Thescaphoidboneispartof
thelateralcolumnofthewristandsupportsforcetransmissionfromthehandtothelateralaspectoftheradius.Fracturesofthisbonetypicallypresentwithlateralwristpainandtendernessintheanatomicsnuff-box,whichisthedorsaldepressionbetween
extensorpoMicisiongusandabductorpollicislongus.Thescaphoidhasabloodsupplythatproceedsfromdistaltoproximal.Displacementofafractureofthisbonemayleadtodecreasedbloodsupplyoftheproximalfragment,leadingtoavascularnecrosisand
debilitatingwristpainanddeformity.Becauseofthis,itisimportanttoidentifyandappropriatelytreatscaphoidfractures.
IncorrectAnswers:B.C,D,E.F,G,andH.
Fracturesofthelunate(ChoiceB)areanuncommoninjury.Morecommonlywithhighenergyinjuriestothewrist,thecapitateandtheremainingcarpalbonesmaydislocatefromtheconcavesurfaceofthelunate.Thisisknownasaperilunatedislocationand
mayleadtoacutecompressionofthemediannerve.
Thetriquetrum(ChoiceC)isacarpalbonelocatedintheulnaraspectofthewrist.Fracturesinthislocationareuncommon.Iffractured,itwouldpresentwithtendernessalongtheulnaraspectofthewrist.
Thepisiform(ChoiceD)isacarpalboneintheulnaraspectofthewrist.Itisapea-shapedsesamoidbonethatcanbemistakenforafracturefragment.Fracturesofthepisiformareuncommon.
Thetrapezium(ChoiceE)istheboneatthebaseofthethumbthatcanalsobepalpatedintheflooroftheanatomicsnuff-box.Fractureofthetrapeziumisuncommonandtypicallypresentswithpainaftertraumatothethumb.
Thetrapezoid(ChoiceF)isawedge-shapedcarpalbonejustproximaltothesecondmetacarpal.Fractureofthetrapezoidisanuncommoncarpalinjury.
Thecapitate(ChoiceG)isalargercarpalboneinthecenterofthewrist.Itarticulateswiththelunateandisdislocatedinaperilunatedislocation.
Thehamate(ChoiceH)isacarpalboneintheulnaraspectofthewristthathasaprocessalongitsvolarsurface,referredtoasthehookofthehamate,thatadsasacoveringforthefingerflexortendonsoftheuJnardigits.Fractureofthehookofthehamatecan
occurwhileplacingahighloadthroughthewristwhileholdingahandle(eg,baseballbat,golfclub,sledgehammer).ThisinjurycanleadtocompressionoftheulnarnerveinGuyoncanalandirritationorimpingementoftheadjacenttendons.
EducationalObjective:Fractureofthescaphoidpresentswithradialwristpainandtendernessintheanatomicsnuff-box,typicallyfollowingafallonanoutstretchedhand.Identificationandappropriatetreatmentofthisinjuryisimportantastheretrogradeblood
supplytothisboneplacesitatriskforavascularnecrosisoftheproximalfragment.
Om m
rts
Previous Next ScoreReport LabValues Calculator Help Pause

ExamSection1:Item17of50 NationalBoardofMedicalExaminers^
ComprehensiveBasicScienceSelf-Assessment
Y
17.A76-year-oldmanundergoeslaparotomyforresectionofanabdominalaorticaneurysm.Duringtheprocedure,anincidentalfindingofacquiredcoionicdiverticulaismade.Thediverticulainthispatientaremostlikelypresentinwhichofthefollowing?
A)Ascendingcolon
B)Cecum
C)Descendingcolon
D)Sigmoidcolon
E)Transversecolon
CorrectAnswer:D.
Thesigmoidcolonisthemostcommonlocationfordiverticulatoform.Diverticulaareoutpouchingsofthemucosalandsubmucosallayersintothemuscularlayerthatoccuratweakpointsinthegutwallwherethesmallarteriolesofthevasarectapenetrate,itis
hypothesizedthatabnormalmotilityinthecoloncausesincreasedintraluminalpressurewithsubsequentherniationofthemucosathroughweakpointsinthecolonicwall.Thismayhappeninthesigmoidcotonbecausethediameterofthesigmoidissmallerthan
otherpartsofthecolon,soabnormalperistalsisinthisareacauseshigherintraluminalpressurecomparedtoothersegments.Riskfactorsfordevelopmentofdiverticulaincludeobesityandadiethighinredmeatandlowinfiber.Diverticulosispredisposestolower
gastrointestinalbleeding(Gl).Italsopredisposestodiverticulitis,abacterialinfectionofadiverticulumthatleadstoalocalinflammatoryresponse.Diverticulitispresentsclassicallywithfever,leftlowerquadrantabdominalpain,andoccasionallywithbloodydiarrhea.
IncorrectAnswers:A,B.C.andE.
Theascendingcolon(ChoiceA)canbeasiteofcolorectalcarcinoma(CRC).BecausestoolisgenerallyliquidintheascendingcolonandtumorsinthislocationtendnottobeexophyticC:RCcanpresent[ateassymptomsarelesscommon.Whilediverticulamay
occurintheascendingcolon,itisalesscommonlocationthanthesigmoid.Similarly,thetransversecolon(ChoiceE}anddescendingcolon(ChoiceC)maydevelopdiverticula,butthesesitesarelesscommonthanthesigmoidcolon.
Cecum(ChoiceB)isthejunctionbetweentheileumandtheascendingcolon.ItisnotacommonsiteofdiverticuEosis,butsincetheappendixliesincloseproximitytoit,thececumcanoccasionallybecomeinflamedinsevere,acuteappendicitis.
EducationalObjective:Diverficufosisreferstoaconditioninwhichthecolonicmucosaandsubmucosaherniateintothemuscularlayeratweakpointswherethevasarectapenetratethecolonicwall.Riskfactorsincludechronicconstipationandalowfiberdiet.The
sigmoidcolonisthemostfrequentlyinvolvedpartofthecolonasitssmallerdiameterpredisposestogreaterintraluminalpressureduringperistalsis,therebyincreasingtheriskfordiverticulaformation.
*
It
Previous Next ScoreReport LabValues Calculator Help Pause

ExamSection1:Item13ol50 NationalBoardofMedicalExaminers^
ComprehensiveBasicScienceSelf-Assessment
Y
18.Duringaperiodof36hours,an80-year-oldwomanhasincreasinglysevereabdominalpainfollowedbyfeverchills,tachycardia,hypotensionand.finally,shock.BloodculturesgrowEscherichiacoiiHerconditionworsensand.despitesupportivetherapy
andantibiotics
;shedies4daysaftertheonsetoftheillness.Whichofthefollowingisthemostlikelycauseoftheinitialhypotension?
A)Excessiveproductionofnitricoxide
B)Generationofhydrogenperoxide
G)Hemorrhage
D}InductionofendothelialadhesionmolecuEes
E)Plateletaggregation
CorrectAnswer:A.
Sepsisisasystemicinflammatorysyndromethatresultsfromadysregulatedandexaggeratedimmuneresponsetoaninfection.Sepsiscanbecomplicatedbyshockandmultiorganfailurewithahighmortalityrate.Septicshockischaracterizedbyanimpaired
responseofthevasculaturetovasoconstrictingstimuliwithmarkedlydecreasedsystemicvascularresistance,tachycardia
:increasedcardiacoutput,oliguria
;andlacticacidosis.Excessiveproductionofnitricoxideisassociatedwithhypotensioninthesettingof
sepsis.Induciblenitricoxidesynthetaseisanitricoxide-producingenzymethatisupregulatedthroughtyrosinekinaseactivationinresponsetoproinflammatorycytokinesandbindingbyHpopolysaccharides.Nitricoxideactivatesguanyiatecyclaseinvascular
smoothmuscleresultinginmusclerelaxationandvasodilationbecauseofincreasedintracellularcyclicguanosinemonophosphateconcentration.
IncorrectAnswers:B,C
:D
:andE.
Generationofhydrogenperoxide(ChoiceB)occursinphagolysosomesbytheenzymesuperoxidedismutase
:whichutilizesfreeoxygenradicalsproducedbyNADPHoxidase.Excessfreeradicalproductionisassociatedwithhosttissueinjury.
Hemorrhage(ChoiceC)mayresultinshocksecondarytohypovolemiaanddecreasedoxygencarryingcapacityofthebloodsecondarytolossofhemoglobin.Thispatient'sabdominalpainislikelytheresultofaninfectiousenteritis,colitis,orperitonitis,andisless
likelyfromarupturedabdominalaorticaneurysm,whichmaycauseshockfrominternalexsanguination.
Inductionofendothelialadhesionmolecules(ChoiceD)isakeystepintherecruitmentandmigrationofleukocytestositesofinfectionandinjury.Theactionofnitricoxideonthevascularsmoothmuscleresultsinhypotension,whereastheexpressionofendothelial
adhesionmolecuEesisinvolvedintheimmuneresponsetolocalizedinfection.
Plateletaggregation(ChoiceE)andactivationoccursinresponsetoinflammationandhelpspromotetheinnateimmuneresponse,althoughitdoesnotdirectlycausethepatient
'shypotension.Diffuseplateletaggregationmayresultinthrombocytopeniainthe
settingofsepsis.
EducationalObjective:Sepsisisadysregulatedsystemicinflammatorysyndromethatmayoccurinresponsetoinfection.Itmayprogresstohypotensionandsepticshock.Excessivenitricoxideproductioncausingdiffusevasodilationisoneofthemechanismsof
hypotensioninsepsis.
O0 © 00
Previous Next ScoreReport LabValues Calculator Help Pause

ExamSection1:Item19of50 NationalBoardofMedicalExaminers^
ComprehensiveBasicScienceSelf-Assessment
Y
19.AG5-year-oldwomancomestothephysicianbecauseofa3-monthhistoryofheadache,weaknessofherarms,andleftflankpain:shealsohashada14-kg(31-lb)weightlossduringthisperiod.Physicalexaminationshowsweaknessoftheproximalupper
andlowerextremitymuscles.Thereisaugmentationofstrengthwithrepetitivetestingofthedeltoidmuscles.AnMRIofthebrainshowsasinglewell-demarcatedmasssurroundedbyedemaintherightfrontallobe.Astereotacticbiopsyspecimenofthe
:esionshowsamalignantsmallbluecellneoplasmthatexpressescytokeratin,chromogranirvandsynaptophysin.Whichofthefollowingisthemostlikelydiagnosis?
A)Anaplasticependymoma
B)Extranodalprimarycentralnervoussystemlymphoma
C)Glioblastomamultiforme
D)Primarycerebralneuroblastoma
E)Pulmonarysmaltcel:carcinomametastatictothebrain
CorrectAnswer:E
Thepatient'spresentationismostconsistentwithpulmonarysmalicellcarcinomametastatictothebrain.Pulmonarysmallcelltumorsaretypicallycentrallylocatedinthelungsandassociatedwithtobaccouse.Theyareneoplasmsofneuroendocrineceilsandmay
beassociatedwithnumerousparaneoplasticsyndromes,inctudingCushingsyndromeduetoadrenocorticotropichormoneproduction,syndromeofinappropriateantidiuretichormone,Lambert-Eatonmyasthenicsyndromeduetopresynapticcalciumchannel
antibodyproduction,andparaneoplasticmyelitis,encephalitis,andsubacutecerebellardegeneration.ProximalextremityweaknessandaugmentationofstrengthwithrepetitivetestingofthedeltoidmusclesaresuggestiveofLambert-Eatonmyasthenicsyndrome.
Histologicfeaturesofpulmonarysmallcellcarcinomaincludesmalldarkbluetumorcellslackingnucleoliwithahighnucleartocytoplasmratio.Thebrainisacommonsiteformetastaticdisease.
IncorrectAnswers:A.B,C.and0.
Anaplasticependymoma(ChoiceA)isacentralnervoussystemneoplasmformedfromependymalcells.Locationtypicallyinvolvesthefourthventricle.Histologiccharacteristicsincludeperivascularpseudorosettesformedbymalignantcellsarrangedarounda
bloodvessel.
Extranodalprimarycentralnervoussystemlymphoma(ChoiceB)isararetypeofmalignantnon-Hadgkinlymphoma,oftenassociatedwithanunderlyingimmunodeficiencysyndrome(eg,AIDS).Histologicexaminationmayreveallarge,atypicallymphocytes.
Glioblastomamultiforme(ChoiceC)isamalignantprimarybraintumorcharacterizedbycentralnecrosisonhistology.Imagingfeaturesincludeanexpansilemasscrossingthecorpuscallosumwithsurroundingvasogenicedema.Seizuresareacommonpresenting
symptom,asareheadachesandfocalneurologicdeficits.
Primarycerebralneuroblastoma(ChoiceD)isamalignancyofneuroendocrinecellsassociatedwithsympatheticnervoustissue.Neuroblastomasmayarisefromtheadrenalglands
;thesympatheticchain,orthecentralnervoussystem,andmaysecrete
catecholamines.Metastaticpulmonarysmallcellcarcinomaismorecommoninadultsthanaprimarycerebralneuroblastoma.
EducationalObjective:Solitarybrainlesionsmaybesecondarytoaprimarycentralnervoussystemmalignancy
:metastaticdisease,infectionorabscess.Histologicanalysisandclinicalfeaturescanhelpnarrowthediagnosis.Pulmonarysmallcellcarcinomasare
neuroendocrinetumorsassociatedwithparaneoplasticsyndromesandbrainmetastases.
© © © ©
f*
Previous Next ScoreReport LabValues Calculator Help Pause

ExamSection1:Item20ol50 NationalBoardofMedicalExaminers^
ComprehensiveBasicScienceSelf-Assessment
Y
20.Failureofnormaldifferentiationoftheendodermintheembryoniclungbudismostlikelytoaffectthedevelopmentofwhichofthefollowing?
A)Capillarypatterns
B)Cartilageinbronchi
C)Smoothmuscleonthebronchi
D)Surfactantsecretion
E)Trachealrings
CorrectAnswer:D.
Endodermisoneofthethreeprimaryembryonicgermlayersandcomposestheinnermostlayeroftheearlydevelopingorganism.Endodermderivativesincludetheepithelialliningsoftherespiratorytract,gastrointestinaltractbiliarysystem,genitourinarytract,
vagina,andmiddleear.Organsthatarisefromtheendodermincludetheliver,parathyroidglands,,thymus,pancreas,andthefollicularandparafollicularcellsofthethyroid.SurfactantsecretioninthematurelungisachievedbytypeIIpneumocytes,acomponentof
therespiratoryepithelium.DefectivedifferentiationoftheendodermintheembryoniclungbudwouldmostikelyresultinimpaireddevelopmentoftypeIIpneumocytesandreducedsecretionofsurfactant.
IncorrectAnswers:A,B.C,andE.
Capillarypatterns(ChoiceA),cartilageinbronchi(ChoiceB),smoothmuscleonthebronchi(ChoiceC),andtrachealrings(Choiceb}areallderivativesofthemesoderm.Mesodermisthemiddleembryonicgermlayerandprimarilyresponsiblefordevelopmentof
connectivetissuestructures,includingmuscle,dermis,bone,cartilage,duramater,thecardiovascularsystem,lymphaticsystem,bloodcomponents,kidneys,adrenalcortex,andreproductiveorgans.
EducationaObjective:Ceilsoftherespiratoryepitheliumarisefromtheembryonicendodermgermlayer.ThisincludestypeIpneumocytes.whichformthesimplesquamousepitheliumofthealveoli,andtypeIIpneumocytes,whichsecretesurfactant.
0 0 & % 0
*
Previous Next ScoreReport LabValues Calculator Help Pause

ExamSection1:Item21ol50 NationalBoardofMedicalExaminers^
ComprehensiveBasicScienceSelf-Assessment
Y
21.AG7-year-oldwomancomestothephysicianforahealthmaintenanceexamination.Herbrotherandmotherhaveahistoryofcoloncancer.Thephysicianrecommendscolonoscopy
:tautthepatientsaysthatshewouldpreferonlyforherstooltobetested
forblood.Thephysicianexplainsthattestingthestoolforoccultbloodisnotappropriateinthiscase.Thephysicianismostlikelyconcernedaboutwhichofthefollowingregardingthistest?
A)Lowsensitivity
B)Lowspecificity
C)Potentialforafalse-positiveresult
D}Uncertainnegativepredictivevalue
E)Uncertainpositivepredictivevalue
CorrectAnswer:A.
Sensitivityistheabilityofatesttodetectadiseaseifitispresent.Atestisdescribedassensitiveifithasahighlikelihoodofdiseasedetection,andthereforealowlikelihoodoffalsenegativity.Highsensitivityisthereforeusefulinrulingoutadisease.Thisis
becauseanegativeresultfromahighsensitivitytestindicatesalowlikelihoodthatthediseaseispresent.Becauseofthis,highsensitivitytestsareusefulforscreeninginwhichprovingtheabsenceofdiseaseandlimitingthenumberoffalsenegativeresultsareof
utmostimportance.Inthecaseofcancerscreening,ahighlysensitivetestallowsthecliniciantobeconfidentthatanegativetestmeansthatthepatientisdisease-free.Incontrast,atestwithpoorsensitivity,ifnegative,doesnotprovidestrongevidenceor
confidencethatthepatientdoesnothavethedisease.Inthecasedescribed,thefecaloccultbloodtestdemonstratesbothalowsensitivityandspecificityandisoflimitedusetothephysicianandthepatientforrulingoutcancer.
incorrectAnswers:B,C
:D
:andE.
Lowspecificity(ChoiceB)describesatestthatissubjecttofalse-positiveerrors,meaningthatapositivetestdoesnothaveahighlikelihoodofdisease.Highspecificityisrequiredtoconfirmadiagnosis.Anexampleofhighspecificitytestingforcoloncancerwould
beacolonoscopywithbiopsyofalesionwithstainingandmoleculartestingundermicroscopy.Directvisualizationofmalignantcellsonmicroscopyishighlyspecificforacancerdiagnosis,thatis,thereisalowlikelihoodofafalsecancerdiagnosis.
Potentialforafalse-positiveresult(ChoiceC)ispossiblewithafecaloccultbloodtest.Forexample,ableedinghemorrhoidortakinganironsupplementcancauseapositivefecaloccultbloodtest.Thisisunlikely,however,tocauseconcernbythephysicianinthis
scenarioasafalsepositivetestdoesnotnecessarilyputthepatientatriskforanundiagnosedcolorectalcancer,althoughitmayleadtounnecessaryfurtherdiagnosticexaminations.Missinganearlycancerdiagnosisduetopoortestsensitivitycouldleadtoearly
mortalityandisamorepressingissue.
Uncertainnegativepredictivevalue(ChoiceD)isnotofimmediateconcerninthisscenario.Negativepredictivevalueisbasedonboththetestsensitivityandthepretestprobabilityofthepatienthavingthedisease.Althoughcertainindividualshavehigherorlower
riskofcoloncancerbasedonlifestyleandgenetics,pretestprobabilityandalterationsinthenegativepredictivevaluedonotaltertheneedforasensitivetestforscreeningpurposes.
Uncertainpositivepredictivevalue(ChoiceE)isalsonotanimmediateconcern.Positivepredictivevalueisbasedonbothtestspecificityandthepretestprobabilityofdisease.Positivepredictivevalueisthelikelihoodthatapersonhasadisease,givenapositive
test.Apositivepredictivevalueisofgreaterimportanceinconfirmatorytesting.
EducationalObjective:Highsensitivitytestsarerequiredforeffectivediseasescreening.Highspecificitytestsarenecessaryforconfirmationofthedisease.Negativeandpositivepredictivevaluesarefunctionsnotonlyofsensitivityandspecificity,butalsoofthe
pretestprobabilityofthedisease.
O0 ©
Previous Next ScoreReport LabValues Calculator Help Pause

ExamSection1:Item22ol50 NationalBoardofMedicalExaminers^
ComprehensiveBasicScienceSelf-Assessment
Y
22.A5-year-oldboyisbroughttotheemergencydepartmentafteringesting10ozofahouseholdcleaningsolvent.Heistreatedforacutehepaticandrenalfailurefor1weekandthendischarged.Duringthenextmonth
;regenerationofthisboy'smature
hepatocytesandrenaltubularepithelialcellswillbeaccomplishedmostlybywhichofthefollowingmechanisms?
A)ActivationofstemcellstoenterG
1
phaseofthecellcycle
B)DecreasedapoptosisatGrMtransitionofthecellcycle
C)RecruitmentofcellsfromG
0intothecellcycle
D}Shortenedtimeforprogressionofcellsthroughthecellcycle
E)Terminaldifferentiationbycellsexitingfromthecellcycle
CorrectAnswer:C.
Whenhepatocytesorrenaltubularepithelialcellsaredestroyed,theremainingcellsarerecruitedfromquiescence(thephase)tore-enterthecellcycle.Inhealthypatients,themajorityofhepatocytesandrenaltubularepithelialcellsareintheGQphase.When
cellsaredestroyedsuchasinacuteliverorrenalfailure,genesareinducedthatprimeremainingceilstore-enterthecellcyclefromquiescence.ThesecellstransitionfromGQphasetoG
1
phase,wheregrowthfactorsengendercellgrowth.Ceilsthatgrow
sufficientlysurpasstherestrictionpoint,atwhichpointtheyarecommittedtoDNAreplicationandcelldivisionviamitosis.AftertheG-jrestrictionpoint,thecellstransitiontotheSphase
:whenDNAreplicates.Thecellsenteranothergrowthphase
:theG
2
phase,
andfinallytheMphase,whenmitosisoccursandhepatocytesandrenaltubularepithelialcellsregenerate.
IncorrectAnswers:A.BfD.andE.
ActivationofstemcellstoentertheG
1
phaseofthecellcycle(ChoiceA)doesnotplayamajorroleintheregenerationofhepatocytesorrenaltubularepithelialcells.Stemcellsfromthebonemarroworwithintheliver/kidneyitselfmayminorlycontributeto
regeneration,buttherecruitmentofthelargepopulationofquiescentcellsintothecellcycleismorecrucial.
DecreasedapoptosisatG
^
-Mtransitionofthecellcycle(ChoiceB)andshortenedtimeforprogressionofcellsthroughthecellcycle(ChoiceD)wouldnotleadtoregenerationofhepatocytesorrenaltubularepithelialcells.Mostcellsarequiescent(notinthecell
cycle)atbaselinesodecreasingapoptosisorshorteningthecellcycletimewouldnotleadtoanappreciableincreaseincells.
Terminaldifferentiationbycellsexitingfromthecellcycle(ChoiceE)doesnotoccurafterhepaticorrenaldamageandwouldpreventcells'futureabilitytoregenerate.Manycelltypes(eg,skeletalmusclecells)terminallydifferentiateandlosetheirabilityto
regenerate.
EducationalObjective:ThevastmajorityofhepatocytesandrenaltubularepithelialcellsareintheGQphase(quiescence)atbaseline.Whencellsaredestroyed,theremainingcellsre-enterthecellcycleattheG
1
phasetogrowanddivide,leadingtoregeneration.
0 0 ^
Previous Next ScoreReport LabValues Calculator Help Pause

ExamSection1:Item23of50 NationalBoardofMedicalExaminers^
ComprehensiveBasicScienceSelf-Assessment
Y
23.A17-year-oldboyisbroughttothephysicianbyhismotherbecausesheisconcernedthathispubertyisdelayed.Themotherstates
:"Heissoshort.Hisfatheris6feet5inchestall.Idon'tunderstandwhyhehasnothadhisgrowthspurt."Whenthemother
leavestheroom,thepatientstates
:
"I
'mfine.!don'tknowwhat'sthematterwithher.Shewantsmetobetalllikemydad."Thepatientis175cm(5ft9in)tallandweighs70kg(155lb);BMIis23kg/m
2SexualdevelopmentisTannerstage4.inadditionto
reassuringthemotherthathersonisfine,whichofthefollowingisthemostappropriateinitialstatementbythephysiciantothemother?
A)"Sinceyoursonisfinewithhisheightyoushouldtrytoaccepthimasheis."
B)"Tel:memoreaboutyourconcernsaboutyourson'sheight.
"
C)"We’lldosomebloodtestsjusttobesurethatallyourson
'shormonelevelsareokay."
D)"Yoursonisaverageforhisheightandweight."
E)"Yoursonisnotgoingtobeanytaller."
CorrectAnswer:B.
Whenpatientsorpatients
'familymembersexpressmedicalconcems
:physiciansshouldinitiallyaskopen-endedquestionstoexploretheunderstandingandfearsofthepatientorfamilymember.Thephysiciancanthentailorfurtherdiscussionandreassurancesto
addresstheseknowledgegapsandfears.Askingopen-endedquestionsalsoinvitesthefamilymembertoelaborateontheirconcerns,astheremaybemedicallyorpsychfatricallyrelevantdetailsthatthefamilymemberrevealsonfurtherdiscussion.Further,
listeningtothespecificconcernsofthepatientorfamilymemberswillimprovetherapeuticalliance.
IncorrectAnswers:A.C
;D,andE.
Givingparentaladvice(ChoiceA)wouldbeunwarrantedinthissituationandoutsideofthephysician'sscopeofpractice.Ifaparent'sbehaviorisclearlyaffectingthementaorphysicalhealthofthepatient,thephysicianmaytactfullybringtheissuetotheparent's
attention.Howeverthispatient'shealthisnotclearlyimpactedbyhismother'sconcern.
Physiciansshouldrefrainfromorderingteststhataremedicallyunnecessarybasedonpatientorfamilyconcern(ChoiceC).Thephysicianshouldinsteadreassureandeducateafterlisteningtothepatientorfamily
'sspecificconcerns.
Sayingthatthepatientisaverageforhisheightandweight(ChoiceD)wouldnotaddressthismother'sconcernaboutthepatientbeingshorterthanhisfatherTnisstatementwouldalsopreventelaborationofthemother'sspecificconcerns,understanding,and
fears.
Informingthepatient'smotherthathersonisnotgoingtobeanytaller(ChoiceE)wouldnotbeaccurateorreassuring.Thisstatementwouldalsopreventelaborationofthemother’sspecificconcerns,understanding,andfears.
EducationalObjective:Whenpatientsorfamiliesexpressmedicalconcernsphysiciansshouldaskopen-endedquestionstoelucidatethespecificnatureoftheconcernandthepatient'sorfamilymember'sunderstanding.Thephysiciancanthentailorfurther
discussiontoaddressknowledgegapsandspecificfears,andmaylearnadditional,medicallyrelevantdetailsabouttheconcern.
© © ©
Previous Next ScoreReport LabValues Calculator Help Pause

ExamSection1:Item24ol50 NationalBoardofMedicalExaminers^
ComprehensiveBasicScienceSelf-Assessment
y
24.A34-year-oldwomanisadmittedtothehospitalfortreatmentofpulmonarytuberculosis.InfliximabtherapywasinitiatedGmonthsagoforsevereCrohndisease.Thispharmacotherapymostlikelyinhibitedwhichofthefollowingimmunologicfunctionsinthis
patient?
A)ActivationofnuclearfactorKBtoinduceexpressionofinterleukin-10(IL-10)
B)Directtoxicitytothecausalorganism
C)Maintenanceofgranulomas
D}Recruitmentofsegmentedneutrophilstoingestandkillthebacteria
E)StimulationofBlymphocytestoproduceneutralizingantibodiesagainstthecausalorganism
CorrectAnswer:C.
Tumornecrosisfactor-a(TNF-a)isacytokinesecretedbymacrophages,whichsupportsgranulomaformation.Granulomasarecollectionsofhistiocytes,ormacrophageswithabundantpinkcytoplasmthatoftencontainmulti-nucleatedgiantcellsandare
surroundedbylymphocytesOnepurposeofgranulomaformationistosequesteraninfectionorforeignbody.MonoclonalantibodytherapytargetedagainstTNF-aincreasesapatientsriskforactivemycobacterialinfectionbecauseitstopstheproductionofTNF-a,
leadingtobreakdownofthegranulomaandreleaseofanycontainedorganism.MonoclonalantibodiesagainstTNF-athatarespecificallyusedtotreatCrohndiseaseincludeinfliximab,adalimumab,andcerfclizumab.Thesemedicationsareknowntoincreasethe
riskofreactivatinglatentMycobacteriumtuberculosisbecauseoftheirdeleteriouseffectongranulomaformationandmaintenance.AllpatientswhoareconsideredfortreatmentwiththeseagentsmustundergoscreeningforlatentMtuberculosisinfection.If
positive,treatmentforlatentM.tuberculosiswithninemonthsofisoniazidisindicated.
IncorrectAnswers:A.B,D.andE.
ActivationofnuclearfactorKBtoinduceexpressionofinterleukin-10(IL-10){ChoiceA)occurswithintheTh
2celltype.1L-1GattenuatestheimmuneresponsebyinhibitingactivatedmacrophagesanddecreasingexpressionofTh
1
cytokines.InhibitionofthisIL-1D
wouldthereforestimulatetheimmuneprocessagainstinfectiousorganisms,notdecreaseit.
TNF-aisacytokinethatmaintainsgranulomasandassistsinwallingoffinfectionssuchastuberculosis.Itdoesnothaveanydirecttoxicitytothecausalorganism(ChoiceB),andthusthisisnotthemechanismbywhichanti-TNF-amonoclonalantibodiesincrease
theriskoflatenttuberculosisreactivation.
IL-8,notTNF-a,isresponsiblefortherecruitmentofsegmentedneutrophilstoingestandkillthebacteria(ChoiceD).ThisprocessisnotaffectedbytheinhibitionofTNF-a.
Rituximab,anairti-CD20monoclonalantibody,diminishesBlymphocytesandisusedinchemotherapyregimensforlymphoma.Itisalsousedinthetreatmentofautoimmuneconditionsincludingvasculitis,rheumatoidarthritis,andpemphigusvulgaris.This
monoclonalantibody,notaTNF-ainhibitor,preventsstimulationofBlymphocytestoproduceneutralizingantibodiesagainstthecausalorganism(ChoiceE).
EducationalObjective:TheuseofmonoclonalantibodytherapyagainstTNF-aisassociatedwithanincreasedriskofreactivatedlatentM.tuberculosissecondarytoinadequategranulomamaintenance.Becauseofthis,allprospectivecandidatesforthistherapy
shouldbescreenedforlatentM.tuberculosisinfectionandaccordinglytreatedifpositive.
© © © ©
f*
Previous Next ScoreReport LabValues Calculator Help Pause

ExamSection1:Item25ol50 NationalBoardofMetlicaiExaminers^
ComprehensiveBasicScienceSelf-Assessment
y
25.An8-year-oldboycontinuestobleedexcessivelyaftertoothextraction.Prothrombintime,bleedingtime,andplateletcountarewithinthereferencerange.Partialthromboplastintimeisprolongedbutcorrectsafteradditiontotheassaychamberofplasma
fromapatientwithhemophiliaA.Whichofthefollowingisthemostlikelydiagnosis?
A)Acutedisseminatedintravascularcoagulation
B)FactorV(proaccelenn)deficiency
C)FactorVt!(proconvertin)deficiency
D)HemophiliaA
E)HemophiliaB
F)Immunethrombocytopenicpurpura
G)vonWillebranddisease
CorrectAnswer:E
HemophiliaBisanX-linkedbleedingdisordercausedbyabsent,decreased,ordysfunctionalfactorIXandmostlikelyexplainsthispatient
'sprolongedbleedingandincreasedpartialthromboplastintime(PIT).AspatientswithhemophiliaAlackfactorVIIIbut
havenormallevelsoffactorIXfadditionofplasmafromapatientwithhemophiliaAwouldcorrectthePTTasdemonstratedmthispatientwithhemophiliaB.FactorIXisacomponentoftheintrinsicclottingcascadeandservestoactivatefactorXtoXa.which
subsequentlyconvertsprothrombintothrombinandfacilitatestheformationofafibrinclot.TheactivityofthecoagulationfactorsintheintrinsiccoagulationcascadeismeasuredbythePTTwhiletheactivityoftheextrinsicpathwayismeasuredbythe
prothrombintime(PT)
fwhichisnormalinthispatient.Theclinicalseverityofhemophilia6isvariable.Patientswithseverediseasepresentearlyinlifewitheasybruising,bleedingfollowingaminorprocedure,orhemarthrosis.Patientswithlessseveredisease
maynotpresentuntiltheyexperienceaneventsuchastraumaorsurgeryTreatmentincludesreplacementofthedeficientfactor.
IncorrectAnswers:A.R,C.D,F,andG.
Acutedisseminatedintravascularcoagulation(ChoiceA)isasyndromecharacterizedbyoverwhelmingactivationoftheclottingcascadeoftenprecipitatedbymalignancy,sepsis,orobstetricalemergencies.Endothelialdysfunctionleadstotheformationof
microthrombianddepletionofcoagulationfactors.Microthrombicauseshearingstressonerythrocytesleadingtomicroangiopathichemolyticanemia,whilethedepletionofcoagulationfactorsmanifestsasaprolongedFTandPTTandincreasestheriskofmajor
bleeding.
FactorV(proaccelerin)deficiency(ChoiceB)isarareinheritedbleedingdisorderthatmaypresentwithmucocutaneousbleedingormajorbleedingfollowingtraumaorsurgery.FactorVisrequiredasacofactorfortheformationofthrombininthecommon
pathwayofthecoagulationcascade.Deficiencyistreatedwithfreshfrozenplasma(FFP)
fwhichcontainsfactorV.
FactorVII(proconvertin)deficiency(ChoiceC)IsararebleedingdisorderwithaspectrumofclinicalseverityPatientswhoaremostaffectedpresentwithheavymenstrualbleedingorbleedingfollowinginvasiveprocedures.Treatmentiswithfactorreplacement,
prothrombincomplexconcentrate,orFFP.
HemophiliaA(ChoiceD)isanX-linkedbleedingdisorderthatpresentssimilarlytohemophiliaBandiscausedbyanabsentorreducedleveloffactorVIII.AdditionofplasmafromanotherpatientwithhemophiliaAwouldnotcorrectthePTT.
Immunethrombocytopenicpurpura(ChoiceF)iscausedbycirculatingantibodiesagainstplateletsthatleadstothrombocytopenia.ThePTandPTTarenormalBleeding,ifitoccurs,tendstobemucocutaneous.
vonWillebranddisease(ChoiceG)isoneofthemostcommonhereditarybleedingdisordersandisduetoquantitativeorqualitativeabnormalityofvonWilebrandfactor,whichbindsplateletsandsubendothekaicollageninprimaryhemostasis.Impairedplatelet
adherenceleadstoaprolongedbleedingtime.Itcanpresentwithepistaxis,gingivalbleeding,petechiae,easybruising,andmenorrhagia.
EducationalObjective:HemophiliaBisanX-linkedbleedingdisorderthatiscausedbyadeficiencyinfactorIXleadingtoanincreasedPTT.Symptomsdependonseveritybutincludeprolongedbleedingfollowinginvasiveproceduresortrauma,easybruising
andhemarthrosis.TreatmentiswithrecombinantfactorIX.
o m m
Previous Next ScoreReport LabValues Calculator Help Pause

ExamSection1:Item26ol50 NationalBoardofMetlicaiExaminers^
ComprehensiveBasicScienceSelf-Assessment
y
26.Thediagramshowsthemajorfactorsthatdeterminebloodpressure.Whichofthefallowinglabeledfactorsisaffectedmostbyanaradrenergtcantagonist?
^
Centralnervoussystem
'

Peripheralresistance
(arteriolar)1=fCardiacoutputjxBloodpressure
©
^
Strokevolume]f
Heartrate
CD
"
Contractilityj£
Venousreturn
"
^
Bloodvolume!Capacitancevesseltone
'
(venolar)
©
A)
B)
C)
D)
El
CorrectAnswer:C.
Theautonomicnervoussystemplaysaprimaryroleinthemaintenanceofbloodpressureviaitseffectsonperipheralarteriolarresistance
:heartrate,myocardialcontractility
:venouscapacitance,andtoanindirectdegree.,bloodvolume.Thesympatheticnervous
systemprimarilyactsthrougha-and(3-adrenergicreceptors,whicharestimulatedbydopamine,epinephrine
:andnorepinephrine.Inrelationtobloodpressureregulation,thea
t
receptorleadstosmoothmusclecontractionespeciallyofthevasculature,which
increasesbloodpressurebyincreasingsystemicvascularresistance.Thep
;receptorincomparison,causesincreasesiinheartrate,myocardiatcontractility,andreninrelease,whichallleadtoincreasesinbloodpressure.Anantagonistatthea
1
receptorwould
decreasevascularsmoothmusclecontractiontoagreaterextentaroundarteries,leadingtovasculardilationanddecreasedbloodpressureasmeanarterialpressureisdirectlyrelatedtocardiacoutputandsystemicvascularresistance.
IncorrectAnswers:A.Ei.D,andE.
Centralnervoussysteminputstocardiacoutput(ChoiceA)canoccurthroughthelimbicsysteminresponsetostrongemotionsoranticipationofphysicalactivityorthroughcentralcomponentsoftheautonomicnervoussystem(eg.brainstem,spinalautonomic
preganglionicneurons).Theseautonomicstimulitotheheartwouldbelessaffectedbyana1receptorantagonistthanperipheralresistance,asthestimuliwouldalsoincludeeffectsonthep,receptorBycontrast,centrala2-adrenoreceptorsarethetargetofthe
agonistclonidine,whichresultsindiminishedsympathetictone,,formingthebasisfortheuseofclonidineinhypertensiveemergency.
Heartrate(ChoiceB)isprimarilyaffectedbythepireceptor,withagonismleadingtoincreasesinheartrate.Aspartoftheparasympatheticnervoussystem,heartrateisdecreasedbyM2receptors.a
1
receptorantagonistsdonotaffectheartratedirectly,though
cancausereflextachycardia.
Bloodvolume(ChoiceD)isaffectedbychangesinredbloodcellsandplasmavolume,suchthatanemiaorhemorrhagewoulddecreasebloodvolume.Howeverchangesinplasmavolumecanalsobemediatedbydiuretics(decreasingplasmavolume)orrenin
release(increasingplasmavolume).a
1
receptorsdonotmoderatebloodvolume.
Capacitancevesseltone(ChoiceE}describestheabilityofthevesseltoholdavolumeofbloodataspecificbloodpressure.Thecapacityofthevenularcompartmentincreaseswithdecreasedsomaticmusclemovement,valvulardysfunction,andnitroglycerin
administration.Venuletonedecreaseswith,receptorblockade,buttoalesserextentthanperipheraarteriolarresistanceasthereisminimalvascularsmoothmuscleinvenulescomparedtoarterioles.
EducationalObjective:Theautonomicnervoussystemplaysaprimaryroleinthemaintenanceofbloodpressureviaitseffectsonperipheralarteriolarresistance,heartrate,myocardialcontractility,venouscapacitance,andtoanindirectdegree,bloodvolume.
aradrenergicreceptorsprimarilymodulatemeanarterial
-
pressurebyincreasingperipheralarteriolarresistance.
© © m
-
"
s
7
Previous Next SconeReport LabValues Calculator Help Pause

ExamSection1:Item27of50 NationalBoardofMetlicafExaminers^
ComprehensiveBasicScienceSelf-Assessment
y
-Y
'NT
VI
V
5 iVft
3!
V EZI
aVL V2 v?
3I J PW VS
R
!\
/I
:
VI
I1
v__
._.J
II
U\._
l
'
V...l~A....
VS
27.Apreviouslyhealthy21-year-oldwomancomestotheofficebecauseofa2-monthhistoryofshortnessofbreathandfatigue.Hermostrecentmenstrualperiodwas3monthsago.Mensespreviouslyhadoccurredatregular28-dayintervals.Shetellsthe
physicianthatshethinksshemaybepregnant.Shetakesnomedicationsandhasnotseenaphysicianforseveralyears.Sheappearshealthy.Sheis160cm(5ft3in)tallandweighs54kg(120lb);BMIis21kg/m
2Vitalsignsarewithinnormallimits.The
lungsaredearCardiacexaminationshowsanormalS
1
;awidelysplitS
2thatdoesnotchangewithrespiration,andagrade3/6holosystoMcmurmurthatisloudestatthelowerleftsternalborderandradiatestotheupperleftsternalborderEGGisshown.
Themostlikelycauseofthesefindingsisdysfunctionofwhichofthefollowingstructures?
A)Atrialseptum
B)Ductusarteriosus
C)interventricularseptum
D)Pulmonicvalve
E)Tricuspidvalve
CorrectAnswer:A.
AtrialseptaldefectisacommoncongenitalmalformationoftheinteratrialseptumThemostcommontypeisanostiumsecundumdefectalthoughostiumprimumdefectsareassociatedwithtrisomy21.Theatrialseptaldefectresultsinaleft-to-rightshuntwith
abnormalflowofbloodfromtheleftatriumtotherightatrium,resultinginrelativevolumeoverloadoftherightatriumandventricle.Thisincreasedstrokevolumeoftherightventricleresultsindelayedclosureofthepulmonicvalve,whichpresentsasafixed,split
S
2landlow-gradephysiologicejectionmurmuroncardiacauscultation.Theincreasedrightheartvolumesalsoresultinaprominentrightventricularimpulseonphysicalexamandmaypresentanincreasedriskforthedevelopmentofarightbundlebranchblock,
whichmaybepresentonECGasseeninthiscase.Iftheatrialseptaldefectremainsuncorrected,itcanresultinthedevelopmentofEisenmengersyndromesecondarytoprolongedpulmonaryvasculatureremodelingresultinginpulmonaryarterialhypertension
andshuntreversalleadingtocyanosis.Asymptomaticatrialseptaldefectsmaybecomeclinicallysignificantinthesettingofincreasedbloodflow(suchasduringpregnancy).
IncorrectAnswers:B,C
:D.andE.
Apatentductusarteriosus(ChoiceB)isapersistentextracardiacconduitbetweentheaortaandthepulmonaryarterythathasfailedtoobliterateafterbirth.Itresultsinacontinuous,machine-likemurmurbestheardintheleftsecondintercostalspace,radiatingto
theclavicle.
Defectsoftheinterventricularseptum(ChoiceC)arecharacterizedbyahoiosystolicmurmurbestheardintheleftlowersternalborderTheydonotclassicallyresultinafixed,splitS
2
increasedflowacrossthepulmonicvalve(ChoiceD)duetoshuntingofbloodfromtheleftatriumtotherightatriumresultsinthetixedS
2associatedwithanatrialseptaldefect.Thepulmonicvalveitselfistypicallynorma!.
Tricuspidvalve(ChoiceE)dysfunctionresultsintricuspidregurgitation,whichpresentsasahoiosystolicmurmurbestheardintheleftlowersternalborder.
EducationalObjective:Afixed,widelysplitS
2ischaracteristicofanatrialseptaldefectduetoincreasedbloodflowthroughthepulmonicvalve.SeveredefectscanresultinpulmonaryhypertensionanddevelopmentofEisenmengersyndromeovertime,with
reversalofthelefttorightshunt.
o© 0
*
0
*
K
Previous Next ScoreReport LabValues Calculator Help Pause

ExamSection1:Item28ol50 NationalBoardofMetlicaiExaminers^
ComprehensiveBasicScienceSelf-Assessment
y
MJW
_:
4
*
m
i «
-
V>
I
LQJI.fc*
L
'
.i
i
HI
"
T
^W.
JP
TP
r
1
*1?.£
p
. L
s'm
-f
i“
- —
1
'
A
.>CJ
*
-
c fc
“VO
i.
..Ip
r
SC
"
fi
J
p
Ol
IT
*r
£ T
m
a
.-i-
E
1
i
a p
tr
h.
d
k
^p
«
J-
»
R
a
I
i
a.
a.
£
"
a J
-
K
28.A53-year-oldmanhashadprogressivedifficultyswallowingforthepast3months.Hehasa10-yearhistoryofheartburnwithesophagealregurgitationofgastriccontents.Tissueobtainedonbiopsyofthelowerthirdoftheesophagusisshown.Whichofthe
followingbestdescribesthenatureofthislesion?
A}Basalzonehyperplasiaofsubmucosalglands
B)Intestinalmetaplasiaofsquamousepithelium
C)Malignanttransformationofepitheliumintosquamouscarcinoma
D)Squamousmetaplasiaofsubmucosalglands
CorrectAnswer:B.
Jntestina
1
metaplasiaofsquamousepitheliumintheesophagus,alsoknownasBarrettesophagus,canbeaconsequenceofprolongedgastroesophageairefluxdisease(GERD)
:whichoccurswhenacidicgastriccontentsrefluxbackwardthroughthelower
esophagealsphincterintotheesophagus.Themucosaoftheesophagusiscomprisedofsquamousepithefiumanddoesnottraditionallyencountersuchanacidicenvironment.Constantexposuretoacidicintraluminalcontentsinducesachangeincelltypefrom
squamousepitheliumtothecolumnarglandularepitheliumfoundintheintestinesasanadaptiveresponse.Thesemetaplasticcellswillexhibitabrushborderandgobletcells.Metaplasiacaneventuallyleadtodysplasia,whichispremalignant.Patientswith
confirmedBarrettesophagusshouldbeevaluatedatregularintervalsdeterminedbythepresenceand/orgradeofdysplasia.TreatmentinvolvesablationofthedysplasticcellsviaendoscopyandmanagementoftheunderlyingGERDwithaprotonpumpinhibitor,
dietarymodification,andsmokingcessation.
IncorrectAnswers:A,C
;andD.
Basalzonehyperplasiaofsubmucosalglands(ChoiceA)isnotthepathologicchangeobservedinBarrettesophagus,althoughsubmucosalglandsecretionsdoneutralizeacidicluminaicontents.Theyalsolubricatetheesophaguswhichallowsforthefoodbolus
topass.
Malignanttransformationofepitheliumintosquamouscarcinoma(ChoiceC)occurswithesophagealsquamouscarcinoma,whichismorecommoninpatientswhoconsumealcoholandsmokecigarettes.Barrettesophagusprimarifypredisposesto
adenocarcinoma,nottosquamouscarcinoma.
Squamousmetaplasiaofsubmucosalglands(ChoiceD)isalsoassociatedwiththedevelopmentofesophagealadenocarcinoma.Submucosalglandscontainprogenitorcellsthatmayplayaroleinthepathogenesisofdysplasiaastheseprogenitorcellsserveas
asourceofpotentiallydysplasticorneoplasticcells,however,thepathophysiologyofBarrettesophagusinvolvesintestinalmetaplasia.
EducationalObjective:BarrettesophagusdevelopsinindividualswithchronicGERDandishistologicallycharacterizedbyintestinalmetaplasiawherebythenormalsquamousepitheliumisreplacedbycolumnarepithelium.Overtime,,dysplasiacandevelop,
predisposingtoesophagealadenocarcinoma.
O0 m (E
T
Previous Next SconeReport LabValues Calculator Help Pause

ExamSection1:Item29of50 NationalBoardofMedicalExaminers^
ComprehensiveBasicScienceSelf-Assessment
Y
29.A50-year-oldmancomestotheemergencydepartmentbecauseofa2-weekhistoryofprogressiveshortnessofbreath.Hispulseis90/min,respirationsare26/mrn.andbloodpressureis120/80mmHg.Physicalexaminationshowsnootherabnormalities.
Laboratorystudiesshow:
ArterialPco
2
ArterialPo
2
Arterial02content
MixedvenousPo
2
Mixedvenous02content8vol%(N=10%-16%)
30mmHg
96mmHg
12vol%(N=17%-21%)
36mmHg
Whichofthefollowingisthemostliketyexplanationforthesefindings?
A)Anemia
B)Drug-inducedalveolarhypoventilation
C)Residenceatahighaltitude
D)Severeregionalmismatchingofalveolarventilationandpulmonarycapillaryperfusion
E)Voluntaryhyperventillation
CorrectAnswer:A.
Thedifferentia!fordyspneaisbroadandencompassesarangeofdisordersthatinvolveimpaireddeliveryofoxygentotissueand/orreducedeliminationofcarbondioxidefromthebody.Thelaboratorystudiesinthiscaseindicatesareducedarterialandvenous
oxygencontent.Theoxygencontentofthebloodisafunctionoftheoxygencarryingcapacity(essentiallythehemoglobinconcentration),percentsaturationofhemoglobin,andpartialpressureofdissolvedmolecularoxygen(Po
^
Theequationtocompute
oxygencontentisthus:Oxygencontent=1.34*[Hemaglobin]*(ArterialOxygenSaturation)+G.003*(ArterialPoJ
.Theamountofdissolvedoxygenisnegligiblecomparedtotheoxygentransportedbyhemoglobin.Thepatientinthiscasehasreducedoxygen
contentinthearterialandmixedvenouscirculationwithanormalarterialPo
2Intheabsenceofahemoglobinopathy(eg.methemoglobinemia,carboxyhemoglobinemia),thepatient'soxygensaturationisexpectedtobenormal.Themostlikelydiagnosisis
anemiawithadecreasedhemoglobinconcentration.ThearteriaiPco
2
isdecreasedindicatinghyperventilation,whichisexpectedinthesettingofdecreasedoxygendeliverytotissue.
incorrectAnswers:B,C
:D
:andE.
Drug-inducedalveolarhypoventilation(ChoiceB)wouldresultinanincreasedarterialPco
^
Potentialetiologiesincludecentralnervoussystemdepressantssuchasopioidanalgesicsandbenzodiazepines.
Residenceatahighaltitude(ChoiceC)wouldbeexpectedtoresultinadecreasedPo
2
withadaptivechangesthatmaintainanadequateoxygencarryingcapacity.Thesechangesincludesecondaryerythrocytosiswithincreasedhemoglobinconcentrationand
increasedlevelsof2,3-bisphosphoglycericacid
:whichstabilizesthedeoxygenatedstateofhemoglobinandpromotesincreasedoxygenreieasetotissue.
Severeregionalmismatchingofalveolarventilationandpulmonarycapillaryperfusion(ChoiceD)occurswheneitherventilationorperfusiontoaregionoflungisimpaired.Anexampleispulmonaryembolism,inwhicharegionofventilatedlunghasobstructed
bloodflow.
Voluntaryhyperventilation(ChoiceE)resultsinarespiratoryalkalosis,withadecreasedPco
^
.Symptomsincludedizziness,weakness,andsyncope.Theoxygencarryingcapacityofthebloodwouldnotbereducedintheabsenceofotherfactors
EducationalObjective:Deliveryofoxygentotissueislargelydependentonthehemoglobinconcentrationasitistheprimarytransporterofoxygenintheblood.Othercontributingfactorsincludetheoxygensaturationofhemoglobin,and,lessSignificantly,the
partialpressureofdissolvedmolecularoxygen.
O0
Previous Next SconeReport LabValues Calculator Help Pause

ExamSection1:Item30ol50 NationalBoardofMedicalExaminers^
ComprehensiveBasicScienceSelf-Assessment
Y
30.Aninvestigatorisstudyingtheeffectsoftriiodothyronine(TJandthyroxine(TJinhepatocytesinanexperimentalanimalmodel.Whichofthefollowingbestdescribestheactionofthesethyroidhormonesonthistargettissue?
A)BothT
3
andT
4
bindtothemelanocortin2receptoronthecellsurface
B)BothT
5
andT
4
enterthenucleus
C)T
3isconvertedtoI
4inthecytosol
D)ThyroidhormonereceptorspreferentiallybindT
4
overT
3
CorrectAnswer:B.
BothT
3
(triiodothyronine)andT
4
(thyroxine)arehormonesthatactonnuclearreceptors,requiringthemtoenterthetargetceiltoexerteffects.Unlikeotherlipophilichormones,thyroidhormonescontainchargedaminoacidsthatpreventpassivediffusionacrossthe
cellularmembraneandthusenterbyfacilitateddiffusion.ThyroidhormonetransporterstransportbothT
3andT
4
intothecelltoreachtheirreceptors.ThethyroidhormonereceptorsarenuclearreceptorsthatcontainDNA-bindingdomains.Nuclearreceptorscan
initiallybeineitherthecytosolornucleus.Oncenuclearreceptorsbindtheirrespectivehormones
:theytranslocateintothenucleus,ifnotalreadythere,wheretheyactasDNAtranscriptionfactorstoregulatetheexpressionoftargetgenes.
IncorrectAnswers:A.C
;andD.
Bindingtothemelanocortin2receptoronthecellsurface(ChoiceA)doesnotoccurwitheitherT
3orT
4
.Themelanocortin2receptorisalsoknownastheadrenocorticotropichormone(AGTH)receptorandisspecificforACTH.ThisreceptorisaGprotein-coupled
receptoranddoesnotactivelytransportACTHinsidethecell.
ConversionofT
3toT
4inthecytosoi(ChoiceC)doesnotoccur.T
4isthelessactiveformofthyroidhormoneandisconvertedtoT
3Intargetcells.Onceinthecellnucleus,T
3
preferentiallybindsthereceptorwithgreateraffinitythanT
4
(ChoiceD)
:althoughboth
hormonesarecapableofbindingandactivatingthereceptor.
EducationalObjective:T
3andT
4
actonnuclearreceptors,requiringthemtoenterthetargetcelltoexerteffects.Unlikeotherlipophilichormones,thyroidhormonescontainchargedaminoacidsthatpreventpassivediffusionacrossthecellularmembrane.Thyroid
hormonetransporterstransportbothT
3andT
4intothecelltoreachtheirreceptors.
0 0 # 0
f*
Previous Next ScoreReport LabValues Calculator Help Pause

ExamSection1:Item31ol50 NationalBoardofMedicalExaminers^
ComprehensiveBasicScienceSelf-Assessment
Y
31.Afemalenewborndeliveredat38weeks'gestationtoa28-year-oldwoman,gravida1,para1,developsrespiratorydistress.Pregnancyanddeliverywereuncomplicated:amnioticfluidwasclearandtheplacentawasnormal.Feta!ultrasonographyand
MRIat34weeks'gestationshowedacongenitaldiaphragmatichernia,withevidenceofsmallbowelandstomachherniationinto[efthemithorax.Sheis52cm(20.4in)longandweighs3500g(7lb11oz).Hertemperatureis37.5CC(99.5aF),pulseis
133/min,respirationsare50/minandbloodpressureis70/55mmHg.Physicalexaminationshowsperipheralcyanosisthatimprovesafteradministrationofoxygenviaendotrachealintubation.Breathsoundsaredecreasedontheleft.Cardiacexamination
showsnormalheartsoundswithoutmurmurs.Whichofthefollowingcomplicationsofthedescribedpathologyismostlikelytobelifethreateningtothisnewborn?
A)Activepneumonia
B)Alveolaredema
C)Amnioticembolism
D)Inadequatesurfactantsynthesis
E)Pulmonaryhypoplasia
CorrectAnswer:E
Thisnewbornpresentswithacongenitaldiaphragmatichernia
;whichplacesthenewbornatriskforpulmonaryhypoplasia.Herniationofabdominalorgansintothethoraxcompressesthelungswhiletheyaredevelopinginutero.Pulmonaryhypoplasiaresults
throughacomplexmechanismthatinvolvesdirectcompressionofabdominalorgansuponthedevelopinglungs,whichaffectsbronchialandvasculararchitecturewithreducedbranching,bfoodsupply,andincreasedthicknessofalveolarsepta.Newbornswith
pulmonaryhypoplasiacommonlydeveloppersistentpumonaryhypertensionduetoincreasedpulmonaryvascularresistance,resultinginpotentiallysevereheartfailure.Thiscollectionofrespiratoryandcirculatorydifficultiesmayberapidlylife-threateningtothe
newbornTreatmentinvolvessurgicalrepairofthehernia,althoughthemanagementofcongenitaldiaphragmaticherniamustalsoconsiderthemultitudeofsecondarypulmonaryandcardiovascularproblemsthatresultfromtheherniationofabdominalviscerainto
thedevelopingthorax.
IncorrectAnswers:A,B.C,andD.
Activepneumonia(ChoiceA)maydevelopinnewbornswhosurvivetheimmediatepostpartumperiod.Thislatecomplication,whilepossible,islessimportantthantheimmediatelylife-threateningconsequencesofpulmonaryhypoplasia.
Alveolaredema(ChoiceB)occursinneonatalrespiratoiydistresssyndromeandincongenitalheartdiseaseAlveolaredemadoesnotnecessarilyresultfromcongenitaldiaphragmatichernia.
Amnioticembolism(ChoiceC)isapotentiallydangerousobstetriccomplicationwhereinamnioticfluidentersthematernalcirculation.Amnioticembolismmaycausematernaldisseminatedintravascularcoagulation.
Inadequatesurfactantsynthesis(ChoiceD)occursinprematurenewbornsandunderliesthepathogenesisofneonatalrespiratorydistresssyndrome.InadequatesurfactantproducedbytypeIIpneumocytesleadstoalveolarcollapse.Thisisaseparatedisease
processfromthepulmonarychangesthatoccurinpulmonaryhypoplasia.
EducationalObjective:Pulmonaryhypoplasiaoccursasasequelaofcongenitaldiaphragmaticherniaandresultsinseriousandpotentiallylife-threateningconsequencestopulmonaryandcardiacfunction.Pulmonaryhypoplasiaiscausedbycompressionofthe
lungsbyherniatedabdominalorganswithsecondarychangesinbronchialandvasculardevelopment.
© © © ©
f*
Previous Next ScoreReport LabValues Calculator Help Pause

ExamSection1:Item32ol50 NationalBoardofMedicalExaminers^
ComprehensiveBasicScienceSelf-Assessment
Y
32.An18-hour-oldmalenewbornis61cm(24in)longandweighs5443g(12lb).Hismotherhastype1diabetesmellitus.Hisserumglucoseconcentrationis20mg/dLWhichofthefollowingfetalconditionsimmediatelypriortobirthmostlikelyprecipitated
thenewborn'spostnatalhypoglycemia?
A)Decreasedgliuconeogenesis
B)Decreasedglycogenconcentration
C)Decreasedglycogensynthetaseactivity
D)Decreasedseruminsulinconcentration
E)Increasedseruminsufin-likegrowthfactor
CorrectAnswer:A.
Excessivefetalexposuretoinsulininuterocancausemacrosomia
;neonatalrespiratorydistresssyndromeorhyalinemembranedisease,andhypoglycemiainthenewborn.Humanplacentallactogenproducedbytheplacentaincreasesmaternalinsulinresistance.
Thisleadstoincreasedlevelsofglucoseinthematernalcirculation,andsubsequentlythefetalcirculation.Fetalhyperglycemiainturncausesthefetustoproduceexcessinsulin,aswellasdecreasetherateofgluconeogenesis.Theinsulinthenactsasagrowth
factorleadingtomacrosomia,andinhibitstheproductionofsurfactant,whichleadstorespiratorydistresssyndrome.Afterbirth,thenewborncontinuestosynthesizeinsulinbutisnolongerexposedtomaternalbloodglucoselevels,whichleadstohypoglycemia.
Treatmentissupportiveandincludesadministrationofsupplementalglucosewhileneonatalendocrineregulationself-corrects.
incorrectAnswers:B,C,D,andE.
Decreasedglycogenconcentration(ChoiceB)wouldleadtofastinghypoglycemiaastherewouldbedeficientglycogenolysistomaintainadequatebloodglucoseconcentration.Thispatientwouldlikelyhaveincreasedglycogenconcentrationduetoitsprolonged
exposuretoinsulininutero.
Decreasedglycogensynthetaseactivity(ChoiceC)wouldleadtofastinghypoglycemiaduetoinadequateglycogensynthesisandinsufficientstores.Howeverglycogensynthetaseisstimulatedbyinsuin,whichisincreasedinthisnewborn.
Decreasedseruminsulinconcentration(ChoiceD)wouldleadtohyperglycemia:thispatienthasincreasedinsulinconcentrationduetoprolongedexposuretomaternalhyperglycemiawithresultantisletcelhypertrophy.
Increasedseruminsulin-likegrowthfactor(ChoiceE)playsaroleinthedevelopmentoflongbonesandmusclemass,ratherthanbloodglucoseleve:s.Itisincreasedingigantismandacromegaly.
EducationalObjective:Maternaldiabetesexposesafetustohighbloodglucoseconcentration,inresponsetowhichthefetusincreasesproductionofinsulinanddecreasesgluconeogenesis.Followingbirth,thiscanleadtopostnatalhypoglycemiaasthenewbornis
nolongerexposedtotheincreasedbloodglucoseconcentrationinthematernalcirculation.
© © © ©
f*
Previous Next ScoreReport LabValues Calculator Help Pause

ExamSection1:Item33of50 NationalBoardofMetlicafExaminers^
ComprehensiveBasicScienceSelf-Assessment
y
33.Thesynthesisoftheenzymesnecessaryforthereplicationofthegenomeoccursduringwhichofthefollowingphasesofthece
1
!cycle?
E
G
2 M
C
G,JGJB
DS
A)
B)
C)
0)
E)
CorrectAnswer:C.
Thenormalcellcycleinsomaticceilsinvolvesseveralstages:G
1
;S.G
2
;andMphase.GOphasedenotescellcyclearrest.DuringtheG1phase,thecellularcontents,exceptthechromosomes,areduplicated.InSphase,chromosomalduplicationoccurs;inG2
phase,thefidelityofrepEicationischecked.Mphaseinvolvestheattachmentofchromosomestospindlesfollowedbytheirseparationanddivisionintotwoidenticalcells,withthestagesofMphasedenotedasprophase,prometaphase,metaphase,anaphase,
andtelophase.ThesynthesisofenzymesnecessaryforreplicationofthegenomeoccursduringtheG1phaseinpreparationfortheSphaseofgenomeduplication.Suchenzymesincludehelicase(unwindsDNAatthereplicationfork),primase(createsanRNA
primerforreplicationtostartwith),DNApolymerase(elongatestheDNAstrandbeingcreatedandremovestheRNAprimer),DNAtopoisomerases(removesupercohsintheDNA),ligase(connectssmallfragmentsofDNAorOkazakifragments),andtelomerase
(addsDNAatthe3'endwitheachduplicationtoprotectthegeneticmaterial)Thephasesoftheceilcycleareregulatedbycheckpointsandregulatoryproteinscalledcyctinsandcyclin-dependentkinasestopreventindefinitecelpro
!iteration.
incorrectAnswers:A.B,D,andE.
M,ormitosis,(ChoiceA)istheprocessbywhichtheduplicatedgenomeisdividedintotwocells.Thephasesofmitosisareprophase,prometaphase,metaphase,anaphase,andtelophase.Proteinsynthesisdoesnotoccurduringthisphase.Mitosisisfollowed
bycytokinesisduringwhichthecell,itscontents,anditscytoplasmaredividedintwo.
G
0
(ChoiceB)isthearrestphaseofthecell.CeilsthatarenotactivelydividingresideinG
&andthusdonothaveaneedtosynthesizetheenzymesnecessaryforDNAreplication
S(ChoiceD)isthephaseofthecellinwhichDNAreplicationoccursTheenzymesnecessaryforreplicationmustbeproducedbeforethisphasebegins.
G
2
(ChoiceE)isthephaseinwhichtheintegrityoftheduplicatedgenomeischeckedandcorrected.Itisalsoaphaseofgrowthandproteinsynthesisinpreparationforthephasesofmitosisandcelldivision
EducationalObjective:Thenormalcellcycleinsomaticcellsinvolvesseveralstages:G
0
,G
l3S, andMphase.DNAduplicationoccursduringtheSphaseandthustheenzymesnecessaryforduplicationmustbesynthesizedpriortothestartatthisphase
duringG
t
0 m r
Previous Next SconeReport LabValues Calculator Help Pause

ExamSection1:Item34ol50 NationalBoardofMetlicaiExaminers^
ComprehensiveBasicScienceSelf-Assessment
y
34.A22-year-oldwomanisbroughttotheemergencydepartmentinasemicomatoseconditionaftercollapsingneartheendofrunningamarathon.Herpreraceweightwas47kg(103lb).Shenowweighs50kg(110lb).Herpulseis115/miri
*
respirationsare
15/min
;andbloodpressureis90/50mmHg.Physicalexaminationshowscool,dryskin.Sheisresponsivetopainfulstimuli.Laboratorystudiesshow:
Serum
Na
+ 116mEq/L
4.3mEq/L
89mEq/L
22mEq/L
22mg/dL
101mg/dL
1mg/dL
K
+
ci
-
HCGj
-
Ureanitrogen
Glucose
Creatinine
Thispatient'sconditionismostlikelyduetowhichofthefollowing?
A)DecreasedADM(vasopressin)
B)Decreasedaldosterone
C)Excessivefluidintake
D)Inadequatefluidintake
E)Increasedaldosterone
CorrectAnswer:C.
Hyponatremiaisoftenasymptomaticifchronicandslowlydeveloping,althoughwhenacute,itisassociatedwithcentralnervoussystemsymptomssuchasheadache,nausea,vomiting,confusion,delirium,weakness,seizures,andcoma.Volumestatusand
osmolalityshouldbedeterminedtoevaluatetheetiologyofhyponatremia.Inthispatientwhoishyponatremiaandhypervolemicasevidencedbyacuteweightgain,etiologiescanincludesyndromesoffluidoverload(eg,cirrhosis,congestiveheartfailure,
nephroticsyndrome)orexcessivefluidintake.Excessivefluidintakeandhypervoemialeadstodilutionalhyponatremia.Theabruptdecreaseinserumosmolalityleadstotranscellularshiftingoffluidbyosmosis,withneuronalswelling(cerebraledema)inthe
centralnervoussystem,whichcanleadtobrainherniation.Emergencytreatmentinvolvesadministrationoflypertonicsalinetocorrectosmolarshiftsandaddresssevereneurologicsymptomsandpreventseizuresduetohyponatremia.Managementfor
hypervolemichyponatremiainvolveswaterrestriction,diuretics,anddialysisinseverecases.
IncorrectAnswers:A,B.D,andE.
ADH(vasopressin)actsatthelevelofthedistalconvolutedtubuleandcollectingducttoincreasewaterreabsorption.Waterreabsorptioninabsenceofsolutereabsorption.specificallysodium,willleadtoeuvolemic,hypoosmolarhyponatremia.DecreasedADH
(ChoiceA)secretionwouldleadtodecreasedwaterreabsorptionandhyperosmolarhypernatremia,nothyponatremia.
Aldosteronepromotesreabsorptionofsodiuminthedistalconvolutedtubuleandcollectingducts,causingindirectreabsorptionofwater,andtoafesserextentthanthewaterreabsorptionthatoccurswithADH.Duetotsactionsonasodium-potassiumpump,it
alsoleadstothesecretionofpotassiumintheurine.Thisgenerallyleadstosodiumreabsorptioninexcessofwaterreabsorption.Decreasedaldosterone(ChoiceB)wouldleadtodecreasedsodiumandwaterreabsorptionandwouldalsopresentwithmetabolic
acidosis.ItwouIdbeunlikelytocauseacute,severehyponatremiaasseeninthispatientduetheproximateeffectofincreasedfluidintakewhilerunning,thoughcouldcausechronichyponatremia.Increasedaldosterone(ChoiceE)wouldleadtotheopposite
effect(eg,mildhypernatremia,hypertension,andhypokalemia).
Inadequatefluidintake(ChoiceD)wouldleadtoweightiossandhypernatremia.Duringmarathons,runnerslosefluidthroughdiaphoresisandrespirationsandrequireadequatehydrationTheselossescombinedwithinadequatehydrationwouldleadto
hypernatremiaandhyperosmolarserum.
EducationalObjective:Acutehyponatremiaisassociatedwithcentralnervoussystemsymptomssuchasheadache,nausea,vomiting,confusion,delirium,weakness,seizures,andcoma.Excessivefluidintakecanleadtohypervolemichyponatremiaand
cerebraledema.
IS
Previous Next ScoreReport LabValues Calculator Help Pause

ExamSection1:Item35ol50 NationalBoardofMedicalExaminers^
ComprehensiveBasicScienceSelf-Assessment
Y
35.A36-year-oldwomanwithtype2diabetesmellituscomestothephysicianforafollow-upexamination.Currentmedicationsincludeasulfonylurea.Sheis173cm(5ft3in)talandweighs95kg(2101b);BMIis32kg/m
2
.Physicalexaminationshows
acanthosisnigricans.Treatmentwithmetforminismostlikelytoproducewhichofthefollowingeffectsinthispatient?
A)Decreaseintestinalcarbohydratedigestion
B)Increasebeta-cellinsulinsecretion
C)Increasedepositionofadipocytefat
D)increasehepatictriglyceridesynthesis
E)Inhibithepaticgluconeogenesis
CorrectAnswer:E
Type2diabetesmellitusisacommondisordercharacterizedbyperipheralinsulinresistanceandhyperglycemia.Uncontrolleddiabetesincreasestheriskformacrovascularcomplications(eg
:coronaryarterydisease,peripheralarterialdisease,andstroke)and
microvascularcomplications(eg.nephropathy,retinopathy,neuropathy).Treatmentwithmetforminisafirst-linetherapyalongwithdietmodification,increasedactivity,andweightloss.Metforminisanoralbiguanideantihyperglycemicthathasmultipleglucose-
loweringeffects.Stinhibitshepaticgluconeogenesisandincreasesperipheralsensitivitytoinsulinresultinginincreasedglucoseuptakefromthecirculation.Metforminisgenerallywell-toleratedandnotassociatedwithhypoglycemicepisodesorweightgain.Adverse
effectsincludegastrointestinaldisturbanceandlessfrequently,lacticacidosis,whichmaybeexacerbatedinrenalfailure.
IncorrectAnswers:A.B,C.andD.
Decreasedintestinalcarbohydratedigestior(ChoiceA)isachievedthroughtheadministrationofa-glucosidaseinhibitorssuchasacarboseandmiglitoiThesemedicationsfunctiontodecreasecarbohydratehydrolysisattheintestinalbrushborder,resultingin
decreasedglucoseabsorption.
Sulfonylureas.glucagon-likepeptide-1(GLP-1)agonists,anddipeptidylpeptidase-4-(DPP-4)inhibitorsincreasebeta-cellinsulinsecretion(ChoiceB)throughdifferentmechanisms.Sulfonylureasinhibitpancreaticbeta-celipotassiumchannelsresultingin
membranedepolarizationandinsulinrelease.GLP-1agonistsmimictheactivityofGLP-1,whichisanincretinproducedbythegastrointestinaltractthatstimulatesinsulinrelease.DPP-4inhibitorsdecreasetheenzymaticclearanceofendogenousGLP-1.
Metformindoesnotincreasethedepositionofadipocytefat(ChoiceG)orincreasehepatictriglyceridesynthesis(ChoiceD).InsulinstimulatesfattyacidsynthesisinadipocytesandtriglyceridesynthesisintheliverTreatmentwithinsulinorantihyperglycemic
agentsthatpromoteinsulinreleaseareassociatedwithweightgain.
EducationalObjective:Metforminisafirst-linetherapyforthetreatmentoftype2diabetesmellitusinadditiontodietandactivitymodification.Metformininhibitshepaticgluconeogenesisandincreasesglucoseuptakebyperipheraltissues.
0
Previous Next ScoreReport LabValues Calculator Help Pause

ExamSection1:Item36ol50 NationalBoardofMedicalExaminers^
ComprehensiveBasicScienceSelf-Assessment
Y
36.A23-year-oldmanintheemergencydepartmenthasapneaandpinpointpupils.Needletracksarepresentonhisarms.Activationofwhichofthefoliowingopioidreceptorsinthecentralnervoussystemismostlikelytoberesponsiblefortheapnea?
A)5
B)K
C)u
D)a
CorrectAnswer:C.
IJLopioidreceptoragonismlikelyexplainsthispatient'sapnea.OpioidreceptorsareG-protein-coupledreceptorslocatedthroughoutthecentralnervoussystem(CNS)andperipheralnervoussystem(PNS).Opioidintoxicationcauseseuphoria,alteredmentalstatus,
sedation,bradycardiaandhypotension,depressedrespiratorydrive(orapnea)
;andconstrictedpupils.popioidreceptoractivitymediatesanalgesia,reward,andadverseCNSandPNSeffects.Specifically,theP2
opioidreceptor,locatedintheCNSandon
periphera!chemoreceptorsandbaroreceptors.controlsrespiratorydepression.MiosisisadistinctivefindingthatislesscommoninintoxicationwithotherCNSdepressantsandiscausedbydirectp
2
opioidreceptoractivityinbrainareasresponsibleforpupillary
ControlOpioidsalsoactonp
2
receptorswithintheentericnervoussystem,reducinggutmotilityandcausingconstipation.Thetreatmentofopioidtoxicityincludessupportivecare(eg,respiratorysupport)andnaloxone,ashort-actingopioidreceptorantagonist.
IncorrectAnswers:A.B,andD.
5opioidreceptors(ChoiceA)mediateanalgesiaandrewardalongsidepopioidreceptorsandmayberesponsibleforneuronaladaptationsthatEeadtoaddiction.5opioidreceptorshavebeenimplicatedinotherdiverseCNSrolessuchasmodulationofmotor
function,epileptogenesisinabsenceseizures,andpost-ischemicneuroprotection.5opioidreceptorsarenotknowntomediaterespiratorydepression.
Kopioidreceptors(ChoiceB)synergisticallyalleviatepainwithpopioidreceptorsbutopposetherewardsignalingofpopioidreceptors,leadingtodysphoriaunderstressfulconditions.AntagonismofKopioidreceptorsisbeinginvestigatedasatherapeutictargetin
mooddisorders,Kopioidreceptorsarenotknowntomediaterespiratorydepression.
areceptors(ChoiceD)wereoriginallythoughttobeasubtypeofopioidreceptorbutarenowtheorizedtorepresenttheirownreceptorclass,oreceptorsarethoughttoregulatehigher-orderfunctionssuchasmemoryanddrugdependence.Forexample,
upregulationoforeceptorshaspromotedstimulant-seekingbehavior.
EducationalObjective:Inopioidintoxication,popioidreceptorsmediateanalgesia,reward,andadverseCNSandPNSeffects.Specificallythep
2
opioidreceptorlocatedintheCNSandonperipheralchemoreceptorsandbaroreceptors.controlsrespiratory
depression.
00 &
Previous Next ScoreReport LabValues Calculator Help Pause

ExamSection1:Item37of50 NationalBoardofMedicalExaminers^
ComprehensiveBasicScienceSelf-Assessment
Y
37.A5-year-oldgirlwithprematuresexualdevelopmentisdiagnosedwithprecociouspuberty.Pelvicexaminationshowsamassconsistentwithanovariantumor.Laboratorystudiesshowdecreasedserumconcentrationsofgonadotropinsandamarked
increaseErrcirculatingestrogens.Theovariantumorismostlikelyderivedfromwhichofthefollowingcelltypes?
A)Endothelialcells
B)Germinalepithelium
C)Granulosacells
D)Stromalfibroblasts
E)Thecalcells
CorrectAnswer:C.
Granulosacelltumorsareatypeofmalignantsex-cordstromaltumorTheygenerallyoccurinwomenintheirsixthdecadeoflife,thoughcanoccuratanyage.includingduringchildhood.Theirpresentationismarkedbytheeffectoftheirfunctionalproductionof
estrogen
;atypicalproductofgranulosacells.Thus,itmaypresentwithprecociouspuberty
:asinthispatient,orvaginalbleedinginyoungerorpremenarchealwomen.Precociouspubertyissuspectedwhengirlsyoungerthan8yearsoldorboysyoungerthan9
yearsolddevelopsecondarysexual
'
characteristics.Sexualmaturityratingstage3characteristics,includingthickeningofpubicandaxillaryhairandbreastenlargement,aretypicallyseeningirlsages11to13.Thepresenceofthesefindingsatasignificantlyearlier
agesuggestsprecociouspubertyandwarrantsevaluation.Granulosacelltumorsaretypicallyindolentandmaynotbedetecteduntillargeoradvanced.Onhistology,granulosacelltumorsdemonstrateCall-Exnerbodies,whicharegranulosacellsarrangedaround
eosinophilicfluid,resemblingovarianfollicles.Treatmentisthroughsurgicalexcision,andifthetumorisearlystage,prognosisisgenerallyfavorable.Inthepostmenopausalpatient,granulosacelltumorsoftenpresentwithpostmenopausalvaginalbleeding,which
shouldpromptinvestigation.
IncorrectAnswers:A.B,D,andE.
Proliferationofendothelialcells(ChoiceA)leadstovascularneoplasms,suchasangiosarcomaandKaposisarcoma.Amatureorimmatureteratomacontainstissuederivedfromalthreeembryologicaltissuelines,whichcouldincludevasculartissue,buttheseare
notprimarymalignanciesoftheendothelialcells.
Germinalepithelium(ChoiceB)isthelayerofcellsthatcoverthesurfaceoftheovary.Mostovariantumors,includingthecommonserouscystadenocarcinoma.arisefromgerminalepithelium.Howeverthesedonottypicallyproducehormones.
Theneoplasticproliferationofstromalfibroblasts(ChoiceD)resultsintheformationofabenignovarianfibroma,whichisthemostcommontypeofovariansex-cordstromaltumor.Theyareoftenasymptomaticanddonotresultintheproductionofactivehormones.
Meigssyndromeconsistsofanovarianfibromainassociationwithascitesandapleuraleffusion.
Aneoplasticproliferationofthecalcells(ChoiceE)iscalledathecoma.Thesetumorsmayalsoproduceestrogenorandrogenandmaypresentwithpostmenopausalbleedingbutarelesscommonthangranulosacelltumors.
EducationalObjective:Granulosacelltumorsaremalignantsex-cordstromaltumorsthattypicallyoccurinwomenandresultintheunregulatedproductionofestrogen.Inchildrenandyoungwomen,theyoftenresultinprecociouspubertyorpremenstrualvaginal
bleeding.
© © © ©
f*
Previous Next ScoreReport LabValues Calculator Help Pause

ExamSection1:Item33ol50 NationalBoardofMedicalExaminers^
ComprehensiveBasicScienceSelf-Assessment
Y
38.Ahealthy22-year-oldwomanundergoestestingtodeterminewhethersheisasuitablekidneydonorforher25-year-oldbrotherwithend-stagerenaldiseasecausedbytype1diabetesmellitus.Immunologicstudiesshowthatsheishuman
'
eukocyte
antigen(HLA)-DR3/DR6positive
:andherbrotherisHLA-DR3/DR4positive.Todeterminetheextentofalloreactivity
:amixedlymphocytereactionisdoneusingirradiatedstimulatorceilsisolatedfromthedonorandrespondercellsisolatedfromthe
recipient.TheTlymphocytesthatproliferateinthesecultureswillmostlikelyreactwithwhichofthefollowingHLAtypes?
A)DR3only
B)DR3andDR4
C)DR3andDR6
D)DR4only
E)DR4andDR6
F)DR6only
CorrectAnswer:F
Withallogeneicdonation,ideally,donorsandrecipientsshouldmatchatallhumanleukocyteantigen(HLA)loci(sixoutofsix).Thepatientreceivingthetransplantandthesiblinginthequestionhaveasinglemismatch.TheyarebothHLA-DR3positive,butthe
donorsibling!sHLA-DR6positive,whereasthepatientisnot.Duetothisdifference,immunecelEsfromtherecipientwilllikelyreacttoHLA-DR6.Tlymphocytesdevelopinthethymus,wheretheyareselectedagainstiftheyreacttoself-antigens(negative
selection).Theyarepermittedtopersistiftheyreacttonon-self-antigenswhichpromotesimmunityagainstnon-nativeproteins.BecausetherecipientisalreadyHLADR3/DR4positive,hisTlymphocyteswouldnotreacttotheseHLAtypesassumingthatno
autoimmuneconditionispresent.MismatchesofanykindpredisposetoT-lymphocytemediatedtransplantrejection,althoughHLA-A,B.andDRappeartobethemostimportant.Inthiscase,animmuneresponsewouldbeexpectedagainstthedonorHLA-DR6by
therecipient'sT-lymphocytesthatbindtheantigen.
IncorrectAnswers:A,B,C,D,andE
BecausetherecipientisalreadyHLA-DR3/DR4positive,hisTlymphocyteswouldnotreacttoHLAtypesDR3orDR4(ChoicesA,BrC.D
;andE).Duringimmunedevelopment,cellscontainingtheseantigenswouldhavebeenpresentedtohisTlymphocytesinthe
thymus.AnyTlymphocytescontainingreceptorsthatbindandpromotereactionagainstthesenativeantigenswouldhavebeendestroyed,preventingautoimmunity(negativeselection).
EducationalObjective:DonorsandrecipientsoftransplantsshouldideallymatchatallHLAloci.Enamixedlymphocytereaction,recipientTlymphocytesWillreacttodonorHLAtypesthatareabsentintherecipient.
00 & oo
Previous Next ScoreReport LabValues Calculator Help Pause

ExamSection1:Item39of50 NationalBoardofMedicalExaminers^
ComprehensiveBasicScienceSelf-Assessment
Y
39.AG7-year-oldmanwithpoorlycontrolledunstableanginaisabouttoundergocoronaryangiographywithstentplacement.Priortotheprocedure,treatmentisinitiatedwithaspirinandadrugthatinhibitsplateletinteractionwithfibrinogen.Thisdrugismost
likelywhichofthefollowing?
A)Abciximab
B)Celecoxib
C)Cilostazol
D)Clopidogre!
E)Dipyridamole
CorrectAnswer:A.
Abciximabisamonoclonalantibodythatinhibitsglycoproteinllb/lllaontheplateletsurface.GlycoproteinItMilanormallybindswithfibrinogenresultinginplateletaggregationandthrombusformation.Acutecoronarysyndromemostcommonlyoccurssecondaryto
atheroscleroticplaqueruptureandthrombusformationinthecoronaryarteries,whichoccludesthevesselDepositionofcholestero!intheendothelialwallspromotesinflammatorycellmigrationandformationofanatheroscleroticplaquecharacterizedbyfibroblasts,
smoothmusclecells,andlipid-ladenmacrophagesthattransformintofoamceils.Afibrouscapformsovertheplaque,whichmayruptureduetostressdependingonitsthicknessandstability.Thisexposessubendothelialcollagenandinitiatesthrombusformation.
Antiplateletagentsareindicatedinthetreatmentofacutecoronarysyndrometoreducetheriskofthrombusprogression.Complicationsofabciximabtherapyincludeacutethrombocytopeniaandprofoundbleeding,andindicationsforusearerestrictedtohigh-risk
patientswithacutecoronarysyndromewithplannedpercutaneouscoronaryinterventionwithin24hours.
JncorrectAnswers:B.C.D
;andE.
Celecoxib(ChoiceB)isaselectivecyclooxygenase2inhibitorthatdemonstratesactivityininflammatorycellsandthevascularendothelium.ItresultsinthedecreasedsynthesisandreleaseofarachidonicacidderivativesincludingthromboxaneA
2Twhich
stimulatesplateletactivation.Thromboxaneisnotinvolvedinplateletbindingtofibrinogen.
Cilostazol(ChoiceC)anddipyridamole(ChoiceE)arephosphodiesteraseIIIinhibitors,whichreducethedegradationofcyclicadenosinemonophosphateinplateletsandvascularsmoothmusclecellsresultingintheinhibitionofplateletaggregationand
vasodilation.Cilostazolisusedinthetreatmentofclaudication.
Clopidogrel(ChoiceD)bindstotheadenosinediphosphatereceptoronplatelets(alsocalledtheP2Y12receptor),whichinhibitsplateletaggregation.Itiscommonlyusedinthetreatmentofacutecoronarysyndromeandcoronaryarterystenting.Abciximabdirectly
inhibitsplateletinteractionwithfibrinogenviaantagonismofglycoproteinllb/llla.
EducationaObjective;Antiplate.ettherapyisindicatedinthesettingofacutecoronarysyndrometoreducethrombusprogression.Abciximabdirectlyinhibitsplateletbindingwithfibrinogenthroughtheblockadeofglycoproteinllb/
'lllaontheplateletsurface.
00# 0
f*
Previous Next ScoreReport LabValues Calculator Help Pause

ExamSection1:Item40ol50 NationalBoardofMedicalExaminers^
ComprehensiveBasicScienceSelf-Assessment
Y
40.Lesch-Nyhansyndrome,anX-!inkedrecessivedisease,isseeninapproximately1/100,000males.Whichofthefollowingistheexpectedprevalenceofheterozygousfemales?
A)1/1000
B)1/10.000
C)1/50,000
D)1/200,000
E)1/10.000.000
CorrectAnswer:C.
TheHardy-Weinbergprinciple,knownasHardy-Weinbergequilibrium,proposesthatallelefrequencieswillremainconstantacrossgenerationsintheabsenceofevolutionarychange.TheHardy-Weinbergequilibriumequationis:p
2
+2pq+q
2
=1.wherepisthe
frequencyofthedominantalleleandqisthefrequencyoftherecessiveallele.2pqistheprobabilityofheterozygosity,p
2
istheprobabilityofdominanthomozygote,andq
2
istheprobabilityofbeingarecessivehomozygote.CalculationofHardy-Weinbergallele
frequenciesforX-linkedrecessivedisordersrequiresspecialconsiderations,asaffectedmalescarryonlyasingleallele.Inthiscircumstance,thefrequencyofaffectedmalesrepresentsq,ratherthanq
2
asinautosomalrecessivedisorders.Thematernalcarrier
frequency,2pq,canthenbecalculatedinastraightforwardmannerasfollows:q=1/100,000-0.00001,p=1-0.00001=0.99999,2pq-2x0.99999x0.00001-0.00002-1/50.000.
IncorrectAnswers:A.El.D,andE.
AssumingthatthecarrierfrequencyESeitheronehundredtimesgreater(ChoiceA),tentimesgreater(ChoiceB),halfasgreat(ChoiceD),oronehundredtimeslesser(ChoiceE)isincorrectandfailstoemploytheHardy-Weinbergprinciple.
ChoicesA.B,andEreflectmathematicalerrorsinvolvingtheuseofq(therecessiveallele)withoutfactoringforthepresenceofthedominantalleleandinvolveerrorsontheorderofmagnitude.
ChoiceD,1/200.000,reflectsamathematicalerrorwhereinpqwasdividedbytwoinsteadofmultipliedpq/2=0.DGO01*.99999/2=0.000005=:1/200,000.
bducationaObjective:WhenX-inkedinheritanceoccurs,theHardy-Weinbergequilibriummustbefactoredfortheimbalanceinalleleexpression.Affectedmalesonlycarryasinglealleleandtherefore,qmustbeadjustedtocalculatethematernalallelefrequency.
00 &
Previous Next ScoreReport LabValues Calculator Help Pause

ExamSection1:Item41ol50 NationalBoardofMedicalExaminers^
ComprehensiveBasicScienceSelf-Assessment
Y
41.Thespecimenshownisfroma65-year-oldman.Whichofthefollowingisthemostlikelydiagnosis?
A)Acuteleukemia
B)Coloniccarcinoma
C)Hepaticcellcarcinoma
D)Leiomyosarcoma
E)Liposarcoma
CorrectAnswer:B.
Coloniccarcinoma,alsoknownascolorectalcarcinoma(CRC),isthemostlikelydiagnosis.Thegrossspecimeninthephotographisacross-sectionoftheliverthandemonstratesinnumerablemetastasesobliteratingthenormalliverparenchyma.Ofthevarious
malignanciesthatmetastasizetotheliver,CRCisoneofthemostcommon.Othercancersthatmetastasizetotheliverincludethoseoforgansthatdrainintotheenterohepaticvenoussystemsuchasthepancreasandthegallbladder,althoughlungandbreast
cancersalsohaveatendencytometastasizetotheliver.Metastasescanbedifferentiatedfromprimarytumorsoftheliversuchashepatocellularcarcinomabytheappearancebothonimagingandgrossexaminationinthecaseofautopsyorresection.Primary
tumorstendtobesolitary,ormultiplebutwithadominantmass,whilemetastasestendtobemultifocalandofvaryingsize.
IncorrectAnswers:A,C_D,andE.
Acuteleukemia(ChoiceA)isamalignancyofwhitebloodcellprogenitorlineswithinthebonemarrow.Clonesarecommonlyfoundinthebonemarrow
;blood
;orlymphaticsystem.Occasionally,certainvarietiesofleukemiasuchasacutemonomyeiocytic
leukemiacanpresentwithinfiltrationofleukemiccellsintheskin(leukemiacutis)orinthegums
:butinfiltrationoftheliverdoesnottypicallyoccur.
Hepaticcellcarcinoma(ChoiceC)typicallypresentsasadominant,singularmass,orasadominantmasswithasmallnumberofadditionalfoci.Itismostcommoninpatientswhohavechronichepatitis6orcirrhosis,andadditionalfindingsongrossexamination
oftheliverwouldincludeanodularcontourconsistentwithcirrhosis.
Leiomyosarcoma(ChoiceD)isacancerofsmoothmusclecellsandismuchlesscommonthanCRC.Whilelivermetastasesfromretroperitonealleiomyosarcomaoccurthesegenerallyresultinonetothreemetastaticlesionsratherthanthediffuseinfiltration
evidentinthegrossphotograph.
Liposarcoma(ChoiceE)isacancerofadiposetissue.ItisexceedinglyrarecomparedtoCRCandrarelymetastasizestoThelivertwouldbeunlikelytocausethelesionsseeninthephotograph.
EducationalObjective:CRCisacommonmalignancythatmetastasizestotheliver.Ascomparedtoprimarymalignanciesoftheliverwhichgenerallyappearassolitarylesions
:metastasesfromCRCareoftenspreaddiffuselythroughouttheliverandappearas
innumerablemassesonimagingandgrossexamination.
^
0 0
Previous Next ScoreReport LabValues Calculator Help Pause

ExamSection1:Item42ol50 NationalBoardofMedicalExaminers^
ComprehensiveBasicScienceSelf-Assessment
Y
42.A45-year-oldmanwhoisabletobicycle45minutesadayhasswitchedtoarowingmachine.After5minutesonthemachine
;heexperiencesvertigo,lightheadedness,andfatigueoftheleftupperextremity.Withinafewminutesofstoppingtherowing
exercise
:allsymptomsresolve.Whichofthefollowingfindingsismostlikelyonphysicalexamination?
A)Diastolicmurmuratthecardiacapex
B)Increasedjuguiarvenouspressure
C)Pansystolicmurmuratthecardiacapex
D)Rightcarotidbruit
E)Supraclavicularbruit
CorrectAnswer:E
Thispatientwithexercise-inducedvertigo,iightheadedness
:andupperextremityfatigueismostlikelyexperiencingsubclavianstealsyndrome,whichmanifestswithasupraclavicularbruitonphysicalexam..Atheroscleroticdiseaseofthesubclavianarteryproximal
totheoriginofthevertebralarteryleadstodecreasedpressureinthedistalsubclavianartery.Duringexerciseoftheipsiiateralupperextremity,atheroscleroticdiseaseoftheipsilateralsubclavianarterymayinitiallypreventsufficientbloodsupplytothearm.The
armmaybeperfusedthroughretrogradeflowintheipsiiateralvertebralartery,diminishingposteriorcerebralcirculationandresultinginsymptomsofvertebrobasilarinsufficiency.Bloodfromthecontralateralsubclavianandvertebralarterymayflowintheretrograde
directionatthevertebrobasilarconfluenceinsteadofcontinuingtoascendthebasilarartery.Symptomsofvertebrobasilarinsufficiencycanincludevertigo,lightheadedness,disequilibrium,ataxia,andnystagmus(duetotransientlydecreasedbloodsupplytothe
vestibularnuclei).Subclavianstealsyndromeisacompensatorymechanismthatistypicallynotinitselfdangerous.Instead,itindicatessignificantatheroscleroticdiseasethatshouldbemanagedwithsurgicalbypassorangioplastywithorwithoutstentingalong
withlifestylechangestopreventfurtheratherosclerosis.
incorrectAnswers:A.B
:C
;andD.
Adiastolicmurmuratthecardiacapex(ChoiceA)istypicalofmitralstenosiswhileapansystolicmurmuratthecardiacapexindicatesmitralregurgitation(ChoiceC).Mitralstenosisormitralregurgitationcanleadtoheartfailure,atrialfibrillation,thromboembolic
events,andsecondarypulmonaryhypertension.Theydonotleadtovertebrobasilarinsufficiencyorexercise-inducedupperextremityfatigue.
Increasedjugularvenouspressure(ChoiceB)isanonspecificphysicalexaminationfindingthatmayindicatestatesoffluidoverload,rightheartfailure,orrestrictionofrightheartfillingasinpulmonaryhypertensionorconstrictivepericarditis,inacutecardiogenic
shock,patientsmaypresentwithfocalneurologicaldeficitsfromischemiaofwatershedareasofthebrainandbrainregionsvulnerabletoanoxicbraininjury{eg,hippocampus).Symptomsincludecorticalblindness,stupor,weaknessofthebilateralproximalupper
andlowerextremities,and/oranamnesicsyndrome.
Arightcarotidbruit(ChoiceD)reflectsatheroscleroticdiseaseoftherightcarotidartery,whichcouldproducetransientischemicattacksthatcouldmanifestastransientlossofmonocularvisionorhemisphericsignsofcerebralinfarction(eg,contralateral
hemiparesis,hemisensoryloss,orhomonymoushemianopsia)ratherthansignsofvertebrobasilarinsufficiency.Patientswithatheroscleroticdiseaseofthesubclavianarterymayadditionallydemonstrateatheroscleroticdiseaseofthecarotidartery,butsubclavian
atherosclerosisisthemostdirectexplanationofthispatient'ssymptoms.
EducationalObjective:Subclavianstealsyndromeoccurswhenatheroscleroticdiseaseofonesubclavianarteryleadstoretrograde,collateralbloodflowfromthevertebrobasilarsystem,thusdecreasingcerebralbloodflowfromthebasilarartery.Symptomsof
transientvertebrobasilarinsufficiencysuchasvertigoandlightheadednessmayresult.
© © © dPI
*
Previous Next ScoreReport LabValues Calculator Help Pause

ExamSection1:Item43of50 NationalBoardofMedicalExaminers^
ComprehensiveBasicScienceSelf-Assessment
Y
43.A55-year-oldmancomestothephysicianbecauseofa2-monthhistoryofincreasingdifficultyswallowingandregurgitationofundigestedfood.Healsohasnoticedunusualrumblingsoundsinhisvoicethathefeelsoriginateinhisneck.Physical
examinationshowshalitosis.Avideofluoroscopicswallowingstudyshowsa4-cm,posteriormidlinepouchprotrudingbetweenthethyropharyngeusandcricopharyngeusportionsoftheinferiorpharyngealconstrictormuscle.Thesemusclesaremostlikely
innervatedbywhichofthefollowingnerves?
A)Glossopharyngealnerve
B)Hypoglossalnerve
C)Motorfibersfromthevagusnerve
D)Parasympatheticfibersfromthevagusnerve
E)Sympatheticfibersfromthesuperiorcervicalganglion
CorrectAnswer:C.
Motorfibersfromthevagusnerveinnervatethethyropharyngeusandcricopharyngeusportionsoftheinferiorpharyngealconstrictormuscle.AZenkerdiverticulumoccursasaresultofanuncoordinatedswallowingmechanism,whichresultsinmusclespasmsof
thecricopharyngeusleadingtopulsiondiverticulaasthewallofthehypopharynxandsuperioresophagusweakens.Itoccursprimarilyinolderpatients.Thevagusnerveinnervatesmostoffhepharynxexceptthestylopharyngeusmuscleandpossessesfibersthat
joinwithnervefibersfromtheexternallaryngealnerve,glossopharyngealnerve,andsympatheticchainstoformthepharyngealplexus.Thepharyngealplexusinnervatesthemusclesofthepharynx.SymptomsofaZenkerdiverticulumincludehalitosis,
regurgitationofundigestedfood,anddysphagia.Bariumswallowrevealsaposterior,midlineoutpouchingaboveoratthelevelofthecricopharyngeus.Treatmentincludessurgicalrepairandresection(diverticulectomy),plusdietarymodifications
toincludesoftor
liquidfoods.
IncorrectAnswers:A.6,D.andE.
Theglossopharyngealnerve{ChoiceA),likethevagusnerve,isamixedsensoryandmotornerve.Itprovidesmotorinneivationtothestylopharyngeusmuscle.Italsostimulatessecretionfromtheparotidgland.
Thehypoglossalnerve(ChoiceB)innervatesboththeintrinsicandextrinsicmusclesofthetongueandisapurelymotornerve.Normalfunctionhelpspropelafoodbolusposteriorlyintothepharynxtoinitiatetheswallowreflex.
Parasympatheticfibersfromthevagusnerve(ChoiceD)innervatethegastrointestinalsystemdowntothetransversecolonaswellastheheart.Thesefibersacttoregulateperistalsisanddigestionandexertcontrolovertheheartrate.Themusclefibersarising
fromthevagusthatinnervatethepharynxarenotparasympatheticfibers.
Sympatheticfibersfromthesuperiorcervicalganglion(ChoiceE)innervatetheeyes,carotidbody,salivarygland,andthyroidgland.Horner
'ssyndromeresultsfromdamagetothesympatheticchainganglionandismanifestasipsilateralmiosis,anhidrosis,and
ptosis.Thesuperiorcervicalgangliondoesnotinnervatethepharyngealmuscles.
EducationalObjective:Motorfibersfromthevagusnerveinnervateallthemusclesofthepharynxexceptthestylopharyngeusmuscle.DisruptionofthisplexusofnervescancontributetopoorcoordinationoftheswalJowreflexleadingtothedevelopmentofa
Zenkerdiverticulum.
© © © ©
f*
Previous Next ScoreReport LabValues Calculator Help Pause

ExamSection1:Item44ol50 NationalBoardofMedicalExaminers^
ComprehensiveBasicScienceSelf-Assessment
Y
44.A5-year-oldgirlsbroughttothephysicianbecauseoflistlessness,fatigue,anddullpainintherightupperquadrantoftheabdomen.Herheightandweightarebelowthe25thpercentile.Laboratoryfindingsindicatethatthecontentofherp-glotarnchainis
15%to20%ofnormal.Sequencingofthe[3-globingeneshowsapointmutationinasequence3'tothecodingregioninwhichAATAAAisconvertedtoAACAAA.ConsequentlytheamountofmRNAfor(3-globinisdecreasedto10%ofnormal.Whichofthe
followingfunctionsinmRNAsynthesisandprocessingismostlikelyencodedbythesequenceAATAAA?
A)CappingwithGTP
B)Cleavageandpolyadenylation
C)Silencingofthepromoter
D)SplicingoftheinitialmRNAtranscriptinthenucleus
E)TransportofthemRNAoutofthenucleus
CorrectAnswer:B.
Cleavageandpolyadenylationofthepre-mRNAmoleculeismostlikelyencodedbythesequenceAATAAA.andapointmutationinthep-globingeneleadingtoanAACAAAsequencewoulddisruptanecessarystepinpre-mRNAprocessingknownas
polyadenylation.Polyadenylationistheadditionofastringofadeninenucleotidestothe3
'endofthemRNAmoleculethatpreventsdegradationandpermitstransportoutthenucleus.TheDNAsequenceAATAAAbecomesAAUAAAonpre-mRNA.During
transcription.thissignalisrecognizedbythecleavageenzymeCPSF(cleavageandpolyadenylationspecificityfactor),whichisboundtoRNApolymeraseIIandsignalsforthepolymerasetoceasetranscription.Thepre-mRNAmoleculeisthencleavedatthe3
'
endandbindingofasecondenzyme
:polyadenylatepolymerase
:catalyzestheadditionofalongstringofadeninenucleotidesontotheendofthepre-mRNAmolecule.Thispolyadenylatetailallowsfortransportoutofthenucleusandintothecytoplasmwherethe
mRNAmoleculecanbetranslatedintoproteins.Anon-functionalpolyadenylationsignalwouldresultinfailuretocleavepre-mRNAorfailuretopolyadenylatethetranscriptleadingtoearlydegradationwithinthenucleus.
IncorrectAnswers:A.C,D,andE.
CappingwithGTP(ChoiceA)occursatthebeginningoftranscriptionatthe5
:
endofthenascentmRNA.Itisrecognizedbyribosomesinthecytoplasmandfacilitatesinitiationoftranslation.Amutationwouldresultinthesynthesisofanorma!mRNAtranscriptbut
failureoftranslation.
Silencingofthepromoter(ChoiceC)wouldresultindecreasedexpressionofthe(3-globingene,butthepromotersequenceisnotencodedbythesequenceAATAAA.Mutationsinthepromoterregionarealsoknowntocausep-thalassemia.
SplicingoftheinitialmRNAtranscriptinthenucleus(ChoiceD)istheprocessbywhichintronsareremovedandexonsareconnected.Splicesitesexistattheendofanintronandthebeginningofanexon.Splicesitesarerecognizedbysmallnuclear
ribonucleoproteins,whichareRNA-proteincomplexesthatcombinewiththepre-mRNAtoformaspliceosome.Mutationsinsplicesitescanresultintheaberrantinclusionofintronsorexclusionofexons.
TransportofthemRNAoutofthenucleus(ChoiceE)occursthroughnuclearpores.Aftertranscription,mRNAmoleculesdiffusefreelythroughthenucleusandintothecytoplasmthroughthesechannels.Whilethepolyadenylatetailaffectsoverallstabilityofthe
moleculeandfacilitatestransport,thesequencethatsignalspolyadenylationisupstreamfromthisprocess.
EducationalObjective:TheAATAAAsequenceencodesthepolyadenylationsignal.Mutationsherewouldresultinfailuretopolyadenylatethe3
'endofthemRNAtranscriptandleadtoearlydegradationwithinthenucleuspriortotranslation.
© © © oo
Previous Next ScoreReport LabValues Calculator Help Pause

ExamSection1:Item45ol50 NationalBoardofMetlicaiExaminers^
ComprehensiveBasicScienceSelf-Assessment
y
45.A76-year-oldmancomestothephysicianforafollow-upexamination.Hehashypertensiontreatedwithap-adrenergicantagonist.HelivesonafarmincentralCaliforniaandsayshehasalwaysdistilledhisownliquor.Beforeretiring10yearsago.he
workedinahatfactoryandsubsequentlyinatextilefactory.Hehassmoked2packsofcigarettesdailyforthepast55years.Hetedsthephysicianthathehashadseveralepisodesofpainfulswellingofhisrightgreattoe.Physicalexaminationshows
severallesionsconsistentwithgoutytophiovertheelbowsbilaterally.Laboratorystudiesshow:
HemoglobinA
1c
Serum
Glucose
Creatinine
Uricacid
5.6%
93mg/dL
3.2mg/dL
7.9mg/dL
Themostlikelycauseofthispatient'sconditioniswhichofthefollowing?
A)Cigarettesmoking
B)Drinkinghome-distilledliquor
C)FarmingincentralCalifornia
D}Workinginahatfactory
E)Workinginatextilefactory
CorrectAnswer:B.
Goutisachronicrecurringinflammatoryarthropathyinwhichdepositionofuricacidcrystalsinbothlargeandsmalljoints,mostcommonlythefirstmetatarsophalangealjoint,leadstopainandinflammationofthejoint.Depositionofuricacidcrystalsmayoccurin
thesurroundingtissues(tophi),increasedintakeoffoodsthatarerichinpurinessuchasseafoodandredmeatshavebeenshowntoincreasetheriskofgoutflares,astheyresultinincreasedproductionofuricacid.Othercausesofincreaseduricacidlevels
includediureticuse,tumorlysissyndrome,alcoholconsumption,andobesity.Itishypothesizedthatalcoholanditsmetabolitescompetewithuricacidtransportersinthekidneyleadingtodecreaseduricacidexcretion.Thus,theingestionofhome-distilledliquor
mayresultindecreaseduricacidexcretionleadingtogoutflares.
IncorrectAnswers:A.C,D,andE.
Cigarettesmoking(ChoiceA)hasbeenlinkedtonumerouspathologiesincludinglungcancerosteoporosis,bladdercancerandchronicobstructivepulmonarydisease.Ithasnotbeendirectiyassociatedwithgoutflaresandalterationofuricacidmetabolismor
excretion.
FarmingincentralCalifornia(ChoiceC)isnotassociatedwithanyspecificpathology.Ingeneral,farmingworkmayleadtoincreasedsunexposureandriskofcutaneousmalignancy,osteoarthritislinkedtoheavymanuallabortraumaticinjuryfromfarm
equipment,andhypersensitivitypneumonitisrelatedtotheinhalationoforganicmaterials.
Workinginahatfactory(ChoiceD)isclassicallyassociatedwithexposuretomercuryusedinthemakingoffelthats.Thiscanleadtoaneurologicaldisorderknownaserethismmercurialis(madhatterdisease)characterizedbypersonalitychanges,memory
loss,depression,apathy,delirium,neuropathy,gastrointestinaldistress,andanemia.
Workinginatextilefactory(ChoiceE)isassociatedwiththeinhalationofcottonparticuatesthatleadstolungscarringandeventualrespiratoryfailure(byssinosis).
EducationalObjective:Goutisarecurrentinflammatoryarthropathyofoneormorejointsthatiscausedbyincreasedserumlevelsofuricaciddepositinginthesynoviumresultinginsynovialinflammation.Theincreasedintakeofpurines(eg,redmeat,seafood)
contributestoincreasedbloodlevelsofuricacid.Alcoholmetabolitesarethoughttocompetewithuricacidforexcretioninthekidney,leadingtogoutyflareswithingestion.
O1» ®
(E e
Previous Next ScoreReport LabValues Calculator Help Pause

ExamSection1:Item46ol50 NationalBoardofMetlicaiExaminers^
ComprehensiveBasicScienceSelf-Assessment
y
46.A22-year-oldwomancomestothephysicianbecauseofa6-monthhistoryofdifficultyswallowingShesaysthatshefeelslikesheischokingonbothsolidsandliquids.Shehasnopainwith
swallowing:Shehashada4.5-kg(10-lb)weightlossduringthistime.Thereisnohistoryoffeverorchills.Sheisnotsexuallyactive.Shedoesnotsmokecigarettesoruseillicitdrugs.Sheis
170cm(5ft7in)tallandweighs59kg(130lb);BMIis20kg/m
2Hervitalsignsarewithinnormallimits.Physicalexaminationshowsnoabnormalities.Anx-rayoftheesophagusisshown.Which
ofthefallowingisthemostlikelyexplanationforthispatient'ssymptoms?
A)Acidrefluxintotheloweresophagus
6}Atrophyofthesmoothmuscleintheesophagus
C)Inflammatorydegenerationofesophagealwall:neurons
D)Longitudinalmucosaltearattheesophagogastricjunction
E)Perforationoftheesophagealwall
CorrectAnswer:C.
Inflammatorydegenerationofesophagealwallneuronshasledtothedevelopmentofachalasiainthispatientwithweightlossanddysphagiatobothsolidsandliquids.Achalasiaisanesophagealdysmotilitydisorderresultingfromdeficientperistalsisasaresult
ofimpairedneuromusculartransmissionandimpairedrelaxationoftheloweresophagealsphincter(LES).ftmanifestsasdysphagia
;odynophagia,weightloss,halitosis,andregurgitationofundigestedfood.Itisdiagnosedwithabariumswallowandesophageal
manometry.Destructionofnervesinthemyentericplexusimpairslocalnitricoxideproduction,preventssmoothmusclerelaxation,anddisproportionatelyaffectsinhibitoryneurons,whichfunctiontorelaxtheLES.AnincreaseinLESpressureanddysfunctional
peristalsiscausegradualdilationoftheesophaguswithretentionoffood,leadingtodysphagiaandregurgitation.Onbariumesophagography.achalasiaclassicallyappearsasadilatedesophaguswitha
L:
birdbeak"distaltaperattheLES.Treatmentincludes
pneumaticdilationorinjectionofbotulinumtoxintorelaxtheLES.
IncorrectAnswers;A.8,DrandE.
Acidrefluxntotheloweresophagus(ChoiceA)describesgastroesophagealrefluxdisease(GERD).GERDcommonlyoccursasaresultofincreasedintra-abdominalpressurerelatedtoobesityordiminishedclosingpressureoftheLESpermittingacidiccontents
ofthestomachtorefluxintotheesophagus.ChronicGERDcanleadtointestinalmetaplasiaoftheloweresophagus,whichpredisposestoesophagealcancer.Esophagealcancercanpresentwithdysphagiaandodynophagiaandcanappearonx-raysassimilar
toachalasiabutwouldbeuncommoninayoungpatient.
Atrophyofthesmoothmuscleintheesophagus(ChoiceB)describesthepathophysiologicmechanismbywhichsclerodermacausesdysphagia.GradualatrophyofsmoothmuscleleadstothelossofLEStoneanddysfunctionalperistalsis.
Longitudinalmucosa!tearattheesophagogastricjunction(ChoiceD)isknownasaMallory-Weisstearandresultsinhematemesisandchestorepigastricpain.Itoftenoccursafterprolongedretchingorvomiting,andgraduallyhealswithoutintervention.
Perforationoftheesophagealwall(ChoiceE),referredtoasBoerhaavesyndrome,developscommonlyafterforcefulvomitingorretching.Itpresentswithsubsternalchestpainandodynophagia:x-raysandCTscanstypicallyrevealpneumomediastinum.
Extraluminalcontrastwouldbeseenonafluoroscopicesophagogram.Dependinguponthelocationofthetear,gastriccontentsmayinvadethemediastinumorthepleura,leadingtomediastinitisorpleuraleffusion.Surgicalrepairisrequiredfordefinitive
managementalthoughstentingviaesophagogastroduodenoscopycanbeusedasatemporizingmeasure.
EducationalObjective:AchalasiaresultsfromthedestructionofneuronsinthemyentericplexusresultingindysfunctionalperistalsisandachronicallyincreasedLEStone.Presentingsymptomsincludedysphagiaandregurgitation;progressivedilationofthe
esophagusisseenonimaging.Treatmentiswithpneumaticdilationorbotulinumtoxin.
o© m
-I-
Previous Next ScoreReport LabValues Calculator Help Pause

ExamSection1:Item47ol50 NationalBoardofMedicalExaminers^
ComprehensiveBasicScienceSelf-Assessment
Y
47.A4S-year-oldwomancomestotheofficebecauseofa4-monthhistoryofheadaches,itchyskimdifficultyswallowing,heartburn
:chesttightness,painnherarmsandlegs,andaburningsensationwithurination.Shehasahistoryofsimilarsymptomssince
theageof14years
:butpreviousexaminationsshowednoabnormalities.Hervitalsignsarewithinnormallimits.Physicalexaminationandlaboratorystudiestodayshownoabnormalities.Whichofthefollowingisthemostlikelydiagnosis?
A)Conversiondisorder
B)Factitiousdisorder
C)Illnessanxietydisorder(hypochondriasis)
D)Malingering
E)Somaticsymptomdisorder
CorrectAnswer:E
Insomaticsymptomdisorder,thepatientispreoccupiedwithoneormoresomaticsymptomssuchthatthesesymptomsdisruptthepatientsdailylife.Thesesymptomsmayormaynotoriginatefromanunderlyingdiseasethougharetypicallyunconsciously
producedorexacerbatedbypsychologicalfactorssuchasstress.Patientswithsomaticsymptomdisorderpersistentlydevoteexcessivetimeandenergytothesesymptomsorrelatedhealthconcerns(eg
;repeatedlygoingtothedoctor).Somaticsymptomdisorder
typicallydemonstratesachronicandfluctuatingcourse.Managementofsomaticsymptomdisordercentersaroundregularprimarycarefollow-upthattargetscopingskills,providesreassurance,andavoidsunnecessarytests.
IncorrectAnswers:A,B.C,andD.
Conversiondisorder(ChoiceA}
;morecommonlycalledfunctionalneurologicdisorder,involvesneurologicsymptomssuchassensoryormotordysfunctionthatarenotfullyexplainedbyobjectivefindingsonphysicalexaminationorimaging.Thoughthesymptoms
disruptdailylife,patientswithfunctionalneurologicdisordermayormaynotbepreoccupiedwiththeirsymptoms.Thispatient
'ssymptomsarenotsolelyneurologic.
Infactitiousdisorder(ChoiceB}
;patientsconsciouslyproducesymptoms(eg,purposelyinjuringthemselves)forprimarygain.Primarygainisthemotivationtobecaredforwhichconstitutesanunconsciousmotivatorforthepatient
'sconsciousproductionof
symptoms.Thispatient'ssymptomsareinsteadunconsciouslyproduced.
Patientswithillnessanxietydisorder(hypochondriasis)(ChoiceC)demonstratepersistentlyexcessiveanxietyandpreoccupationaboutthepossibilitythattheymayhaveoracquireaseriousillnesssuchthattheyperformfrequenthealth-relatedbehaviors(eg,
repeatedlygoingtothedoctor)orexhibitmaladaptiveavoidance(eg,completelyavoidinggoingtothedoctor).Patientswithsomaticsymptomdisorderareanxiousabouttheirexistingsymptoms,whilepatientswithillnessanxietydisordertypicallyhavemilder
somaticsymptomsrelativetotheirconcernaboutdevelopingaseriousillness.
Inmalingering(ChoiceD)
:patientsconsciouslyproducesymptomsduetotheconsciousmotivationofsecondarygain.Secondarygainisanextrinsicmotivatorsuchasprocuringdisabilitypayments.
EducationalObjective:Insomaticsymptomdisorder,thepatientisexcessivelypreoccupiedwithoneormoresomaticsymptomssuchthatthesesymptomsdisruptthepatient'sdailylife.Thesesymptomsmayormaynotoriginatefromanunderlyingdiseasethough
aretypicallyunconsciouslyproducedorexacerbatedbypsychologicalfactorssuchasstress.
© © ©
Previous Next ScoreReport LabValues Calculator Help Pause

ExamSection1:Item43ol50 NationalBoardofMedicalExaminers^
ComprehensiveBasicScienceSelf-Assessment
Y
48.Basedonthegraphofp-aminohippurate(PAM)concentrationversusPAHsecretion,whichofthefollowingisloweratpointYthanatpointX?
80-
A}Glomerularfiltrationrate
B)PAHclearance
C)PAHexcretionrate
D)PAHfilteredload
E)Renalbloodflow
“O
0
C
E
X
O
*f40-
<
CL
0 T T
020406080100
Plasmap-aminohippurate
(PAH}concentration
mg/dL
CorrectAnswer:B.
PAHclearanceisloweratpointYthanatpointX.Renalclearanceisdefinedasthevolumeofplasmathatiscompletelyclearedofagivensubstanceinaspecifiedunitoftime.IncreasedclearanceofPAHwouidmeanthatmoreplasmaisclearedofPAHperunit
oftime,whiledecreasedclearancewouldindicatetheopposite.ThepicturedgraphindicatesthatasplasmaPAHconcentrationsincrease,theamountofPAHclearedbythekidneysintotheurineincreasesinalinearfashionuntilaninflectionpointwheretheslope
ofthegraphbecomeszero.ThisinflectionpointindicatestheplasmaPAHconcentrationatwhichtheabilityofthekidneystoclearhigherconcentrationsofPAHreachesitsmaximum.Beyondthisconcentration,thekidneysareunabletoclearanyadditionalPAH.
AtpointY,thereisahigherplasmaconcentrationofPAHwhencomparedtoX.WhiletheabsolutesecretionofPAHatpointYexceedsthatatpointX,theclearanceislowerbecausethekidneysareunabletoincreasetheirrateofclearancedespiteanincreasingly
highconcentrationofPAH.
IncorrectAnswers:A,C
;D,andE.
Glomerularfiltrationrate(GFR)(ChoiceA)describesthevolumeofrenalbloodflowthroughtheglomerularapparatusperunitoftime.Itishighlyregulatedbychangesinthesizeoftheafferentandefferentarteriolesbutisalsoaffectedbythestateofthe
glomerularapparatus,whichisaffectedinpatientswithchronickidneydisease.TheGFRwouldequalthePAHclearanceifPAHwerenotactivelyexcreted.
PAHexcretionrate(ChoiceC)isnotcorrectastheexcretionrateofPAHishigheratpointYthanatpointX.
PAHfilteredload(ChoiceD)ishigheratpointYthanatpointXThefilteredloadisdependentupontheconcentrationofPAHintheplasma,withincreasedplasmaconcentrationscorrelatingwithanincreasedfilteredload.Thefilteredloadlimitisultimately
determinedbytheGFR.
Renalbloodflow(ChoiceE)cannotbeinterpretedfromthisgraph.
EducationalObjective:Renalclearanceisdefinedasthevolumeofplasmathatiscompletelyclearedofagivensubstanceinaspecifiedunitoftime.Clearanceoftenreachesaninflectionpointbeyondwhichincreasingplasmaconcentrationsresultinnoadditional
urinaryclearanceofthesubstance,whichisdemonstratedinthepicturedgraph.WhiletheexcretionrateandfilteredloadofPAHarehigheratpointYascomparedtopointX,thePAHclearanceislower.
00 & 0f*
Previous Next ScoreReport LabValues Calculator Help Pause

ExamSection1:Item49of50 NationalBoardofMedicalExaminers^
ComprehensiveBasicScienceSelf-Assessment
Y
49.A41-year-oldwomanisevaluatedbecauseofincreasinglysevereheadachesfor6weeks.Herbloodpressureis160/100mmHgwhilestandingandsupine.Abruitisheardovertheleftcostovertebralangle.Urinalysisshowsnoabnormalities.Anangiogram
oftheleftrenalarteryshowsalternatingareasofstenosisandaneurysmaidictation("stringofbeads"sign).Whichofthefollowingconditionsoftherenalarteryisthemostlikelydiagnosis?
A)Fibromusculardysplasia
B)Hyalinearteriolosclerosis
C)Intimalfibroplasia
D)Periarterialfibroplasia
E)Perimecialhyperplasia
CorrectAnswer:A.
Fibromusculardysplasiaisalion-inflammatoryandnon-atheroscleroticangiopathyofmedium-sizedarteries(eg.renal,carotid),thatresultsinmultifocalfibrousandmuscularthickeningofthearterialwall.Theresultantstenosiscausessecondaryhypertensiondue
toabnormalstimulationofthejuxtaglomerularapparatusfromlowafferentbloodflowleadingtoexcessiveproductionofreninandangiotensin.Itcanieadtoseverehypertensioninanotherwisehealthy,youngpatientwithnomedicalcomorbiditiesorlaboratory
abnormalities.PhysicalexaminationfindingsoftenincludesignsofleftventricularhypertrophysuchasS
4
gallopandarenalarterybruitauscultatedlateraltotheumbilicus.Angiographymayrevealabead-likeappearanceoftherenalartery.ACEinhibitorscanbe
consideredforunilateralstenosisbutcanleadtoacuterenalfailureinthesettingofsignificantbilateralrenalarterystenosis.
IncorrectAnswers:B,C
:D.andE.
Hyalinearteriosclerosis(ChoiceB)referstothickeningofarteriolarwallswithhyalinedepositsseenonhematoxylinandeosinstainingthatoccursovertimesecondarytochronichypertension.Narrowarteriolesinthekidneysleadtodecreasedbloodflowand
glomerularfiltrationrate,resultinginincreasedactivationoftherenin-angiotensin-aidosteronesystemandresultanthypertension.
Intimalfibroplasia(ChoiceC)isanuncommonsubtypeoffibromusculardysplasiapresentinlessthan10%ofcasesPeriarterialfibroplasia(ChoiceD)isararesubtypeoffibromusculardysplasiaandperimedialhyperplasia(ChoiceE)isalsouncommon.Themost
commonsubtypeoffibromusculardysplasiaismedialfibroplasia.
EducationalObjective:Fibromusculardysplasiaisanon-inflammatoryandnon-atheroscleroticangiopathyofmedium-sizedarteriesthatcanleadtorenalarterystenosisandsecondaryhypertensionduetotheexcessiveproductionofreninandangiotensin,it
commonlypresentsinayoungpatientwithnomedicalcomorbiditiesorabnormallaboratoryfindings.
00 & ft*
Previous Next ScoreReport LabValues Calculator Help Pause

ExamSection1:Item50ol50 NationalBoardofMedicalExaminers^
ComprehensiveBasicScienceSelf-Assessment
Y
50.Acohortstudyassessingriskfactorsforacquisitionofinfectionwithanewlyidentifiedagentisperformed.Onlynewlydiagnosedsubjectsareeligible,andcontrolsareselectedonthebasisofage.Theresultsofthisstudyareshown:
InfectionPresent InfectionAbsent
Turtleexposure
Noturtleexposure
60 20
40 80
Whichofthefollowingistherelativeriskfortheexposurevariable?
A)0.7
B)1.0
C)1.7
D)2.2
E)3.1
CorrectAnswer:D.
Cohortstudiesareoftenusedtodefinetherelationshipbetweenanexposureandanoutcomeofinterest.Thisisoftencalculatedasrelativerisk.Thedefinitionofrelativeriskistheincidencerateintheexposedgroupdividedbytheincidencerateoftheunexposed
group.Inthisquestion,theincidenceofinfectionintheturtleexposuregroupwouldbecalculatedas60infectionsdividedby30totalpersonsintheturtleexposuregroup(60/80=0.75).Theincidenceofinfectioninthenon-turtleexposuregroupwouldbe40
infectionsdividedby120totalpersonswithoutturtleexposure(40/120=0.33).Relativeriskwouldbecalculatedastheincidenceofinfectionintheturtleexposedgroupdividedbytheincidenceofinfectioninthenon-turtleexposedgroup((6G/3G)/(4Q/12G)=2.25).
Thissignifiesthattheriskofinfectioninthegroupexposedtoturtlesis2.25timestheriskofinfectioninthegroupnotexposedtoturtles.
IncorrectAnswers:A.B,C.andE.
Theseanswerchoices(ChoicesA
;B
:C,andE)representerroneouscalculationsortheuseofrandom,incorrectnumbersthatdonotrepresenttheappropriatecalculationoftherelativeriskorotherepidemiologicmeasures.
EducationalObjective:Reativeriskisameasureofthelikelihoodofanoutcomeinagroupwithaparticularexposureversusthelikelihoodofthesameoutcomeinagroupwithoutthatexposure.Itiscalculatedastheincidenceoftheoutcomeintheexposedgroup
dividedbytheincidenceoftheoutcomeinthenon-exposedgroup.
0 0 & 0
*
&
Previous Next ScoreReport LabValues Calculator Help Pause

ExamSection2:Item1of50 NationalBoardofMedicalExaminers^
ComprehensiveBasicScienceSelf-Assessment
Y
1.A36-year-oldmanwithHIVinfectionhasbeentreatedwithacombinationofantiretroviraldrugs,includingzidovudine(AZT)
;for3years.LaboratorystudiesshowamarkedincreaseinhisplasmaHIVviralloadduringthepast3months.Viralresistanceto
zidovudineissuspected.Amutationinwhichofthefollowingismostlikelytoexplaintheresistancetozidovudineinthispatient?
A)integrase
B)Neuraminidase
C)Protease
D)RNA-dependentDNApolymerase
E)Thymidinekinase
CorrectAnswer:D.
RNA-dependentDNApolymerase(HIVreversetranscriptase)mutationsaremostlikelytoaccountforthispatient
'sresistancetozidovudine(AZT).DrugsthatblockHIVreversetranscriptaseareeithernucleosidereversetranscriptioninhibitors(NRT's)ornon-
nucleosidereversetranscriptioninhibitors(NNRTi).AZTisanNRTiwhiledrugssuchasefavirenzareNNRTIs.TheprocessofHIVreversetranscriptioniserrorproneandcanresultinmutationsoftheHIVDNAasaresultofinaccuratetrainscription,butselective
pressuregarneredbyuseofNRTIscanleadtomutationsoftheHIVreversetranscriptaseenzymethatallowittoevadedrugssuchasAZTAZTisathymidineanalogandthemutationsthatresultinresistancetoAZTandsimilardrugsarereferredtoasthymidine
analogmutations(TAMs).DependinguponthespecificmutationsinHIVreversetranscriptase,themutationcanconferresistancetoalldrugswithintheNRTiclass,butusuallyatleastthreeTAMsarenecessaryforHIVresistancetoemerge.Continueduseofthe
samedrugwiIleadtoaccumulationofadditionalmutationsduetoselectivepressure.Resistanceismostoftenpreventedbyusingacombinationofmedicationsfromdifferentclasses,butpatientswhoweretreatedintheearlydaysoftheHIVpandemicbeforethe
wideavailabilityofcombinationregimensaremorelikelytohavemutationsthatdevelopedasaresultofmonotherapywithdrugssuchasAZT
IncorrectAnswers:A.B
:C
;andE.
Integrase(ChoiceA)inhibitorsincludemedicationssuchasdolutegravirandraltegravir.IntegrationintothehostgenomeoccursviatheactionofanHIV-encodedenzyme,somutationsinthisenzymecanleadtoresistancetointegraseinhibitors.Mutationsoften
conferresistancetoallmedicationswithintheintegraseinhibitorclass.AZTisnotanintegraseinhibitorandwouldbeunlikelytoselectforsuchamutation.
Neuraminidase(Choice6)isanenzymethatbreaksglycosidiclinkagesofneuraminicacidsandismostcommonlyassociatedwiththeinfluenzavirus.
Protease(ChoiceC)inhibitorssuchasdarunavirandloptnavirwillselectformutationsinHIVprotease,whichisnecessaryinthecleavageofimmatureviralproteinsintomatureviralproteinspriortoexitfromthecell.AZTwouldnotselectformutationsofHIV
protease.
Thymidinekinase(ChoiceE)isanenzymethatphosphorylatesguanosineanalogmedicationssuchasacyclovirandvalacyclovirthatareusedtotreatherpeticinfections.ThesedrugsinhibitherpesvirusDNApolymerasebutarenotactivateduntilphosphorylated.
ThesedrugsdonotplayaroleinthetreatmentofHIVunlessthereisconcomitantinfectionwithherpesvirusandwouldnotselectformutationsinHIVreversetranscriptase.
EducationalObjective:HIVmedicationsthatinhibittheRNA-dependentDNApolymerase(HIVreversetranscriptase)areeitherNRTIsorNNRTIs.Whenusedasmonotherapy,theycanpotentiallyinduceresistanceviamutationoftheHIVreversetranscriptase.This
issuehasbeenlargelyattenuatedbyusingcombinationregimenstotreatHIV.
O f? #
Previous Next ScoreReport LabValues Calculator Help Pause

ExamSection2:Item2of50 NationalBoardofMedicalExaminers^
ComprehensiveBasicScienceSelf-Assessment
2.Aninvestigatorisstudying[
^
adrenoreceptorsinfemaleexperimentalanimals.Duringtheexperimentepinephrineisinjectedintramuscularlyintoeachanimal,andtheeffectsonpzadrenoreceptorsarethenobserved.Whichofthefollowingphysiologic
effectsismostlikelytobeobservedintheseanimals?
A)increasedmyocardialcontractility
B)Internalurethralsphinctercontraction
C)Lipolysis
D)Pilomotorcontraction
E)Pupillarydilation
F)Uterinerelaxation
CorrectAnswer:F.
Epinephrineisadirectsympathomimeticthatexertsitseffectsthroughstimulationofadrenergicreceptors,withagreateraffinityforp-adrenergicreceptorsthanfora-adrenergicreceptors,p-adrenergicreceptorshavethreeisotypes
:whichareresponsiblefor
differentactionsoftheautonomicnervoussystem,p-
^
receptorsareprimarilyresponsibleforheartrateandmyocardialcontractilitybothofwhichareincreasedwiththeadministrationofepinephrine,p
2
receptorscausesmoothmuscledilationinthebronchiand
bloodvessels,aswellasdecreaseduterinecontractility.Antagonistsofthisreceptorcanalsobeusedtodecreaseaqueoushumorproductioninglaucoma,whereasagonistscanbeusedtodecreasepotassiumconcentrationinhyperkalemiabyinducingcellular
uptake.Additionally.P2
receptorstimulationincreasesinsulinreleaseandglycogenolysts.pjreceptorsarelesscommon,butstimulationleadstothermcgenesisanddetrusorrelaxation.
IncorrectAnswers:A,6
:C,D,andE
Increasedmyocardialcontractility(ChoiceA)wouldbeseenintheseanimalsduetostimulationofpireceptors.Howeverthisinvestigatorisexaminingtheeffectsofp
2
receptors
Internalurethralsphinctercontraction(ChoiceB:isaffectedbya-adrenergicreceptors;specifically,stimulationofreceptorsleadstoincreasedbladdersphinctercontraction
Lipolysis(ChoiceC)isalsoaffectedbyP
-adrenergicreceptors,withp
2
increasinglipolysisinadiposetissue,althoughtoalesserextentthanp
^
receptors.P2
receptorshaveastrongereffectonsmoothmusclerelaxationandwouldcausemoreprominentuterine
relaxationthanlipolysis.
Pilomotorcontraction(ChoiceD)ismediatedbya-|receptors.Itwouldbeunaffectedbystimulationofp
2
receptors.
Pupillarydilation(ChoiceE)ismediatedbya1receptors.Whileitwouldbeseenwithepinephrineadministration,itisnotadirecteffectofstimulationofp
2
adrenergicreceptors
EducationalObjective:Thestimulationofp
2
adrenergicreceptorscausessmoothmuscledilationinthebronchi,bloodvessels,anduterus,decreasedplasmapotassiumconcentration,andincreasedinsulinrelease
Cl0
4
s
0
f*
Previous Next ScoreReport LabValues Calculator Help Pause

ExamSection2:Item3of50 NationalBoardofMedicalExaminers^
ComprehensiveBasicScienceSelf-Assessment
Y
3.A16-year-oldboyisbroughttotheemergencydepartmentbecauseofa2-dayhistoryofincreasinglysevereabdominalpain.Histemperatureis39X(102.2°-F)
1-pulseis86/min,respirationsare18/min,andbloodpressureis120/60mmHg.Abdominal
examinationshowsexquisitetendernessoftherightlowerquadrant.His[eukocytecountis16,00t
)/mm
3Anappendectomyisdone;theappendixisswollenwithatanexudateontheserosalsurface.Whichofthefollowingbestcharacterizestheleukocytosis
inthispatient?
A)Basophilia
B)Eosinophiiia
C)Lymphocytosis
D)Monocytosis
E)Neutrophilia
CorrectAnswer:E
Appendicitisisacuteinflammationoftheappendixandusuallyresultsfromappendicealobstructionbyafecalithorbylymphoidhyperplasia.Appendicealobstructionleadstobacteria!proliferationwithinthelumenandwalloftheappendix,eventuallyleadingto
appendicealinflammationwithpotentialnecrosisandperforationifuntreated.Bacterialproliferationprovokesaleukocytosis,whichispredominantlyneutrophilic.Appendicitispresentswithrightlowerquadrantabdominalpain,fever,anorexia,nausea
:vomiting,and
leukocytosis.
IncorrectAnswers:A.B
(C.andD.
Basophilia{ChoiceA)Isanon-specifichematologicfeatureofvariousmyeloproliferativedisorders,suchasacutemyeloidleukemia,myelofibrosis,orpolycythemiavera.Itwouldnotbeobservedasapredominantfeatureinthesettingofanacuteinflammatory
response.
bosinophilia(ChoiceB)istypicallyobservedinthesettingofallergicorparasiticresponses,neitherofwhichareMkelytobepresentbecauseofappendicitis.
Lymphocytosis(ChoiceC)isobservedinresponsetoviralinfections,someofwhichfeaturegastrointestinalsymptoms,suchascytomegalovirusorhepatitis.Whileviralinfectionsmaycauselymphoidtissueproliferationthatoccludestheappendix,acute
appendicitisensuesduetobacterialproliferationwithintheappendixandischaracterizedbyaneutrophilicresponse.
Monocytosis(ChoiceD)ischaracteristicofresponsetochronicinfections,includingtuberculosis,ehrlichiosis,andleishmaniasis.Itisnotaprominentfeatureofacuteappendicitis.
EducationalObjective:Appendicitisresultsfromobstructionoftheappendixandresultantbacterialproliferationwithinthelumenandwalloftheappendix.Bacterialproliferationprovokesaleukocytosis,whichispredominantlyneutrophilic.Ifuntreated,appendiceal
necrosisandperforationmayresult.
© © © ©
f*
Previous Next ScoreReport LabValues Calculator Help Pause

ExamSection2:Item4of50 NationalBoardofMedicalExaminers^
ComprehensiveBasicScienceSelf-Assessment
Y
4.A32-year-oldprimigravidwomandeliversahealthy3402-g(7-fb8-oz)malenewbornafteranuncomplicatedcesareandeliverybecauseofanonreassuringfetalstresstest.Twodayspriortodischargefromthehospital,shehaspersistentnumbnessofthe
areasurroundingtheabdominalincisionThephysicianassuresthepatientthatsensationwillgraduallyreturnasthenervesregenerate.Whichofthefollowingbestdescribestherate-limitingstepinthispatient'sreturntonormalsensation?
A)Dorsalrootganglioncellproliferation
B)Fastanterogradeaxonaltransport
C)Fibroblastproliferation
D}Retrogradeaxonaltransport
E)Slowanterogradeaxonaltransport
CorrectAnswer:E
Afterperipheralnerveinjury,nervesregeneratewiththehelpofslowanterogradeaxonaltransport.Whenanaxonisinjuredorsevered,theaxonandassociatedmyelindistaltothesiteofinjuryaredegradedinaprocesscalledWalleriandegeneration.Despitethis
degradation,theSchwanncellsremaininthesameorientationandpositiontoguidenerveregeneration.Proteinsynthesisinthedorsalrootgangliaceilbodiesandaxonsandsubsequentanterogradeaxonalproteintransporttotheaxonterminalmediateperipheral
nerveregeneration.Slowanterogradeaxonaltransportoccurswhenmicrotubulemotorproteinstransportcytoskeletaiproteinssuchasneutofilamentsandcytoplasmicproteinssuchasactinandglycolyticenzymestotheaxonterminal.Slowanterogradeaxonal
transportoccursatapproximately1mmperdayandisslowerthanfastanterogradeaxonaltransportorretrogradeaxonaltransport.Therefore,slowanterogradeaxonaltransportistherate-limitingstepinperipheralnerveregeneration.
IncorrectAnswers:A.B,C.andD.
Dorsalrootganglioncel!proliferation(ChoiceA)isnotaknownmechanismofperipheralnerveregeneration.Alterationsinregeneration-associatedgeneexpressionindorsalrootgangliaassistinaxonalregeneration.
Fastanterogradeaxonaltransport(ChoiceB)inperipheralnerveregenerationinvolvestransportofproteinsalongmicrotubulestotheaxonterminal,similartoslowanterogradeaxonaltransport.Fastanterogradeaxonaltransportmovesvesiclesandmembranous
organelles.Itinvolvesafastertransportratethanslowanterogradeaxonaltransport.
Fibroblastproliferation(ChoiceC)wouldresultintheformationofascar,whichisnotinnervatedandwouldthereforenotexplainthereturnofnormalsensation.Peripheralnerveregenerationshouldideallyovercomefibroblastinfiltration.
Retrogradeaxonaltransport(ChoiceD)referstothemicrotubule-dependenttransportofproteinsthatserveasinjurysignalstothedorsalrootganglioncellbody.Theseinjurysignalsleadthecellbodytoupregulatetranscriptionfactorsthatincreasethetranscription
ofregeneration-associatedgenes.
EducationalObjective:Afterperipheralnerveinjury,microtubulemotorproteinstransportnewlysynthesizedcytoskeletaiandcytoplasmicproteinstotheaxonterminalinaprocesscalledslowanterogradeaxonaltransport.Slowanterogradeaxonaltransportisthe
rate-limitingstepofperipheralnerveregeneration.
O0
It
00
Previous Next ScoreReport LabValues Calculator Help Pause

ExamSection2:Item5of50 NationalBoardofMedicalExaminers^
ComprehensiveBasicScienceSelf-Assessment
5.An18-year-oldwomancomestothephysicianbecauseofprogressivefevergeneralmalaise,andbloodinherurinesinceshebeganoralantibiotictherapyforaurinarytractinfection5daysago.Shealsohasa3-dayhistoryofarash..Hertemperatureis
38°C(100.4TXpulseis75,-min,respirationsare12/min,andbloodpressureis125/80mmHg.Physicalexaminationshowsapetechialrashoverthechest,back
;andupperandlowerextremities.Urinalysisshows:
Blood
Protein
Leukocytes
Eosinophils
3+-
1+
150/hpf
30%
Whichofthefollowingisthemostfikelydiagnosis?
A)Acutetubularnecrosis
B)Glomerulonephritis
C)IgAnephropathy
D)Interstitialnephritis
E)Papillarynecrosis
CorrectAnswer:D.
Acuteinterstitialnephritis(AIN)iscausedbyahypersensitivityreactiontomedications(eg.nonsteroidalanti-inflammatorydrugs,diuretics,sulfonamides,rifampin,protonpumpinhibitors,antibiotics),infections,orautoimmunedisorderssuchassarcoidosisand
systemiclupuserythematosus.Patientsmaybeasymptomatic,butcommonsigns,symptoms,andlaboratoryfindingsincluderash,azotemia,sterilepyuria,hematuria,andeosinophilia.Patientsmayhaveproteinuria,buttypicallynottotheextentofnephrotic
syndrome.AINcanbecomplicatedbyacutekidneyinjuryandadeclineinkidneyfunction.TreatmentofAINincludessupportivecareanddiscontinuingtheoffendingdrugwhenthereisone.Eosinophilsareoftendetectedonurinalysis,asdemonstratedinthiscase.
IncorrectAnswers:A.B,C,andE.
Acutetubularnecrosis(ChoiceA)occursfollowinganischemicornephrotoxicinsulttothekidneys,whichresultsinnecrosisofthetubularepithelium.Granular,muddybrowncastsaretypicalonurinalysis.Itwouldbeunlikelytocausesystemicsymptoms,fever,
andhematuriaasseeninthispatient.
Glomerulonephritis(ChoiceB)referstoavarietyofglomerulardiseases,includingnephriticandnephroticsyndromes.Nephriticsyndromestypicallypresentwithacuterenalfailureassociatedwithhematuria,redbloodcellurinecasts,andhypertension.Nephrotic
syndrometypicallypresentswithexcessiveproteinuria(>3g/day)hyperlipidemia,hypoalbuminemia,.andedema.Antibiotics,rash,andurineeosinophilsaremoreconsistentwithAIN.
IgAimmunecomplexdepositioninsmallvesselscanleadtoIgAnephropathy(ChoiceG).WhenIgAdepositionoccursintherenalmesangium,glomerulonephritismayensue,causingmicroscopichematuria,redcellcasts,andproteinuria.
Renalpapillarynecrosis(RPN)(ChoiceE)occursfollowingischemic,inflammatory,infectious,ortoxin-mediateddamagetotherenalpapillaanddescribesthesloughingandlossofthepapillaeincludingsubstructuressuchasthedistalcollectingtubule.RPNcan
betriggeredbyinfections(eg,acutepyelonephritis),diabetesmeilitus,sicklecelldisease,ornonsteroidalanti-inflammatorydrugs.Ittypicallypresentswithhematuriaandacuteflankpain.
EducationalObjective:AINiscausedbyahypersensitivityreactiontomedications(eg,nonsteroidalanti-inflammatorydrugs,diuretics,sulfonamides,rifampin,protonpumpinhibitors,antibiotics),infections,orautoimmunedisorders.Patientsmaybeasymptomatic,
butcommonsigns,symptoms,andlaboratoryfindingsincluderash,azotemia,sterilepyuria,hematuria,eosinophilia,andurineeosinophils.
© © ©
A
Previous Next ScoreReport LabValues Calculator Help Pause

ExamSection2:Item6of50 NationalBoardofMetlicaiExaminers^
ComprehensiveBasicScienceSelf-Assessment
y
6.A33-year-oldwomanhashadweaknessoftherightlowertwothirdsofthefaceforthepast2months.Whichofthefollowinglabeledregionsinthenormalbrainshownisthemostlikelysiteofthelesioncausingthissymptom?
Centralsufcus
ED F
C G
B
A
J
I
H
A)
B)
C)
D)
E)
F)
G)
H)
I)
J)
CorrectAnswerC.
ChoiceCidentitiestheleftposterolateralfrontallobe,representingtheprimarymotorcortexintheprecentralgyrus.Thisbrainregionmediatesmotorfunctionoftheright-sided(contralateral)face.Lesionsofthisarearesultinweakness,facia
1droop,andanupper
motorneuronpatternofdysfunction(eg
;hyperreflexia).Importantly,theforeheadwouldbespared,asthecranialnerveVIInucleusthatcontrolsforeheadmusculatureisduallyinnervatedbyuppermotorneuronsfrombilateralprecentralgyri.Becauseofthis,the
patientwouldbeexpectedtosymmetricallyraisehereyebrows.Alternatively,lesionsinvolvingthelowermotorneuronsoffacialexpression(eg,facialnerveinflammation
;
nBellpalsy)affecttheforeheadandlowerfacetogether.Alesionoftheprecentralgyrusin
this33-year-oldwomancouldbecausedbyautoimmunediseasesuchasmultiplesclerosis,alongwithmalignancy,trauma,orstroke.
IncorrectAnswers:A.B,D.E,F,G.H,1,andJ.
ChoiceAidentifiestheleftposterior,inferiorfrontallobe.Inthedominanthemisphere(typicallythelefthemisphere),thisbrainregionrepresentstheBrocaarea.LesionscanresultinBrocaaphasia.
ChoiceBidentifiestheprefrontalcortex.Thisareaisassociatedwithfunctionsincludinglearning,reasoning,problemsolving,emotion,behavioralcontrol,memory,self-regulation,andpersonality.Lesionsaffectingthisareamayresultinbehavioraldysregulation
andthedevelopmentofpsychiatricsymptoms(eg,post-strokedepression).
ChoiceDidentifiestheleftposteromedialfrontallobe,representingtheprimarymotorcortexintheprecentralgyrusThisbrainareacontrolsthemotorfunctionoftheright(contralateral)lowerextremity.Lesionsofthisregionwouldleadtoanuppermotorneuron
patternofweaknessoftherightleg.
ChoiceEidentifiestheleftanteromedialparietallobe,representingtheprimarysensorycortexinthepostcentralgyrus.Thisbrainareacontrolssensationoftheright(contralateral)leg.Lesionsofthisregionwouldleadtosensorydeficitsoftherightleg.
ChoiceFidentifiestheleftanterolateralparietallobe,representingtheprimarysensorycortexinthepostcentralgyrus.Thisbrainregionmediatessensationintheright(contralateral)distalupperextremityandface.
ChoiceGidentifiestheleftangulargyrusintheinferiorparietallobe.Thisbrainareamediatesmultimodalsensoryintegrationandassistsinmentalspatialrotation,payingattention,andsolvingproblems.Patientswithleftangulargymsdamagemaydemonstrate
agraphia,acalculia,fingeragnosia,andleft-rightdisorientation(knownasGerstmannsyndrome).
ChoiceFIidentifiestheleftposterior,superiortemporalgyrus,anareathatinthedominanthemispheremakesuponeportionofWernickearea,abrainregioninvolvedintheunderstandingoflanguage.LesionsinvolvingthisareacanresutinWernickeaphasia.
Choiceiidentifiesthemiddletemporalgyms,whichassistsinsemanticmemoryprocessing,visualperception,andsensoryintegration.Lesionsinthisbrainareahavebeenassociatedwithdeficitsincomplexvisualperceptionandsemanticmemoryprocessing
ChoiceJidentifiesthesuperiortemporalgyrus,whichrepresentstheauditoryassociationarea.Lesionsofthisareamaydisruptspokenwordrecognition.
EducationaObjective:Thelateralaspectoftheprecentralgyrusmediatesthemotorfunctionofthecontralateralface.Lesionsaffectingthisregioncanresultinfacialdroop,weakness,andanuppermotorneuron-patternofdysfunction
ft 4P
Tl?
Previous Next SconeReport LabValues Calculator Help Pause

ExamSection2:Item7of50 NationalBoardofMedicalExaminers^
ComprehensiveBasicScienceSelf-Assessment
7.A4G-year-oldwomancomestothephysicianbecauseoftemperaturesof38to39X{1DO.4to102.2aF)andmalaiseforthepast2days.Shehaschronicalcoholdependence.Thereareseveralspiderangiomasontheface,chest,andback;sheisnot
jaundiced.TheabdomenisprotuberantandnontenderThereisshiftingdullnesstopercussion.Afirmliveredgeisballotable3cmbelowtherightcostalmargin.Routinelaboratorystudiesshowmildanemia,mildlyincreasedhepatictransaminaseactivities,
andadecreasedserumalbuminconcentration.Peritonealaspirationyieldsserousfluidwith1100leukocytes/mm
5(80%neutrophils)and100erythrocytes/mm
3Whichofthefollowingprocessesbestaccountsforthepatient'sfebrileillness?
A)Acutecholecystitis
6)Chronicpancreatitis
G}Exacerbationofautoimmunehepatitis
D)Pelvicinflammatorydisease
E)Spontaneousbacterialperitonitis
CorrectAnswer:E
Spontaneousbacterialperitonitis(SBP)isthemostlikelyetiologyofthispatientsfever.SBPreferstothedevelopmentofaperitonealinfectioninpatientswithascitesthatisnotduetoinstrumentationorintroductionofbacteriaintotheperitoneumfroman
alternativesource.Thispatientmostlikelyhascirrhosisfromalcoholusedisorderasevidencedbyafirmfiver,spiderangiomas,andascites.SBPisacommoncomplicationinpatientswithcirrhosisandascitesregardlessoftheetiology.Portalhypertension(PH)
leadstoreducedintestinalmotilitywithbacterialstasisandovergrowthinadditiontoimpairedgastrointestinalimmunity.PHalsocausesgutwalledemaandthisconfluenceoffactorscanpredisposetotranslocationofbacteriafromthegutlumenintotheblood
streamoracrossthewallintotheperitonea!cavity.Thesepathogenscanseedtheperitonealcavitywhich,inpatientswithascites,providesaperfectgrowthmediumforbacteria.Symptomscanincludefever,hepaticencephalopathy,abdominalpain,andsignsand
symptomsofsepsis.RiskishigherinpatientswithlowerserumalbuminconcentrationsandinthosewhohaveexperiencedpriorepisodesofSBP.Diagnosisismadebydiagnosticparacentesisrevealinganabsoluteneutrophilcountofatleast250cells/mm
3
.Gram
stainisnotsensitivefarthediagnosis,butcommonpathogensincludeentericflora,Escherichiacoli.Streptococcusspecies,andKlebsiellapneumoniae.Treatmentiswithantibiotics,followedbyprophylaxiswithciprofloxacinortrimethoprim-sulfamethoxazole.
incorrectAnswers:A.B,C,andD.
Acutecholecystitis(ChoiceA)presentswithfeverandrightupperquadrantpain.Ifgallstonesmigrateintothecommonbileduct,obstructioncanleadtoincreasedtransaminaselevelsandacholestaticpatternofliverinjurywithincreasedalkalinephosphatase.
Rightupperquadrantultrasoundrevealsgallstones,athickenedgallbladderwall,andpericholecysticfluid,althoughthelattertwofindingscanbeseeninpatientswithcirrhosisintheabsenceofcholecystitis.Aperitonealfluidsampleshowing880neutrophilsis
diagnosticofSBPinthiscase.
Chronicpancreatitis(ChoiceB)isoftenaconsequenceofrecurrentacutepancreatitis,whichmaybesecondarytoalcoholuse,hypertriglyceridemia,orautoimmunepancreatitis.Itpresentswithmalabsorptionsteatorrhea,deficienciesoffat-solublevitamins,and
chronicabdominalpain.
Exacerbationofautoimmunehepatitis(AIH)(ChoiceC)wouldrequireapriordiagnosisofAiHandwouldusuallypresentwithjaundiceandliverfailureasevidencedbyanincreasedprothrombintimeandencephalopathy.
Pelvicinflammatorydisease(ChoiceD)isfrequentlycausedbyChlamydiatrachomatisandNeisseriagonorrhoeae,althoughissometimescausedbyE.coirBacteroidesfragilis.StaphylococcusorStreptococcusspecies.Symptomsincludepelvicpainandcervical
discharge;physicalexaminationrevealscervicalmotionoradnexaltenderness.Thispatient'spresentationismoreconsistentwithSBP.
EducationalObjective:SBPisdiagnosedinpatientswithasciteswhohaveanasciticfluidabsoluteneutrophi
:
countofatleast250cells/mm
3
intheabsenceofothercausesofperitonitissuchasbowelperforation.Itcommonlycausesfever,encephalopathy,and
abdominalpain.Treatmentiswithantibioticsdirectedagainstthemostlikelypathogenandmostpatientsaresubsequentlyplacedonantibioticprophylaxis.
© © ©
Previous Next ScoreReport LabValues Calculator Help Pause

ExamSection2:Item8of50 NationalBoardofMedicalExaminers^
ComprehensiveBasicScienceSelf-Assessment
Y
8.Apatientwitha6-monthhistoryofheatintolerance,fatigue,episodesoftachycardia,andweightlosshasadiffuselyenlargedthyroidgland.Serumconcentrationsoftriiodothyronine(T
3
)andthyroxine(TJareincreased:thyroid-stimulatinghormoneis
decreased.Whichofthefollowingisthemostlikelydiagnosis?
A)Autoimmunethyroidhyperplasia
B)Pituitaryneoplasm
C)SurreptitiousingestionofT
4
D}Thyroidneoplasm
CorrectAnswer:A.
Gravesdiseaseisthemostcommoncauseofhyperthyroidismandisanautoimmunethyroidhyperplasia.Itiscausedbyanautoantibodythatstimulatesthethyroid-stimulatinghormonereceptors(TSHR)onthyroidfollicularcells.ExcessivestimulationofTSHR
leadstofollicularthyroidhyperplasiaandadiffuselyenlargedthyroid.SerumlaboratoryevaluationtypicallyrevealsincreasedconcentrationsofT
3andT
4.decreasedthyroid-stimulatinghormone(TSH),andthepresenceofTSHRstimulatingantibodies.Graves
diseasepresentswithsymptomsofhyperthyroidism,includingheatintolerance,weightloss,tremor,hyperreflexia,warm,moistskin,andpretibialmyxedema.SomepatientswithGravesdiseasealsodevelopthyroidophthalmopathy,whichcancausediplopia,
proptosis,andrestrictivestrabismus.
incorrectAnswers:B,C,andD.
Pituitaryneoplasm(ChoiceB)mayleadtohyperthyroidismifthepituitaryneoplasm,mostcommonlyanadenoma,secretesexcessiveTSH.SerumconcentrationsofTSHwouldbemarkedlyincreased,ratherthandecreased.TSH-secretingadenomasarerareand
constituteonlyasmallportionofpatientswithhyperthyroidism.
SurreptitiousingestionofT
4
(ChoiceC),alsoknownasfactitiousthyrotoxicosis,mayresultfromdeliberateingestionofthyroidhormone,suchaslevothyroxine.Factitiousthyrotoxicosiscanbedistinguishedfromothercausesofhyperthyroidism,suchasGraves
disease,bytheabsenceofothersuggestiveclinicalfindings,includinggoiter,thyroidswelling,orbitalinvolvement,orpretibialmyxedema.
Thyroidneoplasm(ChoiceD),suchaspapillaryormedullarythyroidcarcinoma,rarelyleadstohyperthyroidism.Mostthyroidcarcinomaspresentasacoldnoduleonradioactiveiodineuptakescan,whichcandistinguishtheselesionsfromhyperfunctioningthyroid
nodules.
EducationalObjective:GravesdiseaseiscausedbyanautoantibodythatstimulatestheTSHRonthyroidfollicularcells.Excessivestimulationof1SHRleadstofollicularthyroidhyperplasiaandadiffuselyenlargedthyroid.Serumlaboratoryevaluationtypically
revealsincreasedconcentrationsofT
^
andT
4:decreasedTSH,andthepresenceofTSHRstimulatingantibodies.
0 0 #
Previous Next ScoreReport LabValues Calculator Help Pause

ExamSection2:Item9of50 NationalBoardofMedicalExaminers^
ComprehensiveBasicScienceSelf-Assessment
Y
9.A19-year-oldmanwhoisacollegestudentisbroughttotheemergencydepartmentbecauseofthesuddenonsetofright-sidedchestpainanddifficultybreathingafteranaccidentinwhichhewasthrownfromhisbicycle.Hehasdifficultywalkingandcannot
climbstairsbecauseofpainandshortnessofbreath.Heisslightlycyanotic,afebrile
:andtachypneic.Whichofthefollowingismostsuggestivethatfracturedribscausedtherespiratoryproblem?
A)Bronchophony
B)Expiratorystridor
C)Inspiratorystridor
D)Subcutaneouscrepitus
E)Succussionsplash
CorrectAnswer:D.
Chestpain
:shortnessofbreath
;andtachypneaafterablunttraumaticinjuryraisesconcernforribfractureandpneumothorax.Ribfracturescanbecomplicatedbytearingoftheadjacentvisceralorparietalpleura,allowingairinthelungtoescapeintothe
intrapleuralspaceandtosubcutaneoustissues.Airinsubcutaneoustissuesisdemonstratedbysubcutaneouscrepitusuponpalpation.Painfromribfractures,alongwithreducedpulmonaryexpansionandventilationinthesettingofpneumothorax,presentswith
shortnessofbreath,tachypneaandrespiratorydistress.Ribfracturesaretypicallytreatedsupportively,withoxygensupplementationandpaincontroltoreducecomplicationsofatelectasisandpneumonia.Inthepresenceofanassociatedpneumothoraxor
hemothorax,tubethoracostomymaybenecessarytopromotelungexpansion,excludesignificanthemorrhage,andpreventresidualpneumothorax,empyema,orfistula.
IncorrectAnswers:A.B,C.andE.
Bronchophony(ChoiceA}referstoanabnormalorincreasedsoundofvoiceauscultatedwithastethoscopeoveranareaoflungconsolidation.Itisexpectedinpneumonia.Intheabsenceoffeverorproductivecoughsuggestiveofpneumonia,itwouldbeunlikelyin
thispatient.
Expiratorystridor(ChoiceB)canbeseenintracheomalacia,whichresultsinexcessivetrachealend-expiratorycollapseduetoatrophyand/ofreductionofthetrachealelasticfibersanddecreasedintegrityoftrachealcartilage.Inadults.,thismostcommonlyoccurs
afterprolongedendotrachealintubation,whichdamagesthetrachealcartilagedirectly.
Inspiratorystridor(ChoiceC)canbeseeninillnessesobstructingtheairwaysuchascroupandepiglottitisCroupischaracterizedbyacuteviralinflammationofthelarynxcausingupperrespiratorytractsymptomsandabarkingcough.Epiglottitispresentswith
feverandacute,severepharyngitis,drooling,hoarseness,anddysphagia.
Succussionsplash(ChoiceE)describestheauscultationfindinguponback-and-forthmovementoffluidinthestomachorintestines,suchasseeningastroparesis,gastricoutletobstruction,orbowelobstruction.
EducationalObjective:Ribfracturesafterblunttraumaticinjurytothechestcanbecomplicatedbytearingofthevisceralorparietalpleura,allowingairinthelungorpleuralspacetoescapetothesubcutaneoustissues,whichresultsinsubcutaneouscrepitusupon
examination.
o 0
Previous Next ScoreReport LabValues Calculator Help Pause

ExamSection2:Item10o!50 NationalBoardofMedicalExaminers^
ComprehensiveBasicScienceSelf-Assessment
10.AG2-year-oldwomandevelopsdifficultybreathing.Pulmonaryfunctiontestsbeforeandafterbronchoditatortherapyshownochanges.Predictedandpatientvaluesare:
PredictedTest Patient
FVC(L)
FEV -!
(L)
FEV/FVC
Totallungcapacity(L)
Residualvolume(L)
5.0 4.0
4.0 2.4
0.3 0.6
6.0 7.2
1G 2.7
Whichofthefollowingisthemostlikelyexplanationforthesefindings?
AirwayResistance LungCompliance
A) T
B) normalT
C) Normal i
D)
i T
E)
I1
CorrectAnswer:A.
Increasedairwayresistanceandincreasedlungcompliancearefeaturesofchronicobstructivepulmonarydisease(CORD).Lungcomplianceistheabilityofthelungtostretchandexpand;increasedcompliancemeansthatthelunghasagreaterabilityto
expand
:butitdoesnotimplythatthelunghasequalcapabilitytocontract.Airwayresistanceistheresistancetoairflowthroughthebronchiandbronchioles.Spirometryisausefultoolfordifferentiatingvariousformsoflungdisease.Patientscanusuallybe
categorizedasobstructive,restrictive
:ormixedpatterns.PatientswithobstructivelungdiseasehaveanFEVyFVCratiolessthan0.7andanFEV
1
lessthan80%ofpredictedvalueswhilepatientswithrestrictivefungdiseasehaveanFEVyFVCratiogreaterthan
0.8.PatientswithCOF'D
:whichismostcommonlycausedbyprolongedsmokingorexposuretosecondhandsmoke,experiencedynamiccollapseofthebronchiduringexpirationleadingtooutflowobstructionandairtrapping.Outflowobstructionleadsto
increasedairwayresistanceandisreflectedbyareducedFEVt
ThelungparenchymainpatientswithCORDlackselasticityandthenormalrecoilofthelungisimpaired,whichisreflectedasincreasedcompliance.This,inconjunctionwithairtrapping,isusually
demonstratedbyanincreasedresiduallungvolume(RV)andtotallungcapacity(TLC).TheseverityofoutflowobstructionisgradedaccordingtotheFEV
^
withlowerreadingsconsistentwithseveredisease.Responsetobronchodilatorsisimportantin
distinguishingCORDfromasthmaandfromasthma/COPDoverlapsyndromes.Inasthma,improvementinoutflowobstructionasmeasuredbyanincreasein by12%and200-mLabsolutevolumeisexpected.InCOPD
;noresponsetobronchodilatorsis
expected,whichisthecaseinthispatient.
IncorrectAnswers:B.CrD,andE.
increasedairwayresistanceandnormallungcompliance(ChoiceB)isseeninpatientswithasthma.PatientswithasthmawillhaveareducedFEVyFVCbuttheFEV
1willrespondtobronchodilators
:whichrepresentsareversibledefect.Aspatientsdonothave
parenchymaldisease,theirlungcomplianceisnormal.
Normalairwayresistanceanddecreasedcompliance(ChoiceC)isseeninrestrictivelungdisease,whichcanoccurasaresultofconditionssuchasidiopathicpulmonaryfibrosisorneuromusculardisorderssuchasmusculardystrophy.
Decreasedairwayresistanceandincreasedlungcompliance(ChoiceD)mayoccurinthesettingoftreatedCOPDorchronicbronchitis.Thelungcomplianceisincreasedasaresultoflossofalveolarseptaanddestructionofalveoli,andiftherapytoreduce
bronchialinflammationsuchasthroughsuctioningandcorticosteroidsiseffective,airwayresistancefallstopermitventilation.
Decreasedairwayresistanceanddecreasedlungcompliance(ChoiceE)canbeseeninnormalpatientsatmaximuminspiration.Innormallungs,increasingthevolumeofthelungsexpandstheairwaysanddecreasesairwayresistancewhilethecomplianceof
thelungsdecreasesatmaximuminspiration.
EducationalObjective:COPDresultsinlossofthenormalelastanceofthelungwithaconsequentincreaseincompliance.Duringexpiration,dynamicairwaycollapsemanifestsasareducedFEVt
whilechronicairtrappingandlossofelastancearereflectedas
increasedRVandTLC.
o© # m n©
Previous Next ScoreReport LabValues Calculator Help Pause

ExamSection2:Item11of50 NationalBoardofMedicalExaminers^
ComprehensiveBasicScienceSelf-Assessment
Y
11A3-year-oldgirlisbroughttothephysicianbecauseofa2-weekhistoryofdiarrhea.Hertemperatureis37.6X(9&.8°F)
;pulseis7G/min
:respirationsare13/min.andbloodpressureis110/70mmHg.Physicalexaminationshowsgeneralized
[ymphadenopathy.ACTscanofthechestandabdomenshowsenlargedlymphnodesinthemesenteryandpara-aorticregion.Examinationofafymphnodebiopsyspecimenshowsmarkedproliferationofhistiocytesandnumeroussegmentedneutrophils.
Granulomataareabsentandspecialstainsshownumerousacid-fastbacilli.whicharesubsequentlyidentifiedasMycobacteriumavium-intracellulare.SerumstudiesshownormalconcentrationsofIgA,lgG
;IgM,Blymphocytes,Tlymphocytes
:andCD4+
andCDS+Tlymphocytes.Thispatientmostlikelyhasdefectivefunctionorexpressionofwhichofthefollowingproteins?
A)ClassIMHCmolecules
B)Interferon-gammareceptor
C)lnterleukin-2(IL-2)receptor
D)Leukocytefunction-associatedantigen-1
E)NADPHoxidase
CorrectAnswer:B.
Certainintracellularpathogenscanavoiddetectionbytheimmuresystembyreplicatingwithinmacrophages.Tokillintracellularmicrobes,macrophagesrelyoncomplexsignalingbetweentheinnateandadaptiveimmunesystems,initialmacrophageactivation
occurswithbindingofpathogen-associatedmolecularpatternstotoll-likereceptors.Thisstimulatesreleaseofproinflammatarycytokinesincludinginterleukir-12(IL-12),whichstimulatesTlymphocytestoproduceinterferon-gamma(IFJM-y).IFN-ythenbindstothe
JFN-yreceptoronmacrophages,whichtriggersaJAKkinasesignalingcascadethatallowsmacrophagestobegineliminatingintracellularpathogens.ThiscycleisreferredtoastheIL-12/IFN-yaxis.IL-12alsoinduceshelperandcytotoxicT-lymphocyte
differentiation,aswellasactivationofnaturalkillercells.DeficienciesintheIFN-yreceptortypicallypresentinearlychildhoodwithseveremycobacterialand/orsalmonellalinfections.Patientsareunabletoformgranulomasduetotheimpairedstimulationof
macrophages.
IncorrectAnswers:A,C.D.andE.
ClassIMHCmolecules(ChoiceA)arefoundonallnucleatedcellsandplatelets.TheypresentfragmentsofantigensgeneratedbydegradationofproteinsinthecytosoltoTlymphocytes.ForeignantigenswillstimulatecytotoxicTlymphocytestoinitiateapoptosisin
thepresentingcell.
Jnterteukin-2(L-2)receptors(ChoiceC)areprimarilyfoundonTlymphocytes.ActivationstimulateshelperTlymphocytes,cytotoxicTlymphocytes,andregulatoryTlymphocytes.TheyarenotinvolvedintheIL-12/IFN-Yaxis,anddeficiencyisnotassociatedwith
mycobacterialinfections.
Leukocytefunction-associatedantigen-1(LFA-1)(ChoiceD)isanintegrininvolvedinleukocyteadhesiontothevascularendotheliumduringinflammatoryrecruitmentandmigration.MutationorabsenceresultsinleukocyteadhesiondeficiencytypeI
NADPHoxidase(ChoiceE)deficiencyisfoundinchronicgranulomatousdisease,inwhichgranulomasformbuttheabsenceofaneffectiveoxidativeburstimpairsmicrobialkilling.NADPHoxidaseisrequiredintheformationoffreeradicals.
Educationa-Objective:TheIL-12/IFN-ysignalingaxisisrequiredformacrophagestobeabletokiIintracellularpathogens.DeficiencyintheIFN-yreceptorincreasestheriskofseveremycobacterialandsalmonella!infectionsearlyinlife.
O0
It 4
s
&
Previous Next ScoreReport LabValues Calculator Help Pause

ExamSection2:Item12o!50 NationalBoardofMedicalExaminers^
ComprehensiveBasicScienceSelf-Assessment
Y
12.DrugXisusedtotreatpainassociatedwithrheumatoidarthritis.Thedrugisaweakacid,withapKaof4.4:itisabsorbedprincipallythroughthestomach,itisdeterminedthatDrugXisabsorbedefficientlybythebodybecauseoftheionizationconditions
underwhichitexistsatgastricandbloodpH.Whichofthefollowingsetsofphysicalchemicalstatesofthedrugismostlikely?
AtGastricpH
Ionized
Ionized
Nonionized
AtBloodpH
ionizedA)
B) nonionized
C) ionized
D) Nonionized nonionized
CorrectAnswer:C.
pKa
:thenegativelogoftheaciddissociationconstantdescribesthetendencyofagivenacidtoexistinaprotonatedordeprotonatedforminasolutionofgivenpH.pKaindicatesthepHatwhich50%ofthemoleculeswillexistineachprotonatedand
deprotonatedforms.pKaIsinverselyrelatedtothestrengthofanacid,suchthatastrongacidwillhavealowpKavalueandaweakacidwillhaveahighpKavalue.TherelationshipbetweenpKaandthepHofabufferingsolutionisdescribedwiththeHenderson-
Hasselbalchequation.IngeneralifanacidexistsinasolutionwithapHthatislessthanitspKatheacidwillexistinanonionized,protonatedform.IncontrastifanacidexistsinasolutionwithapHthatexceedsitspKathentheacidwillexistpredominantlyinits
ionized,conjugatebaseform.DrugXisaweakacidwithapKaof4.4ThepHofgastricacidislessthan2
;therefore,duetotheexcessofprotonsinsolution,DrugXwillbinditsprotoninthestomachandwillbepredominantlynonionized.whichpermitsits
absorptionacrossthehydrophobicintestinalmembrane.ThepHofthebloodstreamisapproximately7.35-7.45,andundertheseconditionsDrugXwillbepredominantlyionized.
IncorrectAnswers:A,Ei
:andD.
ChoiceAdescribesastrongacidwithessentiallycompletedissociation(pKalessthan1}.AstrongacidwouldbecapableofdissociationevenundertheacidicgastricpH.Examplesincludesulfuricacid,hydrobromicacid
:andhydroiodicacid.
ChoiceBdoesnotdescribethebehaviorofanacidasitisionizedatalowgastricpHbutnonionizedatahigherbloodpH.
ChoiceDdescribesaveryweakacidwithapKagreaterthanthepHofbloodandinsteadlikelydescribesabase,amoleculeacceptingaprotonatanyacidicorneutralpH.
EducationalObjective:pKadescribesthetendencyofagivenacidtoexistinaprotonatedordeprotonatedforminasolutionofgivenpH.AcidicmoleculesdissociateintoprotonsandconjugatebasewhentheyarefoundinsolutionswithapHgreaterthantheir
pKa.
© © © ©f*
Previous Next ScoreReport LabValues Calculator Help Pause

ExamSection2:Item13of50 NationalBoardofMedicalExaminers^
ComprehensiveBasicScienceSelf-Assessment
Y
13.Apreviouslyhealthy10-year-c-ldgirlisbroughttothephysicianbyhermotherbecauseofa6-weekhistoryofheadache
;nausea,anddifficultywalking.AnMRIofthebrainshowsamassintheposteriorfossathatisfoundtobeanastrocytoma.Thistumor
developedfromcellsthatnormallyservewhichofthefollowingfunctionsinthebrain?
A)Formationofmyelinsheathsinthecentra!nervoussystem
B)Phagocytosisofrecycledsynapticterminalmembrane
C)Productionandsecretionofcerebrospinalfluid
D}Terminationofactionpotentials
E)Transportofhormonesfromthecerebralspinalfluidtocapillaries
F)Uptakeofaminoacidneurotransmitters
CorrectAnswer:F.
Astrocytesareasubtypeofglialcellinvolvedintheuptakeofaminoacidneuretransmitterssuchasglutamate,y-aminobutyricacid(GA6A)
:andglycine.Astrocytessupportneuronsbybufferingtheextracellularspace
:regulatingenergymetabolism,andreactingto
neuronalinjury.Inresponsetodamagetothecentralnervoussystem(CNS),glutamatemaybereleasedrapidlyintothesynapticclefts.Whensuchconcentrationspersistathighlevels,neuronalapoptosisensues.High-affinityglutamatetransportersonastrocytes
arethemainmechanismfartheremovalofglutamatefromsynapses,therebyprotectingthebrainfromglutamate-inducedexcitotoxicity.Pilocyticastrocytomas,themostcommonpediatricbraintumor,arebenignbraintumorstypicallyfoundintheposteriorfossa.
Cerebellartumorsmaypresentwithdizzinessandsymptomsofincreasedintracranialpressure(duetocompressionofthefourthventriclebythetumorandconsequentobstructivehydrocephalus)suchasheadacheaswellascerebellarsigns(eg,gaitataxia,
nystagmus,anddysmetria).Treatmentcentersaroundsurgicalresection.
IncorrectAnswers:A.B.CaD,andE
FormationofmyelinsheathsintheCNS(ChoiceA)ismediatedbyoligodendrocytes,whichareglialcellsthatproducemyelin.Oligodendrogliomascanoccurinchildrenbutarelesscommonthanastrocytomasandaretypicallylocatedsupratentorial
Phagocytosisofrecycledsynapticterminalmembrane(ChoiceB)isperformedbymicroglialceLs,whicharederivedfromprecursormonocytes.Tumorsderivedfrommicrogliaarerare.
Theproductionandsecretionofcerebrospinalfluid(CSF)(ChoiceC)isaccomplishedbythechoroidplexus,whichformstheblood-CSFbarrier.Thechoroidplexusisacomplexnetworkofcapillarieslinedbydifferentcellssuchastheependymalcells,whichform
thepermeableliningoftheventricles.Theseperiventricularcellswouldnotformtumorsintheparenchymaoftheposteriorfossa.
Terminationofactionpotentials(ChoiceD)ismediatedbyionchannelsonneurons.Oncetheintracellularspacebecomesdepolarized,sodiumchannelsinactivate,promotinghyperpolarizationoftheintracellularspacebypotassiumeffluxthatisunopposedby
sodiuminflux.Thisdrivestheneuronbacktoitsrestingpotential,terminatingtheactionpotential.
TransportofhormonesfromtheCSFtothecapillaries(ChoiceE)iscontrolledbyarachnoidvilliorbyendothelialcells.ArachnoidvijlbaresmallprotrusionsofthearachnoidmaterthatdrainCSFintotheduralvenoussinuses,whereasendothelialcellslinecapillaries
(substancesthatdiffuseintocapillariesmustcrossthroughorbetweenendothelialcells).ProliferationsofthearachnoidvilJi(eg,arachnoidcysts)wouldappearasextra-axialmassesratherthanmasseswithintheposteriorfossaparenchyma.
EducationalObjective:Astrocytesareasubtypeofglialcellinvolvedintheuptakeofaminoacidneurotransmitterssuchasglutamate,whichprotectsneuronsfromexcitotoxicity.Pilocyticastrocytomasarebenignpediatrictumorsoftheposteriorfossa,whichmay
presentwithsignsofincreasedintracranialpressureandcerebellardysfunction.
© © ©
Previous Next ScoreReport LabValues Calculator Help Pause

ExamSection2:Item1+ol50 NationalBoardofMetlicaiExaminers^
ComprehensiveBasicScienceSelf-Assessment
y
14.A26-year-oldwomancomestothephysicianbecauseofa1-weekhistoryofrectalpainthatismademoreseverebydefecation,andoccasionalbloodonthetoilettissueafterabowelmovement.Shesaysthather
stoolsappearnormalandthatshehasnothadanytrauma.Shehasahistoryofchronicconstipation.Visualrectalexaminationshowsthefindingsinthephotograph.WhichofthefollowingIsthemostlikely
diagnosis?
A)Analfissure
8}Bowencarcinoma
C)Condylomaacuminatum
D)Perianalabscess
E)Prolapsedinternalhemorrhoid
CorrectAnswer:A.
Thispatient
'
sexaminationdemonstratesananalfissureattheposteriormidlinepositionwhichexplainsthepatient'srectalpainandscanthematochezia.Analfissuresareacommoncauseofsevererectalpainwithdefecationandoccurinpatientswithrisk
factorssuchaschronicconstipation,thosewhopartakeinreceptiveanalintercourse,oraftervaginalbirth,althoughdiarrheacanpredisposetofissures.Mostfissuresoccurattheposteriormidlinewiththeremaindertypicallyoccurringintheanteriormidline.A
fissurestartswithaninitialtearintheanodermwithexposureofpartoftheanalsphincterthatsubsequentlyleadstomusclespasmwithcompromiseoflocalbloodflow.Thiscandelayhealingofthefissure.Treatmentiswithsitzbaths,fibersupplements,topical
IkJocainejelly,andtopicalnitroglycerin.FissuresthatpersistforgreaterthantwomonthsorthosethatdonotoccuralongthemidlinecanindicatethepresenceofunderlyingdiseasesuchasCrohndisease,andthesepatientsshouldreceivefurtherevaluation.
IncorrectAnswers:B,C
;D,andE.
Bowencarcinoma(ChoiceB),alsocalledsquamouscarcinomainsitu,describesasquamouscarcinomaoftheepitheliumthatprimarilyaffectsareaswithheavysunexposure.Itpresentswithaslowgrowingscalypatchorplaque.Itwouldbeunlikelytopresentin
theperianalareaorinsuchayoungpatient.
Condylomaacuminatum(ChoiceC)areanogenitalwartsandarecausedbyinfectionwithhumanpapillomavirus.Lesionsmayspontaneouslyresolveorgraduallyincreaseinnumberandenlarge,sotreatmentisusuallyofferedwithimiquimod.Thereisno
evidenceofawart-likelesioninthispatient
'sperianalarea.
Perianalabscess(ChoiceD)presentsasapainful,palpable,fluctuantmassintheperianalregion,occasionallywithsurroundingerythema.Abscessescaninvolvetheana!sphincter,injurytowhichcancausefecalincontinence.Whilethispatienthaspainwith
defecation,herexaminationrevealsafissure.
Prolapsedinternalhemorrhoids(ChoiceE)appearasaredorpurplemassprotrudingfromtheanusduringdefecation.Internalhemorrhoidsaregenerallypainless,unlikeexterna
,hemorrhoids.Ifsevere,hemorrhoidsmayremainprolapsedfollowingabowel
movementrequiringmanualreduction.
EducationalObjective:Analfissuresareacommoncauseofrectalpainandscanthematocheziawithdefecation.Riskfactorsincludepregnancy,chronicconstipation,andlowfiberdiets.Theygenerallyappearintheposteriormidline.Treatmentiswithsitzbaths
andtopicallidocaineornitroglycerin.
o0
*
Previous Next ScoreReport LabValues Calculator Help Pause

ExamSection2:Item15of50 NationalBoardofMedicalExaminers^
ComprehensiveBasicScienceSelf-Assessment
Y
15.Amoderatelyobese55-year-oldmanisbroughttotheemergencydepartmentbecauseofa10-hourhistoryofseverechestpain.Hehasa5-yearhistoryofexercise-inducedangina.Hispulseis109/min,respirationsare15/min,andbloodpressureis
132/92mmHgPhysicalexaminationshowsdiaphoresis.Abloodsampleobtained2hoursafteradmissionshowsincreasedserumactivityofcreatinekinaseMB.Whichofthefollowingisthemostiikelycauseofthislaboratoryfinding?
A)increasedGoigicomplexactivity
B)Increasedpermeabilityoftheplasmamembrane
G)Mitochondrialswelling
D)Nuclearlysis
E)Proliferationoftheendoplasmicreticulum
CorrectAnswer:B.
CreatinekinaseMB(CKMB)isacardiacenzymethatisdetectablewithinsixhoursofmyocardialischemiaandearlycoagulationnecrosis.Increasedplasmamembranepermeabilityisafeatureofearlycoaguiativenecrosisandallows;CKMBandtroponintobe
releasedfromcardiacmyocytes.Otherfeaturesofirreversiblecellulardamageincludepyknosis(chromatincondensation),karyorrhexis(nuclearfragmentation),andcytoplasmicswelling.Ischemicinjurymanifestswithin24hoursaswavinessofmyocardialfibers.
Leakageofintracellularproteinsandtheonsetofnecrosispromptstheinfiltrationofneutrophils.CKMBconcentrationreturntoitsbaselineatapproximately48hoursafteracuteinfarctionandisusefulforthedetectionofre-infarctionafterthistime.Incontrast,
troponinconcentrationremainsincreasedfor7to10daysaftermyocardialinfarction.
IncorrectAnswers:A.C
;D.andE.
WhileincreasedGolgicomplexactivity(ChoiceA}resultsinadditionalsecretionofproteinsatthecellsurfacethroughexocytosis.thisisnotthemechanismofCKMBrelease.CKMBisnotcontainedwithasecretoryvesicle.Rather,itleaksfromthecytoplasm
throughdamagedandpermeablecellularmembraneswithouttheaidofvesicles.
Mitochondria!swelling(ChoiceC)iscausedbyoxidativestressinmyocardialischemiaandmaybereversible,ifsevereenoughtocausemitochondrialrupture,pro-apoptoticmediators,suchascytochromec,arereleasedandleadtoirreversiblecelldeath.CKMB
isnotstoredwithinmitochondria.
Nuclearlysis(ChoiceD)isalsoafeatureofcoaguiativenecrosiswithinthecontextofmyocardialischemia,butdoesnotdirectlyresultinreleaseofCKMB,sinceCKMBisnotstoredinthenucleus.
Proliferationoftheendoplasmicreticulum(ChoiceE)mayoccurinthecellularischemiaduetoalteredfoldingofcellularproteins,whichaccumulateintheendoplasmicreticulum.Thisprocesstriggerstheunfoldedproteinresponse,whichseekstorestorenormal
functionbydegradingunfoldedormisfoldedproteinsandbydownregulatingproteinsynthesis.Ifunsuccessfultheunfoldedproteinresponseinducesapoptosis.
EducationalObjective:CKMBisacardiacenzymethatisdetectablewithinsixhoursofmyocardialischemia.IncreasedplasmamembranepermeabilityisafeatureofearlycoaguiativenecrosisandallowsCKMBandtroponintobereleasedfromcardiacmyocytes.
CKMBconcentrationreturntoitsbaselineatapproximately4Bhoursafteracuteinfarctionandisusefulforthedetectionofre-infarction.
© © ©
Previous Next ScoreReport LabValues Calculator Help Pause

ExamSection2:Item16o!50 NationalBoardofMedicalExaminers^
ComprehensiveBasicScienceSelf-Assessment
Y
16.AGG-year-oldwomancomestothephysicianbecauseofa6-monthhistoryofpaininherhipsandknees.Physicalexaminationshowsfindingsconsistentwithosteoarthritis,andthephysicianrecommendsibuprofen.Thepatientrefusesandasksabout
takingglucosamine.Whichofthefollowingresponsesbythephysicianismostappropriate?
A)"Glucosaminehasn'tbeenstudiedwellenoughformetorecommendit."
B)"Glucosamine'ssideeffectsaren'tlisted.Itmaybemoredangerousthanwerealize."
G)"Ibuprofenhasbeenproveneffectiveforyourcondition."
D)"Whathaveyouheardaboutusingglucosaminetotreatarthritis?"
E)"Whydidyoucometomeifyoudon’twanttotakewhat!recommend?
11
F)"Youshouldreallyseeanaturopathicdoctor."
CorrectAnswer:D.
Whenapatientasksquestionsaboutnaturopathictreatmentssuchasglucosamine,physiciansshouldinitiallyaskopen-endedquestionstoexplorethepatient
'sunderstandingandgoals.Itispossiblethatthispatientdoesnotunderstandtherisksandbenefitsof
ibuprofenversusglucosamine,andthephysiciancantailorfurtherdiscussiontoaddresstheseknowledgegaps.Afterdeterminingthatthepatientunderstandstherisksandbenefits,thephysicianandpatientshouldpartnertodevelopatreatmentplanthataligns
withthepatient'svalues.Thisplanmayultimatelyinvolvereferraltoanaturopathicdoctor.Usingnon-judgmental,open-endedquestioningcanimprovetherapeuticallianceandhelpthephysicianandpatientproblemsolvearoundhowtobestaddressthispatient's
symptoms.
IncorrectAnswers:A.B.C,E.andF.
Immediatelyeducatingthepatientabouttheabsenceofevidencebehindglucosamineorthebenefitsofibuprofen(ChoicesA,BpandC)mayprecludethecollaborativeformationofatreatmentplan.Thisstrategymaypreventthephysicianfromlearningaboutthe
patient'sknowledgegapsandgoals.
Askingthepatientwhytheycametoseethephysicianiftheyarenotgoingtotakewhatthephysicianrecommends(ChoiceE)strikesapaternalisticandjudgmentaltone.Startingtheconversationinthismannermaypreventopen,effectivediscussionaboutthe
options.
Referringthepatienttoanaturopathicdoctor(ChoiceF)maybeultimatelyindicatedifthepatientmaintainsthatsheprefersglucosamine.Howeverthephysicianshouldfirstassessthepatient
'sunderstandingoftherisksandbenefitsofbothtreatmentoptionsso
thatthephysiciancaneducatethepatientandcoElaborativelyformulateatreatmentplanwiththepatient.
EducationalObjective:Whenapatientasksquestionsaboutnaturopathictreatments,physiciansshouldinitiallyaskopen-endedquestionstoexplorethepatient'sunderstandingandgoals.Thephysiciancantailorfurtherdiscussiontoaddressthepatient's
knowledgegapsandgoals.Thephysicianmayultimatelyreferthepatienttoanaturopath.
© © ©
Previous Next ScoreReport LabValues Calculator Help Pause

ExamSection2:Item17of50 NationalBoardofMedicalExaminers^
ComprehensiveBasicScienceSelf-Assessment
y
17.An88-year-oldmanwholivesaloneisbroughttothephysicianbyhisdaughterbecausesheisconcernedthathehasnotbeeneatingawell-balanceddietfor9months.Heis170cm(5ft7in)tallandweighs50kg(110lb);BMIis17kg/m
2Physical
examinationshowsmultipleecchymosesontheupperandlowerextremities.Laboratorystudiesshow:
Plateletcount
Prothrombintime
Serum
VitaminBe
(pyridoxine)
VitaminC(ascorbicacid)
Folicacid
160.000/mms
12sec(I!MR=1)
9no/dL(N=5—30)
0.1mg/dl(N
-0.4-2)
5ng/dL(N=2-20)
Theecchymosesinthispatientaremostlikelyduetoadisorderofwhichofthefollowing?
A)Arachidonicacidproduction
B)Bindingofcarboxyglutamicacidtophospholipid
C)Carboxylationoffactor\\(prothrombin)
D)Prolinehydroxylation
E)Transferofmethylgroupstoorganicacids
CorrectAnswer:D.
VitaminCisanantioxidant,facilitatorofironabsorption,andcoenzymeinthesynthesisofcollagenviaprolylhydroxylaseandneurotransmittersviadopaminehydroxylase.DeficiencyinvitaminCleadstoscurvy,whichpresentswithsignsandsymptomsof
impairedcollagensynthesisincludingswollen,bleedinggums,easybruisingandbleeding(eg.hemarthrosis),petechiae,impairedwoundhealing,andshort,fragile,curlyhair.Itshouldnotbeconfusedwithhemophilia,asitdoesnotresultinadeficiencyof
factors,instead,thecollagenandconnectivetissuedeficiencyweakensbloodvesselwallsresultingineasybruisingandbleeding.Collagenissynthesizedbyfibroblastsandbeginsintheroughendoplasmicreticulumwithtranslationofcollagenchains,whichare
glycineandprolinerich.Prolylhydroxylase,inareactionrequiringvitaminC,hydroxylatesprolineandlysineresidues.Thisstep,alongwithglycosylation,formsalpha-chainsthroughdisulfidebridgingplushydrogenbonding.Procollagenisexocytosed,whereit
formstropocollagenafterremovaloftheterminalends.Itiscross-linkedextraceflularlyinareactionthatrequirescopper.Inthiscase,thepatient'sbruisinganddietlimitedinvitaminCplusnormalprothrombintimesuggestsadiagnosisofscurvyduetoimpaired
collagensynthesis.
IncorrectAnswers:A.B,C.andE.
Arachidonicacidisproducedininflammatorystatesfrommembranephospholipidsandisconvertedtothromboxane-A
2
bycyclooxygenase.Thramboxane-A
2isinvolvedinplateletactivation.Decreasedarachidonicacidproduction(ChoiceA)leadingto
decreasedsynthesisofthromboxane-A
2
wouldresultindiminishedplateletactivation.Anti-inflammatoryagentsthatdecreasearachidonicacidproductionincludecorticosteroidssuchasdexamethasone.hydrocortisone,andprednisone.
Carboxyglutamicacidisamodifiedaminoacidfrequentlyfoundincoagulationfactors.VitaminKisrequiredforitssynthesis.CarboxylationofglutamicacidresiduesoccursonprocoagulantfactorsII,VII,IX,andX.AdeficiencyinvitaminKleadingtodecreased
carboxylationresultsinimpairedbindingofcarboxyglutamicacidtophospholipid(ChoiceB)anddecreasedcarboxylationoffactor.I(prothrombin)(ChoiceC).Theneteffectiscoagulopathy,manifestaseasybruisingwithincreasedprothrombintimeand
activatedpartialthromboplastintime.
VitaminBgisconvertedtotetrahydrofolicacidwhichservesasacoenzymeformethylationreactions.Folatedeficiencyresultinginimpairedtransferofmethylgroupstoorganicacids(ChoiceE)leadstoimpairedDNAandRNAsynthesisandamacrocytic,
megaloblasticanemia.
EducationalObjective:VitaminCisanantioxidantandcoenzymeinthesynthesisofcollagenviaprolylhydroxylase.Deficiencyiscommoninpersonshavingdietspoorinfruitsandvegetables,andresultsinscurvy,whichpresentswithbloodvesselfragility{easy
bruising,petechiae)anddisorderedhairgrowth.
o0 c- 00
ES
Previous Next SconeReport LabValues Calculator Help Pause

ExamSection2:Item13o!50 NationalBoardofMedicalExaminers^
ComprehensiveBasicScienceSelf-Assessment
Y
18.A14-year-oldboyhaspersistentleukocytosisandneutrophiliawithoutevidenceofacurrentinfection.Hehasahistoryofrecurrentinfectionsoftheskin,upperandlowerairways;andperirectalarea.Gram-negativeandgram-positiverodshavebeen
isolated.ThenumberandfunctionofBandTlymphocytesarenormal.Productionofhypochlorousacidbyneutrophilsandthenitrobluetetrazgliumreductiontestarenormal.Neutrophilchemotacticresponsetotheformyl-MetLeuPhe(fMLP)peptideis
diminished.Whichofthefollowingdisordersofneutrophilsisthemostlikelydiagnosis?
A)Chronicgranulomatousdisease
B)Cyclicneutropenia
C)Leukocyteadhesiondeficiency
D)Myeloperoxidasedeficiency
E)Neutrophil-specificgranuledeficiency
CorrectAnswer:C.
Leukocyteadhesiondeficiency(LAD)resultsfromadefectintheattachmentofleukocytestothevascularendothelium,whichconsequentlyresultsintheimpairedrecruitmentandmigrationtositesofextravascularinflammationorinfection,itistypically
characterizedbyrecurrentbacterialinfections,impairedwoundhealing,adelayeddetachmentoftheumbilicalcordafterbirth,andaackofleukocytesatsitesofinfectionswithanabsenceofpus.Theactionsofleukocytephagocytosisandbacterialkillingare
otherwiseunimpaired.LaboratorystudiesiinpatientswithLADwillrevealincreasedleukocytelevelsintheblood.Leukocytemigrationstudiesmayrevealdecreasedresponsivenesstochemotacticagents.
IncorrectAnswers:A.B
(D.andE.
Chronicgranulomatousdisease(ChoiceA)resultsfromadefectinthenicotinamideadeninedinucleotidephosphate{NADPH)oxidasecomplex.DiagnosiscanbemadebyanabnormalnitrobluetetrazoliumreductiontestasfunctionalNADPHoxidaseisneededto
reducenitroblue.
Cyclicneutropenia(ChoiceB)isarareinheritedimmunodeficiencysyndromeassociatedwithdysfunctionalneutrophilelastase.Itischaracterizedbyrecurrentepisodesofneutropeniaandinfection.
Myeloperoxidasedeficiency(ChoiceD)isaninheritedimmunodeficiencysyndromecharacterizedbytheinabilitytoproducehypochlorousacidwithinphagolysosomes.DiseaseistypicallymildandmaypresentwithrecurrentCandidaalbicansinfection.
Neutrophil-specificgranuedeficiency(ChoiceE)resultsfromthedefectiveproductionofgranuleswithinneutrophils.Thedisorderischaracterizedbyrecurrentpyogenicinfectionsthatoccurearlyinchildhoodaswellasimpairedproductionofdefensins.
EducationalObjective:LADisagroupofdisorderscharacterizedbyimpairedieukocyteadhesiontothevascularendothelium.Leukocytesretaintheabilitytophagocytoseandeliminateforeignpathogensbutareunabletomigratetositesofinfectionor
inflammationintheextravascularspace.
© © ©
Previous Next ScoreReport LabValues Calculator Help Pause

ExamSection2:Item19of50 NationalBoardofMedicalExaminers^
ComprehensiveBasicScienceSelf-Assessment
Y
!
1
1i
II 22
23
31
11 1 4 I
O
4
111
22
3
'
3
5 2
G 3
1 2 3
1 1
IV
22
3
'
6 3
2 31 4
Affectedmale
#Affectedfemale
Unaffectedmale
OUnaffectedfemale
0Affectedmale,deceased
0
Unaffectedfemale,deceased
19.Aninvestigatorisstudyingalargefamilywithmanymemberswhoareaffectedbyadisordercausedbyafullypenetrantautosomaldominantinheritedgenemutation.ApedigreeisshownMostaffectedmembersalsohaveararealleleatalocusthoughtto
becloselylinkedtothediseaselocus.Afather(individuallra-3)andhisdaughter(individualIV-3)havethedisorder,buttheyhavethewild-typealleleatthefinkedfocus.Whichofthefollowingisthemostlikelycauseofthesefindings?
A)InsertionofaUNEsequence
B)Randomsegregation
C)Recombination
D}Singlenucleotidepolymorphism
E)Transduction
CorrectAnswer:C.
RecombinationtakesplaceinprophaseIofmeiosisandistheprocessbywhichtwochromatidsexchangeasmallportionofgeneticmaterialandseparate,orunlink
:twotraitspreviouslypresentonthesamechromatid.Inthisexample,alltheindividualshave
onechromosomewithalleles,2.and3atthreeadjacentloci.Mostoftheaffectedindividualshavethealleles4,5
:and6ontheirsecondchromosomeatthesesameloci,indicatingthatitisthischromosomewhichcarriesthegeneticdefect.Twoaffected
individualsstillhavealleles5and6buthavereplaced4with1atthefirstlocusThissuggeststhatrecombinationoccurredbetweenthefirsttwogenelocicausingallele4,whichhadpreviouslybeenJinkedtothelocuswiththemutation,tobecomeunlinked.The
chancethatarecombinationeventwilloccurbetweentwogiverlociiscalledtherecombinationfraction.
IncorrectAnswers:A.B.D.andE.
Insertionofalonginterspersednuclearelement(LINE)sequence(ChoiceA)isamethodofbacterialgeneticvariationthroughtransposableelements.ThisisseeninbacteriawhenasegmentofDNAmovesfromachromosometoaplasmid.Whenitoccurs,
regionsofDNAoneithersidemayalsobetransposed,conferringnewpropertiestotherecipient.
Randomsegregation(ChoiceB)isaprincipleapplyingtomeiosis,theseparationofDNAintohaploidgametes,whichstatesthatparentalgenesseparaterandomlyandequallyintogametes.Eachgametehasanequalchanceofgettingeachparentalallele.It
doesnotimpactthelinkageofallelesattwocloselylocatedgeneloci.
Singlenucleotidepolymorphism(ChoiceD)isthemostcommontypeofgeneticvariationamongindividualsandischaracterizedbyasinglenucleotidechangingfromonebasetoanother,alteringthatsiteontheDNA.Asinglenucleotidepolymorphismmight
causeanalleleatagivengenelocustobealteredbutdoesnotseparatelinkedallelesfromeachotherasoccursduringrecombination.
Transduction(ChoiceE)isamethodbywhichbacterialgenesaretakenfromonebacteriumandgiventoanother.Itoccurswhenalyticphageinfectsabacterium,cleavesbacterialDNA,andthenpackagessomeofthosecleavedpiecesintoanewphage
capsid.Thatphagetheninfectsanotherbacterium,ieadingtotransferofthebacterialgenesThisisnotamechanismofgeneticvariationineukaryoticcells.
EducationalObjective:Recombinationtakesplaceinprophase[ofmeiosisandistheprocessbywhichtwochromatidsexchangeasmallportionofgeneticmaterialandseparate,orunlink,twotraitspreviouslypresentonthesamechromatid.
m m
Previous Next ScoreReport LabValues Calculator Help Pause

ExamSection2:Item20ol50 NationalBoardofMedicalExaminers^
ComprehensiveBasicScienceSelf-Assessment
Y
20.A45-year-oldwomancomestothephysicianbecauseofa6-monthhistoryofshortnessofbreathwithexertionandanonproductivecough.Shesometimeshasdifficultyswallowingandoftenhasheartburn,especiallyifshefiesdownafterameaLSheadds
thatherfingershaveswollen,andshehashadtogetherweddingringresized.Herfingersalsobecomewhiteandpainfulincoldweatherorcofdwater.HerpuEseis75/min
:respirationsare20/min,andbloodpressureis150/100mmHg.Physical
examinationshowstight,smoothfacialskinwithoutwrinkles.Additionaltestingismostlikelytoshowwhichofthefollowingsetsofcardiovascularchangesinthispatient?
LeftVentricular
DiastolicCompliance
MeanPulmonary
ArteryPressure
CoronaryVascular
Resistance
A)
A A A
B)
A A
l
C)
A A
v
D) 4 4
E)
t t
v
F)
A.
4 4
G)
A.
V
H)
4 4 4
CorrectAnswer:E
Systemicsclerosis(scleroderma)isanautoimmunedisordercharacterizedbycollagendepositionandprogressivefibrosisoftheskin,softtissues,andinternalorgansaswellasnoninflammatoryvasculopathy.Therearemultiplephenotypesrangingfromlocalized
sclerodermatodiffusedisease.Multipleorgansystemsmaybeaffected,withinvolvementoftherenal,pulmonary,gastrointestinal,andcardiovascularsystemsbeingcommon.Cardiacfibrosisresultsindecreasedleftventriculardiastoliccomplianceandincreased
coronaryvascularresistance.Interstitialpulmonaryfibrosisclassicallypresentswithdyspneaonexertionandanonproductivecough.Patientsoftenpresentwithanincreasedmeanpulmonaryarterypressuresecondarytochronichypoxicvasoconstrictionandleft
ventriculardysfunction.Gastrointestinalmanifestationsincludeesophagealdysmotilitywithsymptomsofdysphagiaandacidreftux.VascularmanifestationsincludeRaynaudphenomenonduetocutaneousvasospasmTheskinmaybetautwithoutwrinkleson
physicalexamination.Femalesaretypicallyaffectedmorethanmales.
IncorrectAnswers:A.B,C.DF,G,andH.
ChoicesA
:6,C,andDareincorrectasleftventriculardiastoliccompliancedecreasesinsystemicsclerosiswithcardiacinvolvement.Compliancereferstothechangeinvolumeofasystemrelativetoachangeinpressure.Replacementofnorma!cardiomyocyte
fiberswithcollagendepositionandfibrosisresultsinastiffandnoncompliantventricle.
ChoicesC,D
;G,andHareincorrectasboththecardiacandpulmonaryinvolvementinsystemicsclerosisresultsinpulmonaryhypertensionwithanincreasedmeanpulmonaryarterypressure.Pulmonaryhypertensionmayresultfrompulmonaryfibrosisand
hypoxia-inducedpulmonaryvasoconstriction,leftheartdisease,and/orconcentrichypertrophyofthetunicaintimaofthepulmonaryvasculature.
Similarly,choicesEl,D,F.andHare =ncorrectascoronaryvascuarresistancetendstoincrease,notdecrease,insystemicsclerosiswithcardiacinvolvementduetoconcentrichypertrophyofthecoronaryarterywall
EducationalObjective:Systemicscierosis(scleroderma)isanautoimmunedisorderthatmayaffectmultipleorgansystems,ftischaracterizedbycollagendepositionandprogressivefibrosisoftissueaswellasnoninflammatoryvasculopathy,whichresultsinstiff,
noncompliantorgansandincreasedvascularresistance.
oi? & £ A
Previous Next ScoreReport LabValues Calculator Help Pause

ExamSection2:Item21ol50 NationalBoardofMedicalExaminers^
ComprehensiveBasicScienceSelf-Assessment
Y
21.A25-year-oldwomancomestothephysicianforaroutinehealthmaintenanceexamination.Sheiscurrentlypreparingforan8-km(5-mile)race,andshesprinttrainstwiceweekly.Curingthistraining
:sheruns200-metersprintsintwogroupsof10sprints
separatedbya30-secondrestbetweeneachsprintinagroup
:anda2-minuterestbetweeneachgroupof10.Aftertheworkoutherlegsfeelweak
:andhermusclesburnandsometimescramp.Whichofthefollowingbestexplainshersymptoms?
A)Decreasedactivityofthesodium-protonantiporter,resultinginanacidicsarcoplasm
B)Increasedactivityofthesodium-protonantiporter,resultinginanacidicsarcoplasm
C)increasedoxygendeliverytothemuscle,leadingtoincreasedmetabolismandacidproduction
D)RegenerationofNAD^fromNADH
:whichproducesacid
CorrectAnswer:D.
Duringvigorousexerciseactivity,adenosinetriphosphate(ATP)storesinskeletalmusclearedepletedduringthecyclicmechanicalexcitation-contractioncouplingofmyosinandactin.ThereareanumberofmechanismsbywhichskeletalmuscleregeneratesATP.
Whenoxygenconcentrationsarehigh,aerobicrespirationallowsfartheconversionofglucosetopyruvate,whichentersthecitricacidcycleproducingATPandNADH.NADHshuttlesprotonsandelectronstotheelectrontransportchain,whereoxidative
phosphorylationproducesATP.Whenoxygenconcentrationsarelow,cellsundergoanaerobicrespiration,whichregeneratesNAD*fromNADHthroughtheproductionoflacticacidfrompyruvate.Thisisaquicker,althoughlessefficientprocessthanaerobic
respiration.Aswell,therunnerisutilizingprimarilytype2musclefibersduringhersprints(fasttwitch),whichexhibitalowerconcentrationofmitochondria,makingthemdependentonanaerobicglycolysis.Thesubsequentbuildupoflacticacidresuitsinincreased
acidityofthetissueenvironment,whichleadstotissuevasodilationandincreasedoxygendeliverytothemuscle.Clinically,thismanifestsasmusclepainandaburningsensation.
IncorrectAnswers:A.B,andC.
Activityofthesodium-protonantiporter(ChoiceA)doesnotdecreaseduringintenseexercise.Thesodium-protonantiporteractivityincreasesduringrigorousexerciseinordertoremoveH*fromthesarcoplasmandmaintainhomeostasis.
Increasedactivityofthesodium-protonantiporter(ChoiceB)doesoccurinexercisestates,howeverthisfunctionservestoreducetheacidityofthesarcoplasm,notincreasesarcoplasmicacidity.
Increasedoxygendeliverytothemusclewouldfacilitateincreasedaerobicrespiration,whichwoulddecreaselacticacidproduction(ChoiceC)asthemusclecellswouldoptimizeenergyefficiencybyusingaerobiccellularrespiration.
EducationalObjective:Duringvigorousexerciseandtimesofincreasedmetabolicdemandofmyocytes,theremaybeinsufficientoxygenforaerobicrespiration.Myocytesconsequentlydependonanaerobicrespiration,whichresultsintheregenerationofNAD
fromNADHthroughtheproductionoflacticacid.Thisincreasedsarcoplasmicacidityresultsinmusclepainandburning.
Previous Next ScoreReport LabValues Calculator Help Pause

ExamSection2:Item22ol50 NationalBoardofMedicalExaminers^
ComprehensiveBasicScienceSelf-Assessment
Y
22.A14-year-oldgirlisbroughttothephysicianbecauseofa1-monthhistoryofmigraine-likeheadaches,vomiting,andmultipleleft-sidedfocalseizures.Shehashadhearinglosssincetheageof11years.Hermotherandmaternalgrandmotherhavehigh-
tonedeafness.Physicalexaminationshowslossofvisioninonehalfofthevisualfieldoftherighteyeandweaknessoftherightupperandlowerextremities.Serumandcerebrospinalfluidconcentrationsoflacticacidareincreased.Thispatientmostlikely
hasamutationofwhichofthefollowing?
A)Endoplasmicreticulumglycosyltransferase
B)Lysosomala-glucosidase
C)MitochondrialtRNALei
1
D)Nuclearproteasomeactivator
E)Peroxisomalcatalase
CorrectAnswer:C.
Mitochondria!encephalopathy,acticacidosis,andstroke-likeepisodes(MELAS)syndromeisafamilyofmitochondriallyinheriteddiseasesthatarecausedbymutationsinmultiplemitochondrialgenes,oftenthoseencodingmitochondria!tRNA.Patientspresentin
childhoodwithmuscularweaknessandmyalgia,stroke-likeepisodescharacterizedbyhemiparesisandvisionloss,seizures,headache,andlacticacidosis.Afamilyhistorysuggestiveofmitochondrialinheritanceissuggestiveofthediagnosis.Likeother
mitochondrialmyopathies,musclebiopsymaydemonstrateraggedredfibers,althoughgenetictestingconfirmsthediagnosis.Treatmentissupportivebutmayinvolvesupplementationofvitaminsandcarnitine.OthermitochondrialdiseasesincludeLeberhereditary
opticneuropathy,maternallyinheriteddiabetesanddeafness,andmyoclonicepilepsywithraggedredfibers.
IncorrectAnswers:A.B,D,andE.
Endoplasmicreticulumglycosyltransferase(ChoiceA)isimportantforconjugationofsugarmoietieswithproteinspriortotransporttothecellularsurface.ThisprocessplaysaroteinthedeterminationoftheA,B,and0bloodgroupsbutisnotacauseofMELAS
syndrome.
Lysosomala-glucosidase(ChoiceB)istheenzymewhosedeficiencyunderliesglycogenstoragedisease,typeII(Pompedisease).Glycogenstoragedisease,typefl(Pompedisease)leadstocardiomegaly.cardiomyopathyhepatomegaly,andhypotonia.
Nuclearproteasomeactivation(ChoiceD)occursinthesettingoftheaccumulationofmisfoldedproteinrequiringdegradation.Proteasomes.whenfailingtoactivate,areimplicatedinmanyneurodegenerativediseasessuchasAlzheimerandHuntingtondisease.
Peroxisomalcatalase(ChoiceE)deficiencyischaracteristicofacatalasia,aperoxisomaldisordercharacterizedbyanaccumulationofhydrogenperoxidewithinceils.
EducationalObjective:MELASsyndromeiscausedbymitochondrialmutationsinmanygenes,includingthosethatencodemitochondriaEtRNALeu.Mitochondrialinheritancepatternssuggestthediagnosis,whichcanbeconfirmedwithgeneticsequencing.Patients
presentinchildhoodwithmuscularweaknessandmyalgia,stroke-likeepisodescharacterizedbyhemiparesisandvisionloss,seizures,headache,andlacticacidosis.
oo
Previous Next ScoreReport LabValues Calculator Help Pause

ExamSection2:Item23of50 NationalBoardofMedicalExaminers^
ComprehensiveBasicScienceSelf-Assessment
Y
23.Acase-controlstudyisconductedtodetermineifobesityisariskfactorforgastroesophagealrefluxdisease(GERD).AquestionnaireisusedtoselectsubjectswithseveresymptomsofGERDandsubjectswithnosymptoms.ABMfiscalculatedforeach
subjectTheresults(inkg/m
2)areshown:
BMK25 25<BMI<30 30<BMI<35 BM035
SubjectswithGERDsymptoms
Subjectswithnosymptoms
300 900 200 5D
700 900 200 30
WhichofthefollowingrepresentstheoddsratioforGERDsymptomsinsubjectswithBMIsgreaterthan35comparedwithsubjectswithBMIslessthan25?
A)(50x30)7(300^
700)
B)(50
*
700)/(30x300)
C)[50/(30+50)]/[300/(300-1-700)]
D}[50/(50+300)]i[30/(30+700)]
E)[50/(50+200+900+300)3/[30/(30+200+900+700)]
CorrectAnswer:B.
Oddsratioisacomparisonoftheoddsofanoutcomeoccurringintheexposedgroupwiththeoddsofthatoutcomeoccurringinanonexposedcomparisongroup.Itiscalculatedastheoddsoftheoutcomeofinterestintheexposedgroupdividedbytheoddsofthe
outcomeofinterestinthenonexposedgroup.Inthiscase-controlstudy,theoddsratiowouldbecalculatedasfollows.Theoddsofgastroesophagealrefluxdisease(GERD)inpatientswithaBMI>35wouldbe50(numberofsubjectswithGERDsymptoms)divided
by30(numberofsubjectswithnosymptoms)(50/30=1.567)indicatingthatapersoninthiscategoryismorelikelythannottohavesymptomsofGERD.TheoddsofhavingGERDinapatientwithaBMI<25wouldbe300dividedby700(300/700=0.429).
Calculatingtheratiobetweenthesetwoodds(oddsratio)wouldbe:(50/30)/(300/700)=3.830.Thisequationcanberearrangedto(50x700)/(30x300).
IncorrectAnswers:A.C,D,andE.
(50x30)/(300x700)(ChoiceA)isanerroneousexpressionofthetrueoddsratio((50/30)/(300/700)),whichincorrectlysubstitutesmultiplicationfordivision.
50/(30+50)]>[300/(300+700)1(ChoiceC)isacalculationofarelativeriskratio.Thisisaninappropriatecalculationasacase-controlstudyhasadefinednumberofoutcomesofinterest(eg,GERD)astheseindividualcaseswereselectedduringthestudydesign.
BecausethenumberofGERDcaseshasbeenpredetermined,calculatingtheriskofdiseaseisinappropriate.
[50/(50+300)]/[30/(30+700)](ChoiceD)isalsoanerroneouscalculation.Jtdoesnotcalculaterelativeriskoroddsratio.
[50/(50+200+900+300)3[30/(30+200+900+700)](ChoiceE)calculatestnenumberofpatientswithaBMI>35intheGERDgroupdividedbyallpatientswithGERDsymptoms(riskofBMI>35givenGERDsymptoms).Alsocalculatedisthenumberofpatientswith
aBMI>35withnosymptomsdividedbythetotanumberofpatientswithoutsymptoms(riskofBMI>35givennoGERDsymptoms).Thesetwonumbersarethendividedtoformaratio(0.0344/0.0164=2.098),meaningthatpatientswithGERDaretwiceaslikely
tohaveaBMI>35.Althoughthiscalculationistechnicallyreasonable,itisnotthefocusofthestudyandisanexpandedratio.
EducationalObjective:Oddsratioiscalculatedcommonlyincase-controlstudiestoevaluatethelikelihoodofexposurefoacertainriskfactorgivenadiseaseornon-diseasestate.Relativeriskratiosarenotcalculatedincase-controlstudiesasthediseaseor
outcomerate(cases)hasbeenpredeterminedbytheinvestigator.
oi? & &
Previous Next ScoreReport LabValues Calculator Help Pause

ExamSection2:Item24ol50 NationalBoardofMedicalExaminers^
ComprehensiveBasicScienceSelf-Assessment
y
24.A14-year-oldboyisbroughttothephysicianbyhismotherbecauseofdailyheadachesfor2months.Theheadachesaredescribedasabilateralachinginthetemples.Mismotherstatesthathealso’'hasnotbeenhimself'forthepastfewmonths.He
seemsmoreconfused,oftenforgettingnamesrdates
:andplaces,andheisclumsywithfrequentfails.Hisschoolperformancealsohasdeclinedoverthepastquarter.Physicalexaminationshowsabroad-basedataxicgait.Heisaiertandorientedto
person
:place
:andtime,butheisslowtoanswerquestions.Chronicabuseofwhichofthefollowingsubstancesisthemostlikelycauseofthispatientscondition?
A)Cocaine
B)Ethanoi
C)Inhaledglue
D)Methamphetamines
E)PCP(phencyclidine)
CorrectAnswer:C.
Chronicinhalantabuse(eg
:glue,spraypaintshoepolish,toluene,nitrousoxide)iscommoninchildrenandadolescentsandhaseffectsonmultipleorgansystemsneuropsychiatric(headache
;cognitiveimpairment,anosmia.cerebeElardysfunction,moodswings,
irritability,hallucinations),dermatologic(perioralorperinasaldermatitis),otolaryngologic(nosebleeds,halitosis),ocular(conjunctivalinjection),cardiac(dysrhythmia,tachycardia),gastrointestinal(nausea,anorexia),andrespiratory(wheezing,coughing,sneezing).
Thispatient’srecentconfusion,broad-basedataxiaconsistentwithcerebellardysfunction,andpoorschoolfunctioningareEikelyrelatedtotheneuropsychiatriceffectsofinhalants.Treatmentisprimarilysupportive,thoughsomeinhalantshavespecificantidotes(eg.
methylenebluefornitrites).Preventioniskey:schoolsshouldmonitortheuseofsolvent-basedproductsandparentsandchildrenshouldbeeducatedabouttherisksofinhalants.
IncorrectAnswers:A.B,D,andE.
Chroniccocaineabuse(ChoiceA)mayleadtootolaryngologicsymptomssuchasnasalseptalperforationandnosebleedsaswellasneuropsychiatricsymptomssuchasanosmia,cognitiveimpairment,andpsychoticsymptoms(delusions,hallucinations).
Cerebellardysfunctionandheadacheswouldbeatypicalofchroniccocaineuse.
Chronicethanolabuse(ChoiceB)canleadtoventricularandsulcalenlargementandconsequentcognitiveimpairment.Wernicke-Korsakoffsyndromemaycauseseverecognitiveimpairmentaswellasophthalmoplegiaandataxia,anddecadesofethanolusecan
leadtocerebellardegeneration.Howevercerebellardysfunctionisunlikelytobeseenafteronlytwomonthsofethanolabuse,andheadachesaremoretypicalofinhalantabuse.
Chronicmethamphetamineabuse(ChoiceD)canleadtomethamphetamine-inducedpsychoticdisorder,whichfeatureschronicdelusions,paranoia,andhallucinations.Chronicmethamphetamineusemayalsoleadtocognitiveimpairment.However,cerebellar
dysfunctionandheadacheswouldbeatypical.
ChronicPCP(phencyclidine)abuse(ChoiceE)canleadtodepression,psychoticsymptoms,memoryloss,anddysarthria.Headachesandcerebellardysfunctionwouldbeatypical
EducationalObjective:Chronicinhalantabusecanleadtoseveralneuropsychiatricmanifestationssuchasheadache,cognitiveimpairment,anosmia,cerebellardysfunction,moodswings,irritability,andhallucinations.Cerebellardysfunctionandheadaches
distinguishchronicinhalantabusefromthechronicabuseofmanyothersubstances,althoughethanolalsoleadstocerebellardysfunctionaftermanyyearsofabuse.
© © © ©
f*
Previous Next ScoreReport LabValues Calculator Help Pause

ExamSection2:Item25ol50 NationalBoardofMedicalExaminers^
ComprehensiveBasicScienceSelf-Assessment
25.Astudyisdesignedtomeasuretheimpactofexerciseontheincidenceofmyocardialinfarction.Subjectsareenrolledinthestudyanddividedintotwogroupsbasedontheirself-reportedexercisehabits.Attheendofthestudy
;subjectswhoreported
exercisinghavehalftheincidenceofmyocardialinfarctioncomparedwiththesubjectswhodidnotexercise.Whichofthefoliowingbestdescribesthisstudydesign?
A)Case-control
B)Caseseries
C)Cohort
D)Cross-sectional
E)Randomizedclinicaltrial
CorrectAnswer:C.
Acohortstudyidentifiesagroupofpatientsandfollowsthemovertimetoidentifywhetheranexposureisassociatedwithanoutcomeofinterest.Cohortstudiesmayberetrospectiveorprospectiveindesign.Inaprospectivedesign,thehypothesisandanalysis
protocolsareestablishedpriortothestartofthestudyperiod,fnaretrospectivedesignthehypothesisorquestionisdesignedafterthestudytimeperiodhaspassed.Inthisstudy,patientsareobservedovertimeanddividedintogroupsbasedonexercise
(exposure)andarethenfollowedovertime.Ratesofmyocardialinfarction(outcome)arethencomparedbetweenthetwogroups.Therelativeriskofmyocardialinfarction(acomparisonofincidence)isthencalculatedtobetwiceashighinthenon-exercisegroup.
Thisrepresentsaprospectivecohortstudydesign.
IncorrectAnswers:A.B,D.andE.
Acase-controtstudy(ChoiceA)investigatesanassociationbetweenanexposureandanoutcome.Inthisstudydesigmagroupofpatientswiththedisease(cases)areidentified.Agroupofpatientswithoutthedisease(controls)arematchedonbaseline
characteristicstothecases.Exposuredataforthetwogroupsiscoflected
:andthesedataarecomparedtodetermineassociationwiththeoutcome(disease)inquestion.Anoddsratiomaybecalculatedtocompareexposuresbetweengroups.
Acaseseries(ChoiceB)isadescriptivestudydesigninwhichanumberofconsecutiveornonconsecutivecasesofadiseaseand/ortreatmentaredescribedindetail,withinformationaboutexposure
:demographics,andcomorbidities.Caseseriesdonotimplya
cause-and-effectrelationship.Theydonottestahypothesisnoraretheyrandomized.Theyareusefulincharacterizingthenaturalhistoryofadiseaseorresponsetotreatment.Theyarealsousefulindescribingrarediseases,asthesmallpopulationsizemaynot
permitconductingoflargercohortstudiesorrandomizedtrialswithsufficientpower.
Across-sectionalstudy(ChoiceD)seekstoidentifytheprevalenceofaconditionataspecificpointintime.Inaddition,theriskfactorandtheoutcomesaremeasuredsimultaneously.Across-sectionalstudydoesnotfollowpatientsovertime.Allinformationis
collectedatasingletimepoint.
Arandomizedclinicaltrial(ChoiceE)isanexperimentalstudydesign.Patientsarerandomlyallocatedtotwoormoreinterventionalarmsorcontrolarms
:andthesepatientsarefollowedovertimetoevaluateanoutcomeofinterest.Randomizeddesignminimizes
opportunityforbias;thus,arandomizedinterventionalstudycanbeusedtoimplycausation.Commonexamplesofrandomizedtrialsincludetherapeuticcomparisonsbetweenanewdrugandthepreviousstandardofcare.
EducationalObjective:Acohortstudyfollowspatientsoveraperiodoftime,eitherretrospectivelyorprospectively,andseekstoinvestigatetheimpactofexposuresorriskfactorsonanoutcomeofinterest.
Previous Next ScoreReport LabValues Calculator Help Pause

ExamSection2:Item26ol50 NationalBoardofMedicalExaminers^
ComprehensiveBasicScienceSelf-Assessment
Y
26.An18-year-oldwomanisbeingevaluatedforamenorrhea.Shehasneverhadamenstrualperiod.Sheis183cm(6ft)tall.Breastdevelopmentandexternalgenitaliaarenormal.Thereisnoaxillaryorpubichair.Whichofthefollowingkaryotypesismost
likely?
A)45.X
B)46,XX
C)46,XY
D)46,X,i(Xq)
E)47.XXX
CorrectAnswer:C.
Androgeninsensitivitysyndromeisduetoadefectintheandrogenreceptorcomplexresultinginagenetically46
:XYmaledevelopingafemalephenotype.Testesarepresentandproducetestosteronenormally,butabsenceofafunctioningandrogenreceptor
preventshormonebindingandtherebypreventsdevelopmentofmalesexualcharacteristics.Patientspresentwithfemaleexternalgenitalia,scantpubicandaxillaryhair,absentuterusandfallopiantubes,andarudimentaryvagina.Patientsdemonstrateincreased
eveJsoftestosterone,estrogen,andluteinizinghormone.Menseswillnotoccurduetothelackofcycledprogesteroneandestrogen,andthelackofafunctionaluteruswithendometriallining.
IncorrectAnswers:A,B
:D,andE.
45,X(ChoiceA),alsoknownasTurnersyndrome,presentswithcharacteristicphysicalfeaturesincludingshortstature,awide,webbedneck,andabroadchestwithwidelyspacednipples.PatientswithTurnersyndromemayalsopresentwithbicuspidaorticvalve,
aorticcoarctation,orafusedkidney.TurnersyndromecommonlyresultsfrommonosomyoftheXchromosome,butmayalsoresultfrompartialdeletionofthechromosome,asinpatientswithisochromosomeXq,46,X,i(Xq)(ChoiceD).
46,XX(ChoiceB)representsanormalfemalephenotype.Thispatient
'sheight,amenorrhea,andabsenceofaxillaryandpubichairaremoresuggestiveofandrogeninsensitivity.
47,XXX(ChoiceE),alsoknownastrisomyXortripleXsyndrome,presentswithtallstaturebutalsowithepicanthalfoldsandintellectualdisability.Absenceofaxillaryandpubichairisnotafeatureofthissyndrome.
bducationaObjective:Androgeninsensitivitysyndromeoccursduetoadefectintheandrogenreceptorandresultsinpersonswith46.XYchromosomesdevelopingphenotypicallyfemalecharacteristics.
00#
Previous Next ScoreReport LabValues Calculator Help Pause

ExamSection2:Item27of50 NationalBoardofMedicalExaminers^
ComprehensiveBasicScienceSelf-Assessment
y
27.A50-year-oldmancomestothephysicianbecauseofa2-monthhistoryofpainofhiswrists
:changesinskincolor,andprogressivefatigue.Hisbrotherhastype2diabetesmellitusandcirrhosis.Physicaiexaminationshowsbronze-coloredskin,tenderness
ofthemetacarpophalangealjointsinbothhands,andhepatosplenomegaly.Serumstudiesshow:
AST
ALT
Ferritin
100U/L
110U/L
1200ng/mL
Totaliron-bindingcapacity200pg/dl(N=250-4G0)
Transferrinsaturation 80%(N=20—50)
Analysisofaliverbiopsyspecimenshowsamarkedlyincreasedironconcentrationandcirrhosis.Whichofthefollowingisthemostlikelycauseofthefindingsinthispatient?
A}increasederythropoietinaction
B)Increasedintestinalironabsorption
C)Increasedoralironintake
D)Decreasederythropoiesis
E)Decreasedironexcretion
F)Decreasedserumtransferrinconcentration
CorrectAnswer:B.
HemochromatosismaybeacquiredorinheritedsecondarytomutationsintheHFEgene,leadingtoabnormallyincreasedintestinalironabsorption.Thisresultsinaccumulationofironinthebody,increasedserumiron,andincreasedferritin.Ironcanaccumulate
inseveralorgans,includingtheliver,pancreas,skin,heart,andjoints.Duetoincreasedfreeradicalgenerationandoxidativedamage,hemochromatosiscanmanifestwithfailureoftheaffectedorgans.Ittypicallypresentsafterdecadesofironaccumulationwith
liverfailuremanifestascirrhosisandportalhypertension,diabetesmellitus,arthritissecondarytocalciumpyrophosphatedeposition,cardiomyopathywithresultantsymptomsofheartfailure,darkeningoftheskin,and/orgonadaiatrophy.Hemochromatosis,when
acquired,oftenoccursinthesettingoftransfusion-dependentanemiassuchasthalassemia.Diagnosticstudiesmayincludeliverbiopsy,whichcommonlydemonstratesexcessirondepositionseeninhepatocytesonPrussianbluestain.Treatmentinvolvesserial
phlebotomy.
IncorrectAnswers:A.C,D,E,andF.
Increasederythropoietinaction{ChoiceA)resultsinerythrocytosisandcanbeassociatedwithcertaincancersincludingrenalceilcarcinomaasaparaneoplasticsyndrome.Erythropoietinisproducedintheinterstitialceilsofthekidneyandstimulateserythrocyte
productioninthebonemarrow.Itistriggeredbyhypoxiatoincreaseproductionofredbloodcells.Polycythemiacanleadtoitchinganderythromelalgiabutdoesnotinvolveincreasedironlevelsandirondepositionintissues.
increasedoralironintake(ChoiceC)anddecreasedironexcretion{ChoiceE)arenotthepathologicmechanismsinhereditaryhemochromatosis.Normalironsensingregulatestheamountofintestinalabsorptionofiron.DefectsinironsensingintheHFEgene
leadstohemochromatosis.
Decreasederythropoiesis(ChoiceD)isseeninchronickidneydiseaseandend-stagerenaldisease,whichresultsinanemia.Insevererenaldisease,erythropoietinmayneedtobesupplemented.
Decreasedserumtransferrinconcentration(ChoiceF)isnotacauseofhemochromatosis.Transferrinfunctionstotransportandsequesterirontovarioustissues,andtransferrinissaturatedwithironinhemochromatosis.Increasedintestinalabsorptionofironin
hemochromatosisleadstoincreasedserumiron,ferritin,andtransferrinsaturation,aswellasdecreasedtotalironbindingcapacity.
EducationalObjective:HemochromatosismaybeacquiredorinheritedsecondarytomutationsintheHFEgene,leadingtoabnormallyincreasedintestinalironabsorption.Hemochromatosispresentswithliverfailure,diabetesmellitus,arthritis,heartfailure,
darkeningoftheskin,and/orgonadalatrophysecondarytoexcesstotalbodyiron.
o© m c- ©©
Previous Next SconeReport LabValues Calculator Help Pause

ExamSection2:Item28ol50 NationalBoardofMedicalExaminers^
ComprehensiveBasicScienceSelf-Assessment
Y
28.Thepresenceofargininosuccinateintheurineindicatesadefectintheconversionofwhichofthefollowing?
A)Ammoniatourea
B)LysinetoglutarylCoA
C)MethioninetosuccinylCoA
D}Phenylalaninetofumarate
E)Tryptophantoindole
CorrectAnswer:A.
Argininosuccinateisanimportant.ntermediatewithintheureacycle,ttisformedfromcifruIJinebyargininosuccinatesynthetaseandisconvertedtoargininebyargininosuccinatelyase(ASL).MutationsinthegeneencodingASLresultinargininosuccinicaciduria,
characterizedbytheaccumulationofargininosuccinate,citrulline,andammonia.Patientstypicallypresentininfancywithlethargy,vomiting,poorfeeding,hepatomegaly,seizures,andcoma.Deficiencyofthisenzymeisinheritedinanautosomalrecessivemanner.
IncorrectAnswers:B,C.D,andE.
ConversionoflysinetoglutaryiCoA(ChoiceB)takesplaceduringcatabolismoflysine.GlutarylCoAissubsequentlymetabolizedbyglutarylCoAdehydrogenasetocrotonylCoA.
ConversionofmethioninetosuccinylCoA(ChoiceC)occursduringthecatabolismofmethionineviamultipleintermediates,includinghomocysteine,cystathionine,andpropionylCoA.SuccinylCoAisanintermediatewithinthecitricacidcycle.
Conversionofphenylalaninetofumarate(ChoiceD}involvesmultiplestepsandoccursviaintermediatesincludingtyrosineandhomogentisicacid.Disordersinthispathwayincludealkaptonuria,whichiscausedbydeftciencyoftheenzymehomogentisateoxidase.
Conversionoftryptophantoindole(ChoiceE)occursviaareactioncatalyzedbytryptophanasewithadditionalproductspyruvateandammonia.
EducationalObjective:Deficiencyoftheenzymeargininosuccinatelyaseresultsintheautosomalrecessivediseaseargininosuccinicaciduria,whichischaracterizedbytheaccumulationofargininosuccinate,citruiline,andammonia.Patientstypicallypresentin
infancywithlethargy,vomiting,poorfeeding,hepatomegaly,seizures,andcoma.
000 0
*
&
Previous Next ScoreReport LabValues Calculator Help Pause

ExamSection2:Item29of50 NationalBoardofMedicalExaminers^
ComprehensiveBasicScienceSelf-Assessment
Y
29.A55-year-oldwomanisbroughttotheemergencydepartmentafterbeinginjuredinamotorvehiclecollision.Shehasaninjuryofthesofttissueofthefacethatpreventsherfromdrinkingfromaglasswithout
spillingthecontents.Severaldayslatershestillhasthisproblem.Thephotographshowsherattemptingtopurseherlipstowhistle.Whichofthefollowingnervesismostlikelydamaged?
A)Buccalbranchofthefacialnerve
B}Inferioralveolarbranchofthemandibulardivisionofthetrigeminalnerve
C)Infraorbitalbranchofthemaxillarydivisionofthetrigeminalnerve
D)Mandibularbranchofthefacialnerve
E}Pharyngealbranchesofthevagusnerve
CorrectAnswer:A.
Damagetothebuccalbranchofthefacialnerveismostlikelyresponsibleforthispatientsinabilitytopurseherlipsordrinkfromaglass.Thefacialnerveperformsdiverseafferentandefferentfunctionsandhasfivemainextracranialbranches:thetemporal
zygomatic,buccalmarginalmandibularandcervicalbranches.Thebucca!branchofthefaciainerverunsalongtheparotidductandinnervatestheorbicularisoris(purseslips),buccinator(holdscheeksflattoaidinchewingandblowing),levatorlabiisuperiors
(elevatesupperlip),andangufioris(assistsinsmiling).Damagetothebuccalbranchthereforeleadstodifficultyinperformingtasksthatrequireclosingorpursingthelipssuchasdrinkingfromaglassorwhistlingandmayalsoleadtothedecreasedabilityto
masticate.
IncorrectAnswers:B,C,D.andE.
Theinferioralveolarbranchofthemandibulardivisionofthetrigeminalnerve(ChoiceB)mediatessensationofthelowerteethandgingivaealongwiththelowerlipandskinofthechinTheinfraorbitalbranchofthemaxillarydivisionofthetrigeminalnerve(Choice
C)providessensoryinnervationtothelowereyelid,lateralinferiorportionofthenose,andupperlip..Thispatientdemonstratesmotorratherthansensorydysfunction.
Themandibularbranchofthefacialnerve(ChoiceD)suppliesthedepressormusclesofthelowerlip(thedepressorangulioris,depressorfabiiinferioris,andmentals).Consequently,damagetothemandibularbranchleadstothedecreasedabilitytoopenthe
mouthorsmileThispatientdemonstratesadecreasedabilitytoclosethemouth(pursethelips).
Thepharyngealbranchesofthevagusnerve(ChoiceE)innervatethepharyngealconstrictormusclesandpalatinemuscles,whichfunctiontoinitiateswallowingandmoveafoodbolusfromthepharynxtotheesophagus.Thispatientwithdamagetothebuccal
branchofthefacialnervewoulddemonstratedifficultiesinmasticating,notswallowing.
EducationalObjective:Thebucca'branchofthefacialnerveinnervatesseveralperioralmusclessuchastheorbicularisorisandbuccinatormuscles.Damagetothebuccalbranchofthefacialnerveleadstodifficultypursingthelipsandmasticating.
O0
It 4
s
& tt*
It?
Previous Next ScoreReport LabValues Calculator Help Pause

ExamSection2:Item30ol50 NationalBoardofMedicalExaminers^
ComprehensiveBasicScienceSelf-Assessment
Y
30.Apreviouslyhealthy6-month-oldboyisbroughttothephysicianbecauseofacoughfor1week.Initiallyhehadalow-gradefeversneezing
:congestion,andrunnynose.Hethendevelopedadryintermittentcough.Theparentsnownotethatwithany
startlethebabychokesandgasps.Hehasnothadanyimmunizations.Physicalexaminationshowsparoxysmsof''machinegun"-likecoughingwithaforcedexpiratorygruntattheendofcoughing.Leukocytecountis30
;000/mm
3(73%lymphocytes).
Neutrophilchemotaxisandoxidativemetabolismaredefectiveduetoincreasedactivityofwhichofthefollowingenzymes?
A)Adenylylcyclase
B)Myeloperoxidase
C)NADPHoxidase
D)PhospholipaseC
E)ProteinkinaseC
CorrectAnswer:A.
Episodesofsevereparoxysmalcoughinginthesettingofarespiratoryillnessisconcerningforpertussis.PertussisoccursduetoaninfectionfromBordeteflapertussisandclassicallypresentsinthreestages.Thecatarrhalstageisfirst,whichpresentswithfever
rhinorrhea,andamildcough.Thisprogressestotheparoxysmalstageafteronetotwoweeks,inwhichpatientsexperienceviolentboutsofextremecoughing
:attimesviolentenoughtocausesecondarytraumaticinjurysuchasribfractures,urinaryincontinence,
andpneumothorax.Often,suchcoughingfitsarefallowedbysyncope
:emesis,orapnea.Theparoxysmalstagecanlastforweekstomonths.Symptomsresolveduringtheconvalescentstage,whichlastsfromonetofourweeks.Pertussiscanbepreventedwith
immunization,andistreatedwithantibiotics,typicallymacrolides.PertussistoxinisamajorvirulencefactorthatinactivatestheGjsubunitsofGprotein-coupledreceptorsintherespiratoryepitheliumandsystemically,leadingtouninhibitedadenylylcyclaseactivity
andincreasedintracellularcyclicadenosinemonophosphate(cAMP)concentration.Inneutrophilsandmacrophages,thisresultsinimpairedrecruitment,migration,andproductionofinflammatorymediators.
IncorrectAnswers:B,C,D
;andE.
Myeloperoxidase(ChoiceB)deficiencyisacommoninheritedimmunodeficiencysyndromecharacterizedbytheinabilitytoproducehypochlorousacidwithinphagolysosomesaspartoftheoxidativeburst.Diseaseistypicallymildandmaypresentwithrecurrent
Candidaalbicansinfection.
NADPHoxidase(ChoiceC)deficiencyisfoundinchronicgranulomatousdisease,inwhichtheabsenceofaneffectiveoxidativeburstpredisposestorecurrentinfectionswithcatalase-positiveorganisms.
PhospholipaseC(ChoiceD)cleavesmembraneboundphospholipids,leadingtotheformationofinositoltriphosphateanddiacylglycerolwithfurtherdownstreamsignalingeffects,includingactivationofproteinkinaseC(ChoiceE).Pertussistoxindoesnotaffect
thissecondmessengersystem.
EducationalObjective:PertussistoxinisamajorvirulencefactorofB.pertussis.
,thecausativeagentofpertussis.PertussistoxincausesincreasedintracellularcAMPconcentrationthroughtheinactivationofGjsubunitsofGprotein-coupledreceptorsinthe
respiratoryepitheliumandsystemicaIlyleadingtouninhibitedadenylylcyclaseactivity.Thisresultsinimpairedrecruitmentmigration,andproductionofinflammatorymediatorsbyinflammatorycells,includingneutrophils.
© © ©
Previous Next ScoreReport LabValues Calculator Help Pause

ExamSection2:Item31of50 NationalBoardofMedicalExaminers^
ComprehensiveBasicScienceSelf-Assessment
31.A22-year-oldwomancomestotheofficebecauseofa3-dayhistoryofnonproductivecough.Shealsohasa1-weekhistoryoffatigue
:progressiveshortnessofbreathwithexertionandwhiletyingdown
;andswellingofherlegsandfeet.Shedelivereda
malenewbornviauncomplicatedvaginaldelivery1monthago.Shehasnohistoryofmajormedicaiillnessandtakesnomedications.Hertemperatureis37.7X(99.8aF)
rpulseis104/min,respirationsare20/min,andbloodpressureis126/80mmHg.
Bilateralbasilarcracktesareheard.Thereis1+edemaofthelowerextremitiesbilaterally.Whichofthefollowingisthemostlikelydiagnosis?
A)Amnioticfluidembolism
B)Cardiomyopathy
C)Majordepressivedisorder
D)Pneumonia
E)Pulmonaryembolism
F)Pulmonaryfibrosis
CorrectAnswer:B.
Heartfailurepresentswithdyspneaonexertion,reducedexercisetolerance,paroxysmalnocturnaldyspnea,orthopnea,shortnessofbreath,fatigue,weightgain,andperipheraledema.Physicalexaminationoftenrevealsjugularvenousdistension,pittingperipheral
edema,bibasilarrales,adisplacedapicalimpulse,andanS
3
orS
4
gallop.Laboratorystudiestypicallyshowincreasedconcentrationofbrainnatriureticpeptide,whilex-raymayrevealcardiomegalyandpulmonaryedema.Itcanresultfromischemicheartdisease
ornon-ischemiccardiomyopathy(eg,alcoholusedisorder,beriberi,Chagasdisease,adverseeffectofchemotherapy).Thispatienthasperipartumcardiomyopathy,whichisaformofnon-ischemiccardiomyopathyleadingtoleftventricularfailure.Peripartum
cardiomyopathytypicallypresentsafter36weeks'gestationorwithin5monthsofdeliveryandisassociatedwithaeftventricularejectionfractionoflessthan45%.Itsdevelopmentismultifactorialbutincludesoxidativestressandimpairedvascularendothelial
growthfactorsignaling,athoughtheetiologyisnotcompletelyelucidatedTreatmentissimilartotreatmentforotherformsofheartfailure,withsupplementaloxygen,diuretics,inotropesorvasopressorsasneeded.Manypatientshaveresolutionoftheir
cardiomyopathy.Complicationsincludeformationofaleftventricularthrombus,developmentofatrialfibrillationordysrhythmia,worseningcardiacfunctionnecessitatingcardiactransplant,andincreasesinmaternalmorbidityandmortality.
IncorrectAnswers:A,C,D,E,andF.
Amnioticfluidembolism(ChoiceA)presentswithacuteonsetofshortnessofbreath,alteredmentalstatusorunconsciousness,diffusebleedingfromvenipuncturesitesorthevagina,hypoxia,hypotension,andsignsoffetaldistressduringtheintrapartumor
immediatepostpartumperiod,itiscausedbyamnioticfluidenteringthematernalcirculationandisassociatedwithhighlevelsofmaternalmortality.Itwouldnotpresent1monthafterdelivery.
Majordepressivedisorder(ChoiceC)presentswithdepressedmood,anhedonia,insomnia,weightloss,fatigue,impairmentsinconcentration,guilt,psychomotorslowing,andsuicidalideation.Itiscommon ,inthepostpartumperiodbutdoesnotcauseshortnessof
breathorperipheraledema.
Pneumonia(Choice0)typicallypresentswithfever,coughproductiveofpurulentsputum,shortnessofbreath,andoccasionallychestpain.Hypoxia,leukocytosis,andanewinfiltrateonchestx-rayareoftenseen.Itisnotassociatedwithperipheraledema.
Pulmonaryembolism(ChoiceE)classicallypresentswithshortnessofbreath,hemoptysis,tachycardia,andhypoxiainapatientwithacoagulopathy,malignancy,recentsurgery,orprolongedimmobilization.Itdoesnottypicallycauseorthopneaorbibasilar
crackles.
Pulmonaryfibrosis(ChoiceF)mostcommonlypresentsinolderpatientswithexertionaldyspneaandnonproductivecough,oftengradualinonsetandprogression.Drybibasilarcracklesmaybeaudibleonphysicalexamination,andhigh-resolutionCTscanofthe
chestwillrevealhoneycombingandtractionbronchiectasis.Itisesslikelyinthisyoungpatientwithperipheraledemaandrelativelyacuteonsetofsymptoms.
EducationalObjective:Peripartumcardiomyopathypresentsafter36weeks
'gestationorwithin5monthsofdeliverywithsignsofcongestiveheartfailuresuchasshortnessofbreath,weightgain,orthopnea,paroxysmalnocturnaldyspnea,cough,andperipheral
edema.Ittypicallyresolveswithheartfailuretreatment.
0 0 ^
tf?
Previous Next ScoreReport LabValues Calculator Help Pause

ExamSection2:Item32ol50 NationalBoardofMedicalExaminers^
ComprehensiveBasicScienceSelf-Assessment
Y
32.AG3-year-oldmanisscheduledtoundergocoronaryarterybypassgraftingwithaportionofthegreatsaphenousvein.Anincisiontoremoveaportionoftheveinshouldbegininwhichofthefollowinglocations?
A)Alongthelateralsurfaceoftheleg
B)Alongthemedialsideoftheanklejoint
C)Alongtheplantarsurfaceofthefoot
D}Anteriortothekneejoint
E}Posteriortothehipjoint
CorrectAnswer:B.
Thegreatsaphenousveinisthelongestveininthebodyandrunssubcutaneoustyalongthemedialaspectoftheleg.Itstartsjustproximaltothearchofthefootandpassesanteriortothemedialmalleolusoftheankle.OnpersonswithlowerBMI,itcanbe
palpatedatthislocation.Itcoursesproximallyalongthemedialaspectofthekneeandcontinuesalongthemedialaspectofthethighuntilitjoinsthecommonfemoralveinintheregionofthefemoraltriangleinthegroin.Thisveinisoftenusedforbypassgraftingby
cardiothoracicandvascularsurgeons,andtoremoveaportion,theincisionshouldbeginalongthemedialsideoftheanklejoint.Itssubcutaneouslocationmakesiteasytoharvestandredundancyofthevenousoutflowoftheleg(eg,tibia!andpoplitealveins)
makesitanon-essentialveinfordrainageoftheSowerextremity.
IncorrectAnswers:A,C
;D,andE.
Thelateralsurfaceofthelowerleg(ChoiceA)FSdrainedbythesmaltsaphenousvein,whichbridgesofffromthefemoralveinatthelevelofthepoplitealfossa.Thisveinislesscommonlyusedasabypassgraft.
Theplantarsurfaceofthefoot(ChoiceC)hasavenousnetworkthatincludesthemedialplantarvein,thelateralplantarvein
;andthedeepplantarvenousarch,whichconnectsthemedialandlateralveins.Thisvascularnetworkisnotusedforgraftingbecausethe
veinsareshortandsurgerytotheplantarsurfaceofthefootcancauseong-termpainduetoscarringonaweightbearingsurface.
Anteriortothekneejoint(ChoiceD)arethegenicularveinsThesearesmallveinsthatdraintheareaaroundthekneejointintothepoplitealvein.Theyareshortandarenotusedforbypassgrafting.
Posteriortothehipjoint(ChoiceE)arethesuperiorandinferiorglutealveins.Theseveinsdraintheglutealmusculatureposteriortothehipanddrainintotheinternaliliacvein.
EducationalObjective:Thegreatsaphenousveinisthelongestveininthebody.Itrunsalongthemedia,aspectofthelowerextremity.Itcanbeeasilyidentifiedinsomepatientsafongtheanterioraspectofthemedialmalleolusoftheankle.Itnotessentialfor
venousdrainageofthelowerextremity,makingitoptima!forbypassgrafting.
00#
ft*
Previous Next ScoreReport LabValues Calculator Help Pause

ExamSection2:Item33of50 NationalBoardofMedicalExaminers^
ComprehensiveBasicScienceSelf-Assessment
Y
33.AG2-year-oldmanisbroughttotheemergencydepartment2hoursafterthesuddenonsetofpainandcoolnessofhisrightleg.Heisotherwisehealthyexceptformildhyperthyroidismtreatedwithpropylthiouracil.Examinationofthefowerextremitiesshows
normalskin,nails,andhairgrowthpatterns.Pulsesareabsentintherightlowerextremityandnormalontheleft.Whichofthefollowingisthemostlikelydiagnosis?
A)Cellulitis
B)Deepvenousthrombosis
C)Embolicarterialocclusion
D)Lumbardischerniation
E)Rhabdomyolysis
CorrectAnswer:C.
Thromboemboliceventscancausecriticaliimbischemiainpersonswithotherwiserelativelynormalperipheralvascuiature.Themostcommoncauseofembolitothelowerextremityisfromtheheartrelatedtoavarietyofunderlyingpathology.Thromboticemboli
fromtheleftatriumtothelowerextremityareassociatedwithatrialfibrillation,whichcausesuncoordinated,inadequatecontractionsoftheleftatriumandconsequentimpairedbloodflow,stasisofwhichservesasatriggerforthrombusformation.Thromboticemboli
fromtheleftventriclehavebeenassociatedwithabnormalflowpatternsintheleftventriclesecondarytomyocardialinfarctionandventricularaneurysmformation,whichalsoresultsinimpairedcontractilityandpoolingofblood.Lesscommonly,cardiactumorsand
thromboticdebrisfromthevalves(endocarditis)cancauseacutearterialocclusion.Atheroscleroticpiaquefromadiseasedaortamayalsoembolizecausingadistalischemicevent.Symptomsofathromboemboliceventoftheextremityincludesacutepain,
swelling,pallorandpulselessness.Latemanifestationsincludemuscleandnervedeathwithextremepainandpassiblecompartmentsyndrome.
IncorrectAnswers:A.R,D.andE.
Cellulitis(ChoiceA)isasuperficialinfectionoftheskinandsubcutaneoustissue.Ittypicallypresentsasawarm,erythematous,painfulregionofskinthatmayenlargeinsizeandmayoriginateatanareaoftraumaorskinbreakdown.Itdoesnotresultindiminished
pulses.
Deepvenousthrombosis(ChoiceB)commonlyoccursinthelowerextremitiesaftersurgery,duringperiodsofimmobilization,andinpatientswithincreasedinflammatorystatessuchascancer.Althoughthiswouldpresentwithacutepainintheextremitywith
associatedswelling,avenousthrombosiswouldnotcausepuiselessnessunlessadvancedandlarge(eg,phlegmasiaceruleadolens).
Lumbardischerniation(ChoiceD)occursmostoftenfollowingchronicdiscdegenerationcombinedwithatraumaticeventsuchasbendingandliftingaheavyobjectcausinganacuteincreaseintheintradiscalpressure.Acutedischerniationpresentsasacuteback
painalongwithacuteextremitypainand/orweaknessinthedistributionoftheimpingedspinalnerve.
Rhabdomyolysis(ChoiceE)isanacutebreakdownofmuscletissueleadingtoincreasedserumlevelsofmyoglobinandelectrolytederangements(eg,hyperkalemia).Thismayoccurinhealthyadultsafterperiodsofintensephysicalactivity.Itmayalsooccurin
patientswithseizuredisordersandmelderlypatientswhohavefallenandsustainedprolongedcompressionofmuscle.
EducationalObjective:Acuteembolicarterialocclusioncanoccurintheextremitiesandintheinternalorganssuchasthekidneyandintestine.Embolizationofthrombus,cholesterolplaque,ortumortissue,mostcommonlyfromtheheart,leadstoarterialocclusion
thatcanbedetectedonphysicalexaminationaspulselessness,pallor,andpain.
© © © ©
f*
Previous Next ScoreReport LabValues Calculator Help Pause

ExamSection2:Item34ol50 NationalBoardofMedicalExaminers^
ComprehensiveBasicScienceSelf-Assessment
Y
34.A32-year-oldwomancomestothephysicianforafollow-upexaminationafteratypicalcellswerenotedonarecentPapsmear.Physicalexaminationshowsa1x1-cmareaofleukoplakiaonthecervix.Abiopsyspecimenofthelesionshowsinvasive
squamouscellcarcinoma.Malignantcellsfromthissitewillmostlikelydrainfirsttowhichofthefallowinglymphnodesinthispatient?
A)Femoral
B)Inferiormesenteric
C)Internaliliac
D)Lumbar
E)Superficialinguinal
CorrectAnswer:C.
Theinternaliliaclymphnodesareacomponentofthelymphaticsystem,anetworkofvesselswhichfollowapredictablepatternofdrainagetolymphnodebeds.Lymphisgeneratedbyhydrostaticpressureinthetissuescausingfluidtoleakoutofvascular
structuresandintotheinterstitium.Itisthencollectedbythelymphaticsalongwithlymphocytesandanymalignantcellsexitingthetissues.Internaliliaclymphnodesdrainthelowerrectumtotheanaicanalabovethepectinateline,bladdermiddlethirdofthe
vagina
:cervix,andprostate.Afterreachingtheinternaliiiaclymphnodes
;thelymphwillbechanneledthroughthecisternachyliandthoracicductandultimatelydrainintotheleftsubclavianvein.Thisisthesamefinalpathwayforlymphfromeitherlowerextremity,
thepelvis,ortheleftupperextremity.Incontrast,thelymphaticnetworkoftherightsideofthebodyabovethediaphragmisdrainedbytherightlymphaticduct,whichenterstherightsubclavianvein.Knowledgeofthefirstdraininglymphnodeofananatomicalsite,
orsentinelnode,isimportantinpatientswithinvasiveneoplasmsasthesentinelnodeisoftenbiopsiedtogenerateprognosticdataandguidetreatment.
IncorrectAnswers:A.R,D.andE.
Thelymphnodesinthefemoral(ChoiceA)triangleincludethesuperficialanddeepinguinallymphnodes.Thesuperficialinguinal{ChoiceE)lymphnodesdrainlymphaticfluidfromthelabiamajors,vulva,scrotum,analcanalbelowthepectinateline,andtheskin
belowtheumbilicusexceptthepoplitealfossa.Thedeepinguinallymphnodesdraintheglanspenis.
Theinferiormesenteric(ChoiceB)lymphnodesdrainlymphfromthelargebowel,extendingfromtheregionofthesplenicflexuretotheupperrectum.Theportionofthelargebowelproximaltothesplenicflexureandsmallbowelaredrainedbythesuperior
mesentericlymphnodes.
Thelumbar{ChoiceD)lymphnodes,orperiaorticlymphnodes,arecomprisedofthepreaortic,paraaortic,andretroaorticgroups.Theinternaliliacnodesdraintotheparaaortic,asdotheovaries,testes,uterus,andkidneys.However,itisnotthefirstdrainagesite
ofthecervix.
EducationalObjective:Lymphfromthelowerrectumtotheanalcanalabovethepectinatefine,bladder,middlethirdofthevagina,cervix,andprostatedramstotheinternaliliaclymphnodes
00# 00
Previous Next ScoreReport LabValues Calculator Help Pause

ExamSection2:Item35ol50 NationalBoardofMetlicaiExaminers^
ComprehensiveBasicScienceSelf-Assessment
35.A70-year-oldmandiesinamotorvehiclecollision.Hehadbeenundergoingevaluationforoccultbloodinthestool.Aphotographofasectionofthetransversecolonobtainedatautopsyisshown.Whichofthe
followingisthemostlikelydiagnosis?
A)Hyperplasticpolyp
B)Inflammatorypseudopolyp
C)Juvenilepofyp
D)Peutz-Jegherssyndrome
E)Tubularadenoma
CorrectAnswer:E
Atubularadenomaisshowninthegrosspictureofthetransversecolonobtainedduringautopsy.Chronicoccultgastrointestinal(Gl.)bleedingisacommonpresentingsymptomofcoforectalcarcinoma(CRC)andforthisreason
:allpatientswithunexplainedGl
bleedingshouldundergocolonoscopy1oruleoutCRC.Colonicpolypspresentinavarietyofsubtypes
:fromnon-neoplasticpolyps(eg
:hamartomatous
:mucosal,inflammatory,hyperplastic)topotentiallymalignantpolyps(eg.adenomatous,serrated).
Adenomatouspolypsmaybeoftubularorvillousarchitectureonhistology,withvillousarchitectureusuallyhavinggreatermalignantpotential,althoughtubularadenomasaremorecommon.Alladenomasdiscoveredoncolonoscopyshouldbeeithercompletely
removediflessthantwocentimetersorbiopsied.Largeradenomasmustberemovedinapiecemealfashionorsurgically.Histologically,theseadenomasareclassifiedasexhibitingfeaturesofhigh-gradeorlow-gradedysplasiaifthelesionsdonotextendinto
themuscularismucosa,andasinvasiveadenocarcinomaiftheydo.
IncorrectAnswers:A,8.C,andD.
Hyperplasticpolyps(ChoiceA)arebenignpolypsthatdonotcauseanincreasedriskofmalignancy.Theyareusuallysmalllocatedintherectosigmoidcolon,andarenotassociatedwithanypolyposissyndromes.
Inflammatorypseudopolyps(ChoiceB)maybeseenwithinflammatoryboweldiseasesincludingulcerativecolitisorCrohndiseaseandrepresentnormal,non-inflamedtissuesurroundedbyerodedmucosaandgranulationtissue.Theydevelopsecondaryto
chronicinflammationofthecolon.Thispatienthasnoreportedhistoryofunderlyinginflammatoryboweldisease,andonthegrossspecimenthesurroundingcolonicmucosaappearsnormal.
Juvenilepolyps(ChoiceC)constitutebenignhamartomasandaremostcommoninchildrenbetweenagetwoandten.Painlessrectalbleedingischaracteristic.Inpatientswithmorethantenpolyps,considerationshouldbegiventothepresenceofafamilial
polyposissyndrome,butisolatedpolypsaregenerallybenignandconfernoincreasedriskofmalignancy.Thispatient'sageessentiallyeliminatesthisdiagnosis.
Peutz-Jegherssyndrome(ChoiceD)isanautosomaldominantsyndromethatischaracterizedbyhamartomatouspolypsinthecolonandpigmentedmaculesinthemouth,lips,hands,andgenitalia.Itisassociatedwithincreasedriskofbreastand
gastrointestinaltractcancers(eg,colorectal,stomach,smallbowel,pancreatic).ThisolderpatientwithasolitarymassisunlikelytohavePeutz-Jegherssyndrome.
EducationalObjective:TubularadenomascanpredisposetothedevelopmentofCRC.whichmaypresentwithoccult,chroniclowerGlbleeding.Tubularadenomasareclassifiedbysizeandhistologicfeatures,withthetypeofintervention(eg.colonoscopic
removalvssurgical)baseduponthesefeatures.
E3
Previous Next SconeReport LabValues Calculator Help Pause

ExamSection2:Item36ol50 NationalBoardofMetlicaiExaminers^
ComprehensiveBasicScienceSelf-Assessment
y
36.A61-year-oldmanhaserectiledysfunctionduetospinalcordinjuryatL-2.Sildenafilislikelytomarkedlycorrectthedysfunctionbyactingatwhichofthefollowinglabeledstructuresinthetransversesectionofthepenis?
1
A)
B)
C)
D)
CorrectAnswerD.
Sildenafilisaselectiveinhibitorofcyclicguarosinemonophosphate(cGMP)-specificphosphodiesterase-5(PDE5).Itssiteofeffectintreatingerectiledysfunctionisprimarilyatthecorporacavernosa
;apairofrichlyvascularizederectilebodieswithintheshaftof
thepenis.ByinhibitingPDE5
;sildenafilincreasestheconcentrationofnitricoxidewithinthecorpuscavernosum(indicatedbyletterD),whichinturnpromotescorpuscavernosumvasculardilationandtumescence.Adverseeffectsofsildenafilincludeheadache
;
flushing,cyanopsia(atransientbluetinttovision),andpriapism.Sildenafiliscontraindicatedinpatientswhoareusingnitratemedications,asthesetwodrugclassesmayactsynergisticallyandproducelife-threateninghypotension.
IncorrectAnswers:A.B.andC.
ChoiceAindicatesthesuperficialdorsalveinofthepenis.Thisveindrainsmanyofthesuperficialstructuresofthepenisbutplaysnoroleintheinitiationoferection.
ChoiceBindicatestheconnectivetissuessurroundingthecorporacavernosa,whichincludethetunicaalbugineaandtheBuckfascia.Connectivetissueslimittheegressofvenousbloodfromtheerectpenisbutarenotthesiteofactionofsildenafil.
ChoiceCindicatestheurethra,whichplaysnoroleinerections.
EducationalObjective:SildenafilselectivelyinhibitscGMP-spectficphosphodiesterase-5(PDE5),leadingtoanincreaseinnitricoxidewithinthecorporacavernosa,whichresultsinvasculardilationandtumescenceoftheerectilebodies.Sildenafilshouldnotbe
prescribedtopatientsusingnitratesduetothepotentialforlife-threateninghypotension.
000
r-
Previous Next ScoreReport LabValues Calculator Help Pause

ExamSection2:Item37of50 NationalBoardofMedicalExaminers^
ComprehensiveBasicScienceSelf-Assessment
Y 40-i
37.Whichofthefollowingisthemeannumberofepisodesofurinarytractinfectionsforchildren(n=100)inthesampleshownin
^
hegraph?
A)1 ®30-
XJ
B)1.55
C)2.07
D)Cannotbedeterminedfromthisgraph
-
:
u
o
20-
<15
JD
I
IQ
"
30 1 2
Numberofurinarytract
infections
CorrectAnswer:B.
Thisgraphisabarchartthatdemonstratescounts(numberofchildren)ontheY-axisandnumberofurinarytractinfections(LJTIs)ontheX-axis.Thecount(numberofchildren)representedbyeachbarisalsoprovidedatthetopofthebar.Thus
:thisgraph
communicatesthatwithinthisstudy
:25childrenhadnoUTIinfections
*
30childrenhadoneUTI
:10childrenhadtwoUTIs,and35childrenhadthreeUTIs.ThisinformationissufficientforcalculatingtheoverallmeannumbercfUTIsinthestudysample.
CalculationofthemeannumberofUTIswouldbeequaltothetotalnumberofUTIsdividedbythetotalnumberofchildren.Formally,thisiscalculatedas:((25x0)+.(30x1)+(10x2)+(35x3))/(25+30+1Ch+35)=1.55.
IncorrectAnswers:A,C
;andD.
1(ChoiceA)and2.07(ChoiceC)areincorrectnumericcalculations.Theyarenotthemeannumberofinfectionsperchild.2.07wouldbethemeanifonlynon-zerodatapaintswereconsidered(iftheaveragewerecomputedconsideringchildrenthathadatleast
oneUTI).
Cannotbedeterminedfromthisgraph(ChoiceD)isincorrectasthegraphandtheadditionalnumericlabelsonthebarsprovidesufficientinformationforcalculationofthemeannumberofinfections.
EducationalObjective:Meanisabasicmeasureofcentraltendencyandiscalculatedasasimpleaverage.Whencalculatingthemeannumberofoutcomesperperson,thetotalnumberofoutcomesisdividedbythetotalnumberofindividuals.
0 0 & % 0
*
&
Previous Next ScoreReport LabValues Calculator Help Pause

ExamSection2:Item33ol50 NationalBoardofMetlicaiExaminers^
ComprehensiveBasicScienceSelf-Assessment
38.A3-year-oldboyisbroughttothephysicianbecauseofa1-monthhistoryofpaleskin.HisparentsareofEuropeandescent.Hehasnopersonalorfamilyhistoryofmajormedicalillness.Physical
examinationshowspallor.Laboratorystudiesshow:
Hemoglobin
Hematocrit
Meancorpuscularhemoglobinconcentration
Meancorpuscularvolume
Sg/dl(N=11-15)
24%(N=28-45)
34%Hb/cel!(N=31-36)
90pm^(N=77-98)
i
m
Aphotomicrographofaperipheralbloodsmearisshown.Genetictestingismostlikelytoshowwhichofthefollowingfindingsinthispatient?
A)Heterozygousmutationintheankyringene
B)Heterozygousmutationinthea-gfobingene
C)Heterozygousmutationinthep-globingene
D)Homozygousmutationintheankyringene
E)Homozygousmutationinthea-globingene
F)HomozygousmutationintheP
-globingene
CorrectAnswer:A.
Aheterozygousmutationintheankyringeneleadingtohereditaryspherocytosis(HS)mostlikelyexplainsthispatient'
snormocyticanemiaandperipheralsmearshowingspherocytesandreticulocytes.HScanoccuriftherearemutationsinoneormoreproteins
thatlinktheerythrocyteinnermembraneskeletontotheouterlipidbilayer,andincludemutationsinspectrin,ankyrin.band3,andband4.2.Thesemutationsareinheritedinartautosomaldominantpattern,whichexplainswhythispatientwouldonlyrequireone
mutationtoexhibitthephenotype.Spherocytosisresultsfromtheprogressivelossoftheerythrocytemembraneovertimeandgivestheerythrocytesasphericalshapeasthesurfaceareadecreasesbutthevolumeofthecellremainsunchanged.Thisimpairsthe
erythrocyte'sabilitytoconforminsmallvesselsofthemicrocirculationespeciallywithinthespleen
;wherespherocytesaccumulate.Patientsoftenpresentwithsplenomegaly,hemolyticanemia
:andpigmentedgallstones.Peripheralsmearwillshowspherocytes
and.ifthereisahighdegreeofhemofysis
:reticulocytes,whichappearaslarger,paleblueerythrocytes.Definitivemanagementinvolvessplenectomy.
IncorrectAnswers:B,C.D.E.andF.
Heterozygousmutationinthea-globingene{ChoiceB)describesa-tha^
assemiatrait(silentcarrier),whichpresentswithnormalhemoglobinevels,normalhemoglobinelectrophoresis,andmicrocytosis.Therearefoura-globingenesorganizedaspairson
chromosome16,withonepairinheritedfromeachparent.Theseverityofanemiaisdeterminedbythenumberandthenatureofthemutations,withagreaternumberofdeletionsresultinginmoresevereclinicalphenotypes.
Heterozygousmutationinthep-giobingene(ChoiceC)causesp-thalassemiaminor.Incomparisontothefourallelesofthea-giobingene,thereareonlytwop-globinalleles.Patientsaretypicallyasymptomaticoronlymildlysymptomatic.Peripneralsmear
demonstratesmicrocytosisandtargetcellsbutnotspherocytes.
Homozygousmutationintheankyringene(ChoiceD)isassociatedwiththesporadicformofHS,althoughmutationscanbeinheritedinanautosomalrecessivemanner.Thesearelesscommonlyencounteredthanautosomaldominantmutations.
Homozygousmutationinthea-globingene(ChoiceE)causesa-thalassemia,whichinthecaseofatwo-aleledeletionischaracterizedbymildmicrocyticanemiaandhemoglobinelectrophoresiswithasmallconcentrationofHb-Barts(y4).Thesepatientsare
mostlyasymptomatic.Therewouldnotlikelybespherocytesonperipheralsmear.
Homozygousmutationinthep-globingene(ChoiceF)causesp-thalassemiamajor,whichischaracterizedbysevere,transfusiondependent,microcyticanemiaandsignsofextramedullaryhematopoiesissuchasfrontalbossing.
EducationalObjective:Autosomaldominantmutationsinankyrin,spectrin,andbanderythrocytemembraneproteinscausehereditaryspherocytosisthroughlossofthenormalconnectionbetweentheinnermembraneskeletonandtheouterlipidbilayer.Clinical
symptomscanincludejaundice,splenomegaly,andhemolyticanemia.Aperipheralbloodsmeartypicallyexhibitsspherocytesandreticulocytes.Definitivetreatmentiswithsplenectomy.
000
<=IS
Previous Next ScoreReport LabValues Calculator Help Pause

ExamSection2:Item39of50 NationalBoardofMedicalExaminers^
ComprehensiveBasicScienceSelf-Assessment
39.A24-year-oldwomanisbroughttotheemergencydepartment15minutesafterthesuddenonsetofshortnessofbreathfollowingabeesting.Herpulseis130/min,andbloodpressureis70/30mmHg.Threehourslaterherbloodpressurereturnsto
normalfollowingadministrationofintravenousfluids
:corticosteroids,antihistamines
:andepinephrine.Thenextday
:shehasminimalurineoutput.Whichofthefollowingareasofihekidneyislikelytobemostaffectedwiththispatient'sprolonged
hypotension?
A)Glomerularepithelialcells
B)LoopofHenle
G}Medullaryinterstitium
D)Mesangialcells
E)Proximaltubules
CorrectAnswer:E
Decreasedurinaryoutputfollowinganepisodeofhypotensionisconsistentwithadiagnosisofacutetubularnecrosis(ATN)Thecellsoftheproximalconvolutedtubule,whichareresponsibleforalargeportionofionresorptionfromthefiltrate,arehighly
metabolicallyactiveandthereforeespeciallysensitivetoischemicinjury.ATNprogressesthroughaninitialoliguricphasewhichmaylastforseveralweeksbeforerecovery,which,ifoccurring,ismarkedbyasubsequentpolyurscphase.IschemicATNmayoccur
secondarytoanytypeofshockstate,inthispatient,anaphylaxisleadtodisseminatedvasodilationandhypotension,impairingrenalperfusion.Cardiogenic,obstructive,hypovolemic,anddistributiveshocketiologiescanailresultinrenalhypoperfusion.ATNmay
alsooccursecondarytonephrotoxicexposuressuchasheavymetals,antibiotics,andtoxicalcohols.Urinalysistypicallydemonstratesmuddybrowncasts.
IncorrectAnswers:A.ELC.andD.
Glomerularepithelialcells(ChoiceA)
;alsoknownaspodocytes,formpartofthefiltrationbarrieroftheglomerulus.Podocytelossoccursindiabeticnephropathyandisahallmarkofminimalchangedisease.PodocytelossisnotcharacteristicofATN.
TheloopofHenle(ChoiceBYespeciallythethickascendinglimb,isalsometabolicallyactiveandmaybeinjuredbyischemicinjury.However,injuryoftheloopofHenleoccurslessfrequentlythaninjuryoftheproximalconvolutedtubule.
Themedullaryinterstitium(ChoiceC}isthesiteofinterstitialnephritis,whichfrequentlyoccurssubsequenttomedicationexposureandtopyelonephritis.
Mesangialcells(ChoiceD)formthejuxtaglomerularapparatusandareimportantintheregulationoftherenin-angiotensinsystem.Mesangialce.lsareclassicallyobservedtoexpandandproliferateinthecontextofdiabeticnephropathy.
EducationalObjective:Acutetubularnecrosisfrequentlyresultsfromischemicinjurytothenephronandischaracterizedbydamagetotheproximalconvolutedtubule.Thesloughingoftubularceilsleadstotheformationofmuddybrowncasts.Acutetubular
necrosispresentsinitiallywitholiguriabeforeprogressingtopolyuriawhenthecellsregenerate.
O0
It &
Previous Next ScoreReport LabValues Calculator Help Pause

ExamSection2:Item40ol50 NationalBoardofMedicalExaminers^
ComprehensiveBasicScienceSelf-Assessment
Y
40.AG5-year-oldmandevelopsurinaryincontinenceimmediatelyafteranoperationforprostatecancer.Themostlikelycauseofhisconditionisdamagetotheprostaticnerveplexusthatresultedindenervationoftheinternalurethralsphincter.Thefunctionof
whichofthefollowingtypesoftissueismostlikelyimpairedasaresultofthisdamage?
A)Denseirregularconnectivetissue
B)Denseregularconnectivetissue
C)Skeletalmuscle
D)Smoothmuscle
E)Transitionalepithelium
CorrectAnswer:D.
Pelvicparasympatheticnervesinthepelvicplexusfunctiontoexciteandcontractthedetrusormuscle
:asmoothmuscleofthebladder,viamuscarinicacetylcholinereceptors,whilesympatheticnervesmediaterelaxationoftheinternalurethralsphincteralso
smoothmuscle,viact-|-adrenergicreceptors,leadingtonormalurination.Damagetothepelvicplexuscanaffectbothbladdercontractionandurethralsphincterrelaxation,leadingtooverflowincontinence.Overflowincontinence\£characterizedbychronicurinary
retentionandachronicallydistendedbladder.Whenintravesicalpressureexceedsoutletclosingpressure,incontinenceresultsasurineflows.Overflowincontinenceistreatedwithmanagingincitingconditions,timedvoiding,orplacementofaurethralcatheter.
IncorrectAnswers:A,B.CaandE.
Denseirregularconnectivetissue(ChoiceA)isfoundinthedermisandconsistsofalargeproportionofcollagenfibers.Itconsistsoffibersthatarearrangedinamatrix,whereasdenseregularconnectivetissue(ChoiceB)isarrangedinparallelfibers.Dense
regularconnectivetissueisfoundinligamentsandtendons,areasofhightensilestrength.Neitherareinvolvedincontrolofurinationorurinaryincontinence.
Thepudendalnervesuppliessensoryneuronstotheexternalgenitaliaandsomaticmotorneuronstothepelvicmusclessuchastheexternalurethralsphincter.Theexterna!urethralsphincterisaskeletalmuscle(ChoiceC)thatprovidesvoluntarycontrolof
urination.Damagetothepudendalnervewouldresultinlossofvoluntarycontrolovervoiding.
Thetransitionalepithelium(ChoiceE)ofthebladderisspecializedstratifiedepitheliumthatisabletoexpandandcontractdependingonthebladdervolume.Thisallowsforbladderdistension.Itdoesnothavearoleincontrolofurinationorincontinence.
EducationalObjective:Theinternalurethralsphinctersasmoothmuscleinnervatedbythepelvicplexus.Damagetothepelvicplexuscanaffectbothbladdercontractionandinternalurethralsphincterrelaxation,leadingtooverflowincontinence.
O0
It 4
s
ft*
Previous Next ScoreReport LabValues Calculator Help Pause

ExamSection2:Item41ol50 NationalBoardofMedicalExaminers^
ComprehensiveBasicScienceSelf-Assessment
Y
41.A10-year-oldboywhohasbegunchemotherapyforacutemyelogenous.eukemiaawakensatnightwithfeverandseverepainintheankles.Treatmentwithover-the-counteranalgesicsdoesnotresolvethepain.Thenextmorning,hehaspainwith
urinationandbloodintheurine.Anincreasedserumconcentrationofwhichofthefollowingcompoundsisthemostlikelycauseofthispatient'ssymptoms?
A)Cystine
B)Glycine
C)Magnesium
D)Urea
E)Uricacid
CorrectAnswer:E
Tumorlysissyndromemostcommonlyoccursfollowingchemotherapyinitiationforleukemiaorlymphoma.Theabruptdestructionofalargenumberoftumorcellsresultsininterstitialandserumreleaseoftheircontents,manifestingashyperphosphatemia,
hypocalcemia,hyperkalemia,andhyperuricemia.Patientswithtumorlysissyndromecanpresentwithfever,nausea,vomiting
:diarrhea,lethargy,cardiacdysrhythmias,seizures,tetany,musclecramps
:and/orsyncope.Theyareatriskforsuddendeathsecondary
totheelectrolyteandmetabolicabnormalities.Purinenucleicacidsaremetabolizedtohypoxanthineandxanthine,whichareconvertedtouricacidbyxanthineoxidase,resultinginhyperuricemia.Uricacidcancrystallizeintherenaltubuies,leadingtonephropathy
andacutekidneyinjury.Itcanalsobedepositedinthejoints,leadingtoarthralgias(eg.gout}.Treatmentoftumorlysissyndromeinvolvessupportivecareandcorrectionofelectrolyteandmetabolicabnormalities.Rasburicaseisusedtopromotethedegradationof
uricacidandisusedinbothpreventionandtreatmentoftumorlysissyndrome.Inpatientswithseverekidneyinjuryandrefractoryelectrolytederangements,renalreplacementtherapywithdialysismaybeindicated.
IncorrectAnswers:A.B,C.andD.
Cystine(ChoiceA)isanaminoacidthatismetabolizedtourea(ChoiceD)anddoesnotresultinhyperuricemia.Urea
:incontrasttouricacid,issolubleinwaterandisnormallyreabsorbediinthenephron,contributingtothehyperosmolargradientsurroundingthe
loopofHenle.Ureadoesnotcauseacutekidneyinjuryorcontributetohyperuricemia.
Glycine(ChoiceB)isanaminoacidthatisfoundinmanypolypeptides,especiallycollagen,andisalsoaninhibitoryneurotransmitterinthecentralnervoussystem.Itisnotappreciablyincreasedintumorlysissyndrome.
Tumorlysissyndromecanresultinhighlevelsofmagnesium(ChoiceC)
:buthypermagnesemiadoesnotresultincrystalnephropathyorarthropathy.
EducationalObjective:Tumorlysissyndromemostcommonlyoccursfollowingchemotherapyinitiationforleukemiaorlymphoma.Theabruptdestructionofalargenumberoftumorcellsresultsininterstitialandserumreleaseoftheircontents,manifestingas
hyperphosphatemia,hypocalcemia,hyperkalemia,andhyperuricemia,thelatterofwhichresultsinacutekidneyinjuryandgout.
O0
Previous Next ScoreReport LabValues Calculator Help Pause

ExamSection2:Item42ol50 NationalBoardofMedicalExaminers^
ComprehensiveBasicScienceSelf-Assessment
Y
42.A27-year-oldwomanisbroughttothephysicianbyherfamilybecauseshehasbeenprogressivelylethargicandunwillingtoleaveherapartmentoverthepastweek.Shehasbeenreceivingtreatmentinamentalhealthcenterfor10yearsbutmissedher
lastappointment18daysagobecauseofasnowstorm.Sheisnowreluctanttoreturnbecauseshebelievesthestaffisinvolvedinanextraterrestrialplot.Threeyearsago,shehadsimilarsymptomstreatedwithelectroconvulsivetherapySheappears
disheveled.Sheishavingauditoryhallucinationsofseveralpeopletalkingabouther.Physicalexaminationshowsnormalfindings.Shehaspooreyecontactaflataffect,andslowspeech.Shedescribesanelaboratedelusionalsystemabouttheplotatthe
mentalhealthcenter.Thoughtcontentisotherwiseimpoverished.Whichofthefollowingisthemostlikelydiagnosis?
A)Bipolardisorderdepressed
B)Borderlinepersonalitydisorder
C)Delusionaldisorder
D)Schizophrenia
E)Schizotypalpersonalitydisorder
CorrectAnswer:D.
Patientswithchronicpsychoticdisorderssuchasschizophreniaorschizoaffectivedisordercommonlydemonstratedelusionalthinking,auditoryhallucinations,disheveledappearance,aflataffect(severelydecreasedrangeofaffect),animpoverishedthought
content(decreasedincidenceofthoughts),andavolition(decreasedabilitytoperformtasks).AccordingtotheDSM-5
;patientswithschizophreniahavetwoclustersofsymptoms:positivesymptoms(additionofmentaldysfunctionincludingdelusions,hallucinations,
anddisorganizedthinkingandbehavior)andnegativesymptoms(absenceofcertainmentalfunctionssuchasaffect,volition,thought,andspeech).Thesesymptomsleadtofunctionalimpairment.Thispatientillustratesseveralpositiveandnegativesymptomsand
functionalimpairment(eg,inabilitytoieavethehome).Schizophreniasymptomsaretypicallychronicbutcanfeatureepisodicexacerbationsofbothpositiveandnegativesymptomsinthesettingofstressormedicationnonadherence(asinthispatientwhomissed
herappointment).Treatmentcentersaroundantipsychoticmedications,butinsevereortreatment-refractorycases,electroconvulsivetherapymaybeconsidered.
IncorrectAnswers:A,B.CaandE.
Depressiveepisodesinbipolardisorder(ChoiceA)wouldpresentwithdepressedmood,anhedonia,psychomotorretardation,andneurovegetativesymptomssuchassleep,appetite,andenergydisturbances.Severedepressiveepisodesmaybeassociatedwith
mood-congruentpsychoticsymptomssuchasdelusionsofguiltasapposedtothispatient'scomplexparanoiddelusionsandauditoryhallucinations.Additionally,mentalstatusexaminationwouldlikelydemonstrateadysphoricaffectratherthanaflataffect.A
disheveledappearanceisalsomoretypicalofachronicpsychoticdisordersuchasschizophreniathanadepressiveepisodeofbipolardisorder.
Borderlinepersonalitydisorder(ChoiceB)isaclusterBpersonalitydisorder,theemotionalordramaticcluster,thatfeaturesanunstablesenseofselfandtumultuousrelationships.Thoughpatientswithborderlinepersonalitydisordermayexperiencetransient
psychosiswhenemotionallydysregulated,severaldaysofadelusionalbeliefsystemandauditoryhallucinationswouldbeatypical..Aflataffectisalsoatypicalofborderlinepersonalitydisorder.
Delusionaldisorder(ChoiceC)featuresthepresenceofoneormoredelusionsforamonthorlongerwithoutotherpsychoticsymptoms.Prominenthallucinations,negativesymptoms,andfunctionalimpairment,asseeninthispatient,wouldbeatypicalofdelusional
disorder.
Schizofypalpersonalitydisorder(ChoiceE)soneoftheclusterApersonalitydisorders,theoddoreccentriccluster.Thedisorderischaracterizedbyoddormagicalbehaviorandthinking,aswellasaconstrictedaffect.Complex,rigidparanoiddelusionsasinthis
patientwouldbeatypicalforpatientswithschizotypalpersonalitydisorder.Additionally,thispatientsnegativesymptomsandfunctionalimpairmentaremoreconsistentwithschizophrenia.
EducationalObjective:Patientswithschizophreniahavetwoclustersofsymptoms:positivesymptoms(additionofmentaldysfunctionincludingdelusions,hallucinations,anddisorganizedthinkingandbehavior)andnegativesymptoms(absenceofcertainmental
functionssuchasaffect,volition,thought,andspeech).Schizophreniasymptomsaretypicallychronicbutcanfeatureepisodicexacerbationsofbothpositiveandnegativesymptomsinthesettingofmedicationnonadherence.
© © ©
Previous Next ScoreReport LabValues Calculator Help Pause

ExamSection2:Item43of50 NationalBoardofMedicalExaminers^
ComprehensiveBasicScienceSelf-Assessment
Y
43.AviralgeneproductisfoundtodecreaseexpressionofclassiMHCmoleculesonthesurfacesofinfectedcells.Amutantstrainofvirusisisolatedthathasanonfunctionalformofthisgene.Whichofthefollowingtypesofcellsarelikelytocontributemore
effectivelytocontroloftheparentalstrainofthevirusthantocontrolofthemutantvirus?
A)CD4+-T-ymphocytes
B)CD8+Tymphocytes
C)Folliculardendriticcells
D)Naturalkillercells
E)Plasmacells
CorrectAnswer:D.
Naturalkiller(NK)cellsaremorelikelytoeradicatetheparentalstrainofthevirusasthisstraindecreasesclassiMHCexpressiononthesurfaceoninfectedcells.NKcellstargetvirallyinfectedcellsinresponsetoanabsenceofsurfaceclassIMHCmolecules.
Whenactivated,NKcellssynthesizeperforinandgranzyme.whicharebothproapoptoticThemutantstrainofthevirusdoesnotchangetheexpressionofclassIMHConvirallyinfectedcellsandisthereforeunlikelytoresultinactivationofNKcells
:effectively
leadingtoevasionofthispartoftheinnateimmunesystem.InadditiontoactivationasaresultofabsentclassEMHCmolecules
:NKcellsarealsoactivatedbyanantibody-dependentmechanisminwhichpathogensopsonizedbyimmunoglobulin(Ig)bindtoCD16
onthesurfaceofNKcells.
IncorrectAnswers:A.B
:C.andE.
CD4+-Tlymphocytes(ChoiceA)recognizeclassIIMHCmolecules
:notclassIMHCmolecules.Theyrecruitmacrophages,cytotoxicTcells,plasmacells,andeosinophils.
CD8+-Tlymphocytes(ChoiceB)induceapoptosisincellsthatexpressclassIMHC.Theywouldbemorelikelytoplayaroleincontrolofcellsaffectedbythemutantvirus
:whichdoesnotattenuatetheexpressionofclassIMHCmolecuiesonthesurfaceofinfected
ceils.
Folliculardendriticcells(FDCs)(ChoiceC)havereceptorsthatbindandendocytoseantigensviaspecificreceptors(egrGR1andCR2)toallowpresentationoftheseantigenstoBlymphocytes.8lymphocytesthatrecognizeparticu
'
arantigensareinducedto
proliferateandproduceantibodies.FDCsdonotdirectlyinteractwithTlymphocytesviaMHCmolecules.
Plasmacells(ChoiceE)areterminallydifferentiatedBlymphocytesthatsecreteaspecificimmunoglobulinagainstaparticularantigen.InteractionwithclassIIMHCmoleculesisacomponentofBlymphocyteactivation,proliferation
;anddifferentiationbutB
lymphocytesdonotrespondtoanabsenceofclassIMHCmolecules.
EducationalObjective:NKcellsareactivatedtoinduceapoptosisince:sthatdonotexpressclassiMHCmoleculesontheircellsurfaceTheparentalstrainofthevirusdescribedinducesinfectedcellstodownregulateexpressionofclassIMHCmolecules,which
wouldresultinNKcell-inducedapoptosis.
Cl00
Previous Next ScoreReport LabValues Calculator Help Pause

ExamSection2:Item44ol50 NationalBoardofMedicalExaminers^
ComprehensiveBasicScienceSelf-Assessment
Y
44.Apreviouslyhealthy40-year-oldmanisbroughttotheemergencydepartment1hourafterthesuddenonsetofseverepaininhisHeft[egwhileplayingtennis.HeisfoundtohaverupturedtheleftAchillestendonandundergoesoperativerepairandlongleg
castimmobilization.Sixmonthslatertheleftcalfshowsa2-cmdecreaseincircumferencecomparedwiththerightcalf.Whichofthefollowingisthemostlikelycauseofthisdecrease?
A)Decreasedglycogensynthesis
B)Decreasedmyosintightchainphosphataseactivity
C)Increasedphosphatidyldegradation
D)Increasedproteindegradation
E)Mitochondriadamage
F)Necrosisofmusclefibers
CorrectAnswer:D.
Musclesaredynamictissuesthatrespondtouseovertimewithhypertrophyanddisuseovertimewithatrophy.Muscledisuseatrophyoccurswhenanextremityisnon-weight-bearingforanextendedengthoftime,incasesoflimitedmobilityduetofrailty,orduring
periodsofgrossimmobilizationsuchaslongperiodsoftimeintheintensivecareunit.Inthechronicdisuse,therewillbedecreasedinputfromthemotorneuronandconsequentdecreasedmusclecontractions.Thisdiminishedactivityleadstoadecreaseinmuscle
proteinsynthesis.Typically,thereisabalancebetweenproteinsynthesisandproteindegradationinordertomaintainappropriatesarcomerearchitectureandfunctionality,however,duringdisuseatrophythereispreferentialmuscleproteindegradation.Ithasbeen
shownthattheAkt-mTORpathwayisinvolvedinattenuatedproteinsynthesisinmuscledisuse.Similarly,theubiquitin-proteasomepathwayishypothesizedtobeinvolvedintheincreasedproteindegradationindisuseatrophy.
IncorrectAnswers:A.B.C,E.andF.
Decreasedgtycogensynthesis(ChoiceA]occursintimesofrelativeenergydeficiencyandstrenuousexerciseinskeletalmuscle.Decreasedinsulinlevelsandincreasedievelsofglucagonandepinephrineduringtimesofstresssignalthebreakdownofglycogenin
skeletalmuscletoprovideglucoseforglycolyticmetabolism.
Myosinlightchainphosphatase(ChoiceB)dephosphorylatesthelightchainofmyosininsmoothmuscleleadingtorelaxation.Thisenzymedoesnotplayaroleinthepathophysiologyofdisuseatrophy.
PhospholipaseCisanenzymethatcatalyzesthehydrolysisofphosphatidylinositol4
:5-bisphosphateleadingtotheformationofdiacylglycerolThisprocessisregulatedbytheGqprotein-coupfedreceptorandplaysaroleinsmoothmusclecontraction,notskeletal
muscle.Increasedphosphatidyldegradation(ChoiceC)doesnotplayarofeinthepathophysiologyofdisuseatrophy.
Duringnormalcellrespiration,theelectrontransportchainproducessuperoxideradicalsandreactiveoxygenspeciessuchashydrogenperoxide.Underconditionsofinadequateantioxidantconcentration,thesereactiveoxygenspeciesmayleadtodamageof
mitochondria!DNAaswellasmembranelipidsandproteins(ChoiceE).Thismayeventuallyleadtoceldeathbutisnotthemainmechanismofdisuseatrophy
Necrosisofmusclefibers(ChoiceF)isseeninconditionssuchasmusculardystrophy,autoimmunemyopathies,andrhabdomyolysis.Itisnotacausalfactorindisuseatrophy.
EducationalObjective:Muscledisuseatrophyoccurswhenanextremityisnon-weight-bearingforanextendedlengthoftime,incasesoflimitedmobilityduetofrailty,orduringperiodsofgrossimmobilizationsuchaslongperiodsoftimeintheintensivecareunit.
Thepathophysiologyofdisuseatrophyisrelatedtoanimbalancebetweenproteinproductionandproteindegradation.
© © ©
Previous Next ScoreReport LabValues Calculator Help Pause

ExamSection2:Item45ol50 NationalBoardofMetlicaiExaminers^
ComprehensiveBasicScienceSelf-Assessment
45.A56-year-oldwomanhasrecentlydiagnosedcarcinomaofthebreast.Anx-rayofthechestshowsatumornexttotherightsideoftheheart.AnenhancedCTscanwiththetumorinvadingthe
pericardiumisshown.Whichofthefollowingstructuresismostlikelyinvolved?
A)Coronarysinus
B)Greatersplanchnicvein
C)Rightphrenicnerve
D)Rightvagusnerve
E)Thoracicduct
CorrectAnswer:C.
TherightphrenicnerveoriginatesfromthecervicalspinalcordatthelevelsofC3-C5
;coursesinteriorlywiththeinternaljugularvein
;andthencrossesanteriortothesubclavianarterybeforedescendingthroughthesuperiorthoracicaperturetoenterthethorax,
inthethorax,itdescendsanteriorlyalongtherightlungrootandalongthepericardiumoverlyingtherightatriumoftheheart.Itexitsthethoraxthroughtheinferiorvenacavahiatustoprovidemotorinnervationtothediaphragm.Thetumorasseeninthiscaseis
invadingthepericardiumattheleveioftherightatrium,whichcanbeidentifiedbyitsanteriorandlaterallocationinthemediastinum.TherightatriumabutstherightEungandmakesuptherightheartborderItislocatedposteriortoandtotherightofthe
sternum,whereastherightventricleholdsananteriorandinferiorposition,directlybehindthesternum.
IncorrectAnswers:A.B,D.andE.
Thecoronarysinus(ChoiceA)islocatedontheposteriorsurfaceoftheheartandrunswithintheleftatrioventriculargroovebeforedrainingintotherightatrium,litoriginatesfromtheobliqueveinoftheleftatriumandthegreatcardiacvein.Theiesioninthiscase
isanteriorandtotherightsideoftheheart.
Greatersplanchnicvein(ChoiceB)involvementisunlikelygiventhetumorlocation.Thesplanchnicvenouscirculationreferstodrainageofthegastrointestinaltractfromtheloweresophagustotheuppertwo-thirdsoftherectum,whichincludesthehepaticportal
vein,mesentericveins,andsplenicveinThesplanchniccirculationdrainsintotheliverinferiortothediaphragm.
Therightvagusnerve(ChoiceD)originatesinthemedullaandexitsthecraniumthroughthejugularforamenbeforepassinganteriortothesubclavianarteryandintothethorax.Therightrecurrentlaryngealnervebranchesoffandloopsunderneaththeright
subcfavianartery.Thevagusnervecontinuesitsdescentthroughthethoraxadjacenttotheesophagusandenterstheabdomenthroughtheesophagealhiatus.Itslocationismoreposteriorthanthelesionintheradiograph.
Thethoracicduct(ChoiceE)passesthroughtheaorticapertureofthediaphragmandascendsadjacenttothethoracicaortaandazygosvein,itdrainsintothevenoussystemnearthejunctionoftheleftsubclavianandleftinternaljugularveins.
EducationalObjective:Therightphrenicnervecoursesalongthepericardiumintheregionoftherightatrium.Infiltrativelesionsinthisareamayresultinphrenicnervedysfunction.
C O o
r- & CT
Previous Next ScoreReport LabValues Calculator Help Pause

ExamSection2:Item46ol50 NationalBoardofMetlicaiExaminers^
ComprehensiveBasicScienceSelf-Assessment
y
46.A25-year-oldwomanwithahistoryofrheumaticfeverandmitralvalvedysfunctioncomestothephysicianbecauseofa2-weekhistoryoffeverandfatigue.Sheunderwentarootcanalprocedure1month
ago,beforewhichshehadtakenasingledoseofamoxicillin.Hertemperatureis38.4X(101.2T).Agrade4/6blowingmurmurisheardonauscultationundertheleftaxilla.AphotomicrographofaGramstain
oftheorganismrecoveredfromabloodculturespecimenisshown.Onbloodagarplates,theorganismshowsalphahemolysis.Whichofthefollowingisthemostlikelycausalorganism?
A)Enterococcusfaecalis
B}GroupAbeta-hemolyticstreptococci
C)Staphylococcusaureus
D)Streptococcusmitis
E)Streptococcuspneumoniae
r
CorrectAnswer:D.
Bacterialendocarditisreferstoacuteorsubacutebacterialinfectionofthecardiacvalvularendocardium,mostcommonlyfromanacuteinfectionwithStaphylococcusaureusand/orasubacuteinfectionwithviridansstreptococci.Itcanpresentwithanarrayof
symptoms,mostcommonlyfever,newcardiacmurmur,anemia,glomerulonephritis,Rothspots
:Osiernodes
:andsplinterhemorrhagesinthenaiibedsandconjunctiva.Riskfactorsforinfectionincludepriorvalvulardamage(eg,rheumaticheartdisease),
intravenousdruguse
:immunosuppression,prostheticcardiacvalves,andcongenitalheartdisease.Themitralvalveismostcommonlyaffected,andmitralregurgitationmaypresentwithaholosystolicmurmurbestheardintheleftfourthorfifthintercostalspace
alongthemidclavicularlinewithradiationtotheleftaxilla.Viridansstreptococci,suchasStreptococcusmills,arecommensalorganismsofthehumanoropharynx.Dentalproceduresmaycausetransientbacteremia,wheretheyreachtheheartvalvesvia
hematogenousspread.Viridansstreptococciarea-hemolytic(partialorincompletelysisofredcells)coccithatgrowinchains.TheymaybedistinguishedfromStr.pneumoniaebydemonstrationofoptochin-resistanceonantibioticsensitivityanalysis.
IncorrectAnswers:A,6,C,andE.
Enterococcusfaecalis(ChoiceA)areGram-positivecoccithatnormallyresideinthecolonandmaycausediseaseinthesettingofgenitourinaryorgastrointestinaldisruption.Theydemonstrateavariablehemolyticpattern(aory)onbloodagar.Theyare
classicallyassociatedwithurinarytractinfections,biliarytreeinfections,andendocarditis.
GroupAbeta-hemolyticstreptococci(ChoiceB)causeavarietyofhumandisease,mostnotablyskinandsofttissueinfections,streptococcalpharyngitis,andrheumaticfever.Theydemonstrateap-hemolyficpattern(completelysisofredcells)onbloodagar.
Staphylococcusaureus(ChoiceC)commensailyoccupyhumanskinandthenasopharynxbuthavevirulentpotential.TheyareGram-positivecoccithattendtogrowinclusters.S.aureusisacommoncauseofacute,severeinfectiveendocarditis,bacteremia,and
post-viralpneumonia.
Streptococcuspneumoniae(ChoiceE)area-hemolytic,Gram-positivecoccithatgrowinchains.Theyarealesscommoncauseofinfectiveendocarditisthanviridansstreptococci,especiallyinthesettingofarecentdentalprocedure.
EducationalObjective:Bacterialendocarditiswithviridansstreptococciisariskfollowingdentalproceduresduetotransientbacteremia.Thepresenceofpreviouslydamagedorprostheticvalvesincreasestheriskofinfection.
o1» ®
r-
Previous Next ScoreReport LabValues Calculator Help Pause

ExamSection2:Item47ol50 NationalBoardofMedicalExaminers^
ComprehensiveBasicScienceSelf-Assessment
Y
47.A24-year-oldwomancomestothephysicianforadviceaboutcontraceptivemethods.Sheisrecentlymarriedandisnotinterestedinhavingchildrenuntilhermid30s.Whichofthefollowingcontraceptivescarriesthehighestriskforinterferencewithfertility
inthispatientlaterinlife?
A)Cervicalcapwithspermicidaljelly
B)Condomsandspermicidalfoam
C)IUD
D)Oralcontraceptive
E)Progestinimplant
CorrectAnswer:C.
AnIUDisalong-actingreversiblecontraceptiveoptionthatdoesnotrequireactivecomplianceonthepartofthepatient.Bothcopperandlevonorgestre!ILJDsareavailable.ThecopperIUDprovidesacontraceptivemethodwithouttheuseofhormones.Copperis
spermicidalandinhibitsspermmotilityandtheacrosomalreactionnecessaryforfertilization.Aswell,itinducesEocalinflammatorychangesoftheendometriumandincreasescervicalmucusproduction,furtherdecreasingthechanceofsuccessfulspermpassageor
embryonicimplantationintheendometrium.Potentialcomplicationsincludeincreasedmenstrualflowandpainduringmenses.HormonallUDsalsocausecervicalmucousthickeningandinduceglandularatrophyoftheendometrium,preventingimplantation.fUDs
areareliableformofcontraceptionwithfailureratesaslowas1%orless.Dependingonthetype,theymaybekeptinplaceforuptotenyears.TheuseofanIUDmaysEightlyincreasethechanceofinfertilitylater,butmostwomenareabletoconceiveafter
removalofthedevice.
IncorrectAnswers:A.R,D.andE.
Cervicalcapwithspermicidaljelly(ChoiceA)andcondomsandspermicidalfoam(ChoiceB)arebothformsofabarriercontraceptivecombinedwithaspermicide.Acervicalcaporadiaphragmfitsontopofthecervixandanteriorwaiofthevagina.Itmustbe
placedpriortointercourseandleftinforsixtoeighthoursafteruse.DiaphragmsandcondomsarenotaseffectiveatpreventingpregnancycomparedtoanIUDbuthavenohormonalcomponentandnoimpactonfuturefertility.Likewise,acondommustbeplaced
priortointercoursetobeeffectiveandhasnoimpactonfertility.
Oralcontraceptives(ChoiceD),whicharetakendailyareaformofhormonalcontraception.Hormonalcontraceptivesarecontraindicatedinpatientswithahistoryofthromboembolicdisorders,liverdysfunction,andbreastmalignancywithestrogenorprogesterone
receptorpositivity.Theydonotaffectfuturefertility.
Progestinimplant(ChoiceE)isalong-actinghormonalcontraceptivecontainingprogestin,whichisplacedsubcutaneouslyinthearm.(1hishasthesamecontraindicationsasanoralhormonalcontraceptiveandleavesfuturefertilityunaffected.
EducationalObjective:lUDsareoneofthemostreliableformsofcontraceptionbecausetheydonotdependonpatientcompliance
'
1heuseofanIUDmayslightlyincreasethechanceofinfertilitylaterbutmostwomenareabletoconceiveafterremovalofthe
device.Othersideeffectsincludecrampingandirregularmenstrualbleeding.
©0 & 0
Previous Next ScoreReport LabValues Calculator Help Pause

ExamSection2:Item43ol50 NationalBoardofMetlicaiExaminers^
ComprehensiveBasicScienceSelf-Assessment
y
48.Topicalcorticosteroidcreamsandphototherapyareofteneffectiveinthetreatmentofpsoriasis.Thiseffectivenesssuggestsaroleformetabolismornuclearbindingofwhichofthefollowingvitaminsinthetreatmentofpsoriaticlesions?
A)Niacin
B)VitaminB
1
(thiamine)
C)VitaminB
2
(riboflavin)
D)VitaminB
6
(pyridoxine)
E)VitaminC
F)VitaminD
G)VitaminE
H)VitaminK
CorrectAnswer:F.
Psoriasisvulgarisisacommoninflammatoryskinconditioncausedbyimmunedysregulationleadingtokeratinocyteproliferation.TheTh
17
lymphocyteiscentraltothepathophysiologyofpsoriasis,whichoccurssecondarytotheeffectsofsomecytokines
:such
asinterleukin-23andtumornecrosisfactor-o.Recentdevelopmentsinthetreatmentofpsoriasishavetargetedthesespecificimmunepathwayswithbiologicmedications.Howeverphototherapywithnarrow-bandultravioletbandlightandtopicalcalcipotriene
:a
vitaminDanalog
:arefrequentlyusedinthemanagementofpsoriasis.ActivevitaminD{1,25-OH
2
vitaminD)bindsanucleartranscriptionfactor,whichsubsequentlyimpactsgenetranscription.VitaminDplaysaroleinimmunemodulationandthusimprovesthe
immunedysregulationseeninpsoriasis.Clinically,plaque-typepsoriasisvulgarispresentswiththick,salmon-coloredplaquesontheextensorextremitieswithoverlyingsilvery,whitescale.Otherformsofpsoriasisincludepustularpsoriasispsoriaticarthritis,and
guttatepsoriasis.
IncorrectAnswers:A,B.C.D,E.G
;andH.
Niacirt(ChoiceA)isacomponentofnicotinamideadeninedinucleotideandnicotinamideadeninedinucleotidephosphate,whichareusedinvariousbiochemicalreduction-oxidationreactions.Niacinlowersserumcholesterol-VLDLconcentrationandraisesserum
HDL-cholesterolconcentration,makingitapotentialtreatmentfordyslipidemia.Jthasalsobeenusedforskincancerpreventioninat-riskindividuals.Itisnotatreatmentofpsoriasis.
Vitamin(Choice6),orthiamine,causesvariablemanifestationswhendeficient.Dryberiberipresentsasperipheralneuropathywithmentalstatuschange.Wetberiberiexhibitsadilatedcardiomyopathyleadingtoheartfailure.Wernickeencephalopathy
classicallypresentsasconfusion,ataxia,andophthalmoplegia.Adeficiencyofthiaminedoesnotcausepsoriasis
VitaminB
2
(ChoiceC),orriboflavin,isanothercofactorinreductionoxidationreactionsintheprocessofglucosemetabolism.Adeficiencyofriboflavinmaycausecheilitisandcornealvascularization.Itdoesnothavearoleinpsoriasispathogenesisor
management.
VitaminB
$(ChoiceD)
rorpyridoxine,contributestothesynthesisofhistamine,hemoglobin,andneurotransmittersincludingepinephrine,norepinephrine,dopamine,serotonin,andy-aminobutyricacid.Deficiencycommonlypresentswithperipheralneuropathy,
sideroblasticanemia,glossitis,andseizures(especiallyinisoniaziduse).Deficiencymayalsocausedermatitis,butnotpsoriasis.
VitaminC(ChoiceE),orascorbicacid,isawater-solubleantioxidantfoundinfruitsandvegetables.Deficiencyofthisvitamincausestheconstellationofsymptomsknownasscurvy,whichischaracterizedbypetechiae,perifollicularhemorrhage,bruising,poor
woundhealing,andsmall,curly,fragilehairs.Itisnotcurrentlythoughttopiayaroleinthetreatmentofpsoriasis.
VitaminE(ChoiceG)isanantioxidantthatprotectscellsfromfreeradicaldamage.Itisincludedinmanytopicalproductsintendedforskincaretohelpmitigatetheeffectsofultravioletradiation.Itdoesnothavearoleinpsoriasismanagement.
VitaminK(ChoiceH)playsacriticalroleinthesynthesisofhepaticcoagulationproteins;itisoxidizedintheliverduringthecarboxylationofglutamicacidresiduesoncoagulationfactorsII,VILIX,X.andproteinsCandS.Itdoesnotcontributetothe
pathogenesisofpsoriasis.
EducationalObjective:VitaminDbindstoitsnuclearreceptorcausingincreasedtranscriptionofimmuneregulatorygenes,itisusedinthetreatmentofpsoriasis,whichiscausedbyimmunedysregulationleadingtokeratinocyteproliferation,intheformof
phototherapyandtopicalcalcipotriene.
om m 9Iff
Previous Next ScoreReport LabValues Calculator Help Pause

ExamSection2:Item49of50 NationalBoardofMedicalExaminers^
ComprehensiveBasicScienceSelf-Assessment
Y
49.A21-year-oldwomanofJapanesedescentcomestotheemergencydepartmentbecauseofa3-hourhistoryoffacialflushing.Hersymptomsbeganshortlyaftershedrankaglassofchampagneforthefirsttimeataweddingreception.Physicalexamination
showsprofounderythemaovertheface.Themostlikelycauseofthesefindingsisamutationinthegeneforwhichofthefollowingenzymes?
A)Acetyl-CoAreductase
B)Alcoholcatalase
C)Alcoholdehydrogenase
D}Alcoholreductase
E)Aldehydedehydrogenase
CorrectAnswer:E
Facialflushingduetoalcoholconsumptioniscausedbydeficientactivityoftheenzymealdehydedehydrogenase,whichconvertsacetaldehydetoaceticadd.Deficiencyofaldehydedehydrogenaseleadstoanaccumulationofacetaldehyde
:whichstimulatestothe
releaseofhistaminefrommastcellsandcausessymptomsoffacialflushing.Inseverecases,histaminereleasemaycausesymptomsthatmimicasthma,includingwheezingandcough.Aldehydedehydrogenaseisalsothetargetofthemedicationdisulfiram,
whichutilizestheunpleasantphysicaleffectsofacetaldehydeaccumulationtotreatalcoholusedisorder.Additionalsymptomsofaldehydetoxicityincludeheadache,fatigue,nausea,vomiting,tachycardia,andhypotension.Antagonismofaldehydedehydrogenase
isalsothemechanismunderlyingthedisulfiram-likesideeffectsofdrugssuchasmetronidazole,ketoconazole,andnitrofurantoin.
IncorrectAnswers:A.B,C.andD.
Acetyl-CoAreductase(ChoiceA)andsimilarenzymesinthefamilyoffattyacylCoAreductasesactinfattyacidmetabolism.Theydonotplayaroleinthemetabolismofalcohol.
Alcoholcatalase(ChoiceB)convertsethanoltoacetaldehydeusinghydrogenperoxideasanoxygendonor.Thisisaninfrequentmechanismofalcoholmetabolismascomparedtoalcoholdehydrogenaseandoccursinperoxisomes.Thisstepprecedesthe
conversionofacetaldehydetoaceticacidbyaldehydedehydrogenase.
Alcoholdehydrogenase(ChoiceC}playstheprimaryroleinthemetabolismofalcoholssuchasethanol,methanol,andethyleneglycol.Alcoholdehydrogenaseisthetargetofthedrugfomepizole,whichisusedtotreatmethanolorethyleneglycoJpoisoning.
Alcoholreductase(ChoiceD)mayrefertoaromaticalcoholreductase,agenethatreducesalcoholgroupsassociatedwitharomatichydrocarbons.Itisnotsignificantinthepathophysiologyoffacialflushinginaldehydemetabolism.
EducationalObjective:Alcohol-associatedfacialflushingiscausedbyadeficiencyofaldehydedehydrogenase,whichconvertsacetaldehydetoaceticacid.Deficiencyofaldehydedehydrogenaseeadstoaccumulationofacetadehyde,whichstimulatestothe
releaseofhistaminefrommastcells.
© © © oo
Previous Next ScoreReport LabValues Calculator Help Pause

ExamSection2:Item50ol50 NationalBoardofMedicalExaminers^
ComprehensiveBasicScienceSelf-Assessment
Y
50.A35-year-oldwomanisbroughttotheemergencydepartmentaftershesustainsafractureoftheneckofthefibulaofherrightleg.Shewasstruckbyacarwhitecrossingthestreet.Whichofthefollowingfindingsismostlikelyonexaminationoftheaffected
leg?
PainOver
ProximalFibula
PainOver
DistalFibulaDorsiflexionPlantarFlexionAchillesReflex
absent
present
absent
A) Absent 4/5 1/5 1+
B) Absent 0/5 4/5 1+
C) Present 1/5 4/5 2+
D) Present absent
present
5/5 0/5 absent
E) Present 4/5 4/5 2+
CorrectAnswer:C.
Therearenumberofkeystructuresalongthelateralaspectofthelowerlegthatcanbedamagedduringtrauma.Thefibulaprovideslateralsupportandstructuretothelowerleg.Thecommonperonealnerveislocatedsuperficially,justdistaltotheheadofthe
fibulabelowthekneejoint.Thisnervebranchesintothesuperficialperonealnerveanddeepperonealnerveasittravelsinthelateralcompartmentofthelowerleg.Thesuperficialperonealnerveprovidesmotorinnervationtoperoneusbrevisandlongus,aswef;
assensoryinnervationtothedorsolateralaspectofthefootThedeepperonea]nerveprovidesmotorinnervationtotheankledorstffexorsaswellassensoryinnervationtoThefirstdorsalwebspaceofthefoot.Otherimportantstructuresofthelaterallowerleg
includethelateralorfibularcollateralligamentwhichprovidesstabilitytothekneeaswellasthesyndesmosisligaments,whichsupportthedistaltibiofibularjoint,providingstabilitytotheankle.
Inthiscase,thepatienthasafractureofthefibularneck
:whichislocatedattheproximalaspectofthefibula,justdistaltothefibularheadandadjacenttothecommonperonealnerveasitwrapsaroundthefibula.Injuryinthislocationtypicallypresentswithpain
attheproximalfibulaandweaknessandnumbnessinthedistributionoftheperonealnerve.Withthisinjury,painoverthedistalfibulawouldnotbepresent.Functionofthegastrocnemiusandsoleusmuscleswouldbeunaffectedasthesemusclesareinnervated
bythetibialnerve,whichisintheposterioraspectoftheEowerleg(poplitealfossa)leadingfoanintactAchillesreflex.
IncorrectAnswers:A.BfD,andE.
Plantarflexionweaknesswithoutbonypaininthelowerleg(ChoiceA)mayrepresentacompressiveesionofthetibialnerveorcompressionoftheS1sacralnerveroot.1heseinjuriesarenotconsistentwiththeinjurymechanismofthispatient.
Painoverthedistalfibulaandweaknessindorsiflexion(ChoiceB)canbeseeninapenetratingtraumaticinjurytothefibulathatalsoseveredtheextensortendonsofthetoesanddorsiflexorsoftheankle.Suchaninjuryisunlikelytoleadtoseveringofthemotor
nervesoftheanteriorcompartmentasthemusclebelliesofthesemusclesareproximaltotheleveloftheinjury.
Painovertheproximalfibulawithweaknessinplantarflexion(ChoiceD)isanuncommoninjurypatternastheproximalfibulaisnotlocatednearthetibia!nerve.Although,itshouldbenotedthatinhighenergyinjuriessuchashighway-speedmotorcycleaccidents,
massivedamagetobone,softtissue,andnervesmayoccurleadingtoinjuriestostructuresthatarespatiallyseparated.
Atraumaticinjurycausingpainovertheproximalfibulaanddistalfibula(ChoiceE)wouldbeconcerningforaMaisonneuveinjury,whichconsistsofaninjuryofthedistaltibiofibularsyndesmosis,typicallyassociatedwithafractureofthemedialorlateralmalleolus,
combinedwithaninjuryoftheproximalfibularshaftorneck.
EducationalObjective:Aninjurytothelaterallowerextremitycanresultinaproximalfibularfracture.Thecommonperonealnervewrapsaroundthefibularneckasitentersthelateralcompartmentofthelowerleg.Injurytothisareacanleadtoperonealnerve
injuryandaconsequentfootdrop.
© ©
*
r- ©
Previous Next ScoreReport LabValues Calculator Help Pause

ExamSection3:Item1of50 NationalBoardofMedicalExaminers^
ComprehensiveBasicScienceSelf-Assessment
Y
1.AG5-year-oldmancomestothephysicianbecauseoffeverandaworseningcoughproductiveofapproximately%cupofblood-tingedsputumdaily.Hehassmoked1.14packsofcigarettesdailyfor45years.Histemperatureis38.4X(1012aF)
:respirations
are20/min.andbloodpressureis140/90mmHg.Physicalexaminationshowsbilateralclubbingofthedigits.Pulmonaryexaminationshowsegophony,whisperedpectoriloquy,anddullnesstopercussionintheareaoverlying"he2ndand3rdribsontheright
anteriorlyjustinferiortotherightclavicle.Whichofthefollowingstructuresisthemostlikelysiteofanobstructingcarcinoma?
A)Maincarina
B)Rightlowerlobebronchus
C)Rightmainbronchus
D)Rightmiddlelobebronchus
E)Rightupperlobebronchus
CorrectAnswer:E
Thegrosslobaranatomyofthelungcontainsthreelobesoftherightlung(uppermiddle,andlower)andtwolobesoftheleftlung(upperandlower).Thelingulaisadistinctprojectionoftheupperlobeoftheleftlungandishomologoustothemiddlelobeoftheright
lung.Therightupperlobeliesdeeptotheanterior2
nd
and3
rd
ribsontherightsideofthethoraxjustinferiortotheclavicle,andtherightupperlobebronchusisthemostlikelysiteofthispatient
'sobstructingcarcinoma.Riskfactorsforallmajortypesoflungcancer
includetobaccouse.secondhandsmoke,asbestosorradonexposure,andafamilyhistoryoflungcancer.Lungcancer,ingeneral,typicallypresentswithcough,unintentionalweightloss,hemoptysis,chestpain,dyspnea,andhoarseness.Occasionally,wheezing,
focalrhonchi,orhypertrophicosteoarthropathymaybenotedonexamination.Amassthatoccludesasegmentoftheairwayincreasestheriskforpost-obstructivepneumonia.Onphysicalexamination,thismanifestswithegophony,whisperedpectoriloquy,
increasedtactilefremitus,anddullnesstopercussionintheregionofthepneumonia,whichinthispatient,correlateswiththerightupperlobe.Definitivediagnosisismadebychestimagingandbiopsy.Prognosisisafunctionofthecancertypealongwithgrading
andstagingofthedisease.ItFSoftendetectedoncemetastatic,atwhichpointtheprognosisispoor.
incorrectAnswers:A.B,C,andD.
Themaincarina(ChoiceA)marksthedivergenceofthetracheaintotherightandleftmainstembronchi,itistypicallylocatedatfhelevelofthefourththoracicvertebralbody.Anobstructingmassinthislocationmayresultincompleteairwayobstructionofboth
lungs.
Therightlowerlobebronchus(Choice6)branchesfromthebronchusintermediusandsuppiestherightlowerlobe,whichoccupiesmostoftheinferiorandposterioraspectsoftherighthemithorax.Apost-obstructivepneumoniainvolvingtherightlowerlobewould
bebestappreciatedbythepresenceofpneumoniaexamfindingsalongtheanterior-inferiororposteriorchestwall.
Therightmainbronchus(ChoiceC)branchesfromthetracheaatthecarinaandliesatthelevelofthefifththoracicvertebralbodyalongtherightparasternalborder.Anobstructingmassoftherightmainbronchusmayresultincompleterightlungcollapse.
Therightmiddlelobebronchus(ChoiceD)branchesfromthebronchusintermediusandsuppliestherightmiddlelobe,whichformsatriangularwedgeintheanterior-medialrighthemithorax.
EducationalObjective:Post-obstructivepneumoniaisacommoncomplicationofmassesthatoccludethelargeandmedium-sizedairways.Localizationofthepneumoniaandpotentialunderlyingmaybepossibleonphysicalexamination,althoughdefinitive
diagnosisisachievedwithchestimagingfollowedbyabiopsyofthemass.
© © ©
Previous Next ScoreReport LabValues Calculator Help Pause

ExamSection3:Item2of50 NationalBoardofMedicalExaminers^
ComprehensiveBasicScienceSelf-Assessment
Y
2.A43-year-oldwomancomestothephysicianbecauseofa4-weekhistoryofintermittentepisodesofsevereabdominalpain
;especiallyaftereatingfattyfoods.Physicalexaminationshowsnoabnormalities.Abdominalultrasonographyshowsseveral
3-to4-mmgallstonescontainedwithinthegallbfadder.Thepatientdeclinessurgery.Adrugwithwhichofthefollowingmechanismsofactionismostappropriateforthispatient?
A)Decreasedgallbladdermusclecontractility
B)Increasedgastrinproduction
C)Increasedmetabolismofbilesalts
D)Inhibitionofbiliarycholesterolsecretion
E)InhibitionofcytochromeP450activity
CorrectAnswer:D.
Inhibitionofbiliarycholesterolsecretionisachievedwithursodeoxycholicacid(ursodiol),amedicationusedtotreatbiliarycolic
:primarybiliarycirrhosis,andintrahepaticcholestasisofpregnancy,inpatientswhohaveatypicalsymptomsthatarenotdefinitively
attributabletogallstones,improvementwithursodiolsuggeststhatcholecystectomymaybebeneficial,butinpatientswithclassicbiliarycolicasinthispatient,ursodiolisconsideredasecondlinetosurgery.Jtworksbyreducingcholesterolsecretionfromthetiver
by40-60%,andtherebyreducingthecholesterolcontentofthebile.Italsoreducestheamountofcholesterolabsorbedinthegut.Asmostgallstonesaremadeofcholesterol,withaminorityformedfromhemeasaresultofchronichemolysis,graduallyreducingthe
amountofcholesterolinthebilepreventsformationofnewcholesterolgallstones.
IncorrectAnswers:A.B,C.andE.
Decreasedgallbladdermusclecontractility(ChoiceA)isafeatureofmanymedicationsincludingatropine(cholinergiceffect),somatostatin,calciumchannelblockers,ondansetronandopiates.Thesemedicationsarenotusedtotreatcholelithiasis.
Increasedgastrinproduction(ChoiceB)isacharacteristicofmedicationsthatincreasethepHofthestomach,includingproton-pumpinhibitorsand
^
-blockingmedicationsGastrinsecretionisunderinhibitorycontrolbygastricpHandsecretionislowwhenthepH
islow.ExcessgastrinsecretionisalsoadefiningfeatureofZollinger-Ellisonsyndrome.
Increasedmetabolismofbilesalts(ChoiceC)isachievedwiththeuseofbileacidsequestrantssuchascholestyramine.Thisclassofmedicationsformsanonabsorbablecomplexwithbileacidsinthegastrointestinaltractandfacilitatestheirexcretioninthestool.
InhibitionofcytochromeP450activity(ChoiceE)istheresultofnumerousmedicationsandcansometimesaccountforseveremedicationinteractionsasinhibitionofthisenzymeaffectsthemetabolismofcertaindrugs.CytochromeP450referstonumerous
subtypesofenzymes,includingCYP3A4andCYP2D6.CommonmedicationsknowntointeractwiththisclassofenzymesIncludefluconazole,ciprofloxacin,andfluoxetine.
EducationaObjective:Ursodioldecreasesthecholesterolcontentofbileandtherebypreventstheformationofcholesterolgallstones.Itisconsideredsecondlinetherapywhencomparedtocholecystectomyinpatientswithcholelithiasisandbiliarycolic.
© © ©
Previous Next ScoreReport LabValues Calculator Help Pause

ExamSection3:Item3of50 NationalBoardofMedicalExaminers^
ComprehensiveBasicScienceSelf-Assessment
Y
3.A24-year-oldwomancomestothephysicianforanexaminationpriortoemployment.Physicalexaminationshowsnoabnormalities.AnEGGshowsasinusrhythmof70/minPRintervalof152msecandQRScomplexof84msec:therearenoectopic
beats.Whichofthefollowingbestrepresentsthelongestconductiontimeinthispatient?
A)Anteriorleftbundlebranch
B)Intra-atrial
C)LowrightatriumtobundleofHis
D}ProximalbundleofHistoventricularmyocardium
E)Rightbundlebranch
CorrectAnswer:C.
Thenormalelectricalconductionpathwayofheartstartsinthesinoatrial(SA)nodewithspontaneousslowdepolarizationoftheSAnodalpacemakercellslocatedinthesuperiorrightatrium.TheatriathendepolarizewhichresultsintheP-waveontheEGG.The
actionpotentialspreadstotheatrioventricular(AV)node
;wherethesignalisslowedandconductedtothebundleofHiswhichrunsalongtheinterventricularseptum.SlowingofconductionthroughtheAVnodeallowstheatriatofullycontractandcompletefillingof
theventriclespriortosystole.TheactionpotentialisconductedalongfascicularbranchesandPurkinjefiberstostimulatethecoordinatedcontractionofventricularcardiomyocytes.DepolarizationoftheventriclescreatestheQRScomplexontheEGG.ThePR
intervalrepresentsthetimefrominitialdepolarizationoftheatriatothestartofventriculardepolarization.Thenormalrangeis120-200msec.ConductiontimefromthelowrightatriumtothebundleofHisthereforetakesthelongestduetothephysiologicdelayin
conductionthroughtheAVnode.
IncorrectAnswers:A.R,D.andE.
Conductionthroughtheanteriorleftbundlebranch(ChoiceA),proximalbundleofHistoventricularmyocardium(ChoiceD)
;andrightbundlebranch(ChoiceE)israpidandoccurswithintheinitialperiodoftheQRScomplexinorderforcoordinatedcontractionof
theventriclestooccur.Delaysintheseconductionpathwaysmaybeduetostructuralabnormalities,fibrosisfromischemicorinflammatoryinjury,toxic-metabolicsyndromes,electrolytederangements,ornormalanatomicvariants,andappearasawidenedQRS
complex(>120msec)ontheECG.
Intra-atrial(ChoiceB)conductionisrepresentedbythedurationoftheP-wave.Conductionisrapidinorderforcoordinatedatrialcontractiontooccur.
EducationalObjective:TheAVnodedelaystheelectricalimpulseconductedfromtheatriainorderforatrialcontractiontocompleteandtheventriclestofillentirelypriortoventriculardepolarization.
00# 0f*
Previous Next ScoreReport LabValues Calculator Help Pause

ExamSection3:Item4of50 NationalBoardofMedicalExaminers^
ComprehensiveBasicScienceSelf-Assessment
Y
4.An18-year-oldwomanwithmildintellectualdisabilityisbroughttothephysicianbecauseofa3-dayhistoryofdecreasedabilitytoseeinreducedfight.Shehasalifelonghistoryofchronicdiarrhea.Twoyearsago.shedevelopedalackofmusclecontrolof
herarmsandlegs,andgeneralizedweakness.Her16-year-oldbrotherhashadsimilarsymptoms.Ophthalmologicexaminationshowsbilateralretinitispigmentosa.Thereisataxiaandlossofdeeptendonreflexes.Laboratorystudiesshowerythrocyteswith
spinyprojectionsandaserumtotalcholesterolconcentrationof40rng/djlWhichofthefollowingapolipoproteinsismostlikelydeficientinthispatient?
A)ApoA4
B)ApoA-lll
C)ApoB
D}ApoG
E)ApoE
CorrectAnswer:C.
ApoBdeficiencyresultinginabetalipoproteinemia,arareautosomalrecessivedisorderthatiscausedbyamutationinthemicrosomaltriglyceridetransferprotein(MTPjgene,isthemostlikelyexplanationforthispatient'ssymptoms.Mostmutationsresultin
reducedfunctionorcompleteabsence.AsMTPisrequiredfortheassemblyandsecretionofApoBintheliverandgastrointestinaltract,anabsenceofthisproteinresultsinanabsenceofApoB.ApoBisanecessarycomponentofintermediate-densitylipoprotein
(IDL)
:VLDL.andLDL,sodeficiencyresultsinaninabilitytoassemblethesecriticallipoproteinparticles.Thiscausesseveremalabsorptionofdietaryfatsandfat-solublevitaminssuchasvitaminA
:D,E.andK.Symptomsarepresentininfancyasfailuretothrive
andmentalretardation.Subsequentsymptomscanincludeperipheralneuropathies,retinitispigmentosaleadingtocompleteblindness,andmuscularweakness
.Additionally,chronicmaiabsorptivediarrheaischaracteristicandisacomponentofthe
pathophysiologyofthisdisease.
IncorrectAnswers:A.6
:D.andE.
ApoA-1(ChoiceA)isrequiredforassemblyofHDLandactsasaligandforABCAlbinding.DeficienciesresultinabsentHDL,cornealopacities,xanthomas,andearlyatherosclerosis.ApoA-Jt(ChoiceB)isalsoacomponentofHDL.Deficiencywouldresultina
reducedamountofcirculatingHDL.Neitherdeficiencyresultsinthesymptomsofthispatient.
ApoC(ChoiceD)consistsofthreeseparateproteins:C-LC-ll
:andC-lll.DeficiencyofO-llresultsinhyperlipoproteinemiatypeIB.Clinicalmanifestationscloselyresemblelipoproteinlipasedeficiencywithprofoundhypertriglyceridemia.
ApoE(ChoiceE)deficiency,suchaslimitedApoE2.canleadtodysbetalipoproteinemiaasaresultofreducedclearanceofVLDLandchylomicronswhereasApoE4deficiencyresultsinhypercholesterolemiaandprematureatheroscleroticheartdisease.
EducationalObjective:ApoBisanessentialcomponentofLDL
;1DL
;andVLDLItisrequiredforthenormalabsorptionandtraffickingoflipidstoandfromtheintestinesandtheliver.MutationsintheMTPgeneleadtoabsentorreducedlevelsofApoB,with
consequentchronicmaiabsorptivediarrhea,fat-soiublevitamindeficiencies,andneurologicmanifestationssuchasperipheralneuropathy,retinitis,andmentalretardation.
© © © 0
*
0*
Previous Next ScoreReport LabValues Calculator Help Pause

ExamSection3:Item5of50 NationalBoardofMedicalExaminers^
ComprehensiveBasicScienceSelf-Assessment
Y
5.Aninvestigatorisstudyingobesityinanadolescentpopulation.Fivethousandnormal-weightpatientsarefollowedfromtheageof10yearstotheageof15years.Attheconclusionofthestudy,1100patientsmeetthecriteriaforobesity.Whichofthe
followingbestrepresentstheincidenceper1000patient-yearsofobesityduringthecourseofthisstudy?
A)10
B)25
C)44
D)50
E)64
CorrectAnswer:C.
Incidencerateisdefinedasthenumberofnewcasesofdiseasedividedbythetotalpopulationatriskperunitoftime.Itisanimportantepidemiologicmeasurethatprovidesanunderstandingofhowquicklythenumberofnewcasesofadiseaseareoccurring.In
thiscase
;theincidencerateofobesityper1000patientyearswouldbecalculatedasfollows.1100casesofobesitydividedby5000patientsatrisk
;dividedbythe5-yeardurationofthestudyperiod(1100cases/5000patientsatrisk/5years=0.044
cases/patient/year).Thisnumbercanthenbemultipliedby1000inordertoconverttheunitsfromcasesperpatientperyeartocasesper1000patient-years(0.044x1000=44casesper1000patient-years).
IncorrectAnswers:A,0D,andE.
Eachofthesenumbers(ChoicesA.B
;D
;andE)representerroneouscalculationsortheuseofrandom
:incorrectnumbersthatdonotrepresenttheappropriatecalculationoftheincidencerate.
EducationalObjective:Incidenceper1000patient-yearsisacommonmethodofpresentingincidencedata.Itcanbecalculatedbydividingthenumberofnewcasesbythetotalnumberofpersonsinthepopulationatriskdividedbythestudydurationinyears
:and
thenmultiplyingby1000.
© © © r- &
Previous Next ScoreReport LabValues Calculator Help Pause

ExamSection3:Item6of50 NationalBoardofMetlicaiExaminers^
ComprehensiveBasicScienceSelf-Assessment
y
—nr- m
«
*
o
*
iftew_
* I
i
J #r
6.A23-year-oldwomanat32weeks'gestationconiestothephysicianbecauseofa4-dayhistoryoffeverandbackpain.Shesaysthatduringthistimeshealsohasbeencryingfrequently.Hertemperatureis38CC
(1GG.4°F).Physicalexaminationshowscostophrenicangletenderness.Thephotomicrographshownrepresentsherdisease.Whichofthefollowingmechanismsisthemostlikelycause?
MAI *
H i
#
ir
vi^
I
v-
.ii
l
*
J
r
«i
a
AY*-
J!
*
***
r
*
.t
it#
*
*
vJn? V:
Wii
*
2
i
T0
>
v.
'
s

.
*
A}Chorioamnionitis
B)Endometritis
C)Glomerulonephritis
D)Hematogenousinfection
E)Obstructiveuropathy
F)Pelvicinflammatorydisease
r
Jtfr
!
"4
»
-
V
%
-
r
"
P
'
4.
*’MLia
’^
"4Mf
*JJ
*
li-
Ok
I
-
l'
V
p
M
m>1
*ri*-
Cl#?.-VrJ
--
*
I
«-
m
^
5v
i
5
r
* *v *

i
r
r
|*
r
w
*
i
f
'
l
m.
'pi
*-
;»V-
r
*
*.
47«

£
.-
V
H
*
^
I#
«*
#
%
j
1
?'
I'-HI®
i-it:
*
»
04
J
\m
CorrectAnswer:E.
Feverchills,andflankpainareclassicfindingsinthediagnosisofpyelonephritis,aninfectionofthekidneythatcommonlyresultsfromretrogradespreadofaurinarytractinfection.Urinarytractinfectionsaremuchmorecommoninwomenduetotheshorter
urethraandfavorableregionalenvironmentforbacterialgrowth.Pregnantpatientsareathigherriskforurinarytractinfectionsincludingpyelonephritisduetothestagnationofurinefromprogesterone-induceddecreasedureteraltoneandmotility,alongwith
mechanicalcompressionoftheuretersatthepelvicbrim,bladderandureteralorificesbythegraviduterus,resultinginobstructiveuropathy.Ifuntreated,bacteriacantraveluptheuretertoinfectthekidney,resultinginpyelonephritis.Pyelonephritistypically
presentswithdysuria,flankpain,andassociatedsystemicsymptomssuchasfever,rigors,nausea,vomiting,myalgias,arthralgias,andfatigue.Onexamination,patientswithpyelonephritistypicallydemonstratehighfevers,tachycardia,leukocytosis,and
costovertebralangletenderness.Urinalysisinpyelonephritisoftenshows>10leukocytesperhighpowerfield,mayshowredcellsasevidenceofmucosalinflammation,andmayshowwhitebloodcellcasts.Bacteriaaregenerallyseenonmicroscopy,withGram-
negativerodsbeingthemostcommonpathogen.
IncorrectAnswers:A,B.C,D,andF.
Chorioamnionitis(ChoiceA)isabacterialinfectionofthefetalmembranesandcommonlyoccursinthesettingofprematureorprolongedruptureofmembranes.Maternalfeverandleukocytosisarecharacteristic,andtheuterusmaybefendertopalpation
Endometritis(ChoiceB)isanacute,typicallypolymicrobial,infectionoftheuterineendometriuminvolvingamixtureofaerobesandanaerobesfromthegenitaltract.Duringlaboranddelivery,microbesfromthegenitaltractcanentertheuterinecavityandresultin
infection.Patientstypicallypresentwithfever,severetendernessoftheuterinefundus,andmucopurulentvaginaidischarge.
Glomerulonephritis(ChoiceC)referstoavarietyofglomerulardiseases,includingnephriticandnephroticsyndromes.Nephriticsyndromestypicallypresentwithacuterenalfailureassociatedwithhematuria,redbfoodcellurinecasts,andhypertension.Nephrotic
syndrometypicallypresentswithexcessiveproteinuria(>3g/day)hyperlipidemia,hypoalbuminemia,andedema.Itwouldnottypicallycauseureteralobstruction.
Hematogenousinfection(ChoiceD)canresultfrompyelonephritisorotherinfectionsthatspreadintothebloodstream,causingsepsis.Patientswithsepsiswouldtypicallypresentwithtachycardia,systemicsymptoms,andmaybeill-appearing.
Pelvicinflammatorydisease(ChoiceF)canbecausedbysexuallytransmittedinfectionssuchasChlamydiatrachomatisandNeisseriagonorrhoeae.Ittypicallypresentswithfever,mucopurulentcervicadischarge,andcervicaloradnexaltenderness.Adnexal
tendernessmaybepresentincasesoftubo-ovarianabscess.
EducationalObjective:Pyelonephritistypicallypresentswithfevers,nausea,vomiting,andflankpainwithassociatedcostovertebralangletendernessonphysicalexamination.Thismostcommonlyoccursfromascendingurinarytractinfections.Pregnantpatients
areatahigherriskforpyelonephritisduetothestagnationofurinefromprogesterone-induceddecreasedureteraltoneandmotility,alongwithmechanicalcompressionoftheuretersatthepelvicbrim,bladderandureteralorificesbythegraviduterus.
r© m
m onIE
Previous Next ScoreReport LabValues Calculator Help Pause

ExamSection3:Item7of50 NationalBoardofMedicalExaminers^
ComprehensiveBasicScienceSelf-Assessment
Y
7.A54-year-oldmanwitha1-yearhistoryofmultiplesclerosishasincreasinglypainfulmusclespasms.Anagonistthatactsatwhichofthefollowingreceptorsismostlikelytoalleviatetheincreasedextensortoneandclonus?
A)y-Aminobutyricacid
B)Histamine-1(H
^
)
C)Histamine-2(HJ
D)Muscarinic-1(M,)
E)Muscarinic-2(Mg
F)Nicotinic
CorrectAnswer:A.
E
y-aminobutyricacid(GABA)
E
receptorsaremetabotropicG-proteincoupledreceptorsthatincreaseeffluxo!potassiumandthushyperpolarizeskeletalmusclecellsanddecreaseactionpotentialfrequency.BaclofenisaGABAanalogthatactsasanagonistat
GABA-
B
receptorsandincreasesmusclerelaxation.Therefore,baclofenisafirst-linetreatmentformusclespasticityinmultiplesclerosisandfollowingneurologicinjury.GABA-
^
receptorsarealsofoundinthecentralnervoussystem(CNS):therefore,baclofencan
causesedation,especiallywhenusedinconjunctionwithotherCNSdepressants.
IncorrectAnswers:B.C
:D
:E.andF.
Histamine-1(h
^
)(ChoiceB)receptorsareinvolvedinacuteinflammatoryreactions
:sedation,andallergicresponsesandaredistributedwidelyirtthebrain,vascularendothelium,andsmoothmuscleintheheartandlungs.H-
f
agonismresultsinincreasedcentral
pain,vascularpermeability,andnasalandbronchialmucusproductionalongwithpruritus.Histamine-2(H
2
)(ChoiceC)receptoragonismresultsinincreasedgastricacidsecretion.H-jandH
2
receptorsarenothighlyexpressedonskeletalmuscle.
Muscarinic-1(M
^
(ChoiceD)receptorsareprimarilyfoundiintheCNSandentericneivoussystem.MireceptoragonistsareunderinvestigationinAlzheimerdementia.M
1
receptorsarenotexpressedonskeletalmuscle
Muscarinic-2(M
2
)(ChoiceE)receptorsareprimarilydemonstratedbysinoatrialandatrioventricularnodalcells.M
2
receptoragonistsincreasepotassiumeffluxanddecreaseheartrate.M
2
receptorsarenothighlyexpressedinskeletalmusclecells
Nicotinic(ChoiceF)acetylcholinereceptorsarehighlyexpressedonskeletalmuscleandrespondtoacetylcholinereleasebylowermotorneurons.Varenicline,whichtargetstobaccodependence,isanicotinicacetylcholinereceptoragonist.Asnicotinic
acetylcholinereceptorstimulationeadstomusclecontraction,nicotinicacetylcholinereceptoragonistswouldnotbehelpfulforchronicmusclespasticity.
EducationalObjective:Baclofenisa.GABAanalogthatactsasanagonistatGABA-greceptors,whichhyperpolarizesmusclecellsandrelievesmusclespasticity.
0 0 #
Previous Next ScoreReport LabValues Calculator Help Pause

ExamSection3:Item8of50 NationalBoardofMetlicaiExaminers^
ComprehensiveBasicScienceSelf-Assessment
y
8.A27-year-oldprimigravidwomanat36weeks'gestationcomestothephysicianforaprenatalvisitPhysicalexaminationshowsauterusconsistentinsizewitha33-weekgestation,sothepatientisreferredforultrasonography,whichshowsnormalfetal
measurements.Theultrasonographyshowrnisanimageofthemalefetalscrotum:thetestesareindicatedbythearrows.Whichofthefollowingisthemostlikelyunderlyingcauseofthisfinding?
A)Highattachmentofthetunicaalbuginea
B)Lackofscrotalattachmentofthegubernaculum
C)Obstructionoftheposteriorurethra
D)Patentprocessusvaginalis
E)Twistingofthespermaticcord
CorrectAnswer:D.
Theultrasonographicfindingsdemonstratetestesthataresurroundedbyhypoechoicfluid,consistentwithadiagnosisofhydroceleinthefetus.Thetestesdescendfromtheabdomenintothescrotumbypassingthroughtheanteriorabdominalwalland
processusvaginalis,guidedbythegubernaculum.Hydroceleresultsfromapatentprocessusvaginalisthatdoesnotobliteratefollowingdescentofthetestis,allowingperitonealfluidtocollectinthescrotalspace.Whiletypicallyunilateral,casesmaybebilateral.
Mostcasesresolvespontaneouslypriortobirth.Persistentcasesmayrequiresurgicalresection.
IncorrectAnswers:A,B
(C.andE.
Highattachmentofthetunicaalbuginea(ChoiceA)whenreferencingthefibroustissuethatinveststhetestis(thepenisalsocontainsatunicaalbuginea),resultsinincreasedmotilityofthetestisandspermaticcordwithinthetunicavaginalis,whichmayleadto
testiculartorsion.
Lackofscrotalattachmentofthegubernaculum(ChoiceB)mayleadtoanundescendedtestisduetoinabilityofthegubernaculumtoguidethetestisthroughtheinguinalcanaltothescrotum.Ultrasonographywouldnotdemonstratethetestesinsidethe
scrotum,asisfoundinthisfetus.
Obstructionoftheposteriorurethra(ChoiceC}wouldresultinoligohydramniosandPottersequence.Fluidwouldnotaccumulateinthescrotalspace.
Twistingofthespermaticcord(ChoiceE)wouldresultininuterotesticulartorsionandinfarctionofthetestes.Bilateralinuterotesticulartorsionisrare:apatentprocessusvaginalisleadingtohydroceleismorelikely.However,itwouldpresentwithalarge
accumulationoffluidwithinthescrotum.
EducationalObjective:Fetalhydroceleresultsfrompatencyoftheprocessusvaginalis,whichallowsperitonealfluidtocollectinthescrotum.Ultrasonographydemonstratesscrotalaccumulationofhypoechoicfluid,whichmaybebilateral.Mostcasesresolve
spontaneously.
©0
t
* *
if
*
Previous Next ScoreReport LabValues Calculator Help Pause

ExamSection3:Item9of50 NationalBoardofMedicalExaminers^
ComprehensiveBasicScienceSelf-Assessment
Y
9.A93-year-oldwomandiesofrespiratoryfailure.Atautopsy
;grossexaminationshowsasmallheartwithabrowndiscoloration.Histologicexaminationofthecardiactissuebyfightmicroscopyshowslightbrownpigmentwithinthecardiacmyocytes,
particularlyintheperinuclearregion.Thismaterialdoesnotstainpositivelyforiron.Whichofthefollowingbestdescribesthispigment?
A)Anthracoticpigment
B)Freefattyacids
C)Hemosiderin
D)Laminin
E)Lipofuscin
CorrectAnswer:E
Lipofuscinisayellow-brownpigmentthatisdepositedwithtimeandindicateswearonthetissues.Itisassociatedwithnormalagingandanautopsyofanagedindividual,asinthiscase
;willdemonstratelipofuscindepositsinmanyorgans
:Includingtheheart
colon
;liver,andkidney.Lipofuscinisformedbyoxidationoftheorganellarmembranes,whichhavebeenautophagocytosed.Anotherbrownpigmentthatmaybeseenintissuesishemosiderin.Hemosiderinispigmentfromironthatdepositsintissuesafter
hemorrhageorerythrocyteextravasation.Inhemochromatosis,aconditionofincreasedirondeposition,hemosiderinisfoundonliverbiopsy.Hemosiderinandlipofuscincanbedifferentiatedfromeachotherbyironstaining.Hemosiderinstainspositivelyforiron
whilelipofuscindoesnot.
IncorrectAnswers:A.B,C.andD.
Anthracoticpigment(ChoiceA)issecretedbypigment-producingfungi.Italsopresentsascarbonaceousdepositsinthelunginpulmonaryanthracosis.Pigment-producingfungiarerarebutinduceinfectionstikephaeohyphomycosisandchromoblastomycosis.
Theseorganismsareendemicinsoilandcausesuperficialcutaneousinfections.
breefattyacids(ChoiceB)mayappearyellowwhentheyaredepositedtogetherintheskin,suchasinxanthelasma.However,theywouldnotdemonstrateanyyelloworbrowncolorationonlightmicroscopy.
Hemosiderin(ChoiceC)isanirondepositedinthetissuesandthuswillstainpositivelyforirononhistopathologicexamination.Itwouldnotbeexpectedtobepresentwithincardiacmyocytesoutsideofhemochromatosis.
Laminin(ChoiceD)isaproteinfoundincellmembranesandhemidesmosomes .Lipofuscinisformedbyoxidationoforganellarmembranes,whichmayincludelaminin,howeverlamininalonedoesnotappearyellow-brownonlightmicroscopy.
EducationalObjective:Lipofuscinisayellow-brownpigmentthatindicateswearonthetissuesandispresentinmostorgansonautopsyofanagedindividual.Itisderivedfromorganellarmembranes,notiron.
00 &
Previous Next ScoreReport LabValues Calculator Help Pause

ExamSection3:Item10ol50 NationalBoardofMedicalExaminers^
ComprehensiveBasicScienceSelf-Assessment
Y
10.A43-year-oldwomanhashadincreasingfatigue,intolerancetocold
:anddrythickenedskinfor7weeks.Thethyroidglandisfirm,nodularanddiffuselyenlarged.Examinationoftissueobtainedonbiopsyofthethyroidglandshowsadiffuseinfiltrationof
lymphocyteswithoccasionallymphoidfollicles.Whichofthefollowingisthemostlikelydiagnosis?
A)Chronicautoimmune(Hashimoto)thyroiditis
B)Diffusetoxicgoiter(Gravesdisease)
C)Granulomatousthyroiditis
D)Papillarycarcinoma
E)Toxicmultinodulargoiter
F)Viralthyroiditis
CorrectAnswer:A.
Chronicautoimmune(Hashimoto)thyroiditisisthemostcommonformofthyroiditisandischaracterizedbythepresenceofantithyroidperoxidaseandantithyroglobulinantibodies.Womenaredisproportionatelyaffectedcomparedtomen.Theearlystagesof
chroniclymphocytic(Hashimoto)thyroiditiscanpresentwithneckswellingandadiffuselyenlarged,nontenderthyroidgland.Thepatientmaydisplayevidenceofhyperthyroidismfromdestructionofthyroidfollicularceilsandtheunregulatedreleaseofthyroid
hormone.Duringthechronicstagesofchroniclymphocytic(Hashimoto)thyroiditis
;thepatienttypicallydevelopsprimaryhypothyroidismfromtheunderlyinginflammatorydestructionofnormalthyroidparenchymaandresultantinsufficientproductionofthyroid
hormone.Signsandsymptomsduringthisstageincludefatigue
;coldintolerance,weightgain,hyporefiexia,myxedema,anddry,coolskin.FineneedlebiopsydemonstratesalymphoidinfiltratewiththepresenceoffolliclesandHurthlecells.Treatmentiswiththe
oralreplacementofthyroidhormone.
IncorrectAnswers:B,C,D
:6andF
Diffusetoxicgoiter(Gravesdisease)(ChoiceB)isthemostcommoncauseofhyperthyroidismandiscausedbyanautoantibodythatactivatesthyroidstimulatinghormonereceptorsonthethyroid.Itpresentswithsymptomsofhyperthyroidism,pretibialmyxedema,
andthyroidophthalmopathy,whichcancausediplopia,proptosis,andrestrictivestrabismus.
Granulomatousthyroiditis(ChoiceC)andviralthyroiditis(ChoiceF)
pbothrefertosubacutegranulomatousthyroiditis,alsoknownassubacute{deQueivain)thyroiditis,whichisaself-limitedthyroiditisthatoftenfollowsanacuteviralillnessandpresentswith
symptomsofhyperthyroidism.Histologydemonstratesgranulomatousinflammation.Apainful,tenderthyroidisacharacteristicfinding.
Papillarycarcinoma(ChoiceD)isthemostcommonthyroidcancer.Ittypicallypresentswithasolitarythyroidnoduleandischaracterizedbythepresenceofnucleargrooveswithempty-appearingnucleiandpsammomabodiesonhistologicalevaluation.Ittypically
carriesagoodprognosis.
Toxicmultinodulargoiter(ChoiceE)presentswithsymptomsofhyperthyroidismandmultiplepalpablenodules.Histologydemonstratesisolatedclustersofenlarged,hyperfunctionalfollicularcells.
EducationalObjective:Chroniclymphocytic(Hashimoto)diseaseisthemostcommonformofthyroiditisandischaracterizedbythepresenceofantithyroidperoxidaseandantithyroglobulinantibodies.Patientswithchronicdiseasepresentwithsignsandsymptoms
ofhypothyroidism,includingfatigue,coldintolerance,weightgain,hyporefiexia.myxedema,anddry,coolskin.FineneedlebiopsydemonstratesalymphoidinfiltratewiththepresenceoffolliclesandHurthlecells.
© © ©
A ftft
Previous Next ScoreReport LabValues Calculator Help Pause

ExamSection3:Item11of50 NationalBoardofMedicalExaminers^
ComprehensiveBasicScienceSelf-Assessment
Y
11A3-year-oldgirlhasahistoryofrecurrentinfections.Envitro,neutrophilsisolatedfromthispatientarecapableofphagocytosisandcankillLactobacillusspeciesbutnotStaphylococcusaureusThispatientmostlikelyhasadefectinvolvingwhichofthe
followingenzymes?
A)Catalase
B)Elastase
C)Myeloperoxidase
D)NADPHoxidase
E)Superoxidedismutase
CorrectAnswer:D.
NADPHoxidasedeficiencyresultinginchronicgranulomatousdisease(COD)mostlikelyexplainsthispatient'srecurrentinfectionsandnormalneutrophilphagocytosisbutwithimpairedkillingofStaphylococcusaureus.NADPHoxidasewithinneutrophilsuses
oxygenasasubstrateforthegenerationoffreeradicals(superoxideanions).Freeradicaloxygenspeciesaresubsequentlyusedforthecreationofhydrogenperoxideandhypochlorousacid.Activationofthispathwayleadstotherespiratoryburstwhichresultsin
bacterialdeath.DeficiencyofNADPHoxidaserendersphagocytesincapableofneutralizingcatalase-positivebacteria(eg,S.aureus)whichcanneutralizetheirownhydrogenperoxide,thusleavingthehostcellswithoutthesubstratenecessarytocompletethe
respiratoryburst.Lactobacillusspeciesarecatalase-negative.Diagnosisismadebyanabnormaldihydrorhodamrnetestoranitrobluetetrazoliumreductiontest.InthelattertestnormalphagocytesusetheactionofNADPHtoreducenitroblue.whichleadstoa
colorchangefromyellowtoblue.PatientswithCGDwillnotdemonstratecolorchange.RecurrentpneumoniaisthemostcommonpresentinginfectioninpatientswithCGD.andthemostcommoninfectingbacteriaincludeStaphylococcusspecies,Aspergillus
species,Burkholderiacepacia,andNocardiaspecies.PatientswithCGDarealsoatriskforfungalinfections,especiallyAspergillusspecies.
incorrectAnswers:A.B
:C
;andE.
Catalase(ChoiceA)isaubiquitousenzymepresentinbotheukaryoticandprokaryoticorganismsthatcatalyzestheconversionofhydrogenperoxidetowaterandoxygen.Catalase-positivebacteriaaremorelikelytocausediseaseinpatientswithCGD,buthuman
catalasedeficiencyisoftenbenign.
Elastase(ChoiceB)isaserineproteasesecretedbythepancreasandfunctionstodigestproteinsinthegastrointestinaltract.Patientswithchronicpancreatitisandpancreaticinsufficiencywilloftenhaveanabsenceofstooielastase,whichindicatesinadequate
pancreaticenzymefunction.
Myeloperoxidase(ChoiceC)isanautosomalrecessiveimmunedisordercausedbymutationsintheMPOgeneonchromosome17.Dysfunctionalmyeloperoxidaseresultsinaninabilitytoproducehydroxy-halideradicals,namelyhypochloritefromhydrogen
peroxideandchloride.Myeloperoxidasedeficiencyresultsinimpaired,butnotabsentbacterialkillingastheenzymaticproductsofNADPHoxidaseandsuperoxidedismutasearealsobactericidal.Patientstypicallypresentwithrecurrentfungalinfections.
Superoxidedismutase(ChoiceE)isanenzymethatconvertsreactiveoxygenspeciestooxygenandhydrogenperoxide,thuspreventingoxidativedamagetocelts.Deficiencyhasbeenlinkedtofamilialamyotrophiclateralsclerosis.
EducationalObjective:NADPHoxidaseisnecessaryfortheformationofreactiveoxygenspeciesinphagocytes,whichallowsforbactericidalactivity.MutationsintheproteinsthatmakeuptheNADPHoxidasecomplexresultinCGD,typicallycharacterizedby
recurrentpyogenicbacterialinfections.
O0
Previous Next ScoreReport LabValues Calculator Help Pause

ExamSection3:Item12ol50 NationalBoardofMedicalExaminers^
ComprehensiveBasicScienceSelf-Assessment
Y
12.A7-year-oldgirlisbroughttothephysicianbecauseofa2-weekhistoryofpainfulswellingunderherrightarm,whichhasbecomeincreasinglysevereduringthepast3days.Physicalexaminationshowsa2x1-cm
:tenderrightaxillarylymphnodeanda
smallpapuleonthedorsumoftherighthand.Asilver-stainedbiopsyspecimenofthepapuleshowspleomorphicbacilli.Whichofthefollowingisthemostlikelycausalorganism?
A)Bartonellahenselae
B)Brucellamelltensis
C)Burkholderiamallei
D)Franciselfatuiarensis
E)Streptobaciflusmoniliformis
CorrectAnswer:A.
CatscratchdiseaseiscausedbyinfectionwiththeGram-negativepleomorphiccoccobacillusBartonellahenselae,infectionisacquiredviascratchesorbitesfromdomesticorferaicats
;maybelocalizedordisseminated,butmostcommonlycauseslocal
lymphadenitisinthelymphaticdrainagepatternofthescratchlocation.Themostcommonlyinvolvedlymphnodesaretheaxillaryandcervicallymphnodes.Histologicexaminationshowsnecrotizinggranulomaswithstellate(star-shaped)abscesses.Multinucleated
giantcellsmayormaynotbepresent.Othercausesoflymphadenitisinchildrenincludeviruses(eg,adenovirus,respiratorysyncytialvirus,rhinovirus,Epstein-Barrvirus),Mycobacteriaspecies,Streptococcusspecies,Staphylococcusspecies,malignancy,fungal
infections,andKawasakidisease.Thesmallpapulealongthedorsumofthehandsuggestsasiteofinoculation,whereasviralinfectionwouldlikelydemonstratenosuchiesion.
IncorrectAnswers:B,C,D,andE.
Brucellamelitensis(ChoiceB)isoneofthespeciesofGram-negativecoccobaciliithatcausebrucellosis,alsoknownasunduiantfever.TheotherspeciesthatinfecthumansareBabortus.8.canis,andB.suls.Brucellosisiscommonlycontractedbythe
consumptionofunpasteurizedmilkandwouldnotbeexpectedtopresentwithadrainingpunctum.
Burkholderiamallei(ChoiceC)isaGram-negativerodthatcausesthezoonoticdiseaseglanders,characterizedbynodularlesionsandulcerationsinmucousmembranes,upperrespiratorytract,andlungs.Itrarelyaffectshumans,andtypicallyisfoundinhorses
mules,donkeys,andotheranimals.
Franciselfatuiarensis(ChoiceD)isaGram-negativecoccobacillusthatcausestularemia.Signsandsymptomsoftularemiaincludefever,skinulcers,andlymphadenopathy.Itcanalsocausepharyngitisandsevere,fatalpneumonia.Itischaracteristically
associatedwithrabbits.
Streptobaciflusmoniliformis(ChoiceE)isaGram-negativerodthatcausesrat-bitefeverazoonoticillnesstransmittedbythebiteorscratchofaninfectedrat,ortheconsumptionoffoodorwatercontaminatedbyrodenturineorfecescarryingthebacteria.
Symptomscanincludefever,vomiting,rash,arthralgias,myalgias,andheadache,ftcanbecomplicatedbydisseminatedspread,andcancauseabscesses,pneumonia,meningitis,endocarditis,orhepatitis.
EducationalObjective:CatscratchdiseaseiscausedbyinfectionwiththeGram-negativepleomorphiccoccobacillusB.henselae.Itisacommoncauseofpediatriclymphadenitis.Histologicexaminationshowsnecrotizinggranulomaswithstellate(star-shaped)
abscesses.Multinucleatedgiantcellsmayormaynotbepresent.
© © © ©f*
Previous Next ScoreReport LabValues Calculator Help Pause

ExamSection3:Item13of50 NationalBoardofMedicalExaminers^
ComprehensiveBasicScienceSelf-Assessment
Y
13.Anewbornbornat3Gweeks'gestationdies3dayslater.Thephotographshownisofasectionofbrainasseenatautopsy.Whichofthefollowingisthemostlikelyunderlyingdisease?
A}Biliaryatresia
B)Cyanoticcongenitalheartdisease
C)Hemolyticdiseaseofthenewborn
D)Neonatalmeningitis
E)Respiratorydistresssyndrome
CorrectAnswer:C.
Kernicterusoccursduetothedepositionofunconjugatedbilirubininanewborn'sbrain,usuallyinthebasalganglia.pons
;andcerebellum.Ittypicallypresentswithpoorfeeding,lethargy,andseizures.Unconjugatedorindirectbilirubiniscirculatinginsoluble
bilirubinthatislargelyboundtoalbumin.Ithasnotyetundergoneconjugationintheliver.Causesofincreasedunccnjugatedbilirubinincludehemolysis,breastmilkjaundice,Crigler-Najjarsyndrome,Gilbertsyndrome,andphysiologicjaundice.Innewborns,
etiologiesalsoincludeexposureofthemothertosulfonamideantibiotics,astheydisplacebilirubinfromalbumin,increasingthefreeplasmaconcentrationofunconjugatedbiiirubin.Kernicterustypicallyoccurswhenthebilirubinlevelexceeds25to30mg/dL,which
iscommonlyduetoglucose-6-phosphatedehydrogenasedeficiencyorRhhemolyticdiseaseofthenewborn.Riskfactorsincludeprematuritycephalohematomaorsignificantbruising,poorbreastfeeding,andEastAsianethnicity.Phototherapyorexchange
transfusioncanpreventsevereunconjugatedhyperbilirubinemiaandsubsequentkernicterus.However,mortalityishighoncekernicterusdevelops.
IncorrectAnswers:A,B.D,andE.
Biliaryatresia(ChoiceA)presentswithworseningjaundiceduetoconjugatedhyperbilirubinemia,palestool,darkurine,poorweightgain,and,inuntreatedcases,cirrhosis,coagulopathy,andporta!hypertension.Itwillnotcausekernicterus,asconjugatedbilirubin
iswatersolubleandunabletodepositinthebrain,anditisunlikelytocausedeathinthefirst3daysoflife.
Cyanoticcongenitalheartdiseases(ChoiceB)includetruncusarteriosus,transpositionofthegreatvessels,tricuspidatresia,tetralogyofFallot,andtotalanomalouspulmonaryvenousreturn.Theytypicallypresentwithacyanoticnewbornandsignsofglobal
hypoxiaonbrainpathologybutdonotpresentwithbilirubindepositsinthebasalganglia.
Neonatalmeningitis(ChoiceD)presentswithfeverorhypothermia,lethargy,irritability,abulgingfontanelle.hypotonia,andpoorfeeding._euikocytosis,aswellasevidenceofbacterialinfectiononlumbarpuncture,aretypicallypresent.Grosspathologymay
revealpurulentexudateintheleptomeninges.
Respiratorydistresssyndrome(ChoiceE)presentswithtachypnea,hypoxia,accessorymuscleuse.andcyanosis,ofteninprematureinfants.Itisduetoasurfactantdeficiencythatleadstoalveolarcollapse,andtypicallydemonstratesgroundglassopacitieson
chestx-ray.Itdoesnottypicallycausepathologicchangeswithinthebrainunlessitleadstoseverehypoxemia.
EducationalObjective:Kernicterusiscausedbyhighlevelsofunconjugatedhyperbilirubinemia,commonlyinthesettingofhemolysisduetoRhalloimmunizationorglucose-6-phosphatedehydrogenasedeficiency.Itisassociatedwithpoorfeeding,lethargy,and
seizuresduetodepositionofbilirubininthebasalganglia,pons,andcerebellum.
© ©
*
L2
Previous Next ScoreReport LabValues Calculator Help Pause

ExamSection3:Item14-olSO NationalBoardofMedicalExaminers^
ComprehensiveBasicScienceSelf-Assessment
Y
14A50-year-oldwomanisbroughttotheemergencydepartment20minutesafterthesuddenonsetofpaininherepigastriumthatradiatestoherbackandisassociatedwithnausea.Shehasahistoryofcholelithiasis.Sheappearsrestfess.Hertemperatureis
37.SX(99.7CF),pulseis120/min,andbloodpressureis115/60mmHg.Abdominalexaminationshowsdistentionwithreboundtendernessandguarding.Thepainispartiallyrelievedwhenthepatientbendsforward.ACTscanoftheabdomenshowsfluid
surroundingthepancreas.Obstructionofwhichofthefollowingisthemostlikelycauseofthefindingsinthispatient?
A)AmpullaofVater
B)Cisternachyil
C)Commonhepaticduct
D)Cysticduct
E)Superiormesentericartery
CorrectAnswer:A.
AmpullaofVaterobstructionsecondarytogallstonesisthemostlikelycauseofthispatient'sacutepancreatitis,whichisdiagnosedwhenatleasttwoofthreecriteriaaremet:epigastricpainthatradiatestotheback
:lipaselevelatleastthreetimestheupperlimitof
normal,and/orevidenceofacutepancreatitisonabdominalimaging.Thispatientmeetstwoofthesecriteria,whichissufficientforadiagnosisofacutepancreatitis.Themostcommoncausesofacutepancreatitisarealcoholusedisorderandgallstones,nthis
instance
:thepatienthasknowngallstonedisease,anditislikelythatoneormoregallstoneshastraveledoutofthegallbladderviathecysticduct,downthecommonbileduct,andhasbecomelodgedattheampullaofVater,wherethemainpancreaticductandthe
commonbileductmeet.Obstructionatthispointblocksboththecommonbileduct,predisposingtoascendingcholangitis,andthemainpancreaticduct,predisposingtoacutepancreatitis.Beatmentiswithendoscopicretrogradecholangiopancreatographyto
removetheobstructingstone.
IncorrectAnswers:B,C,D,andE.
Cisternachyli(ChoiceB)isadilatedareawithinthethoracicductatthelevelofL1-L2thatcollectslymphfromtheintestinalandlumbarlymphatictrunks.Itisnotinvolvedinthepathogenesisofacutepancreatitis.Obstructionwouldinterferewithnormallymphatic
flow.
Commonhepaticduct(ChoiceC)isaconduitforbilereleasedfromtheliver.Itjoinswiththecysticductfromthegallbladdertoformthecommonbileduct.AsthehepaticductissuperiortothepancreaticductandampullaofVater,obstructionbyagallstonewould
notcausepancreatitis.
Cysticduct(ChoiceD)istheconduitforbitetoandfromthegallbladder.Thecysticductmeetsthecommonhepaticducttoformthecommonbileduct.Obstructionofthecysticductmaycausecholecystitis,butitdoesnotcausepancreatitis.
Superiormesentericartery(ChoiceE)isabranchoftheabdominalaortathatsuppliesthegastrointestinaltractfromtheduodenumthroughtheproximaltwo-thirdsofthetransversecolon.Theinferiorpancreaticoduodenalbranchsuppliesbloodtotheheadofthe
pancreas.Obstructioncausesacuteorchronicmesentericischemia.
EducationalObjective:Acutegallstonepancreatitisoccurswhenagallstonebecomeslodgedatapointinthecommonbileductdistaltothemainpancreaticductinsertion,whichleadstoobstructionandpancreaticenzymeautoactivation.Acommonlocationfor
obstructionistheampullaofVater.whichisthesiteofconfluenceofthecommonbileductandthemainpancreaticduct.
© © © ©
f*
Previous Next ScoreReport LabValues Calculator Help Pause

ExamSection3:Item15ol50 NationalBoardofMedicalExaminers^
ComprehensiveBasicScienceSelf-Assessment
Y
15.A74-year-oldmancomestotheofficebecauseofa3-weekhistoryofprogressiveshortnessofbreathwithexertion.Hehashaddifficultyclimbingoneflightofstairs.Healsohasa20-yearhistoryofpoorlycontrolledhypertension,forwhichherarelytakes
hisprescribedmedication,andosteoarthritistreatedwithnaproxendaily.Histemperatureis37.1X(9&.8°F).pulseis76/minarespirationsare2G/mFn,andbloodpressureis180/98mmHg.Pulseoximetryonroomairshowsanoxygensaturationof92%.
Physicalexaminationshows10-cmjugularvenousdistentionabovethesternalangle.CracklesareheardoverbothSungbases.AnS
4
isheard.Echocardiographyshowsanormalejectionfraction.Whichofthefallowingbestexplainsthispatient's
symptom?
A)Decreasedmyocardialoxygensupply
B)Externalcompressionoftherightventricle
C)Impairedleftventricularrelaxation
D)Increasedturbulentflowacrossthemitralvalve
E)Obstructionoftheleftventricularoutflowtract
CorrectAnswer:C.
Hypertensionisacommoncauseofchronicsystolicanddiastolicleft-sidedheartfailureandisconsideredamodifiableriskfactor.Chronichypertensionmayresuitinleftventricularhypertrophyduetoconstantworkagainstanincreasedafterload.Thethickened
myocardiumresultsinimpairedleftventricularrelaxationandincreaseddiastolicfillingpressuresthatleadtoleftatrialenlargementandright-sidedheartfailureovertime.Thispatient
'
shistory,signs,symptoms,andexamfindingsareconsistentwithchronicleft-
sidedandright-sidedheartfailure.Onphysicalexam,thereisanS
4
gallopsuggestiveofastiffandnoncompliantleftventricle.Jugularvenousdistensionandpulmonarycracklesaresuggestiveofimpairedforwardflowofblood.AnEGGmayshow'evidenceofleft
ventricularhypertrophyandleftatrialenlargement.Anormalejectionfractiononechocardiographyindicatesthatthisisdiastolicheartfailure(alsocalledheartfailurewithpreservedejectionfraction).Adequatecontrolofbloodpressuredecreasesafterloadand
reducesthedegreeofcardiacmyocytehypertrophy,whichdecreasestheriskofdevelopingheartfailure.
IncorrectAnswers:A.6,D.andE.
Decreasedmyocardialoxygensupply(ChoiceA)resultsinanginaandmaybeduetoanon-obstructingcoronaryplaque(withluminalnarrowing),acutecoronarysyndrome,orcoronaryarterydissection,aneurysm,orvasospasm.Echocardiographymayshow
hypokineticorakineticregionsintheterritoryexperiencingthedecreasedsupply.
Externalcompressionoftherightventricle(ChoiceB)mayoccurinthesettingofamediastinalmass,pectusexcavatum,constrictivepericarditis,orcardiactamponadeandwouldresultinright-sidedheartfaiureduetotheinabilityoftherightventricletoexpand
andcontractfully.Inanotherwiseunrestrictedpericardium,leftventricularhypertrophydoesnotimpedeonrightventricularexpansion.
Increasedturbulentflowacrossthemitralvalve(ChoiceD)mayoccurinmitralstenosisorregurgitation,generatingthecharacteristicmurmurs..Mitralregurgitationiscommonlyassociatedwithmitralvalveprolapseorpriormyocardialinfarctionandpresentswitha
holosystolicmurmurbestheardintheleftfourthorfifthintercostalspacealongthemidclavicularlinewithradiationtotheleftaxilla.
Obstructionoftheleftventricularoutflowtract(ChoiceE)mayoccurinhypertrophiccardiomyopathy.Diastolicdysfunctioniscommonduetoahypertrophicandnoncompliantleftventricle.Echocardiographywillrevealathickenedinterventricularseptumand
ventricularwalls.Leftventricularhypertrophyduetohypertensiondoesnottypicallyobstructtheoutflowtract.
EducationalObjective:Leftventricularhypertrophyisariskfactorfordiastolicheartfailureduetoastiffandnoncompliantleftventricle.
O0 ©
Previous Next ScoreReport LabValues Calculator Help Pause

ExamSection3:Item16ol50 NationalBoardofMedicalExaminers^
ComprehensiveBasicScienceSelf-Assessment
25i16.Astudyisconductedtoassesstheeffectofanewselectiveestrogenreceptormodulator(SERM}onbonefractures.Twothousandwomenwithahistoryofvertebralfractureand5000womenwith
nofracturehistoryarerandomlyassignedtoaplaceboortoSERMatadoseof60mgor120mg.Thegraphshowrsthepercentofwomenwhohadanewfractureduring5yearsoffollow-up
(±standarderrorofthemean).Whichofthefollowingisthemostaccurateconclusionaboutpreventingnewfracturesbasedonthesedata?
T
£20
-
A)High-doseSERMtreatmentismoreeffectivethanlow-doseSERMtreatment
B)Low-doseSERMtreatmentismoreeffectivethanhigh-doseSERMtreatment
C)Placebotreatmentiseffectiveonlyinwomenwithahistoryofvertebralfracture
D)SERMtreatmentiseffectiveonlyinwomenwithahistoryofvertebralfracture
E)SERMtreatmentiseffectiveonlyinwomenwithnohistoryoffracture
o
15
-
T T
0)
10
-
£ T T T
5
-
o
o
0
NofracturehistoryFracturehistory
Placebo
SERM60mg
SERM120mg
CorrectAnswer:D.
Inthegraph,theleftgroupofbarsdemonstratethatinwomenwithnovertebralfracturehistory,theadministrationofplacebo,60mgselectiveestrogenreceptormodulator(SERM),and120mgSERMdemonstratedequivalentfractureriskoverthe5-yearperiod
ofapproximately4.5%.ThisdatasuggeststhattreatmentwithSERMofeitherdosedoesnotdecreasethe5-yearriskoffractureinpatientswithoutvertebralfracturehistory.Wheninterpretingtherightportionofthegraph,treatmentwithplaceboinpatientswith
ahistoryofvertebralfracturedemonstratesa5-yearriskofnewfractureofapproximately22%.Treatmentwith60mgSERMor120mgSERMinpatientswithahistoryofvertebralfracturedemonstratedariskofnewfractureof14.5%at5years.Thus,ina
randomized,placebo-controlledtrial.SERMdecreasedfractureriskbyapproximately7.5%inpatientswithpriorvertebra!fractures.Therefore,itisaccuratetostatethatSERMtreatmentiseffectiveonlyinwomenwithahistoryofvertebralfracture.
IncorrectAnswers:A.El,C.andE.
High-doseSERMtreatmentismoreeffectivethanlow-doseSERMtreatment(ChoiceA)andlow-doseSERMtreatmentismoreeffectivethanhigh-doseSERMtreatment(ChoiceB)areincorrectashighandlow-doseSERMtreatmentdemonstratedequivalent
riskofnewfractureinpatientsbothwithandwithoutverfebraifracturehistory.
Placebotreatmentiseffectiveonlyinwomenwithahistoryofvertebralfracture(ChoiceC)isnotcorrectasplaceboisgenerallyunderstoodtobeacontrol,notanintervention.Moreover,theSERMwasshowntobemoreeffectivethantheplacebointhepatient
groupwithpriorvertebralfractures.
SERMtreatmentiseffectiveonlyinwomenwithnohistoryoffracture(ChoiceE)isincorrectasfractureriskinthisgroupwasnotshowntobedifferentfromtheriskinpatientstreatedwithplacebo.
EducationalObjective:Placebosareoftenusedasacontrolforinterventionalstudies.Demonstratingastatisticallysignificantreductionintheriskofdiseaserelativetoaplaceboisthegoldstandardfortheevaluationofadrugorintervenfion.Thestratificafionof
patientsintogroupswithinaclinicaltrialisusefulforidentifyingtheeffectofaninterventioninpatientsathighandlowriskofthediseaseunderinvestigation.
*
*©6
- iS
Previous Next ScoreReport LabValues Calculator Help Pause

ExamSection3:Item17of50 NationalBoardofMedicalExaminers^
ComprehensiveBasicScienceSelf-Assessment
17.A23-year-oldmancutshislipinadvertentlywhileshaving.Secondsaftertheinjury
;thebleedingnearlystops.Whichofthefollowingmechanismsisthemostlikelycauseoftheearlyrapidcontrolofbloodlossinthisman?
A)Activationofantithrombinill
B)Localizedsecretionofendothelin
C)Oxygen-stimulatedcleavageofthromboplastin
D)Polymerizationoffibrin
CorrectAnswer:B.
Localizedsecretionofendothelinfromendothelialcellsaftertissueinjuryisstimulatedbythrombin
:whichaccumulatesatthesiteofbleedingasanormalpartofthecoagulationcascade.ThrombinisactivatedbyfactorXaattheconvergenceoftheintrinsicand
extrinsicclottingcascadesafterwhichitconvertsfibrinogentofibrin,allowingforfibrinpolymerizationandclotformation.Thrombinalsostimulatesthereleaseofendothelinfromendothelialcells.Endothelinisashortpeptidemadebythevascularendotheliumvia
theactionofendothelinconvertingenzymeandbindstoitsGprotein-coupledreceptors(ETA
andETB
)onvascularsmoothmuscle.Thisinducesvasoconstrictionviatheintracellularreleaseofcalcium.Vasoconstriction,inadditiontoclotformation,isanimportant
stepinpreventingfurtherbloodlossbyincreasingvascularresistanceandtherebydiminishingflow.
IncorrectAnswers:A,C
;andD.
Activationofantithrombinill(AT-JII)(ChoiceA)allowsAT-IHtobindtofactorsXaandIla(thrombin)
:,aswellastofactorsIXa
:Xla
:XIla.Vila.ka!likrein
;andplasmin
;therebyexertingabroadinhibitoryeffectontheclottingcascade.Heparin
:amedicationused
commonlytotreatbloodclots
;augmentstheactivityofAT-iiTherefore,activationwouldresultinprolongedbleeding.
Oxygen-stimulatedcleavageofthromboplastin(ChoiceC)wouldresultininactivationofthromboplastinandinhibitionoftheclottingcascade.ThromboplastinisacomplexoflipidsandtissuefactorthathasbindingsitesforfactorVIf.Endothelialortissuedamage
exposestissuefactorandsubendothelialcollagemwhichbindfactorVIIleadingtoitsactivationtofactorVila.Activationisstimulatedbyhypoxicconditions,notbythepresenceofoxygen.
Polymerizationoffibrin(ChoiceD)isthefinalstepinthecoagulationcascadeandservestobuildthefibrinsuperstructuretowhichplateletsanderythrocytescanbindtoformamatureclot.Thisisimportantinstoppingbleedingbutoccurslaterthantheimmediate
vasoconstrictionthatresultsfromthereleaseofendothelin.
EducationalObjective:Endothelinreleasefromendothelialcellsinresponsetotissueinjurycausesvasoconstrictionthataidsinreducingbloodflow.Itisanearlyresponsetoinjuryandoccursbeforeothercomponentsoftheclottingcascade
:suchasthe
polymerizationoffibrin.
© © ©
Previous Next ScoreReport LabValues Calculator Help Pause

ExamSection3:Item13ol50 NationalBoardofMedicalExaminers^
ComprehensiveBasicScienceSelf-Assessment
Y
18.A21-year-oldwomancomestothephysicianforaroutinehealthmaintenanceexamination.Her3-year-ofdsisterwasrecentlydiagnosedwithcysticfibrosis.Priortoanygenetictesting,whichofthefollowingbestapproximatesthispatient'sriskofbeinga
heterozygoteforcysticfibrosis?
A)0
B)1/2
C)1/4
D)2/3
E)3/4
CorrectAnswer:D.
Inheritanceofdiseaseisbasedonautosomal,mitochondrial,andX-linkedgeneticsoftheparents,genesdistributedintothegametes,denovomutationsacquiredduringembryoniccelldivisionanddifferentiation,penetranceandexpressivityofthecondition,
mosaicism,andepigeneticmodificationsofexpression.Inthiscase,cysticfibrosisispresentinonesiblingbutnotanother.CysticfibrosisisinheritedinanautosomalrecessivepatternandfollowstheprinciplesofMendeliangenetics.Thewildtypeallelecanbe
representedbyAandthemutatedallelebya.ftheoffspringoftwophenotypicaliyunaffectedindividualshascysticfibrosis,thenbothparentsmustbeheterozygous(Aa).Anoffspringoftwoheterozygousparentshasfourpotentialoutcomes:theycouldreceive
bothmutatedallelesandbehomozygousfortherecessivemutatedgene(aa)inwhichcasetheywoulddemonstratethediseaseassumingcompletepenetrance,theycouldreceiveonewildtypeallelefromthefatherandonemutatedallelefromthemother(Aa)or
viceversa,ortheycouldreceivebothwildtypealleles(AA).inanindividualwhoisknownnottohavethedisease(phenotypicaliynormalasinthispatient),theoutcomeofaaisruledout.leavingthreeotherpossiblegenotypes.Therearetwowaystobe
heterozygous(Aa)andonewaytobehomozygousforthedominantallele(AA).Thus,forthisindividual,thelikelihoodofheterozygosityfortheallelecausingcysticfibrosisis2/3.
IncorrectAnswers:A.B,C,andE.
Theindividualinquestionhasasisterwithcysticfibrosis,meaningherparentsmustbothcarryrecessivealleles.Thismeansthereisanon-zerolikelihoodthattheindividualinquestionisheterozygousatthegeneiocus.Thelikelihoodofherbeingheterozygous
cannotbe0(ChoiceA).
1/2(ChoiceB)isthelikelihoodthatanoffspringwithanunknownphenotypewillbeheterozygous.However,theindividualinthiscaseis21yearsoldandhasnotdemonstratedanymanifestationsofcysticfibrosis,sothepossibilityofherbeinghomozygous
recessive(aa)canberuledout.
1/4(ChoiceC)isthelikelihoodthatanoffspringwithanunknownphenotypewouldbeeitherhomozygousrecessiveorhomozygousdominant.Asthisindividual
'sphenotypeisknown,thereareonlythreepossiblegenotypes,notfour.
3/4(ChoiceE)isthelikelihoodthatanindividualwithanunknownphenotypeisnothomozygousrecessive.Forexample,iftheparentswerewantingtoconceiveagainandaskedthelikelihoodoftheirchildbeinghealthy,thatprobabilitywouldbe3/4.
EducationalObjective:InheritanceofcysticfibrosisfollowsaMendelianpattern.Itisinheritedinanautosomalrecessivepattern.Homozygousrecessiveindividualsdemonstratethediseasewhileheterozygousindividualsdonot.
© © ©
Previous Next ScoreReport LabValues Calculator Help Pause

ExamSection3:Item19of50 NationalBoardofMedicalExaminers^
ComprehensiveBasicScienceSelf-Assessment
Y
19.A25-year-oldwomanwitha3-yearhistoryofceliacdiseasecomestothephysicianbecauseofamarkedlyitchyrashonherscalp,arms,buttocks,andlegsfor5weeks.Examinationoftheskinshowsmanycrustedpapulessurroundedbyexcoriationsover
theupperandlowerextremitiesandafewvesiclesonthebuttocks.Askinbiopsyspecimenshowsthevesiclestobesubepidermal.DirectimmunofluorescenceofthevesiclesshowsgranulardepositsofIgAintheepidermal-dermaljunction.Whichofthe
followingisthemostlikelydiagnosis?
A)Bullouspemphigoid
B)Dermatitisherpetiformis
C)Dyshidroticeczema
D)Epidermolysisbullosaacquisita
E)Pemphigus
CorrectAnswer:B.
Dermatitisherpetiformisisavesiculobullousdisordercharacterizedbysmall,tense,vesicleslocatedonextensorsites(elbows,knees,andbuttocks),whichareextremelypruritic.Often,novesicleswillbepresentastheywillhavebeenintenselyscratchedand
replacedbyerosions.DermatitisherpetiformisiscausedbythedepositionofIgAalongtheepidermalbasementmembrane.Onhistopathologicexamination,smallcollectionsofneutrophilsfocallyseparatetheepidermisfromthedermisatthedermalpapillae,which
correspondtoavesicleclinically.Directimmunofluorescence(DIF)demonstratesagranularpatternofIgAdepositionalongthebasementmembraneattheepidermal-dermaljunctionofthedermalpapillae.Dermatitisherpetiformishasastrongassociationwith
celiacdisease,orgluten-sensitiveenteropathy,andtheprimarytreatmentistheavoidanceofgluten.Inseverepresentations,dapsonecanbeusedtoprovidenearimmediaterelief.
incorrectAnswers:A.C,D,andE.
Bullouspemphigoid(ChoiceA)iscausedbyantibodiesagainstthehemidesmosomeandischaracterizedbytensebullae.Itmostoftenpresentsinelderlymenwithseverepruritus.Themucosaisusuallyspared.Earlyinthediseasecourse,builaemaybeabsent
andonlyurticarialplaquesandsevereitchingwillbepresent.Onhistopathology,thebullaewilldemonstrateeosinophifs.DirectimmunofluorescencedemonstratesalinearpatternofIgGdepositionalongthebasementmembrane.
Dyshidroticeczema(ChoiceC)ischaracterizedbysmaI.firmvesiclesonthepalmsandlateralfingers.Itiscommonlyseenasaformofallergiccontactdermatitisbutcanalsobeinducedbydermatophyteinfection.
Epidermolysisbullosaacquisita(ChoiceD)ischaracterizedbyantibodiesagainsttheanchoringfibrils,whichareanothercomponentofthehemidesmosomecomplexandnecessaryforanchoringtheepidermistothebasementmembraneanddermis.Itismost
commonlyseeninelderlypatientswithtensebullaeandcanbeamarkerofunderlyingmalignancy.
Pemphigus(ChoiceE)iscausedbyantibodiesagainstdesmosomes,theproteincomplexthatmaintainscelltocelladhesionintheepidermis.Becausethetargetofpemphigusvulgarisismoresuperficial,intheepidermis,theresultantblisterswillbefragileand
flaccid.Thus,theclinicalmanifestationsofpemphigusincludeflaccidbullaeanderosions.Itoftenalsoinvolvesmucosalsurfaces.
EducationalObjective:Clinically,dermatitisherpetiformisischaracterizedbyseverelypruriticvesiclesorerosionsontheextensorsurfacesofextremities.Itisstronglyassociatedwithceliacdisease.DIFdemonstratesacoarse,granularpatternofIgAdepositionat
theepidermal-dermaljunctionofthedermalpapillae,whichconfirmsthediagnosis.
O0 © 0
f*
Previous Next ScoreReport LabValues Calculator Help Pause

ExamSection3:Item20ol50 NationalBoardofMedicalExaminers^
ComprehensiveBasicScienceSelf-Assessment
Y
20.A1-year-oldboyisbroughttothephysicianforawell-childexamination.Themotherisconcernedbecauseherson'sfine,palehairhasnotchangedcolorsincebirth.Hiseyesareblue.Duringophthalmologicexamination,thepatientturnsawayfromthe
flashlightandstartscrying.Whichofthefollowingisthemostlikelycauseofthepaleskincolorinthispatient?
A)Aberrantmigrationofneuralcrestcells
B)Decreasednumberofepidermalmelanocytes
C)Immunedestructionofmelanocytes
D)Inabilitytoproducemelanin
E)Melanindropouttothedermis
CorrectAnswer:D.
Oculocutaneousalbinismisthepartialorcompleteabsenceofmelaninpigmentfromthemelanocytesintheskin,hair,andeyes.Thefindingsofalbinismallrelatetothislackofmelaninpigmentwhitehairmilkyskin
:blue-grayeyesatbirth,extremesensitivityto
ultraviolet(UV)radiation,andanincreasedriskofskincancersthroughoutlife.Thefindingsarefoundatbirthandpresentdiffuselythroughoutthebody,unlessthemutationispresentinamosaicdistribution.Thisisincontrasttootherpigmentaryaberrancies,
whichmayaffectmelanocytemigrationtosomeareasofthebodyonabackgroundofnormalskin.Oculocutaneousalbinismhasthreesubtypes,eachofwhichiscausedbyadifferentmutationintheprocessofmelaninproduction.Inthemostcommonform,
oculocutaneousalbinismtypeIdecreasedorabsenttyrosinaseistheprimaryabnormality.
IncorrectAnswers:A.B,C.andE.
Melanocytesarederivedfromtheneuralcrestcells,whichbeginatthedorsaneuraltube.Duringembryologicdevelopmenttheymigratefromdorsaltoventral,andthentotheepidermis.Aberrantmigrationofneuralcrestcells(ChoiceA)presentsaspiebaldism.In
thiscondition,awhiteforelock,orawhitepatchofhairusuallyonthefrontalhairline,occurswhenthemelanocytesdonotfullymigratecephalad.Theremainingskinwilldemonstrateitsusualcoloring.
Decreasedmelaninproduction,ratherthanadecreasednumberofepidermalmelanocytes(ChoiceB),istheprimaryabnormalityofoculocutaneousalbinism..Adecreasednumberofepidermalmelanocytesisdemonstratedinidiopathicguttatehypomelanosis.In
thiscondition,patientswithchronicsundamagedevelopnumeroussmall,benign,depigmentedmaculesinsunexposedareas.
Immunedestructionofmelanocytes(ChoiceC)characterizesvitiligo.Invitiligo,depigmentedpatcheshaveapredilectionfordevelopingaroundthemouth,nose,eyes,hands,orgenitalia,butanyotherlocationofthebodycanbeinvolved.Autoimmunedestruction
ofthemelanocytesinducesthesepatchestoform.Itisalsoassociatedwithotherautoimmuneconditions,suchaschroniclymphocytic(Hashimoto)thyroiditis.
Melanindropouttothedermis(ChoiceE)isthemechanismofpost-inflammatoryhyperpigmentation.Wheninflammationoccursintheepidermisatthebasaljunction,themelanocytesarealsoaffectedanddie.Asmelanocytesandkeratinocytesdie,melaninis
releasedandsettlesinthedermiswhereitcanbechallengingtoremove,resultinginchronichyperpigmentation.
EducationalObjective:OculocutaneousalbinismtypeIiscausedbyadecreasedproductionofmelaninbythemelanocytesintheskin.WithoutmelanintocountertheeffectsofUVradiation,patientsareathighriskforbothmelanomaandnon-melanomaskin
cancers.
© © © oo
Previous Next ScoreReport LabValues Calculator Help Pause

ExamSection3:Item21ol50 NationalBoardofMedicalExaminers^
ComprehensiveBasicScienceSelf-Assessment
Y
21.AGS-year-oldmanwithrecurrenturinarytractinfectionscomestothephysicianbecauseofa2-dayhistoryoffeverandpainwithurination.Urinalysisshowspus.Heistreatedwithfevofloxacin.Hlissymptomsworsen
:andabloodcuFturegrows
Pseudomonasaeruginosathatisresistanttolevofloxacin.AlterationofwhichofthefollowingbestexplainstheresistanceofPaeruginosatolevofloxacininthispatient?
A)Dihydrofolatereductase
B)(3-Lactamase
C)30Sribosomalsubunit
D)Topoisomerase
E)Transpeptidase
CorrectAnswer:D.
FluoroquinoloneantibioticsinhibitbacterialtopoisomeraseIII(DNAgyrase)andtopoisomeraseIV.TheseenzymesfunctiontounwindsupercoilsinDNAthroughpurposefulsingle-ordouble-strandedbreaks,whichpreventdamagetotheDNAandallowittocontinue
itsnormalreplicativeprocessesandfunctionality.Examplesoffluoroquinoloneantibioticsincludeciprofloxacin,ofloxacin,levofloxacin
:andmoxifloxacin.TheyaremosteffectiveagainstGram-negativeinfectionsoftherespiratorytracturinarytract,and
gastrointestinaltractandtheyarealsoareusedforcertaineyeandearinfections.Theyexhibitanti-bacterialactivityagainstPseudomonasaeruginosa.Bacteriadevelopresistancetofluoroquinolonesthroughbacterialchromosome-encodedmutationsin
topoisomerase,whichdisruptsthetypicalbindingoffluoroquinolonestotheenzyme.Apotentialsecondarymechanismofresistanceistheoverexpressionofcellulareffluxpumps,whichactivelyremovefluoroquinolonemoleculesfromthebacteriacells,reducing
theirintracellularconcentrationandeffectiveness.
IncorrectAnswers:A.R,C.andE.
Dihydrofolatereductase(ChoiceA}functionstoconvertdihydrofolatetotetrahydrofolateinordertoshuttlemethylgroupsforthesynthesisofpurinenucleotides.Trimethoprimhasanextremelyhighaffinityforbacterialdihydrofolatereductaseandamuchlower
affinityforhumandihydrofolatereductase.thus
;itisaneffectiveantibacterialagent.
Themainmodalityofresistanceto(3-lactamantibioticsinGram-negativeorganismsistheproductionof(3-lactamases(ChoiceB)
;alsoknownaspenicillinases.These(3-lactamasescleavethe(3-lactamringintheseantibiotics,renderingtheminactive.
The30Sribosomalsubunit(ChoiceC)isthebindingsitefortetracyclineandaminoglycosideantibiotics.Themainmodeofresistancetotetracyclineantibioticsistheproductionofbacterialeffluxpumps.Aminoglycosideresistanceistypicallyderivedfrombacterial
transferaseenzymesthatinactivetheantibioticthroughacetylation,adenylation,orphosphorylation.
Themainmodalityofresistanceto(3-lactamantibioticsinGram-positiveorganismsisanalteredstructureoftranspeptidasepenicillin-bindingproteins(ChoiceE}
:rendering(3-lactamantibioticsunabletobindandinactivatethem.
EducationalObjective:FluoroquinoloneantibioticsactbybindingDNAgyraseandtopoisomeraseIV.bothofwhicharetopoisomerases.Geneticalterationsintheseenzymes,whichreducetheabilityofthefluoroquinolonetobind,facilitateresistancetothisclassof
antibiotics.
© © ©
Previous Next ScoreReport LabValues Calculator Help Pause

ExamSection3:Item22ol50 NationalBoardofMetlicaiExaminers^
ComprehensiveBasicScienceSelf-Assessment
y
22Asexuallyactive20-year-cldwomanhashadpainful,erythematous,vesicularlesionsonthevulvafor2days.Examinationbyspeculumalsoshowslesionsinthevaginalvaultandonthecervix.Aphotomicrographofascrapingofthebaseofanunroofed
vesicleisshown.Whichofthefollowingisthemostlikelydiagnosis?
A)Candidalvaginitis
B)Condylomataacuminata
C)Genitalherpes
D)Syphilis
E)Trichomonasvaginalisvaginitis
CorrectAnswer:C.
Thispatient'svesicularlesionsonthevulvaarelikelycausedbyherpessimplexvirus(HSV)intheformofgenitalherpes,whichalsocommonlycausesfever,chills,andinguinallymphadenopathy.Infectiontypicallybeginswiththeformationofvesicles
:whichiyse
andprogresstoshallow,painfululcerswithanerythematousborder.Genital,herpesoftencausesvulvarpainwithurinationduetoirritationofthevesiclesandlaterulcers.ThediagnosismaybeconfirmedbyPCRtestingbutperformingaTzancksmearisanother
methodofdiagnosis.OnTzancksmearherpes-
nfededmultinucleatedgiantceIsmaybeseen,asshowninthisphotomicrograph.Thenucleiareclosetogetheranddemonstratenuclearmolding.GenitalherpesismostoftencausedbyHSV-2tautcanalsobe
causedbyHSV-1.HSV-1andHSV-2arebothmembersoftheherpesvirusfamilyofdouble-strandedDNAenvelopedviruses.Genitalherpesistransmittedthroughsexualcontactandperinatally.HSVmayremainlatentinthesacra!gangliauntilreactivation,
whenitcausespainfulvesiclesandpunchedouterosionsonthegenitaliawithassociatedinguinallymphadenopathy.Systemicmanifestationsarepossible,includingviralmeningitisandencephalitisTreatmentforherpeticinfectionsinvolvesdrugsthatinhibit
viralDNApolymerase,classicallybyguanosineanalogssuchasacyclovirvalacyclovir,andfamciclovir.
IncorrectAnswers:A,B.DaandE.
Candidalvaginitis(ChoiceA)iscausedbyCandidaalbicans,afungusthatformspartofthenormalvaginalflora.OvergrowthofCandidaspeciesmayproducevulvovaginitis.Candidalvulvovaginitisresultsinpruritusandathick,whitevaginaldischarge.Vaginal
pHFSnormal,andhyphaemaybevisualizedonwetmount.Vesiclesandulcersarenotseen,anditisnotpainful.
Condylomataacuminata{ChoiceB)
:orgenitalwarts,aresoft,pink,papillarylesionsontheexternalgenitaliaassociatedwithhumanpapillomavirusthatcanappearonthepenis,vagina,vulva,and/oranalcanal.Theyarenotpainful.Diagnosisismadewith
examinationand/orbiopsywhichwillshowpapillomatouslesionswithviralfyinfectedcellscalledkoilocytesintheepidermis,butnotmultinucleatedgiantcells.
Syphilis(ChoiceD),causedbyaninfectionwiththespirocheteTreponemapaftidum,demonstratesmultiplestageswithvaryingsymptoms,includingprimarywithapainlesschancre,secondarywithfever,lymphadenopathy,andcondylomatalata,andtertiarywith
tabesdorsalis,aoriitis,andgummas.Thechancreofprimarysyphilistypicallypresentsasapainlessulcerativegenitallesion,unlikethepainfullesionsseeninthispatient.
Trichomonasvaginalisvaginitis(ChoiceE)isasexuallytransmittedinfectionthatcausesvaginitisandcervicitisinwomenandurethritisinmen.Trichomoniasiscanleadtooveractivebladder(frequency,urgency,urgeincontinence)anddysuria.Trichomoniasis
presentswithgreen,malodorousvaginaldischargeandmotiletrichomonadsonwetmount,notpainfulgenitalvesiclesorulcers.
EducationalObjective:Genitalherpesinfectionspresentwithpainfulvesiclesanderosions.HSV-2followedbyHSV-1arethemostcommonviralcausesandthediagnosiscanbeconfirmedwithaTzancksmearwhichdemonstratesmultinucleatedgiantcellswith
nuclearmolding.
SI©
Previous Next ScoreReport LabValues Calculator Help Pause

ExamSection3:Item23of50 NationalBoardofMedicalExaminers^
ComprehensiveBasicScienceSelf-Assessment
Y
23.AG7-year-oldwomanwithCrohndiseaseisadmittedtothehospitalforbowelrestandparenteralhyperalimentation.Acentralvenouscatheterisplaced.Sevendaysafteradmission,shedevelopsatemperatureof39.2aC(102.6aF}andshakingchilis.
Broad-spectrumantibioticsareadministeredpendingbloodcultures.Shecontinuestohavedailytemperaturespikes.Twodayslaterbloodculturegrowsyeastforms.Whichofthefollowingisthemostlikelycausalorganism?
A}Aspergillusfumigatus
B)fifastomycesdermatitis]is
C)Candidaalbicans
D}Cryptococcusneoformans
E)Mucorramosissimus
CorrectAnswer:C.
Candidaalbicansisthemostlikelypathogenresponsibleforthispatient'sfeverandrigors,especiallygiventhepresenceofyeastformsonbloodculture,whichisdiagnosticoffungemia.C.albicansisaubiquitousdimorphicfungusthatcausesawidespectrumof
clinicaldiseasesinimmunocompetenthostsincludinginfectionsofmoistskinfolds(intertrigo),vagina!infections,andoralthrush.ItalsocausesseverediseaseinpatientswhoareimmunocompromisedsuchasthosewithHIV/AIDSinfection,neutropenicpatients
onchemotherapy,patientswithpoorlycontrolleddiabetesmellitus,andthosewithautoimmunedisorderssuchasCrohndisease,suchasthispatient.PatientsreceivingparenteralnutritionareatsubstantiallyhigherriskforinvasivefungaldiseasefromC.albicans,
andanyfungiisolatedinbloodculturesshouldbetreatedasevidenceofdisseminateddisease.InitialtreatmentmayincludeanechinocandinwithbroadactivityagainstCandidaspecieswithnarrowingofantifungalsfollowingspeciationandsensitivitystudies
Ophthalmologicexaminationinpatientswithcandidemiaisessentialtoruleoutocularinvolvement.
IncorrectAnswers:A.B,D.andE.
Aspergillusfumigatus(ChoiceA)isamoldthatcausesaspergillosis,whichcanpresentwithaspergilloma,allergicbronchopulmonaryaspergillosis,chronicnecrotizingaspergillosis,andairwayinvasiveorangioinvasiveaspergillosis.Aspergillomas(fungalmasses}
arethemostcommonandarisewithinexistingcavitarylesionsofthelungsuchasthosecausedbyprevioustuberculosis,emphysematousbullae,orcavitarysquamouscelllungcarcinoma.Parenteralnutritionisnotariskfactorfordisease.
Blastomycesdermatitidis(ChoiceB)isayeastendemictotheMississippiandOhiorivervalleys.Itcancauseacuteandchronicpneumoniainadditiontoskinlesionswithverrucousborders,osteomyelitis,prostatitis,andmeningoencephalitisin
immunocompromisedhosts.Thispatienthasnoknownriskfactorsandfungemiawithfi.dermatitiswouldberare.
Cryptococcusneoformans(ChoiceD)isanencapsulatedyeastandopportunisticpathogeninimmunocompromisedpatients,especiallypatientswithHIV/AIDSinfectionandCD4+countslessthan200cells/mm
3
.Disseminateddiseasecancausecryptococcal
meningitisandencephalitis.Histologyrevealsfungalcellswithnarrow-basedbuddingandabrightredinnercapsuleonstainingwithmucicarmine.Parenteralnutritionisnotaknownriskfactorfordisease.
Mucorramosissimus(ChoiceE)isaubiquitousfungusfoundcommonlyondecayingvegetation.HumanshavefrequentdailyexposuretoMucorspecieswithoutthedevelopmentofdisease,butinpatientswithimmunocompromisingconditionssuchasdiabetes
mellitusorstemcelltransplant,Mucorspeciescancausedevastatingsinus,orbital,andcentralnervoussystemdisease.Mortalityishigh.Parenteralnutritionisnotknowntopredisposetoinfection.
EducationalObjective:Candidaspeciescancausefungemia.andparenteralnutritionisaknownriskfactor.Yeastformsisolatedonbloodculturesshouldneverbetreatedasacontaminantandtreatmentwithabroad-spectrumantifungalshouldbestarted
immediately.
© © © ©f*
Previous Next ScoreReport LabValues Calculator Help Pause

ExamSection3:Item24ol50 NationalBoardofMedicalExaminers^
ComprehensiveBasicScienceSelf-Assessment
y
24.A25-year-oldwomancomestothephysicianbecauseofa6-monthhistoryofawhitedischargefromherbreasts.Herlastmenstrualperiodwas6monthsago.Shehashadnoheadachesorvisualfieldchanges.Shehasneverbeensexuallyactive.Gentle
palpationofbothbreastsproducesexpressionofamilkydischarge.Serumstudiesshow:
Thyroid-stimulatinghormone
Follicle-stimulatinghormone
Luteinizinghormone
Prolactin
1.5pU/mL
6mlU/mL
6mlU/mL
200ng/mL
A5-daycourseofmedroxyprogesteronefailstoinducemenses.AnMR!ofthebrainshowsan8-mmlesioninthepituitarygland.Themostappropriatetreatmentforthispatientisadrugfromwhichofthefollowingclasses?
A)Dopamineagonist
B)Dopamineantagonist
G)Oxytocinagonist
D)Oxytocinantagonist
E)Serotonin(5-HT
t
)agonist
F)Serotonin(5-HTt
)antagonist
CorrectAnswer:A.
Aninappropriateincreaseinserumprolactinisdefinedashyperprolactinemiaandcausestheproductionofbreastmilk(galactorrhea),growthofglandularbreasttissue,andamenorrhea.Amenorrheaiscausedbythesuppressionofgonadotropin-releasing
hormone,increasesinprolactinoftenarisefromahyperfunctioninganteriorpituitarymicroadenoma.Sincedopamineinhibitsprolactinsecretion,dopamineantagonistscancausegalactorrheaasanadverseeffect,whichmaybeseenwiththeuseoftypical
antipsychoticmedicationssuchashaloperidol.Incontrast,prolactinispositivelyregulatedbythyrotropin-releasinghormone(TRH)and
:instatesofhypothyroidismwithupregulatedTRH,galactorrheacanoccur.Withtheexclusionofthyroidpathologyandany
contributionsfromdopamineantagonists,furtherdiagnosticevaluationshouldincludeanMRIofthebraintoassessforthepresenceofapituitaryadenoma.Ifapituitarymicroadenomaispresent,asisnotedinthisclinicalscenario,treatmentiswithdopamine
agonistssuchasbromocriptineorcabergoline.Ifsuchmedicationsfail,orifamacroadenomaisfound,surgicalresectionoftheadenomaisthenextstep.
incorrectAnswers:B,C,D
:E,andF
Dopamineantagonists(ChoiceB)decreasethelevelofdopamineinthebrain,whichreducestheamountofnegativefeedbackonprolactin.Theywouldleadtofurtherincreasesinthispatient'ssymptoms.
Oxytocinagonists(ChoiceC)wouldleadtouterinecontractionsandwouldalsopromotemilkletdown.Theywouldnotbeusefulintreatmentofaprolactinoma.
Oxytocinantagonists(ChoiceD)wouldcauseuterineatonyanddecreasethemilkletdownreflex.Theywouldnotbeusefulinthetreatmentofaprolactinoma.
Serotonin(5-HT
^
)agonistsandantagonists(ChoicesEandF)includeantidepressantsandatypicalantipsychotics.Thesemedicationsarenoteffectiveinthetreatmentofprolactinoma
EducationalObjective:Prolactinomaspresentwithgalactorrheaandamenorrhea.Ifapituitarymicroadenomaispresent,treatmentiswithdopamineagonistssuchasbromocriptineorcabergoline.Ifsuchmedicationsfail,orifamacroadenomaisfound,surgical
resectionoftheadenomaisthenextstep.
O0 ©
Previous Next ScoreReport LabValues Calculator Help Pause

ExamSection3:Item25ol50 NationalBoardofMedicalExaminers^
ComprehensiveBasicScienceSelf-Assessment
Y
25.A23-year-oldwomanisadmittedtothehospitalaftertakinganoverdoseof16acetaminophentablets.Sheingestedtheacetaminophenaftershebrokeupwithherboyfriendof2months.Shehasbeenadmittedtothehospitalseveraltimesduringthepast
5yearsfortreatmentofsuicideattemptsandotherself-harmattempts.Herhistoryincludeschildhoodsexualabuse,andshehasdifficultyholdingajob.Shetellsthephysicianthatshehasreceivedsubstandardcarefromthishospitalinthepast.Shesays,
"Butnowthatyou'remyphysicianIknowI'mgoingtogetbetter.Icantellbecausewehaveaconnection.
11
Heraffectisbrightandcheerful.Thispatientmostlikelyhaswhichofthefollowingtypesofpersonalitydisorders?
A)Antisocial
B)Borderline
C)Dependent
D)Narcissistic
E)Schizoid
CorrectAnswer:B.
Borderlinepersonalitydisorder(BPD)isadusterBpersonalitydisordertheemotionalordramaticclusterthatfeaturesanunstablesenseofselfandtumultuousrelationships.Likelyduetoacombinationofgeneticpolymorphismsinserotoninanddopamine
receptorsandemotionalinvalidationorabuseduringchildhood,patientswithBPDunconsciouslylearntomakeimpulsiveanddramaticgestures,includingself-harm
:toobtainattentionandemotionalfulfilmentfromothers.Chronicinvalidationalsoleadstopoor
self-esteemandtheconsequentrelianceonothersforself-esteemneeds,resultinginanintensefearofabandonmentandseveredistresswhenabandonmenthappens.Duetoinadequateemotionalattunementfromcaregivers,patientswithBPDdonotlearnto
label,understand,orregulatetheiremotions.TheirnegativeemotionalexperiencesaresointenselypainfulthatpeoplewithBPDmustdissociatetheseexperiencesfromtheirconsciousness.ThissplittingleadspeoplewithBPDtoblack-and-whitethinkingthat
includesseeingpeopleasallgoodandothersasallbad
;asinthispatient.
IncorrectAnswers:A.C,D,andE.
Antisocialpersonalitydisorder(ChoiceA)isaclusterBpersonalitydisorderthatarisesfromadeficitinempathyresultinginpervasiveviolationsofothers'rights,aggression,andahostileandmanipulativeattitudetowardothers.Thesepatientsmayalsobe
chronicallyboredandhencesensationseeking,leadingtoahighrateofsubstanceabuseandgambling.PatientswithBPDmayadoptahostileattitudetowardothersattimesbutaremorelikelytoself-harmthanharmothers.
Dependentpersonalitydisorder(ChoiceC)isaclusterCpersonalitydisorder,whichistheanxiouscluster.Dependentpersonalitydisorderpresentswithanexcessiveneedtobecaredforbyothersthatmanifestsassevereseparationanxietyandclingingbehavior
Thoughthispatientdoesillustrateabandonment-relateddistress,shealsoillustratesdifficultywithemotionalregulation,black-and-whitethinking,andsuicidalgesturesthataremoretypicalofBPD.
Narcissisticpersonalitydisorder(ChoiceD)isaclusterBpersonalitydisorderthatischaracterizedbyfragileself-esteemandcompensatoryarrogant,self-aggrandizingbehaviortogainadmiration,sometimesatothers
'expense.PatientswithBPDdonotseek
admirationfromothers,insteadalternatingbetweenseekingemotionalclosenessandemotionaldistance.PatientswithnarcissisticpersonalitydonottypicallyexperiencetheintenseemotionsorfearofabandonmentexperiencedbypatientswithBPD.
Schizoidpersonalitydisorder(ChoiceE),aclusterApersonalitydisorder,manifestsasextremesocialdetachmentandacold,restrictedaffect.PatientswithBPDareinsteaddependentonothersandpresentwithlabileaffect.
EducationalObjective:BPDisaclusterBpersonalitydisorderthatfeaturesanunstablesenseofself,emotionaldysregulation,andtumultuousrelationships.Tounconsciouslyfulfilltheiremotionalneeds,patientswithBPDmakesuicidalgestures,desperatelyavoid
abandonment,andeitheridealizeordevalueotherpeople.
© © ©
Previous Next ScoreReport LabValues Calculator Help Pause

ExamSection3:Item26ol50 NationalBoardofMedicalExaminers^
ComprehensiveBasicScienceSelf-Assessment
Y
26.A13-year-oldboyisscheduledtoreceivechemotherapyforaleukemiathathasthehistologicfeaturesofmalignantlymphocytesThisneoplasmisfurthertypedforcellsurfaceandintracellularmarkersspecificforlymphocytesubsets.Theneoplasticcells
donotexpressthefollowingmarkers:CD4
:CD3
;surfaceIgM,surfaceIgG,cytoplasmicIgMandp-heavychaincytoplasmiclgG
:andy-heavychainTheleukemiccellsexpressclassIMHCmoleculesandshowrearrangementoftheT-[ymphocytereceptor
P
-chaingeneDandJsegments.Whichofthefallowingisthenormalcounterpartofthesemalignantlymphocytes?
A)Activatedcytolyticeffector!lymphocytesinthecirculation
B)MatureIgM-secretingBlymphocytesinthelymphnode
C)Matureimmunoglobulin-secretingplasmacellsinthefymphnode
D)Pre-BlymphocyteprogenitorofmatureBlymphocytesinthebonemarrow
E)T-lymphocytethymocyteslocalizedtothethymiccortex
CorrectAnswer:E
ThelaboratoryanalysisofthisacutelymphoblasticleukemiaismostsuggestiveofmalignanttransformationofnormalT-lymphocytethymocyteslocalizedtothethymiccortex.Leukemiamayarisefromthelymphoidormyeloidcelllines.Acutelymphoblastic
leukemiaiscomposedoflympho.dprogenitorcellsandisthemostcommonhematologicmalignancyofchildhood.Cellmarkersareusedtoidentifytheunderlyingcelltype.NormalT-lymphocytedevelopmentbeginsinthebonemarrowwithdifferentiationof
hematopoieticstemcellsintoT-lymphocyteprecursors.TheprecursorsthenmigratetothethymiccortexwheretheyundergorearrangementoftheT-lymphocytereceptorp-chaingeneDandJsegments.TlymphocytesthatrecognizeclassMHCmolecules
undergopositiveselectionandproliferate.TheTlymphocytesatthisstagearedoublenegative(donotexpressGD4orCDS).Followingpositiveselection,Tlymphocytesundergorearrangementofthea-chaingeneandenterthedoublepositivestagewith
expressionofCD4andCD8.DoublepositivecellsthendifferentiateintosinglepositiveCD4+orCD3+celsviainteractionwitheitherclassIIMHCorclassIMHCmolecules,respectively.Tnesinglepositivecellsmigratetothethymicmedullatoundergonegative
seection.Tlymphocyteswithahighaffinityforself-antigenswillundergoapoptosistoensureself-tolerance.TlymphocytesthatsurvivenegativeselectionwillenterthecirculationasmatureTlymphocytes.
IncorrectAnswers:A.B.C.andD.
ActivatedcytolyticeffectorTlymphocytesinthecirculation{ChoiceA)isincorrectastheseceilswillexpressCDS,whichbindstoclassIMHContargetcells.EffectorTlymphocyteseliminateneoplasticcellsandvirus-infectedcellsbyreleasingcytotoxicgranules
andinducingapoptosis.
MatureIgM-secretingBlymphocytesinthelymphnode(ChoiceB)expresssurfacelgM
;cytoplasmiclgM
:andp-heavychaincellmarkers.IgM-secretingBlymphocytesareoneofthefirstcellsactivatedintheprimaryantibodyresponsetoaforeignantigen.Pre-B
lymphocyteprogenitorsofmatureBlymphocytesinthebonemarrow(ChoiceD)alsoinitiallyexpressIgMandp-heavychainsbeforeprogressingtothenextstageofdevelopment.
Matureimmunoglobulin-secretingplasmace:lsinthelymphnode(ChoiceC)expresssurfaceIgGandy-heavychains.IgGistheprimaryclassofimmunoglobulinpresentinthecirculation,whicheffectsthesecondaryantibodyresponsetoaforeignantigen.
EducationalObjective:Tlymphocytesundergoselectionandmaturationinthethymuspriortoenteringthecirculation.PriortopositiveselectionforrecognitionofclassiMHCmolecules,TlymphocytesdonotexpressCD4orCDS.Afterpositiveselection,they
expressbothbeforedifferentiatingintosinglepositiveCD4+orCD8+cells.
0 oo
Previous Next ScoreReport LabValues Calculator Help Pause

ExamSection3:Item27of50 NationalBoardofMetlicafExaminers^
ComprehensiveBasicScienceSelf-Assessment
y
27.Thetracingshowscontractileresponsesofaskeletalmusclepreparationtoelectricalstimulationatfivedifferentfrequencies.Theamountofcalciumsequestrationinsarcoplasmicreticulumishighestduringwhichofthefollowinglabeledresponses?
CD EAB
o
CO
0
Stimulus
frequency1-Hz3-Hz6-Hz 12-Hz
»
Time
A)
B)
C)
0:1
E)
CorrectAnswer:A.
Musclefibersutilizeelectromechanicalcouplingtoallowforthelocomotionofthehumanskeleton.Normallythisprocessoccursviaacetylcholinereleasedfromthepostsynapticneuron,whichresultsintheoccurrenceofendplatepotentialsattheneuromuscular
junction.Onceathresholdisreached,musclecelldepolarizationandactionpotentialpropagationoccur.TheTtubules,infoldingsofthesarcolemma
:transmittheactionpotentialdeepintothemusclecell.Dihydropyridinereceptorsarevoltage-gatedcalcium
channelsthataremechanicallycoupledtoryanodinereceptors.Depolarizationresultsinconformationalchangesofthesereceptors,whichresultsinthesubsequentreleaseofcalciumfromthesarcoplasmicreticulum.Calciuminthecytoplasmofthemusclecell
bindstotroponinC,whichinducesaconformationalchangethatmovestropomyosin,allowingformyosin-actinbindingandmechanicalcontraction.
Jnthisexperiment,electricalstimuliareappliedatnumerousfrequenciestoaskeletalmuscleandthetensionismeasured.At1Hz
:musclecontractioncanbedemonstratedafterthestimulusfollowedbyrelaxationofthemuscleasevidencedbythedescending
slopeofthetensionmeasurement.Duringmusclerelaxation,calciumissequesteredbackintothesarcoplasmicreticulum.Thiscyclethenoccursagainasanotherstimulusisappliedandtensiononceagainrisesandfalls.Itcanbenotedthatthetensiondoesnot
fullyreturntobaseline,asnotenoughtimeoccursbetweeneachstimulustoallowforcompleterelaxationofthemuscleandfullcalciumsequestrationintothesarcoplasmicreticulum.Asthefrequencyincreases,theamountoftimeallowedforrelaxationand
calciumsequestrationdecreases,leadingtotheinabilityofthemuscletoapproachitsbaselinetension.Therefore,stimulationofthemuscleat1Hzallowsforthehighestlevelofcalciumsequestrationintothesarcoplasmicreticulumbetweeneachstimulusas
thispermitsthegreatestrelaxationtime.Bycomparison,at12Hz
:thestimuusfrequencydoesnotallowforanyrelaxationofthemuscleandtensioncontinuestobuild.ThisisalsoknownastetanyInthisscenario,thereisminimalsequestrationofcalciumand
norelaxationofthemuscleat12Hz.
IncorrectAnswers:B,C,D
:andE.
Increasingfrequencyofstimulileadstodecreasedtimeforcalciumsequestrationintothesarcoplasmicreticulum.Cytoplasmiccalciumconcentrationsremainincreased,whichpreventsmechanicalrelaxationofthemusclefibers.Stimulationat3Hz(ChoiceB),G
Hz(ChoiceC)
:9Hz(ChoiceD),and12Hz(ChoiceE)sequentiallydecreasesthetimeforcalciumsequestrationintothesarcoplasmicreticulum,resultingineventualtetany.
EducationalObjective:Skeletalmuscleiscomprisedofcontractilecellscalledmyocytes.Myocytesrespondtoactionpotentialsbyreleasingcalciumintothecytoplasmthatresultsintropomyosinconformationchangesandactin-myosinbinding.Thegenerationof
mechanicaltensileforceresults.Betweeneachstimulus,calciumissequesteredintothesarcoplasmicreticulum,allowingformusclerelaxation.Increasingstimuusfrequencyleadstoinadequatetimeforcaiciumsequestrationresultingintetany,whichisastate
ofhighintracellularcalciumconcentrationandconstantforce.
O1» ®
12
Previous Next ScoreReport LabValues Calculator Help Pause

ExamSection3:Item28ol50 NationalBoardofMedicalExaminers^
ComprehensiveBasicScienceSelf-Assessment
Y
28.A83-year-oldmancomestothephysicianbecauseofintermittentnumbnessandpainofhisrightarmduringthepast6months.Hisbloodpressureis120/80mmHginbothupperextremities:thepulseexaminationshowsnoabnormalities.Achestx-ray
showsarightcompletecervicalrib.ACTscanoftheneckandchestshowscompressionofthebrachialplexus.Whichofthefollowingvesselsisatgreatestriskforinjuryduringoperativeremovaloftherightcervicalrib?
A)Brachiocephalicartery
B)Rightcommoncarotidartery
C)Rightinternalcarotidartery
D)Rightsubclavianartery
CorrectAnswer:D.
Therightsubclavianarteryoriginatesfromthebrachiocephalicarteryinthemediastinumandcourseslaterallydeeptotheclavicle.Asthisarterycoursespastthefirstrib
:itbecomestheaxillaryartery,surroundedbythebrachialplexus,thencoursesdistallyto
supplythearm.AcervicalribisanextraribthatarticulateswiththeC7vertebra.Thecervicalribcanextendintotheareabetweentheribofthefirstthoracicvertebraandtheclavicle,whichmayresultinabnormalpressureplacedonthestructurescoursingthrough
thisregionThesestructuresincludethedivisionsofthebrachialplexus,thesubclavianartery
;andthesubclavianvein.Thiscanresultinassociatedextremitynumbness
:tingling,pain,andpotentialmusclewastingifchronic.Thisconstellationoffindingsistermed
thoracicoutletsyndrome.Treatmentrequiresremovalofthecervicalribtoalleviatecompressionofadjacentstructuresandtheassociatedsymptoms.Caremustbetakennottoinjuretheunderlyingsubclavianartery
:whichtypicallyliesadjacenttothecervicalrib.
IncorrectAnswers:A.B.andC.
Thebrachiocephalicartery(ChoiceA)originatesfromtheaortaandcoursesonlyafewcentimeterspriortobifurcatingintotherightsubclavianarteryandtherightcommoncarotidartery.Thisstructureisiintraihoracicandisatloworriskofinjuryduringcervicalrib
surgery.
Therightcommoncarotidartery(ChoiceB)originatesfromthebrachiocephalicarteryandcoursescraniallytosupplystructuresofthehead.Itterminatesbybifurcatingintotheinternalandexternalcarotidarteries.Thisstructureismedialplacingitatlowerriskof
injuryduringcervicalribremoval.
Therightinternalcarotidartery(ChoiceC)originatesfromtherightcommoncarot
'
darteryjustbelowtheangleofthejawandproceedscraniallytoperfusethebrain.Thisstructureistoosuperiortobeatriskduringresectionofaribattheleveloftheseventh
cervicalvertebra.
EducationalObjective:Therightsubclavianarterycoursesfromthebrachiocephalictrunklaterallytotheborderofthefirstrib,deeptotheclavicle.7hisstructurecanbecompressedduringextremerangesofmotionoftheupperextremityandwhenanintervening
cervicalribispresent.Thesubclavianarteryisatriskofinjuryduringremovalofacervicalribduetoitsposition.
© © © 0
*
&
Previous Next ScoreReport LabValues Calculator Help Pause

ExamSection3:Item29of50 NationalBoardofMedicalExaminers^
ComprehensiveBasicScienceSelf-Assessment
Y
29.A24-year-oldmanwhoiscomatoseisadmittedtothehospitalbecauseofadrugoverdoseandpneumonia.Onadmissionheisintubatedandmechanicallyventilated.Whilereceivingatidalvolumeof500mL.thepatient'send-expiratorypressureis+5cm
H
2
Q,andhisend-inspiratoryairwaypressureis+25cmH20
.Anesophagealballoonisinserted,andanend-inspiratorypleuralpressureof+20cmHpismeasured.Whichofthefollowingbestrepresentsthispatient'srespiratorysystemcompliance?
A)0.01cmHp/mL
B)0.04cmHp/mL
C)0.05cmHp/mL
D)20mL/cmHp
E)25mL/cmHp
F)100mL/cmH20
CorrectAnswer:E
Complianceisthechangeinvolumeofasystemperchangeinpressure.Intherespiratorysystem,thecomplianceofthechestwallandthelungsarebothconsideredtodeterminetotalsystemcompliance.Inaventilatedpatient,thetotalcompliancecanbe
calculatedbydividingthetidalvolumebythechangeinpressureneededtodeliverthebreath(egthedifferencebetweenend-inspiratorypressureandend-expiratorypressure).Inthiscase,thecomplianceisequaltothetidalvolumeof500mLdividedbythe
differencebetweenend-inspiratoryandend-expiratorypressure(25cmH
^
O-5cmH
^
O=2DcmH
^
O),whichequals25mL/cmH
2
O.Complicationsofpositive-pressuremechanicalventilationincludebarotrauma(pressure-inducedinjury).Thepressure
experiencedbythelungtissueisthetranspulmonarypressure,whichisthedifferencebetweenthealveolarpressureandthepleuralpressureThealveolarpressureisdeterminedbytheventilator.ThepleuralpressureischallengingtomeasuredirectEyandis
typicallyestimatedbyutilizinganesophagealballoon.
IncorrectAnswers:A.6,C.DrandF.
ChoicesA.B,andCareincorrectastheyrepresenttheinverserelationshipbetweenthechangeinvolumeandthechangeinpressure(pressure/volume=F
^
O/mL)indescribingcompliance,whichshouldbethechangeinvolumedividedbythechangein
pressure.
ChoiceDisobtainedbydividingthetidalvolumebytheend-inspiratorypressureof25cmH
2O.Thisdoesnotrepresentthechangeinpressureofthesystemduringtherespiratorycycle,whichmustaccountfortheend-expiratorypressure.
ChoiceFisobtainedbyusingthedifferencebetweentheend-inspiratorypressureandthepleuralpressureasthechangeinpressureofthesystem.Thisvaluerepresentsthetranspulmonarypressureattheendofinspiration,whichdeterminestheriskofsuffering
barotrauma,butdoesnotprovideinformationaboutthetotalrespiratorysystemcompliance.
EducationalObjective:Complianceisthemeasureofthechangeinvolumeofasysteminresponsetoachangeinpressure.Therespiratorysystemcompliance,inthesettingofmechanicalventilation,canbedeterminedbydividingthetidalvolumedeliveredbythe
differencebetweentheend-expiratoryandend-inspiratorypressures.
0 0 ^
A oo
Previous Next ScoreReport LabValues Calculator Help Pause

ExamSection3:Item30ol50 NationalBoardofMedicalExaminers^
ComprehensiveBasicScienceSelf-Assessment
V
30.A5-month-oldgirlisbroughttothephysicianbecauseofa3-dayhistoryofTloppiness
7
'whenshesits.Shealsohashaddifficultyfeedingandirritabilitysincetheageof1month.Sheisatthe3rdpercentileforlength
;50thpercentileforweight,and
3rdpercentileforheadcircumference.Hertemperatureis36°C(963"F),pulseis100/min,respirationsare29/min
:andbloodpressureis30/50mmHg.Neurologicexaminationshowsbilateralhorizontalnystagmus,decreasedextensortone
:hyperreflexia
andankleclonus.Babinskisignispresentbilaterally.RespiratorychainenzymestudiesonculturedfibroblastsshowdecreasedcomplexI!activity.Themostlikelycauseofthesefindingsisadefectinwhichofthefollowingprocesses?
A)Couplingofelectrontransporttoprotongradientformation
B)CouplingofprotongradienttoATPsynthesis
C)Oxidationofsuccinatetofumarate
D)Reductionofpyruvatetolactate
E)TransferofelectronsfromNADHtocoenzymeQ
F)Transferofelectronsfromubiquinoltocytochromec
CorrectAnswer:C.
Leighsyndrome,orsubacutenecrotizingencephalopathy,iscausedbyavarietyofgeneticalterations,includingmutationsofcomplexesIll
:IV,orV,coenzymeQ
:orthepyruvatedehydrogenasecomplex.Itcommonlypresentsearlyinlifewithsevereneurological
dysfunctionandisassociatedwithseizures,developmentaldelay,psychomotorregression,nystagmus,ataxia,dystonia,andweakness.Laboratorystudiesmayshowlacticacidosis,andCTscanmayshownecrosisinthespinalcord,brainstem,thalamus,and
basalganglia.Mortalityishigh,withlifeexpectancyonlybeingseveralmonthsafterdiagnosisInthispatient'scase,herdiagnosisisduetoageneticmutationincomplexIIorsuccinatedehydrogenase,whichcatalyzestheoxidationofsuccinatetofumaratewith
theassistanceofFAD*Typically,theFADH
2
formedfromtheconversiontransferselectronstoaniron-sulfurprotein,wheretheyarethentransferredtocytochromebandsubsequentlyubiquinone.Ubiquinone(coenzymeQ)carriestheelectronstocomplexIIIof
theelectrontransportchain(ETC).TheelectronsarethenusedtogenerateATPviaATPsynthase.AsthisadditionofelectronsoccurslaterintheelectrontransportcyclethantheelectronsofNADH(whichjoinatcomplexJ),onemoleculeofFADH
2
creates1.5
moleculesofATP.whereasonemoleculeofNADHcreates2.5moleculesofATP.
IncorrectAnswers:A.B,D.E,andF.
Couplingofelectrontransporttoprotongradientformation(ChoiceA)occursatcomplexIandcomplexIII,whichareresponsibleforbuildingtheprotongradientthatallowsATPsynthasetogenerateATP.ItdoesnotoccuratcomplexII,whichaccountsforthe
decreasedenergyproductionfromFADH
2introductiontotheETC.
CouplingofprotongradienttoATPsynthesis(ChoiceB)occursatcomplexV,orATPsynthase.TheflowofprotonsdowntheirelectrochemicalgradientintothemitochondrialmatrixprovidestheenergyrequiredforproductionofATPviaoxidativephosphorylation.
Reductionofpyruvatetolactate(ChoiceD)iscatalyzedbylactatedehydrogenase,utilizingNADHasaprotondonor.Pyruvateisreducedtolactateinanaerobicmetabolism,whereasitentersthetricarboxylicacidcycleinaerobicmetabolismtogenerateNADH,
PADH
2Icarbondioxide,andguanosinetriphosphate.
TransferofelectronsfromNADHtocoenzymeQ(ChoiceE)occursincomplexI,withthetransfergeneratingaprotongradient.NADHdoesnotentertheETCviacomplexII.
Transferofelectronsfromubiquinoltocytochromec(ChoiceF)occursatcomplexIII.withthetransfergeneratingaprotongradientIhispatienthasreducedactivityofcomplex,ratherthancomplexIII.
EducationalObjective:ComplexIIoftheETC,orsuccinatedehydrogenase,generatesFADH
2viatheoxidationofsuccinatetofumarate.TneelectronsfromFADH
2thenentertheETCtogenerateATPviaoxidativephosphorylation.Deficienciesincomplex
causesevereneurologicaldisease,suchasLeighsyndrome.
can
0 1? & &fi
*
Previous Next ScoreReport LabValues Calculator Help Pause

ExamSection3:Item31ol50 NationalBoardofMedicalExaminers^
ComprehensiveBasicScienceSelf-Assessment
Y
31.A28-year-oldwomanwhoworksasaglassblowerhasrecentlysustainedburnsonherhandstautfeelslittlepain.Neurologicexaminationshowsdecreasedpainsensationinbothupperextremitiesandweaknessandatrophyoftheintrinsicmusclesofboth
hands.Whichofthefollowingbestexplainsthesefindings?
A)Cysticlesioninvolvingmidlinestructuresandbothventralhornsinthecervicalenlargement
B)Demyefinationoftheleftcorticospinaltractatalllevelsofthecord
C)LesionoftheleftanterolateralquadrantatC-2
D)Lassofmyelininthedorsalcolumnwhitematter
E)Lossofmyelininthelateralcolumnwhitematterandlossofventralhornmotoneurons
CorrectAnswer:A.
Thispatientpresentingwithhypoalgesia,weakness,andatrophyoftheupperextremitiesmostlikelyhassyringomyelia,whichfeaturesacysticlesioninvolvingmidlinespinalcordstructuresandbothventralhornsinthecervicalenlargement.Syringomyeliainvolves
aprogressivelyexpandingcavityinthecervicalcentralspinalcanalandmaybeassociatedwithcongenitalmalformations(eg,Chiarimalformationtype\)spinalcordinfections,autoimmunediseases(eg,multiplesclerosis),neoplasms,orspinalcordtrauma.
Syringomyeliatypicallydamagesspinothalamicfibersdecussatingintheanteriorwhitecommissure.Consequently,patientstypicallypresentwithdeficitsinpainandtemperaturesensationinthebilateralupperextremities.Sincetheascendingspinothalamictracts
inthelateralspinalcordareunaffected,sensationinthelowerbodyisintact.Thecysticlesioncanexpandtothebilateralanteriorhorns(thelocationofthecellbodiesoflowermotorneurons).Therefore,patientsmayalsopresentwithalowermotorneuron-pattern
ofdysfunctioninthebilateralupperextremities(eg,decreasedmuscletone,hyporeflexia,earlymuscleatrophy).
IncorrectAnswers:B,C,D,andE.
Demyefinationoftheleftcorticospinaltractatalllevelsofthecord(ChoiceB)wouldresultinherniparesisthatsparestheface.Uppermotorneuronsigns(eg,increasedmuscletone,hyperreflexia.delayedatrophy)wouldpredominate,andsensorydeficitswouldnot
bedemonstrated.
AlesionoftheleftanterolateralquadrantatC-2(ChoiceC)wouldaffecttheleftspinothalamictractandleftanteriorhorn.Patientswouldpresentwithhemisensorydeficitsinpainandtemperatureinferiortotheneckandweaknessofthetrapeziusandrectuscapitis
muscles.
Thelossofmyelininthedorsalcolumnwhitematter(ChoiceD)wouldaffectthedorsalcolumn-mediallemniscuspathwayandleadtothelossofsensationoffinetouch,pressure,proprioceptionandvibration.Thispatientinsteaddemonstratedlocalizedmotor
dysfunctionandlossofpainandtemperaturesensation.
Thelossofmyelininthelateralcolumnwhitematterandlossofventralhornmotoneurons(ChoiceE)wouldaffectthespinothalamictractsandlowermotorneuroncellbodies.Patientswouldpresentwithalowermotorneuron-patternofdysfunction(eg,decreased
muscletone,hyporeflexia,earlymuscleatrophy)anddeficitsinpainandtemperaturesensation.
EducationalObjective:Syringomyeliafeaturesacysticcavityinthecervicalcentralspinalcanal,whichaffectsmidlinespinalcordstructuressuchastheanteriorwhitecommissure(decussationofthespinothalamictracts)andtheanteriorhorns(cellbodiesoflower
motorneurons).Patientswithsyringomyeliamaypresentwithdeficitsinpainandtemperaturesensationandalowermotorneuron-patternofdysfunctioninthebilateralupperextremities.
Previous Next ScoreReport LabValues Calculator Help Pause

ExamSection3:Item32ol50 NationalBoardofMetlicaiExaminers^
ComprehensiveBasicScienceSelf-Assessment
y
32.AS7-year-oldwomanhashaddecreasedpainandtemperaturesensationintheleftlowerextremityfor1week.Themostlikelycauseisdamagetowhichofthefollowinglabeledsitesinthecrosssectionofthespinalcord?
Right
C
A)
B)
C)
D)
E)
F)
G}
H)
I)
J)
CorrectAnswer:E
Damagetotheright-sidedspinothalamictract(labelE}inthiscervicalspinalcordsection(asindicatedbytheovoidshapeandpredominanceofwhitematter)wouldcauseleft-sidedimpairmentofpainandtemperaturesensation.Thespinothalamictract
originatesfromthesensorynerveending,travelstothedorsalrootganglionwhereitscellbodyislocated,entersthespinalcord,andsynapseswiththesecond-orderneuronintheipsiateraldorsalhornofthespinalcord.Thesecond-orderneuronthen
decussatesattheanteriorwhitecommissureofthespinalcord(ascendingtwolevelsduringitsdecussation),andthenascendswithinthecontralateralspinothalamictracttotheventralposterolateralnucleusofthethalamus.Thethird-orderneuronthenascends
totheprimarysensorycortex.Aninjurytothespinothalamictractinthespinalcordresultsincontralateralneurologicdeficitsasthesecond-orderneuronhasalreadydecussatedfromthesideofthefirst-orderneuronsensorynerveending.Thespinothalamictract
isorganizedsomatotopicailyinthespinalcordsuchthatthelateralregioncontrolspainandtemperatureandtheanteriorregioncontrolscrudetouchandpressure.
IncorrectAnswers:A.B.C.D,F.G,H,I,andJ.
LabelsAandJrepresentthefasciculigracilisofthedorsalcolumn-mediallemniscustract,whilelabelsBandirepresentthefasciculicuneatusofthedorsalcolumn-mediallemniscustract.Thedorsalcolumn-medialemniscustractoriginatesfromthesensory
nerveendingandascendsipsilaterallywithinthefasciculusgracilisorcuneatusuntilitsynapseswiththesecond-orderneuronintheipsilateralnucleusgracilisorcuneatus,respectively.Thesecond-orderneuronthendecussatesinthemedullaandsynapsesin
thethalamus.Thethird-orderneuronthenascendstoterminateintheprimarysensorycortex.Dorsalcolumn-mediallemniscustractlesionswithinthecervicalspinalcordwillleadtoipsilateraldeficitsinpressure,vibration,finetouch,andproprioception.The
fasciculigraciliscontainascendingsensoryneuronsforthelowerbody,whilethefasciculicuneatuscontainascendingsensoryneuronsfortheupperbody.
LabelsCandHrepresentthelateralcorticospinaltracts.Uppermotorneuronsofthelateralcorticospinaitractoriginateintheprimarymotorcortex,descendipsilaterallythroughtheinternalcapsule,cruscerebriofthemidbrain,ventralpans,andmedullary
pyramids,wheretheythendecussateinthecaudalmedulla,andthendescendcontralaterallyinthelateralcorticospinaltractofthespinalcordtosynapsewiththecontralaterallowermotorneuron.Lateralcorticospinaltractlesionswithinthecervicalspinalcord
willleadtoanipsilateraluppermotorneuronpatternofmotorweakness,includinghyperreflexiaandincreasedmuscletone.
LabelsDandGrepresenttheanteriorhornsofthespinalcord,whichcontainthecellbodiesoflowermotorneurons.Lesionsoftheanteriorhornwouldresultinanipsilaterallowermotorneuronpatternofskeletalmuscleweaknessincludinghyporeflexiaand
decreasedmuscletone.
LabelFrepresentstheleft-sidedspinothalamictract.Lesionsofthisareawouldresultinimpairmentinpainandtemperaturesensationintherightlowerextremity.
EducationalObjective:Spinalcordlesionsofthelateralspinothalamictractcausecontralateraldeficitsinpainandtemperature,whereaslesionsoftheanteriorspinothalamictractcausecontralateraldeficitsincrudetouchandpressure.Alternatively,spinalcord
lesionsofthecorticospinalanddorsalcolumn-mediallemniscustractsproduceipsilateralneurologicdeficits.
4
*
m
gs £
Previous Next ScoreReport LabValues Calculator Help Pause

ExamSection3:Item33of50 NationalBoardofMedicalExaminers^
ComprehensiveBasicScienceSelf-Assessment
Y
33.A7S-year-oldmanisbroughttotheemergencydepartmentafterfallingfromaladder.Onarrivalheislucid.Physicalexaminationshowsaseverescalpcontusionandlaceration.Neurologicexaminationshowsno
abnormalities.Onthesecondday
:heisstuporousandhasaheadache.Examinationshowsslightweakness,hyperreflexia.andanextensorplantarreflex,aliontheright.ACTscanoftheheadisshown.Themost
likelycauseofthesefindingsistraumatowhichofthefollowingvascularstructures?
A)Arteryinthesubarachnoidspace
B)Duralbridgingveins
C)Middlecerebralartery
D}Middlemeningealartery
E)Superiorsagittalsinus
CorrectAnswer:B.
Headtrauma
:theuseofanticoagulants
;physicalabuse
:alcoholusedisordercerebralatrophyandmalignancyareassociatedwiththeriskofdevelopingasubduralhematoma(SDH).OnCTscamSDHappearsasacrescent-shapedfluidcollectionabuttingthe
internalsurfaceoftheskullnotboundbysuturelines.Insomecases,theSDHmaybelargeenoughtocompressadjacentbrainparenchyma;thedisplacementofthecerebralhemispherecanresultinneurologicdeficits,alteredmentalstatus
:seizures,herniation
syndromes,andsignsofincreasedintracranialpressure.Patientsmaymanifestuppermotorneuronexamfindingssuchasspasticity,hyperreflexia,andextensorplantarreflex(Babinsk;sign).Elderlypatientsareatriskforcerebralatrophy,whichplacestraction
onduralbridgingcorticalveins.Inthissetting,minortraumacanresultinshearingoftheveinsleadingtohemorrhage.Untreated,thesubduralhematomapresentstheriskofcontinuingtogrow.Tieintracranialcompartmentisafixedspacewithinarigidskull
pathologiclesionssuchasSDHcanresultinmidlineshiftandherniationofcentralnervoussystemstructuresiflargeenough.Whensuchmasseffectispresentoratriskofdeveloping,surgicalinterventionssuchasadecompressiveburrholeorcraniotomymay
beindicated.
IncorrectAnswers:A,C,D,andE.
Subarachnoidhemorrhagescanbeduetoaneurysmalruptureofanarteryinthesubarachnoidspace{ChoiceA)orfromtraumaticinjury.Patientsmaypresentwithanacute,severeheadache,stupor,orcomainseverecases.CIscanoftheheaddemonstrates
hyperdensebloodproductsinthecerebralsuiciandbasalcisterns,notthecrescent-shapedcollectionseeninSDH.
Themiddlecerebralartery(ChoiceC)isabranchoftheinternalcarotidarterythatsuppliesbloodtothetemporallobe,frontallobe,andparietallobe.Itisunlikelytobeinjuredintrauma.Inthecaseofdissectionorthrombosisofthemiddlecerebralartery,it
classicallypresentswithcontralateralmotorandsensorydeficitsofthefaceandupperextremity,andaphasia(eg,Wernicke,Broca)ifinvolvingthedominanthemisphere.
Epiduralhematomaisclassicallyassociatedwithtraumatothemiddlemeningealartery(ChoiceD).EpiduralhematomaspresentonCTscanaslens-shaped,bi-convexhyperdensecollectionsabuttingtheskult.Theyoftencompressadjacentbrainandarebound
byduralsutures.
Thesuperiorsagittalsinus(ChoiceE)functionsinvenousdrainageofthebrainandliesalongthesuperiormidlinebetweenthetwocerebralhemispheres.Superiorsagittalsinusinjuryisrareandcanbeduetoskullfracture.Itcanhaveasimilarappearancetoan
epiduralhematoma.
EducationalObjective:Elderlypatientsareatriskforcerebralatrophy,whichcausestractiononduralbridgingcortical
'
veinssuchthatminortraumacanresultinvenousinjuryleadingtoasubduralhematoma.Subduralhematomasappearascrescent-shapedfluid
collectionsabuttingtheinternalsurfaceoftheskull,notboundbysuturelines.
si©
E5
Previous Next ScoreReport LabValues Calculator Help Pause

ExamSection3:Item34ol50 NationalBoardofMedicalExaminers^
ComprehensiveBasicScienceSelf-Assessment
Y
34.Ahealthy24-year-oldwomanparticipatesinastudytodeterminetheroleofghrelininappetiie.Sheisgivenfreeaccesstofoodduringthestudy.Thegraphshowsplasmaconcentrationsofghreiininthewomanduringthestudyperiod.Whichofthe
followinglabeledpointsonthecurvemostlikelyrepresentstheconsumptionofameal?
c
o
(0
OJ
c
o
u
<u
OS
E
ifl
(0
CL
Time
A)
B)
C)
D)
E)
CorrectAnswerB.
Ghrelinisancrexigenicpeptidehormoneproducedbygastrointestinalenteroendocrinecells.Ghrelinactivitystimulateshungerandactivateshypothalamicandpituitaryneuralrewardcircuitsbybindingtogrowthhormonesecretagoguereceptor1A.Ghrelin
concentrationincreasesduringthepreprandialperiodandbeginstodeclineimmediatelyaftertheingestionoffoodduringthepostprandialperiod.Ghreliinconcentrationmayalsobeinfluencedbycircadianrhythm.Ghrelinplaysaroleinobesityinpatientswith
Prader-Willisyndrome,inwhomghrelinconcentrationisincreased.Ghrelinisantagonizedbylepfin.whichisproducedbyadipocytesandenterocytesinthefedstateinordertoreducehungerandincreasesatiety.
IncorrectAnswers:A.C,D,andE.
ChoiceAdemonstratesanincreasingbutnotyetmaximalghrelinplasmaconcentration.whichisobservedpriortotheingestionofamealinordertostimulatehunger.
ChoiceCdemonstratesadecreasingghrelinplasmaconcentration,whichIsobservedimmediatelyafteringestionofamealinordertopromotesatiety.
ChoicesDandErepresentnon-maximalfluctuationsinghrelinserumconcentration;ghrelinconcentrationisthehighestinthepreprandialperiod.
EducationalObjective:Ghrelinisanorexigenicpeptidehormonethatstimulateshungerandactivateshypothalamicandpituitaryneuralrewardcircuits.Ghrelinconcentrationincreasesduringthepreprandialperiodpriortoamealandbegintodeclineimmediately
aftertheingestionoffoodduringthepostprandialperiod.
O ©9
&ft? rT:
Previous Next SconeReport LabValues Calculator Help Pause

ExamSection3:Item35ol50 NationalBoardofMetlicaiExaminers^
ComprehensiveBasicScienceSelf-Assessment
y
35.A2-month-oldgirlisbroughttotheemergencydepartmentbyhermotherbecauseofdifficultybreathingforShours.Physicalexaminationshowscyanosis.Aloudsystolicmurmurisheard.Laboratorystudiesshowhypocalcemiaandlymphopenia.
EchocardiographyshowstetralogyofFallot.ThediagnosisofDiGeorgesyndromeismade,andakaryotypeisordered.Chromosomalanalysisbyhigh-resolutionG-bandtngisreportedas46,XX.Whichofthefollowinggeneticmechanismsisthemostlikely
causeofthispatient'sanomalies?
A)Deletion
B)Nondisjunction
C)Paracentricinversion
D)Pericentricinversion
E)Reciprocaltranslocation
F)Robertsoniantranslocation
G)Trinucleotiderepeat
H)Uniparentaldisomy
CorrectAnswer:A.
DiGeorgesyndrome
:a22q11deletionsyndrome,presentsasaresultoffailureofthethirdandfourthbranchialpouchestodevelop.Thethirdbranchialpouchgivesrisetothethymusandinferiorparathyroidglands
:andthefourthbranchialpouchgivesrisetothe
superiorparathyroidglands.Classically,DiGeorgesyndromepresentswithimmunodeficiencyduetofailureofthymusdevelopment(thymicaplasia),hypocalcemiafromfailureofparathyroidglanddevelopment,structuralcardiacdefectssuchastetralogyof
Fallot,andcraniofacialmalformations.Commonphysicalexaminationfindingsincludeacleftpalateandlow-setears:however,theexpressivityvariesandcombinationsofabnormalpalate,facialandcardiacmalformationsareobserved.Anincreased
susceptiblitytopredominantlyviralandfungalinfectionsistheresultofimmunodeficiencyfromaplasiaofthethymus(thesiteofmaturationofTlymphocytes).Thedefinitivediagnosisrequiresfluorescencein-situhybridizationtodetectthedeletedDNAsegment
withafluorescentprobe.Treatmentissupportive,asthereisnocureforthecondition.
IncorrectAnswers:B,C
:D
:E
;F,G
:andH.
Nondisjunction(ChoiceB).sthemechanismbywhicherrorsinchromosomenumbersoccurduetofailureofpairedchromosomestoseparateappropriatelyduringcelldivision.ExamplesofdisordersincludeDownsyndrome(trisomy21).Edwardssyndrome
(trisomy1S)
:andPatausyndrome(trisomy13).
Paracentricinversions(ChoiceC)andpericentricinversions(ChoiceD)Occurwhentwobreaksinachromosomeresult-nanexcisedfragmentthatthenrejoinsthesamechromosomewiththefragmentreversedfromitsoriginalorientation.Aparacentricinversion
doesnotinvolvethecentromere(thefragmentisasectionofoneofthechromosomearms)whiteapericentricversiondoesinvolvethecentromere.
Reciprocaltranslocation(ChoiceE)occurswhenfragmentsfromtwochromosomesexchangeplaces,whichresultsinauniquederivativepairofchromosomes.
Robertsoniantranslocation(ChoiceF)occurswhentwodifferentchromosomesbreakatthecentromereandrecombinewiththebreakfromtheother.Thismostoftenhappenswithacrocentricchromosomes,whichhavealongarmcontainingmostofthegenetic
materia!andashortarm.Aderivativepairformsfromthelongarmsjoiningtogetherandshortarmsjoiningtogether.
Trinucleotiderepeat(ChoiceG)describesthreenucleotidesinrepetitiveseriesthataresusceptibletoreplicationerrorsGeneticconditionsthatarisefromtheseerrorstendtodisplaygeneticanticipation.ExamplesincludeHuntingtondisease,myotonicdystrophy
:
andfragileXsyndrome.
Uniparentaldisomy(ChoiceH)istheinheritanceofapairofnonidenticalchromosomesfromasingleparent.Genomicimprintingresultsinmethylationandsuppressionofgenesdependentontheparent-of-origin.UniparentaldisomyoccursinAngelman
syndromeandPrader-Willisyndrome.
EducationsObjective;Deletionofgeneticmaterialfromachromosomeresultsinmissinggenesthatmaycausecongenitaldisorders.DiGeorgesyndromeisanexamplethatresultsfromamissingsegmentofchromosome22.
oo m e 00
Previous Next SconeReport LabValues Calculator Help Pause

ExamSection3:Item36ol50 NationalBoardofMedicalExaminers^
ComprehensiveBasicScienceSelf-Assessment
Y
36.AGS-year-oldwomanhashadprogressiveshortnessofbreathwithexertionsinceshestartedtreatmentwitheyedropsforglaucoma2weeksago.Pulmonaryfunctiontestsshowadecreasedforcedexpiratoryvolumein1sec(FEV
^
.Thelungsareclearto
auscultation.Theeyedropsaremostlikelyfromwhichofthefollowingclassesofdrugs?
A)a-Adrenergicagonist
B)p-Adrenergicagonist
C)p-Adrenergicblocker
D)Carbonicanhydraseinhibitor
E)Cholinesteraseinhibitor
F)Prostaglandinanalog
CorrectAnswer:C.
Timololisanon-sefectivep-adrenergicreceptorblockerandisusefulforthetreatmentofprimaryopenangleglaucoma.Blockadeofp-adrenergicreceptorsintheciliarybodyleadstothereducedformationofaqueoushumorandadecreasedintraocularpressure,
whichisusefulfarslowingtheprogressionofopticnervedamage.Systemicsideeffectsoftopicaltimololeyedropsincludetheexacerbationofasthma,asinthispatientaswellasbradycardia,decreasedexercisetolerance,erectiledysfunction,andmaskingof
hypoglycemiainpatientswithdiabetesmellitus.
IncorrectAnswers:A.BpD,E,andF.
a-Adrenergicagonists(ChoiceA)includethemedicationsbrimonidineandapraclonidine(whichtargeta
2
receptors),whichareusedinthetreatmentofprimaryopenangleglaucoma.a
2
-Adrenergicagonistsmediatetheireffectsbydecreasingaqueoushumor
production.Systemicsideeffectsofthesemedicationsincludedrymouth,bradycardia,andhypotension.a
2
-Adrenergicagonistscancausethebloodbrainbarrierandcancausehypotoniaandapneainchildren.Bycontrast,a
^
adrenergicagonistswhenusedinthe
eyepromotemydriasis.
p-Adrenergicagonists(ChoiceB)includedrugssuchasalbuterol,whichareusefulforthetreatmentofasthma.P
-Adrenergicagonistsarenotcommonlyusedintopicalformulationsandwouldnotbeusefulforthetreatmentofglaucoma.
Carbonicanhydraseinhibitors(ChoiceD)includetopicalglaucomamedicationssuchasdorzolamideandbrinzolamide,aswellasoralacetazolamide.Thesedrugsreducetheformationofaqueoushumor.Systemicsideeffectsincludeparesthesia,hypokalemia,
acidosis,andincreasedriskofnephrolithiasis.
CholinesteraseInhibitors(ChoiceE)thatareusedforglaucomaincludeechothiophate,whichlowersintraocularpressurebyincreasingoutflowofaqueoushumorthroughthetrabecularmeshwork.Thisdrugclassisrarelyusedforthetreatmentofglaucoma.
Prostaglandinanalogs(ChoiceF)includelatanoprost,bimatoprost,andtravaprost,whichlowerintraocularpressurebyincreasingoutflowofaqueoushumorthroughtheuveosderalpathway.Topicalprostaglandintherapyhasfewsystemicsideeffects.Localocular
sideeffectsincludelengtheningofeyelashes,changeiniriscolor,andatrophyandhyperpigmentationofperiorbitalsofttissues.
EducationalObjective:p-adrenergicreceptorblockers,suchastimolol,areusefulforthetreatmentofglaucomabyloweringintraocularpressure.Systemicsideeffectsincludetheexacerbationofasthma,bradycardia,decreasedexercisetolerance,erectile
dysfunction,andmaskingofhypoglycemia.Non-selectiveP-adrenergicreceptorblockersarerelativelycontraindicatedinpatientswithahistoryofasthma.
© © ©
AIB
Previous Next ScoreReport LabValues Calculator Help Pause

ExamSection3:Item37of50 NationalBoardofMedicalExaminers^
ComprehensiveBasicScienceSelf-Assessment
Y
37.Aphysicianissadbecausehehastoinformapatientofrecenttestresultsthatindicateprogressionofcarcinomatotheterminatphase.Whenthepatientseesthephysician'sface,hebeginstocryandsays,"It'sbadnews,isn'tit?"Whichofthefollowing
responsesbythephysicianismostappropriate?
A)"HowhaveyoubeensincethelasttimeIsawyou?
'
r
B)"Let'stalkaboutthepositiveaspectsfirst."
C)"Lookonthebrightsideofthings."
D)"Tellmehowyouarefeeling."
E)"Thereareotherpeoplewhohaveitalotworsethanyou."
F)"Yes,itis."
G)"You'vehadseveralyearsbetteroffthanmanyotherswiththisdisease."
CorrectAnswer:F.
Inpatientswhoexpressthedesiretobeinformedofaterminaldiagnosis,physiciansshoulddeliverbadnewsdirectlyandcompassionately.Thisphysicianshouldanswerthepatient'sdirectquestion.Oncethepatienthasbeeninformedofthediagnosis,the
physicianshouldvalidatethepatient'semotionsbylisteningreceptively,usingreflectivestatements,inferringemotionsfromtheexpressedcontent,andnormalizingthepatient'sexperience.Thephysicianshouldalsodelivereducationandexplainthenextsteps
usingsmallpiecesofinformation.
IncorrectAnswers:A,6
:C,D,EandG.
SmaItalksuchasaskinghowthepatienthasbeen{ChoiceA)wouldconstituteanavoidantapproachtoansweringthepatient'squestionandelicitingthepatient'semotionsThephysicianshouldinsteadfocusonthemostemotionallysalienttopic,whichFSthebad
news.Thephysiciancanthensupportthepatient'semotions.
Framingthenewspositively(ChoicesB,C
:E,andG)wouldavoidansweringthequestionandmayinvalidatethispatient
'snaturalemotionalresponse.Encouragingwordswouldminimizethesituationandprobablyfeellikeplatitudestothepatientgiventhegravity
ofthenews.Allowingthepatienttofee!sadnessandvalidatingtheseemotionswilhelpthepatientprocessthenewsandultimatelymoveforwardtomakeend-of-lifepreparations.
Invitingthepatenttosharehowheisfeeling(Choice0)wouldbeappropriatelaterintheconversationsothatthephysiciancansupportthepatient'semotions.First,thephysicianshouldanswerthepatient'squestionaboutthenews.
Educationa-Objective:Forpatientswhodesiretobeinformedofaterminaldiagnosis,physiciansshoulddeliverbadnewsdirectly.Oncethepatienthasbeeninformed,thephysicianshouldvalidatethepatient'semotions.
00# 0
f*
Previous Next ScoreReport LabValues Calculator Help Pause

ExamSection3:Item33ol50 NationalBoardofMedicalExaminers^
ComprehensiveBasicScienceSelf-Assessment
Y
38.An18-year-oldmancomestothephysicianbecauseofnonpittingfacialedemaanddifficultybreathingseveralhoursafteradentalprocedure.Hehasahistoryofunexplainedepisodesofabdominalpain.HisserumC2concentrationisfoundtobemarkedly
decreasedWhichofthefollowingbestexplainsthispatient'sillness?
A)Absentpyrin
B)DecreasedC1esteraseinhibitors
C)Increasedurineporphyrins
D)Penicillinallergy
E)Superantigenproduction
CorrectAnswer:B.
HereditaryangioedemaisadisorderofthecomplementsystemthatresultsfromdecreasedG1esteraseinhibitorsandischaracterizedbyoveractivationofthekallikrein-kininpathway.Kalfikreinactivatesbradykinin,whichcausesvasodilationandincreases
vascularpermeability.C1esteraseisresponsibleforkallikreininhibition.Unregulatedkallikreinactivityresultsinanexcessofbradykininandangioedema.Thedisordertypicalypresentsearlyinlifewithrecurrentepisodesofangioedema
:whichmayincludethe
face
;oropharynx
:extremities,orgastrointestinaltract.Symptomsdependupontheaffectedareaandincludedyspneasecondarytoupperairwayobstruction,abdominalpain
;andgastrointestinaldistress.Physicalexaminationdisplaysedemaandtheabsenceof
urticaria,whichcanhelpdistinguishhereditaryangioedemafromananaphylacticreaction.LaboratoryanalysisclassicallyrevealsdecreasedlevelsofC2andC4.DiagnosiscanbeconfirmedbydirectmeasurementofC1esteraseinhibitorconcentrationand
functionalassays.Airwaycompromisecanbelife-threateningandimmediatetreatmentisindicatedtosecuretheairway.Epinephrineandantihistaminesaretypicallyadministereduntilanaphylaxishasbeenruledout.Patientsshouldbetreatedwithplasma-derived
orrecombinantC1inhibitorassoonaspossible.
IncorrectAnswers:A.C
;D,andE.
Absentpyrin(ChoiceA)isthecauseoffamilialMediterraneanfever.Patientstypicallypresentinchildhoodwithrecurrentfever,abdominalpain
;andarthralgias.Itisnotassociatedwithfacialedema.
Increasedurineporphyrins(ChoiceC)suggestthepresenceofporphyria,whichisagroupofdisordersofhemesynthesis.Thesedisordersarecharacterizedbyaccumulationofhemeprecursorsandmaypresentwithdarkcoloredurine,gastrointestinaldistress,
andblisteringoftheskin.C2levelswouldnotbeaffected.
Penicillinalergy(ChoiceD)maypresentwithanaphylaxis,whichcanbedifficulttodistinguishfromhereditaryangioedemaintheacutesetting.ThepresenceofdecreasedC2levelsismoresuggestiveofhereditaryangioedema.
Superantigenproduction(ChoiceE)occursinseverecomplicationsofstaphylococcalandstreptococcalinfectionssuchastoxicshocksyndrome,scaldedskinsyndrome,andscarletfever.Superantigensstimulateamassiveimmuneresponseresultingin
widespreadinflammation.
EducationalObjective:HereditaryangioedemaresultsfromadeficiencyofC1esteraseinhibitorsandmaypresentwithangioedemaoftheface,extremities,oropharynx,andgastrointestinaltractduetoincreasedlevelsofbradykinin.
© © © oo
Previous Next ScoreReport LabValues Calculator Help Pause

ExamSection3:Item39of50 NationalBoardofMedicalExaminers^
ComprehensiveBasicScienceSelf-Assessment
Y
39.A67-year-oldwomanwithacutemyelogenousleukemiadevelopsfeverandchills6daysafterundergoinginductiontherapy.Hertemperatureis38.3aC(100.9T),andbloodpressureis110/60mmHg.Physicalexaminationshowsscatteredpetechiaeover
theupperandlowerextremitiesandtrunk.Laboratorystudiesshowaleukocytecountof100/mm
5(98%lymphocytesand2%monocytes)andplateletcountof4000/mm
5
.BloodculturesgrowPseudomonas
aeruginosaAdeficiencyofwhichofthefollowing
isthemostlikelypredisposingfactorfordevelopmentofbacteremiainthispatient?
A)Complement
B)Immunoglobulin
C)Lymphocytes
D)Macrophages
E)Platelets
F)Segmentedneutrophils
CorrectAnswer:F.
Acutemyelogenousleukemiaisanaggressivebonemarrowmalignancythatprimarilyaffectsadults.Itischaracterizedbythepresenceofcirculatingblastsofthemyeloidlineage.Patientsareclassifiedintodifferentprognostictiersbaseduponthepresenceor
absenceofspecificmutations.Thegoalofinitialtherapyistoinducecancerremission,generallywithcytarabineanddaunorubicin.Complicationsofboththeleukemiaitselfandinductiontherapyincludepancytopeniabecauseofthesuppressionofnormalbone
marrow.Patientsmaypresentwithfatigueanddyspneasecondarytoanemia,bleedingandpetecrraesecondarytothrombocytopenia,andinfectionsduetoneutropenia.Neutropenicfeverisaiife-threateningmedicalconditionthatreferstoanacuteinfectionand
feverpresentinginthesettingofadeficiencyofsegmentedneutrophils,definedasanabsoluteneutrophilcountlessthan500/mm
3
.Patientsmaynotdisplaytypicalsignsofinfectionduetoaseverelyimpairedimmunesystem.Patientsrequirehospitalizationand
broad-spedrumantibiotictherapythatincludesadivityagainstPseudomonasaeruginosa.
IncorrectAnswers:A.B.CaD,andE
Complement(ChoiceA)deficienciesarenottypicallyassociatedwithacutemyelogenousleukemiaascomplementissynthesizedintheliver.Compementdeficienciesareassociatedwithrecurrentneisserialinfections.
Immunoglobulin(ChoiceB)andlymphocyte(ChoiceC)deficienciesareassociatedwithcongenitaldisordersoflymphocytessuchasX-linkedagammagobulinemia,selectiveIgAdeficiency,commonvariableimmunodeficiency,severecombinedimmunodeficiency,
andthymicaplasia.Lymphocytesandmacrophages(ChoiceD)arealsosuppressedinthesettingofcytotoxicchemotherapy,butanimpairedinitialinnateimmuneresponsewithneutropeniaisthemajorpredisposingfadorforbaderemia.
Platelet(ChoiceE)deficienciesresultinbleedingcomplications,whichmaymanifestasbruises,petechiae.mucosalbleedingfromthegingiva,epistaxis,and/ormoreseveregastrointestinalhemorrhageormenorrhagia.
EducationalObjective:Complicationsofacutemyelogenousleukemiaarerelatedtotheimpairedproductionofnormalcelllines.Thisisfurthercomplicatedbytheadministrationofcytotoxicchemotherapy,whichsuppressesnormalbonemarrowfunction.
Neutropenicfeverisamedicalemergencyandisdefinedasthepresenceofafeverinthesettingofanabsoluteneutrophilcountlessthan500/mm
3
.
© © ©
Previous Next ScoreReport LabValues Calculator Help Pause

ExamSection3:Item40ol50 NationalBoardofMedicalExaminers^
ComprehensiveBasicScienceSelf-Assessment
Y
40.A54-year-oldwomanwithchronickidneydiseasecomestothephysicianbecauseofa2-weekhistoryofitchyskin,increasinglyseverenausea
:anddecreasedurination.Currentmedicationsarehydrochlorothiazideandlisinopril.Herpulseis80/min
respirationsare14/min,andbloodpressureis160/90mmHg.Physicalexaminationshowsnoabnormalitiesexceptforexcoriationsfromitching.Laboratorystudiesshow:
Hemoglobin
Hematocrit
Meancorpuscularvolume
Leukocytecount
Plateletcount
9.8g/dL
30.1%
39pm
3
5600/mm
3
237.000/mm
3
Whichofthefollowingisthemostlikelycauseofthefindingsinthispatient?
A)Hypoalbuminemia
B)Hypothyroidism
C)Uremia
D)VitaminCdeficiency
E)VitaminKdeficiency
CorrectAnswer:C.
Gradualdeclineinkidneyfunctioninpatientswithchronickidneydiseasecanbeasymptomaticuntilpatientsmaypresentwithvolumeoverload,electrolyteabnormalities,anemia,hypertension,anduremia.Thepatienttakeshydrochlorothiazide,adiureticthat
inhibitssodiumchloridereabsorptioninthedistalcollectingtubuleofthenephron.Anadverseeffectofhydrochlorothiazideisincreasedlevelsofuricacid.Uremiacanpresentwithnonspecificsymptomsincludingnausea,vomiting,andpruritus.Onphysical
examination,patientsmayhaveasterixis,encephalopathy,andpericardialrubinpericarditis.Plateletcountmaybenormal,buturemialeadstoplateletdysfunctfonandprolongedbleedingtimewhichcanmanifestwitheasybruisingandmucocutaneousbleeding.
IncorrectAnswers:A,B.D,andE.
Hypoalbuminemia(ChoiceA)canbeassociatedwithnephroticsyndrome.Duetotheexcessiveproteinuriainnephroticsyndrome,serumoncoticpressureisdecreased,resultingindependentedemainthebuttocks,lowerback,andlegs.
Hypothyroidism(ChoiceB)presentswithweightgain,fatigue,constipation,coldintolerance,andmenstrualirregularities.Physicalexaminationmayrevealbradycardia,dry,edematousskin,anddelayedrelaxationofdeeptendonreflexes.Ihispatientdoesnot
demonstrateanysignsorsymptomstosuggesthypothyroidism.
VitaminCdeficiency(ChoiceD)causestheconstellationofsymptomsknownasscurvy,whichischaracterizedbypetechiae,perifollicularhemorrhage,bruising,poorwoundhealing,andshort,curly,fragilehairs.VitaminCdeficiencyresultsfromdietary
insufficiency,whichisrareinresource-richsettings.
VitaminKplaysacriticalroleinthesynthesisofhepaticcoagulationproteins;itisoxidizedintheliverduringcarboxylationofglutamicacidresiduesoncoagulationfactorsllrVII,IX,X,andproteinsCandS.VitaminKdeficiency(ChoiceE)leadstocoagulopathy
withincreasedprothrombintimeandactivatedpartialthromboplastintimeandischaracterizedbyeasybruising.
EducationalObjective:Chronickidneydiseaseandagradualdeclineinkidneyfunctioncanleadtouremia.Uremiacanpresentwithnonspecificsymptomsincludingnausea,vomiting,andpruritus.
© © ©
Previous Next ScoreReport LabValues Calculator Help Pause

ExamSection3:Item41ol50 NationalBoardofMedicalExaminers^
ComprehensiveBasicScienceSelf-Assessment
41.An8-year-oldgirlisbroughttothephysiciantorawell-childexamination.Hermothersaysthatshehasbeenwellexceptforanoccasionalcold.Herimmunizationsareuptodate.Sheisatthe50thpercentileforheightand60thpercentileforweight.
Physicalexaminationshowsbreastbuddevelopmentandafewpubichairs.Themotheraskswhetherherdaughter'sdevelopmentisnormal.Whichofthefollowingisthemostappropriateinitialresponsebythephysician?
A)1amconcernedthatyourdaughtermayhaveprecociouspuberty.
B)
:,
lthinkthatyourdaughterislikelytobeginmenstruatinginthenextfewmonths."
G}1wouldliketoorderlaboratorystudiestoensurethatyourdaughter'sdevelopmentisnormal.
1
'
D)"Yourdaughter'sbreastdevelopmentissomewhatlessthanmightbeexpectedforherage.'
1
E)
:
iYourdaughter'sdevelopmentisnormal."
CorrectAnswer:E
This8-year-oldfemalepatientappearstobeundergoingnormalsexualmaturation.Twophysiologicaleventsexplaintheonsetofpuberty:adrenarche,ortheproductionofandrogensbytheadrenalgland,andgonadarche,thestimulationofthegonadstoproduce
estradiolbyfollicle-stimulatinghormoneandluteinizinghormone.Thelarche
;orthedevelopmentofbreastbuds
;ismediatedbygonadalestradiolandistypicallythefirstphysicalchangeinpuberty
:startingasearlyas8yearsold.Pubarche,orthedevelopmentof
pubichairasaresultofadrenalandrogens
;istypicallythesecondphysicalchange.Boththelarcheandpubarcheusuallyoccurinsexualmaturityrating(SMR)stage2ofsexualmaturity.Alternatively,menarche,orthefirstmenstruation,whichresultsfromthe
effectsofgonadalestradiolontheendometriallining,typicallyhappensafterthelarcheinlateSMR3.Thegrowthspurttypicallyoccursafewmonthsbeforemenarche.Thispatient'saverageheightandweightreflectthatshehasnotyetstartedhergrowthspurt.
incorrectAnswers:A.B,C.andD.
Thispatient'ssexualmaturityisnotprecociousordelayed(ChoicesAandD)
;sonolaboratorytestingisindicated(ChoiceG).ThispatienthasenteredSMRstage2.whichmaydevelopasearlyas8yearsold(oraslateas15yearsold).Physiciansshouldconsider
laboratoryevaluationofprecociouspubertyforchildrenpresentingwithsecondarysexualdevelopmentearlierthan7orSyearsold.
Menarchetypicallyfollowsthelarcheafterafewyears,notafewmonths(ChoiceB).Thefirstmenstrualperiodisrelatedtoestradiolstimulationoftheendometrialliningratherthanovulation.whereassubsequent,regularlyoccurringmenstrualperiodstypically
resultfromovulation.
EducationalObjective:Earlysexualmaturation,whichbeginsasearlyasSyearsold,typicallyincludesthelarcheandpubarche.Menarcheusuallyoccursafewyearsafterthelarche.
O0
It
00
Previous Next ScoreReport LabValues Calculator Help Pause

ExamSection3:Item42ol50 NationalBoardofMedicalExaminers^
ComprehensiveBasicScienceSelf-Assessment
42.An11-year-oldboyhashadpersistentpaininhisrightkneesincehetwisteditinasoccergame3weeksago.Physicalexaminationshowsatenderareainthedistalfemurbutnoperceptiblemass.Anx-rayofthefemurshowsanosteolyticmassthathas
erodedthroughthecortexandelevatedtheperiosteum:themassissurroundedbyreactiveboneinsomeareas.BiopsyofthemassshowsatypicalcellswithhyperchromaticpleomorphicnucleiThecefisaresurroundedbyaneosinophilicmatrix
;someof
whichiscalcified.Thistumorismostlikelytospreadfirsttowhichofthefollowingareas?
A)Brain
6)Cervicallymphnodes
C)Liver
0)Lung
E)Vertebrae
CorrectAnswer:D.
OsteosarcomaisthemostcommonosseousmalignancyinchildrenItischaracterizedbycellswithenlargednucleiandadisorganizedtissuestructurethatdemonstratesareasofosteoidproduction(mineralizedcollagenthatprovidesthestructureofbone}.This
malignancyislocallydestructivetoboneandcanerodethroughthecortexresultinginaperiostea!reactionwithelevationoftheperiosteum,whichcanbeseenonx-rays(eg.Codmantriangle).Treatmentistypicallysurgicalresectioncombinedwithchemotherapy.
Osteosarcomahasahighmortalityrateandhighrateofmetastasistothelungs,evenwithadequatesurgicalresection.Evenwheninitialstagingdoesnotdetectmetastaticdisease,individualorsmallgroupsofcellsmayhavealreadymetastasizedyetremain
belowthethresholdofdetectioninstaging.Thismayleadtohighratesoflungmetastasesevenaftertreatment.
IncorrectAnswers:A.B,C.andE.
Malignanciesthatcommonlymetastasizetothebrain(ChoiceA)includelung,breast,kidney,andcoloncancers.Thebrainisnotacommonlocationforosteosarcomametastases.
Thecervicallymphnodes(ChoiceB)areacommonlocationformetastasesfromheadandneckcancerssuchassquamouscellcarcinomaofthefaceandoropharynx.Inilsisnotacommonlocationforosteosarcomametastases.
Theiver(ChoiceC)isacommonlocationformetastasesofcoloncancer.
Thevertebrae(ChoiceE)arecommonsitesofmetastasesofbreastcancerandprostatecancer.Thevalvelessvenoussystem,knownastheBatsonsystem,whichconnectsthepelvicveins,thethoracicveins,andthevenousvertebralplexusmayallowforthe
depositionofmalignantcellsinthevertebralbodies.
EducationalObjective:Osteosarcomaisthemostcommonmalignancyofbone.Itisanaggressivema;gnancythatoccursinabrmodaldistributioninchildrenandinolderadults.Itcommonlymetastasizestothelungs.
00#
Previous Next ScoreReport LabValues Calculator Help Pause

ExamSection3:Item43of50 NationalBoardofMedicalExaminers^
ComprehensiveBasicScienceSelf-Assessment
Y
43.A52-year-oldmanwithlungcancercomestothephysicianforafollow-upexamination.Smokingcessationhasbeenrecommended,butthepatientsaysthatheisunabletoquitbecauseheexperiencesapleasurablesensationasheinhalesthesmoke.He
hassmoked2packsofcigarettesdailyfor30years.Whichofthefollowingmechanismsbestexplainshownicotineproducesthissensationleadingtoaddictioninthispatient?
A)Activationofopioidreceptorsinthemidbrain
B)Increasedreleaseofdopamineinthenucleusaccumbens
C)Inhibitionofy-aminobutyricacid(GABA)releaseinthehypothalamus
D}Inhibitionofglutamatereceptorsintheamygdala
E)PotentiationofGABAreceptorsinthecerebralcortex
CorrectAnswer:B.
Thereleaseofdopamineinthenucleusaccumbensmediatesanticipatoryreward.Thispatient'spleasureimmediatelyafterinhalingislikelyrelatedtothisanticipatorydopaminesignalingratherthantheeffectofthenicotineitself.Theventraltegmentalareainthe
midbrainprojectsdopaminergicfiberstothenucleusaccumbens
;whichispartofthelimbicsystem
;calledthemesolimbicpathway.Aildrugsofabuseincreaseextracellulardopamineinthenucleusaccumbens.Themesolimbicdopaminepathwayistherefore
thoughttounderliealladdictions,asanticipatoryrewardcanmanifestascravingandleadtodrug-seekingbehaviorattheexpenseofapatient'shealthorsocialfunctioning.
IncorrectAnswers:A,C
;D.andE.
Activationofopioidreceptorsinthemidbrain{ChoiceA)mayindirectlymediaterewardbyincreasingmidbraindopaminesignaling.However,dopamineitselfisthedirectmediatorofrewardandaddiction.
Inhibitionofy-aminobutyricacid(GABA)releaseinthehypothalamus(ChoiceC)wouldresultinthemodulationofappetite,sleep,andotherhomeostaticactivities.Therelationshipbetweenenvironmentalstressandrelapseondrugsofabusemaybemediatedby
thehypothalamus.However,themesolimbicpathwayplaysalargerroleinmaintainingaddictivebehaviors.
Theinhibitionofglutamatereceptorsintheamygdala(ChoiceD)wouldleadtodecreasedfearlearning.Repeatedalcoholuseleadstoupregulationofglutamatereceptorsintheamygdala,whichmayincreaseanxietyduringwithdrawalandleadtorelapse.Thus,
inhibitingglutamatesignalingmaydecreasesubstanceuse.
PotentiationofGABAreceptorsinthecerebralcortex(ChoiceE)mediatestheexcitatory-inhibitorybalanceinthecortex(importantforpreventingepilepsy).AddictionispostulatedtoresultfrominsufficientGABAergicsignalinginseveralbrainregions.
EducationalObjective:ThemesolimbicpathwayreferstotheprojectionofdopaminergicneuronsfromtheventraltegmentalareatothenucleusaccumbensThisdopaminergicpathwaymediatestheanticipatoryrewardofdrugsofabuseandaddictivebehaviors.
© © ©
Previous Next ScoreReport LabValues Calculator Help Pause

ExamSection3:Item44ol50 NationalBoardofMedicalExaminers^
ComprehensiveBasicScienceSelf-Assessment
Y
44.AG5-year-oldmandiesofrenalfailure.Urinalysisshows:
Blood
Protein
Glucose
BenceJonesprotein
negative
negative
negative
positive
Atautopsy,thekidneysarefoundtobeenlarged.Ahistologicsectionofrenaltissueisshowninthephotomicrograph.Whichofthefollowingisthemostlikelydiagnosis?
A)Acutetubularnecrosis
B)Amyloidosis
C)Lipoidnephrosis
D)Membranousglomerulonephritis
E)Pyelonephritis
CorrectAnswer:B

saBBgc
*
Multiplemyelomaisamalignancycausedbytheneoplasticproliferationofplasmacells.NeoplasticplasmaceJsoverproducemonoclonalimmunoglobulinandlightorheavychains,whichmayresultinacutekidneyinjury.Patientsalsocommonlypresentwith
symptomsoffatigueandweightloss,symptomsofhypercalcemia(eg.abdominalcramping,kidneystones,psychiatricdisturbance),symptomsofanemia(eg,pallor,lightheadedness,dyspneaonexertion),orwithopportunisticinfectionssecondarytoimmune
dysfunction.Multiplemyelomaresultsinprimaryamyloidosisduetooverproductionanddepositionofimmunoglobulinchainsinthekidneys,heart,brain,andotherorgans.Renalamyloiddepositiononbiopsyrevealsdepositsinthemesangium.Evaluationunder
polarizedlightwithCongoredstainrevealsgreenbirefringentcrystals.EvaluationformultiplemyelomamayalsoincludeurinalysisrevealingBenceJonesprotein,orimmunoglobulinlightchains,peripheralbloodsmearshowingrouleauxformationoferythrocytes
andanMspikeonserumproteinelectrophoresis.
IncorrectAnswers:A,C,D.andE.
Acutetubularnecrosis(ChoiceA)typicallyoccursfollowinganischemicornephrotoxicinsulttothekidneys,whichresultsinnecrosisofthetubularepithelium.Granular,muddybrowncastsaretypicalonurinalysis.
Lipoidnephrosis(ChoiceC)orminimalchangediseaseandmembranousglomerulonephritis(ChoiceD)ormembranousnephropathyarenephroticsyndromes,whicharecharacterizedbyproteinuria(>3g/day),edema,hypoalbuminemia,and
hypercholesterolemia.Minimalchangediseaseappearsnormalonlightmicroscopyandisthemostcommoncauseofnephroticsyndromeinchildren.Itcanbeidiopathicortriggeredbyarecentinfectionorimmunestimulus.Abiopsyofmembranousnephropathy
usuallyshowsdiffusethickeningoftheglomerularcapillarywailonlightmicroscopy.Onimmunofluorescence,IgGdepositionwilloccuralongtheglomerularbasementmembrane.
Pyelonephritis(ChoiceE)typicallypresentswithfever,nausea,vomiting,dysuria,andflankpainwithassociatedcostovertebralangletendernessanphysicalexamination.Thismostcommonlyoccursfromascendingurinarytractinfections.
EducationalObjective:Multiplemyelomaisamalignancycausedbytheneoplasticproliferationofplasmacells,whichresulsintheabnormalproductionofimmunoglobuinlightandheavychains.Depositionofimmunoglobulinleadstorenalamyloidosisand
BenceJonesproteinonurinalysis.
&
Previous Next ScoreReport LabValues Calculator Help Pause

ExamSection3:Item45ol50 NationalBoardofMedicalExaminers^
ComprehensiveBasicScienceSelf-Assessment
Y
45.A5G-year-oldmancomestothephysicianbecauseofa3-monthhistoryofamildnonproductivecough.Hehasa20-yearhistoryofalcoholism.Physical,examinationshowsnoabnormalities.APPDskintestresultispositive.Achestx-rayandaGramstain
ofinducedsputumshownoabnormalities.Treatmentwithisoniazidandadailymultivitaminisbegun.Thepatientiscompliantwiththeisoniazidtherapybutnotwiththemultivitaminsupplementation.Twomonthslaterhedevelopstinglingandnumbnessof
bothfeet.Deficiencyofwhichofthefollowingisthemostlikelycauseofthefootsymptomsinthispatient?
A)FoEicacid
B)Niacin
C)VitaminB
2
(riboflavin)
D)VitaminB
6
(pyridoxine)
E)VitaminB
12
(cobalamin)
CorrectAnswer:D.
ApositivePPDskintestwithnoevidenceofactivetuberculosis(normalchestx-rayandsputumgramstain)isdiagnosticoflatenttuberculosisinfection.Treatmentforlatenttuberculosisinfectioninvolvesninemonthsofisoniazid.isoniazidbindsvitaminBe
(pyridoxine)whichmayresultinearlyexcretionandinactivationofpyridoxine.Isoniazidalsoinhibitspyridoxinephosphokinase,whichisnecessarytoactivatepyridoxine,therebycausingpyridoxinedeficiency.Therefore,allpatientstakingisoniazidalsoconcurrently
takepyridoxinesupplementation.Pyridoxinedeficiencylimitssynthesisofhistamine,hemoglobin,andneurotransmittersincludingepinephrine,norepinephrine,dopamine,serotonin,andy-aminobutyricacid.Deficiencycommonlypresentswithperipheral
neuropathy,dermatitis,sideroblasticanemia,glossitis,andseizures(especiallywithisoniaziduse).
IncorrectAnswers:A.B
(C.andE.
Folicacid(ChoiceA)isconvertedtotetrahydrofolicacidandusedasacoenzymeinthesynthesisofnucleotidesandnucleosides.Folateiscontainedinleafyvegetablesandabsorbedinthejejunum.Folatedeficiencyisoftenseeninpatientswithmalnutrition,
alcoholusedisorder,andpatientstakinganti-folatemedications(eg.phenytoin,methotrexate).MegaloblasticanemiaoccursinthesettingofimpairedDNAsynthesis.
Niacin(ChoiceB)deficiencycanresultinglossitis,diarrhea,neuropsychologicaldisturbancessuchasdementiaandhallucinations,anddermatitis.
VitaminB
2
(riboflavin)(ChoiceC)deficiencyischaracterizedbyinflammationandcrackingofskinaroundthelips,mouth,andtongue.Itisnotassociatedwithisoniaziduse
VitaminB
12
(cobalamin)(ChoiceE)deficiency,similartofolicaciddeficiencyisclassicallycharacterizedbyerythrocytemacrocytosisandhypersegmentedneutrophils.Inaddition,italsoresuftsinposteriorcolumnandspinocerebellartractdemyelination.ftismost
oftenseeninpatientswithmalnutrition,alcoholusedisorder,perniciousanemia,orCrohndisease.
EducationalObjective:Isoniazidtherapyforlatenttuberculosiswithoutsufficientpyridoxinesupplementationcanleadtopyridoxinedeficiency,whichischaracterizedbyperipheralneuropathy,dermatitis,sideroblasticanemia,glossitis,andseizures.
0 0 ^
Previous Next ScoreReport LabValues Calculator Help Pause

ExamSection3:Item46ol50 NationalBoardofMedicalExaminers^
ComprehensiveBasicScienceSelf-Assessment
Y
46.Humoralantibodyproducedagainstmanyvirusesfollowingnaturalinfectionmediatesprotectionagainstreinfectionwiththesamestrainprimarilybywhichofthefollowingmechanisms?
A)Activationofmacrophages
B)Blockingofviralattachmenttoceflreceptors
C)Inhibitionofviralnucleicacidreplication
D)Killingofintracellularvims
E)Preventionofreleaseofvirionsfrominfectedcells
CorrectAnswer:B.
ViralinfectioncauseshumoralantibodyproductionthroughtheprocessofTlymphocyteandBlymphocyteactivation.Thisispartoftheadaptiveimmunesystemresponse.Whenadendriticcellencountersaviralantigen,oracellisinfectedwithavirus
:antigens
arepresentedonmajorhistocompatibilitycomplexes(MHC).TheseMHC-antigencomplexesthenbindtoT-lymphocytereceptors,activatingthoseTlymphocytesandcausingthemtoproliferate,survive,andsecretecytokines.TheactiveTlymphocytesthenhindto
BlymphocytesviatheCD40ligandandreceptorpairing,whichleadstoBlymphocyteactivationandantibodyclassswitching,ultimatelyleadingtohumoralantibodyproduction.Theantibodiesproducedtargetavarietyofviralparticles,someofwhichincludethe
proteinsontheviralsurfacerequiredtobindtoandinfecthostcells.Forexample,influenzacontainstheproteinhemagglutinin,whichbindsthesialicacidreceptorontheoutsideofhostcellsandallowsforviralentry.Whenantibodiesarepresenttohemagglutinin,
:heybindtotheproteinandcreateabarricadebetweentheligandandreceptorpreventingviralattachmenttoceilreceptorsandsubsequentcellentry.
IncorrectAnswers:A,C,D,andE.
Activationofmacrophages(ChoiceA)isacomponentoftheinnateimmunesystem,nottheadaptiveimmunesystem.Activatedmacrophagesmaybecomehistiocytesandmayformgranulomasinthecaseofsomeinfections(eg.Mycobacteriumtuberculosis).
Inhibitionofviralnucleicacidreplication(ChoiceC)isthemechanismutilizedbynucleosidereversetranscriptaseinhibitors(NRTIs)andnon-nucleosidereversetranscriptaseinhibitors(NNRTIs)inthetreatmentofHIV.Thisisnottheeffectofcirculatinghumoral
antibodies.
Killingofintracellularvirus(Choice0)isafeatureofthecytotoxicCD8+Tlymphocytesresponsetoviralinfection.Whenaceflhasbeeninfectedwithvirus,itpresentsintracellularantigenonthecellsurfacethroughclassiMHC.ThisisrecognizedbyCD8+
cytotoxicTlymphocytes,whichthencauseceildeathbyapoptosisandsubsequentdeathtotheintracellularvirus.
Neuraminidaseinhibitors,atypeofmedicationutilizedintreatinginfluenza,preventthereleaseofvirionsfrominfectedcells(ChoiceE).Thisisnotaneffectofantiviralantibodies.
EducationalObjective:HumoralantibodyproductionbyBlymphocytesoccursafterviralinfectionleadstoT-lymphocyteactivationfollowedbyassociatedB-lymphocyteactivation.Onceformed,antibodiessurroundviralparticles,effectivelyblockingthemfrom
bindingtoceilreceptors.Ultimately,thispreventsthevirusfromenteringthecell,thefirststepinviralinfectionofacell.
© © ©
Previous Next ScoreReport LabValues Calculator Help Pause

ExamSection3:Item47ol50 NationalBoardofMedicalExaminers^
ComprehensiveBasicScienceSelf-Assessment
Y
47.AninvestigatorisstudyingaI-lymphocyteclonethatrecognizesapeptidefromthehemagglutininglycoproteinofinfluenzavirusboundbyclassfihumanleukocyteantigen.Thisclonemostlikelyrecognizesapeptidethatwascombinedwithahuman
leukocyteantigeninwhichofthefollowingcellularcompartments?
A)Cytosol
B)Endoplasmicreticulum
C)Endosomes
D)Mitochondria
E)Nucleus
CorrectAnswer:C.
Endosomesarecytoplasmicorganellescomposedmainlyofmicrotubulesandvesiclesthattrafficandsortendocytosedmaterial,includingvaccinesandliveviruses.Endosomesexistinseveralforms(early,late
:recycling)andcantrafficendocytosedmaterialtothe
Golgicomplex,toiysosomes,ortotheextracellularmembraneTheclassIIhumanmajorhistocompatibilitycomplex(MHC)proteins,alsoknownasclassIIhumanleukocyteantigen(HLA)proteins,aresynthesizedintheroughendoplasmicreticulumandare
traffickedtotheGolgicomplexandthentolateendosomes.wheretheyformhigh-affinitycombinationswithdegraded,endocytosedproteins.BoundproteinisthenrecycledtothecellsurfacemembranewherethepeptidefragmentsarepresentedonclassII
HLA/MHCproteinstoTlymphocytes.TheT-lymphocytereceptoridentifiesthepeptideboundtothehumanleukocyteantigenandinitiatesorsustainsanimmuneresponse.
IncorrectAnswers:A.B,D.andE.
Thecytosol(ChoiceA)Isthesiteofdegradationofcytoplasmicpeptidesviatheproteasome,whicharethencombinedwithclassIMHCproteinsintheendoplasmicreticulumandshuttledtotheplasmamembraneforpresentation.AnabsenceofclassIMHC
proteinsonthecellsurfaceisaninitiatingtriggerfornaturalkillercell-inducedceIdestruction.
Endoplasmicreticulum(ChoiceB)isthesiteofcombinationoftargetpeptideswithclassIMHCproteins.
Mitochondria(ChoiceD)donotplayasignificantroleintheprocessingofendocytosedmaterial.
Thenucleus(ChoiceE)isthesiteofmRNAsynthesisbutisnotamajorsiteoftraffickingofendocytosedmaterial.
EducationalObjective:EndosomesarecytoplasmicorganellescomposedmainlyofmicrotubulesandvesiclesandservetotrafficandsortendocytosedmaterialEndocytosedviralproteinactivatestheimmunesystemthroughcombinationwithHLAproteinsand
subsequentpresentationtoTlymphocytes.
00# 0f*
Previous Next ScoreReport LabValues Calculator Help Pause

ExamSection3:Item43ol50 NationalBoardofMedicalExaminers^
ComprehensiveBasicScienceSelf-Assessment
Y
48.A45-year-oldmanisbroughttothephysicianbecauseofa4-weekhistoryofmildconfusion,memorydifficulties,generaizedmuscleaching,andintermittenttinglingofhislipsandfingers.Hesayshehasbeensleepingmorethanusual,andthathefeels
"downinthedumps.”Vitalsignsarenormal.Physicafexaminationshowsnoabnormalitiesexceptfordry,scalyskinanddryhair.Tappingwithareflexhammerovertheleftsideofthefacejustanteriortotheearcausesleftfacialmusclecontractions.Which
ofthefollowingisthemostlikelydiagnosis?
A)Addisondisease
B)Cushingdisease
C)Hyperparathyroidism
D)Hyperthyroidism
E)Hypoparathyroidism
F)Hypothyroidism
CorrectAnswer:E
Thepatient'spresentationofconfusion,memorydifficulties,myalgias,andperioraland/orperipheralparesthesiaraisessuspicionforhypocalcemia.Facialmusclecontractionselicitedbytappingthefacialnerveanteriortotheearonexaminationisaclassicsignof
hypocalcemiaknownasChvosteksign.Patientsmayalsohavecarpalspasmduringbloodpressurecuffmeasurementandarterialocclusion.CommoncausesofhypocalcemiaincludevitaminDdeficiency,hypomagnesemia,acutepancreatitis,citratechelation
frombloodproductadministration,chronickidneyfailure,malnutrition,hypomagnesemia,andpost-surgicaloridiopathichypoparathyroidism.Parathyroidhormoneisapeptidehormonethatincreasesosteoclastactivity,vitaminDsynthesis,andrenalcalcium
reabsorptioninordertomaintaincalciumhomeostasis.Deficiencyofparathyroidhormoneresultsinhypocalcemia.
IncorrectAnswers:A,Ei.C.D,andF.
Addisondisease(ChoiceA)orprimaryadrenalinsufficiencymaybeduetoautoimmunedestructionortuberculosis.Itpresentswithhypotension,hyperkalemia,non-aniongapmetabolicacidosis,andskinhyperpigmentationsecondarytoadeficiencyofaldosterone
andcortisol.
Cushingdisease(ChoiceB)occursduetoincreasedcortisollevelsandpresentswithtruncalweightgain,striae,musc:eweakness,andfattydepositsaroundthefaceandneck.Vitalsignsgenerallydemonstratehypertension,andlaboratorystudiesmayShow
hypokalemiaandhyperglycemia.
Hyperparathyroidism(ChoiceC)andexcessiveproductionofparathyroidhormoneresultsinincreasedserumcalciumfromincreasedosteoclastactivity,intestinalabsorption,andrenaltubularabsorption.Patientswithhypercalcemiawouldpresentwith
nephrolithiasis,chronicbonepain,abdominaldiscomfort,andpsychiatricdisturbances.
Hyperthyroidism(ChoiceD)classicallypresentswithheatintolerance,weightloss,warm,flushedskin,hairloss,loosestools.Femalepatientsmayalsopresentwitholigomenorrheaoramenorrhea.Hypothyroidism(ChoiceF)classicallypresentswithcold
intolerance,weightgain,dryskin,constipation,andmenorrhagiainfemalepatients.
EducationalObjective:Hypoparathyroidismcanresultinhypocalcemia,whichcanpresentwithconfusion,memorydifficultiesmyalgias,perioraland/orperipheralparesthesias,andfacialmusclespasmupontappingthefacialnerveonexamination.
oi? &
Previous Next ScoreReport LabValues Calculator Help Pause

ExamSection3:Item49of50 NationalBoardofMedicalExaminers^
ComprehensiveBasicScienceSelf-Assessment
49.A38-year-oldmanconiestothephysicianbecauseofa1-weekhistoryofadischargefromhispenisandtesticularpain.Heissexuallyactiveandusescondomsinconsistently.Physicalexaminationshowsathick
greenurethraldischargeandexquisitetendernessoftherighttestis.Thepainisrelievedbyelevationofthescrotum.AphotomicrographofaGramstainofthedischargeisshown.Whichofthefollowingisthemost
likelycausalorganism?
A)Chlamydiatrachomatis
B)Mycoplasmagenitallum
C)Neisseriagonorrhoeas
D)Trichomonasvaginalis
E)Ureaplasmaureatyticum
m
CorrectAnswer:C
Thispatientpresentswithsignsandsymptomssuggestiveofepididymitis
:whichfrequentlyoccursduetoinfectionwithNeisseriagonorrhoeasorChlamydiatrachomatisThispatient'ssymptomsofthick,greenurethraldischargeandphotomicrograph
demonstratingaccumulationofGram-negativediplococciwithinneutrophilsismostconsistentwithN.gonorrhoeae.Epididymitistypicallypresentswithlever,purulenturethraldischarge,andpainlocalizedtoonetestis,whichmaybealleviatedwithelevationof
thehemiscrotum(distinguishingthepainofepididymitisfromthatoftesticulartorsion,whichisnotalleviatedbythismaneuver).N.gonorrhoeaeusuallyrespondstotreatmentwithceftriaxone,howeverresistancetomultipleantibioticsisbecomingincreasing
prevalentamongstN.gonorrhoeae.CoinfectionwithC.trachomatisisfrequent,andpatientsshouldbetreatedempiricallyforbothinfections.Sexualpartnersrequiretestingandtreatmentduetothehighrateofasymptomaticinfection.
IncorrectAnswers:A.B,D,andE.
Chlamydiatrachomatis(ChoiceA)isaGram-negativebacteriumthatmaycauseepididymitis.Howeverifislesslikelytoproducegreendischargeandappearsmicroscopicallyasanobligateintracellularorganismwithelementaryandreticulatebodies,rather
thanasadiplococcus.
Mycoplasmagenitallum(ChoiceB)isararecauseofurethritisinmenanddoesnotcommonlycauseepididymitis.M.genitatiumisalsonotableforitssmallgenome
:leadingittobechosenasthefirstorganismtohaveitsentiregenomereproducedsynthetically.
Trichomonasvaginalis(Choice0)isananaerobic,flagellatedprotozoanthatcausesvaginaltrichomoniasis.Trichomoniasispresentswithpruritus,dysuria,dyspareunia,cervicalerythema,andabnormaldischargethatisdescribedasgreen,frothy,and
malodorous.Menaffectedbytrichomoniasisareoftenasymptomatic,andT.vaginalisdoesnotcommonlycauseepididymitis.
Ureaplasmaurealyticum(ChoiceE)maycauseurethritisinmen.ItischaracterizedbyitslackofaceIIwallonmicroscopyandbyitsproductionofurease.
EducationalObjective:N.gonorrhoeaeisaGram-negativediplococcusthatisacommoncauseofepididymitis.Epididymitispresentswithfever,purulenturethraldischarge,andpainlocalizedtoatestisthatisalleviatedwithelevationofthehemiscrotum.
© © ©
Previous Next ScoreReport LabValues Calculator Help Pause

ExamSection3:Item50ol50 NationalBoardofMedicalExaminers^
ComprehensiveBasicScienceSelf-Assessment
Y
50.Onethousandofficeworkersaresurveyedtodeterminewhetherornottheyusecomputersandwhetherornottheyhavesymptomsofcarpaltunnelsyndrome.Whichofthefollowingbestdescribesthisstudy?
A)Case-controlstudy
B)Caseseries
C)Cohortstudy
D)Cross-sectionalsurvey
E)Randomizedclinicaltrial
CorrectAnswer:D.
Across-sectionalsurveyseekstoidentifytheprevalenceofaconditionatasinglepointintime.Theriskfactorandtheoutcomesaremeasuredsimultaneously.Across-sectionalstudydoesnotfollowpatientsovertime.AHinformationiscollectedatasingletime
point.Causalitybetweentheriskfactorandoutcomecannotbeconcluded.Itcanbeusedtoassessassociationswithpriorexposures.Associationswithpriorexposuresmaybesubjecttorecallbiasassurveyparticipantsmaynotaccuratelyrepresentorrecallprior
exposures.Inthestudydescribed,theoutcomedata(carpaltunnelsyndrome)andtheriskfactordata(computeruse)arecollectedatthesametimeandthepatientsarenotfollowedovertime,makingthisacross-sectionalsurveystudydesign.
IncorrectAnswers:A,B.CaandE.
Acase-centraIstudy(ChoiceA)investigatesanassociationbetweenanexposureandanoutcome.Enthisstudydesign,agroupofpatientswiththedisease(cases)areidentified.Agroupofpatientswithoutthedisease(controls)arematchedonbaseline
characteristicstothecases.Exposuredataforthetwogroupsiscollected,andthesedataarecomparedtodetermineassociationwiththeoutcome(disease)inquestion.Anoddsratiomaybecalculatedtocompareexposuresbetweengroups.
Acaseseries(ChoiceB)isadescriptivestudydesigninwhichanumberofconsecutiveornonconsecutivecasesofadiseaseand/ortreatmentaredescribedindetail,withinformationaboutexposure,demographics,andcomorbidities.Caseseriesdonotimplya
cause-and-effectrelationship.TheydonottestahypothesisnoraretheyrandomizedTheyareusefulincharacterizingthenaturalhistoryofadiseaseorresponsetotreatment.Theyarealsousefulindescribingrarediseases,asthesmallpopulationsizemaynot
permitconductionoflargercohortorrandomizedtrialswithsufficientpower.
Acohortstudy(ChoiceC)identifiesagroupofpatientsandfollowsthemovertimetoidentifywhetheranexposureisassociatedwithanoutcomeofinterest.Cohortstudiesmayberetrospectiveorprospectiveindesign.Inaprospectivedesign,thehypothesisand
analysisprotocolsareestablishedpriortothestartofthestudyperiod.Inaretrospectivedesignthehypothesisorquestionisdesignedafterthestudytimeperiodhaspassed.Anexampleofaprospectivecohortstudywouldbefollowingagroupof1,000smokers
foratimeperiodof10yearsandidentifyingtheproportionofthepatientswhodeveloppancreaticcancertoidentifytheriskofpancreaticcancerinsmokersascomparedtoacontrolgroupofnonsmokers.
Arandomizedclinicaltrial(ChoiceE)isanexperimentalstudydesign.Patientsarerandomlyallocatedtotwoormoreinterventionalarmsorcontrolarms,andthesepatientsarefollowedovertimetoevaluateanoutcomeofinterest.Randomizeddesignminimizes
theopportunityforbias;thus,arandomizedinterventionalstudycanbeusedtoimplycausation.Commonexamplesofrandomizedtrialsincludetherapeuticcomparisonsbetweenanewdrugandthepreviousstandardofcare.
EducationalObjective:Across-sectionalsurveystudydesigncollectsdataaboutboththeexposureandoutcomeatthesametimepointanddoesnotfollowpatientsovertime.Itcanbeusedtoassesstheprevalenceofaspecificdiseaseandalsoassess
associationswithpriorexposures.Associationswithpriorexposuresmaybesubjecttorecallbiasassurveyparticipantsmaynotaccuratelyrepresentorrecallpriorexposures.
© © © ©
f*
Previous Next ScoreReport LabValues Calculator Help Pause

ExamSection4:Item1of50 NationalBoardofMedicalExaminers^
ComprehensiveBasicScienceSelf-Assessment
Y
1.Amalenewbornisdeliveredvaginallyat39weeks'gestationtoa24-year-oldprimigravidwoman.Thepregnancywascomplicatedbypolyhydramnios.Soonafterbirth,thenewbornhasfrothingatthemouth,cyanosis,andrespiratorydistress.An
unsuccessfulattemptismadetopassanasogastrictube.Anx-rayshowsanair-distendedstomach.Whichofthefollowingisthemostlikelycauseofthesefindingsinthispatient?
A)Abnormalityofaorticarchresultinginvascularring
B)Deviationofthetracheoesophagealseptum
C)Failureofre-formationofduodenallumen
D)Hypertrophyofpyloricmuscle
E)Incompleteformationofthetrachealcartilagerings
CorrectAnswer:B.
Thetypicalpresentationforanewbornwithesophagealatresiaisaninabilitytofeed,withassociatedgaggingandaninabilitytomanageoralsecretions.Esophagealatresiaoftenpresentswithconcomitanttracheoesophagealfistula(TEF),whichoccursasaresult
ofdeviationofthetracheoesophagealseptumduringembryonicdevelopment.ThepresenceofaTEFconveysincreasedriskofaspirationpneumoniaandrespiratorydistress.Initialevaluationrequiresplacementofanasogastrictubeandevaluationofitsposition
withachestx-ray.Ifthenasogastrictuberemainscoiledwithintheesophagus,thissuggeststhepresenceofesophagealatresia.AirwithinthegastrointestinaltractimmediatelyafterbirthandpriortofeedingsuggeststheexistenceofaTEFdistaltothelevelof
esophagealatresia.Definitivemanagementrequiressurgicalrepair.Aswell,additionalscreeningsforassociatedcongenitalmalformationsshouldoccur.Asthefetusisunabletoswallowamnioticfluid,apregnancycomplicatedbypolyhydramniossuggeststhe
diagnosispriortobirth.
IncorrectAnswers:A.CrD,andE.
Abnormalityofaorticarchresultinginvascularring(ChoiceA)presentswithstridorwheezing.cough
:dysphagia,andvomitingduetocompressionofthetracheaandesophagus.VascularringscanbedistinguishedfromTEFbyimprovementofsymptomsupon
neckrepositioning(eg
:extension)andbyabsenceofanair-distendedstomach
:wtiichismoresuggestiveofTEF.
Failureofre-formationofduodenallumen(ChoiceC)isobservedinduodenalatresia.Duodenalatresia,presentswithvomitingandabdominaldistension,aswellasdistensionofthestomachandduodenumonabdominalradiograph.Duodenalatresiaismore
commoninchildrenwithDownsyndrome.
Hypertrophyofpyloricmuscle(ChoiceD)isobservedinpyloricstenosis.Pyloricstenosispresentsininfancywithnonbiiiousvomitingaround4to8weeksoflifewithapalpableabdominamass.
Incompleteformationofthetrachealcartilagerings(ChoiceE)mayleadtocompressionofthetracheawithassociatedrespiratorydistress
:wheezing,andpositionalstridoroftenwforsewhensupine.Theesophaguswouldnotbeaffectedandtherewouldnotbe
difficultypassinganasogastrictube.
EducationalObjective:Esophagealatresiawithtracheoesophagealfistulapresentswithaninabilitytofeed,gagging,coughing,respiratorydistress,andcyanosisintheneonatalperiod.Inabilitytopassanasogastrictubewiththepresenceofgastricai;risa
characteristicfinding.Definitivemanagementrequiressurgicalrepair.
© © © oo
Previous Next ScoreReport LabValues Calculator Help Pause

ExamSection4:Item2of50 NationalBoardofMedicalExaminers^
ComprehensiveBasicScienceSelf-Assessment
2.A22-year-oldmanisbroughttotheemergencydepartment30minutesafterafriendfoundhimunconsciousonthefloorAdrugoverdoseissuspected.Hispulseis120/min,respirationsare4/minwithapparentlowtidalvolumes,andbloodpressureis
120/100mmHg.Physicalexaminationshowsmildcyanosis.Heisintubatedandmechanicallyventilatedwithpositivepressureventilation{rate=12/min:VT=7mL/kg)using10cmH20positiveend-expiratorypressure(PEEP).Whichofthefollowingsets
offindingsbestdescribestheeffectsofthemechanicalventilationwithPEEPanthispatient'salveolar{Pjandintrapleural(Pip)pressures?
PeakInspiratoryP
p
positive
positive
zero
PeakInspiratoryPA
Positive
Positive
Zero
Zero
Negative
Negative
End-TidalPA
positive
zero
End-TidalPip
positive
negative
negative
zero
positive
negative
A)
8)
C) zero
0) negative
positive
zero
positive
negative
E)
F) zero
CorrectAnswer:A.
Spontaneousbreathingisaccomplishedbyexpansionofthechestwall,whichpullstheparietalpleuraoutward;andgeneratesanegativeintrapleuralpressurerelativetotheatmosphere.Thepressuregradientthatresultsdrivesairintothelungs.Normalend-
tidalintrapleuralpressureisaround-5cmHprelativetoatmosphericpressure.Mechanicalventilationisindicatedforpatientswhoareunabletobreatheontheirown,whichmaybeduetocentralnervoussystemdepression,neuromusculardisease,or
decreasedlungcompliance(requiringgreaterforcetoexpandlungagainstincreasedlungrecoil).Modernmechanicalventilationutilizesapositivepressuresystem.Theventilatorcircuitisconnectedtotheairwayviaanendotrachealtubeandsealedwithacuff.
Positivepressureisgeneratedbytheventilator,whichdrivesairintothelungs.Positiveend-expiratorypressure(PEEP)maybeutilizedtopreventthepressureinthealveolifromdroppingtozeroorbecomingnegativeduringtherespiratorycycleduetochest
wallrecoil.Thishelpskeepalveolistentedopenandabletoparticipateingasexchange.ThepositivepressureIstransmittedtothepleuralspaceThepeakinspiratoryandend-tidalalveolarpressureswillbepositiverelativetoatmosphere,aswellasthepeak-
inspiratoryandend-tidalintrapleuralpressures.
IncorrectAnswers:B,C;D
:E
;andF
ChoicesC,D.E,andFareincorrectasthepeakinspiratorypressurewillbepositivetodriveairintothelungsintheabsenceofspontaneouschestwallmovement.Thepeakinspiratorypressurerepresentsthepressureneededtoovercomeairwayresistance
anddeliverthefulltidalvolumetothelungs.
ChoicesB,C
;DandFareincorrectastheend-tidalalveolarpressurewillbepositiverelativetoatmosphereinapositive-pressureventilatorycircuitutilizingPEEP.Thismayhelppreventatelectasisandcanimproveoxygenationindisorderswithreducedlung
compliancesuchasacuterespiratorydistresssyndrome.
ChoicesC.D
:andFareincorrectinthatthepeakinspiratoryintrapleuralpressurewillbepositiveinthissystem.Anegativepeakinspiratoryintrapleuralpressureoccursinspontaneousbreathing.
Similarly,choicesBrC,0.andFareincorrectinthattheend-tidalintrapleuralpressureispositiveinthissystem.Inspontaneousbreathing,theend-tidalintrapleuralpressureisnormallyaround-5cmH20duetotheoutwardpullfromthechestwallandinward
puflfromlungelasticrecoil.
EducationalObjective:Modernmechanicalventilationutilizesapositivepressuresystemtodriveairintothelungs.Thisisincontrasttonormalspontaneousbreathing,inwhichoutwardexpansionofthechestwallgeneratesnegativeintrapleuralandalveolar
pressuresrelativetoatmosphere,creatingapressuregradientforthemovementofairintothelung.
O0
4
s
00
Previous Next ScoreReport LabValues Calculator Help Pause

ExamSection4:Item3of50 NationalBoardofMedicalExaminers^
ComprehensiveBasicScienceSelf-Assessment
Y
3.A35-year-oldwomangravida1
rpara1
:comestothephysicianforafollow-upexamination1monthafterdeliveringahealthymalenewborn.Duringherpregnancy,shewasdiagnosedwithgestationaldiabetes.Hermotherandsisteralsodevelopedthis
conditionduringpregnancy.Today,laboratorystudiesshowaserumglucoseconcentrationof1DDmg/dLThispatientmostlikelyhasdecreasedactivityofwhichofthefollowingenzymes?
A)D13kinase
B)Glucokinase
G)Mevalonatekinase
D)Phosphofructokinase
E)Pyruvatekinase
CorrectAnswer:B.
Glucokinasecatalyzesthephosphorylationofglucosetoglucose-6-phosphateduringglycolysis.Whileglucosecanfreelydiffuseintoandoutofceils,glucose-6-phosphateissequesteredinthecellduetoitschargedphosphategroup.Glucokinaseispresentin
hepatocytesandthepancreatic(3cellsandisstimulatedbyhighbloodglucoseconcentration,causinguptakewhencirculatingconcentrationsofglucosearehigh..Itsactionisalsostimulatedbyinsulin.Inthepcells,thegenerationofadenosinetriphosphate(ATP)
throughglycolysisleadstoclosureofATF'-dependentpotassiumchannelswhichleadstoanincreaseinmembranepotentialandaninfluxofcalcium.Thecalciumcausescellularsecretorygranulescontaininginsulintofusewiththecellmembrane.Deficienciesof
theenzymeleadtomaturity-onsetdiabetesoftheyoung(MODY)
:withanincreasedthresholdforinsulinreleaseduetolowcytoplasmicglucose.Itisinheritedinanautosomaldominantpatternandpresentswithmildhyperglycemiathatcanoftenbemanagedwith
dietalone.
IncorrectAnswers:A.CrD,andE.
D13kinase(ChoiceA}
;aspartofthekinasefamilyofproteins,actstohydrolyzenucleotidetriphosphatemoleculesinthephosphorylationofproteinsinthecell.D13kinasemayrelatetoK-RasD13,adownstreamproteininthecellgrowthanddifferentiation
pathwaytriggeredbyinsulinsignaling.Thedeficiencyingestationaldiabetesarisesfromintracellularglucoselevelsthatarelessthannormalduetofailuretophosphorylateglucose,nototherproteinsinthesignaltransductioncascade.
Mevalonatekinase(ChoiceC)isanenzymepresentinthecholesterolbiosynthesispathway.Deficienciesinthisenzymeleadtomevalonicaciduria,aninbornerrorofmetabolismmarkedbydevelopmentaldelay,failuretothrive,hepatomegaly,andfebrilecrises.
HyperimmunoglobulinDsyndromealsoresultsfromdeficiency,whichpresentswithincreasedlevelsofimmunoglobulinD,recurrentfever,abdominalpain,andlymphadenopathy.itdoesnotplayaroleinthedevelopmentofdiabetesmeINtus.
Phosphofructokinase(ChoiceD)convertsfructose-6-phosphatetofructose-1,6-bisphosphateorfructose-2
:6-bisphosphate.DeficienciesleadtoglycogenstoragediseaseVIIwhichpresentswithfatigue,myalgias,andrhabdomyolysiswithexercise.Itdoesnotplay
aroleinthedevelopmentofdiabetesmellitus.
Pyruvatekinase(ChoiceE)convertsphosphoenolpyruvatetopyruvate.Itsdeficiencypresentswithextravascularhemolysis.Ittypicallypresentsininfancy,ratherthaninadulthood.
EducationalObjective:GlucokinaseactsasaglucosesensorinhepatocytesandBcellsofthepancreas.DeficienciesoftheenzymeleadtoMGDY,whichpresentswithanautosomaldominantpatternofmildhyperglycemia.
oi? & * IP*
Previous Next ScoreReport LabValues Calculator Help Pause

ExamSection4:Item4of50 NationalBoardofMedicalExaminers^
ComprehensiveBasicScienceSelf-Assessment
Y
4.An18-month-oldboyisbroughttothephysicianbyhisparentsbecauseofeyepainfor3days.Hehasahistoryofmultipleinfectionssincetheageof3weeks,includingupperrespiratorytractinfections,staphylococcalosteomyelitis,andtwoepisodesof
pneumonia(oneduetoStaphylococcusaureusandtheotherduetoAspergillusfumigatus).Ophthalmologicexaminationshowserythemaoftheconjunctivaofthelefteyewithasmaltamountofpurulentdischarge.Physical
'
examinationshowsmultiple
ulceratedlesionswitherythematousbordersovertheface,neck,andback;cervicalandaxillarylymphadenopathy;andhepatosplenomegaty.SerumIgE,lgG
:andIgMconcentrationsarewithinthereferenceranges.Cultureoftheconjunctiva!secretions
growsStaphylococcusepidermidis,andcultureofseveral.skinlesionsgrowsStaphylococcusaureus.Thispatientmostlikelyhaswhichofthefollowingimmunodeficiencydiseases?
A)Chediak-Higashisyndrome
B)Chronicgranulomatousdisease
C)DiGeorgesyndrome
D)PerinatalHIVinfection
E)Wiskott-Aldrichsyndrome
CorrectAnswer:B.
Chronicgranulomatousdisease(CGD),animmunodeficiencysyndromecausedbyadefectinthenicotinamideadeninedinucleotidephosphate(NADPH)oxidasecomplexrequiredforphagocytickillingofcatalase-positiveorganisms,ischaracterizedbyrecurrent
pyogenicinfectionscausedbyGram-positiveandGram-negativebacteria.RecurrentpneumoniaisthemostcommonpresentinginfectioninpatientswithCGD,butskinandsofttissueinfectionsarealsofrequentlyencountered.Commoninfectingbacteriainclude
Staphylococcusspecies,Burkholcieriacepacia.
,andNocardiaspecies.PatientswithCGDareatriskforfungalinfections,especiallyAspergillusspecies.NADPHoxidaseusesoxygenasasubstrateforthegenerationoffreeradicals(superoxideanions).Free
radicaloxygenspeciesaresubsequentlyusedforthecreationofhydrogenperoxideandhypochlorousacid.Activationofthispathwayleadstotherespiratoiyburstwhichresultsinbacterialdeath.DeficiencyofNADPHoxidaserendersphagocytesunableto
neutralizecatalase-positivebacteria,whichcanneutralizetheirownhydrogenperoxide,thusleavingthehostcellswithoutasubstratenecessarytocompletetherespiratoryburst.Diagnosisismadebyanabnormaldihydrorhodaminetestoranifrobluetetrazolium
reductiontest.3nthislattertest,normalphagocytesusetheactionofNADPHtoreducenitroblue,whichleadstoacolorchangefromyellowtoblue.PatientswithCGDwillnotdemonstratecolorchange.
IncorrectAnswers:A,C,D,andE.
Chediak-Higashisyndrome(ChoiceA)isarare,autosomalrecessivedisorderoftheimmunesystemcausedbymutationsinthelysosomaltraffickingregulatorgene(LYST)thatencodesaproteinessentialfornormalformationandtransportationoflysosomes
withinthecell.Theclinicalmanifestationsincludefrequentbacteria!infections,oculocutaneousalbinism,peripheralneuropathy,andprogressiveneurologicdysfunction.
DiGeorgesyndrome(ChoiceC}iscausedbya22q11deletionthatleadstofailureofthethirdandfourthbranchia
:
pouchestodevelop.Thesestructuresgiverisetothethymusandparathyroidglands,andtheirabsenceleadstothymicaplasiaandhypocalcemiain
thesettingofcardiacandcraniofacialdefects.Patientsaresusceptibletoviralandfungalinfections.Thedefinitivediagnosisrequiresfluorescencein-situhybridizationtodetectthemissingDNAsegment.Treatmentissupportive.
PerinatalHIVinfection(ChoiceD)islikelytooccurinsituationsinwhichthemotherdidnotreceiveantiretroviraltherapypriortoorduringpregnancy,orininstanceswhentheHIVviralloadisdetectable.MostpatientsdonotdevelopclinicalmanifestationsofHIV
infectioninresourcedsettingsasdiagnosisismadequicklyandpatientsarestartedonantiretroviraltherapytopreventthedevelopmentofAIDS.
Wiskott-Aldrichsyndrome(ChoiceE)iscausedbyamutationintheWASgeneontheX-chromosomethatencodesaproteinessentialforactincytoskeletonrearrangementthatoccursduringinteractionsbetweenTlymphocytes,antigen-presentingcells,andB
lymphocytes,leadingtoanimpairedinnateandadaptiveimmunesystem.Thephenotypeisvariablebutclassicallyincludeseczema,thrombocytopenia,andnfectionswithencapsulatedbacteriaandopportunisticpathogens.
EducationalObjective:CGDiscausedbyNADPHoxidasedeficiencyandresultsinimpairedintracellularkillingofpathogensbyphagocytes.Thispresentsasrecurrentpyogenicnfectionswithcatalase-positiveorganismssuchasStaphylococcusaureus.
© © ©
A
Previous Next ScoreReport LabValues Calculator Help Pause

ExamSection4:Item5of50 NationalBoardofMedicalExaminers^
ComprehensiveBasicScienceSelf-Assessment
Y
5.A10-year-oldboyisbroughttothephysicianbyhisparentsbecauseofincreasedurinaryfrequencyandprogressivefatigueduringthepast3weeks.Physicalexaminationshowsnoabnormalities.Hisfastingserumglucoseconcentrationis350mg/dL
Urinalysisshows4+glucose.Whichofthefollowingmechanismsintheproximaltubuleisthemostlikelycauseoftheglycosuriainthispatient?
A)Decreasedsodiumdiffusionacrosstheapicalmembrane
B)Inhibitionofthesodium/glucosecotransporter-1(SGLT-1)carrierprotein
G)Saturationoftheglucosetransporterprotein-1(GLUT-1)carrierprotein
D}Saturationofglucosetransporterprotein-2(GLUT-2)carrierprotein
E)Saturationofthesodium/glucosecotransporter-1(SGLT-1)carrierprotein
CorrectAnswer:E
Saturationofthesodium/glucosecotransporter-1(SGLT-1)carrierproteinintherenalproximaltubuleaccountsforthispatient
'sglycosuria
;likelyinthesettingofanewdiagnosisoftypeEdiabetesmellitus.Autoimmunedestructionofpancreaticisletcellscauses
profoundhyperglycemia.Mostcellularglucosetransportproteinsrequirethepresenceofinsulintofunction:thus
;insulindeficiencyleadstohighserumlevelsofglucosebutlowintracellularstoresofglucose.Circulatingglucoseisfilteredthroughtheglomerulusand
resorbedfromfiltrateinthetubulesviatheactionofSGLT-1andSGLT-2,whichtransportglucoseacrosstheapicalmembraneinanadenosinetriphosphate(ATP)-dependentmannerSGLT-1accountsforonlyabout10%ofglucoseresorptionwhileSGLT-2
accountsforabout90%.Thesereceptorshaveamaximumrateatwhichtheycanresorbglucoseinfiltrate,andwhenbloodglucoseconcentrationexceedapproximately180mg/dLtherateofglucosefiltrationexceedstheabilityofthetransporterproteinstoresorb
it.Thisresultsinglycosuria.
IncorrectAnswers:A.R,C.andD.
Decreasedsodiumdiffusionacrosstheapicalmembrane(ChoiceA)wouldnotresultinglycosuria,althoughhyperglycemiacanindirectlyaffecttotalbodysodiumbalance.Sodiumispassivelyreabsorbedalongaconcentrationgradientalongwithwaterandchloride
andisalsoresorbedviatheactionofSGLT-1andasodium-hydrogenantiporter.
Inhibitionofthesodium/glucosecotransporter-1(SGLT-1)carrierprotein(ChoiceB)doesnotaccountforglycosuriainhyperglycemia.SGLT-1isexpressedontheluminalbrushborderinenterocyteswhereitabsorbsglucosefromthelumenandisexpressedinthe
renalproximaltubuleswhereitcontributestogfucoseresorption.InhibitionofSGLT-2.notSGLT-1.isacharacteristicofaclassofmedicationsusedtotreattypeIIdiabetesmellitusandincludesmedicationssuchasempagliflozinandcanagliflozin.These
medicationsarenotusedtotreattypeIdiabetesmellitus,whichoccursasaresultofinsulindeficiency.
Saturationoftheglucosetransporterprotein-1(GLUT-1)carrierprotein(ChoiceC)isnotthemechanismbywhichglycosuriaoccursinpatientswithhyperglycemiaTheGLUT-1transporterismostactiveonthesurfaceoferythrocytesandactstobringglucoseinto
thecytoplasmwhereitisphosphorylatedtoglucose-6-phosphate.Itisalsoexpressediinendothelialcellsandisnottheprimaryglucosetransportproteininthenephron.
Saturationofglucosetransporterprotein-2(GLUT-2)carrierprotein(ChoiceD)occursafterlargeoralglucoseloads.GLUT-2isexpressedintheliverpancreas,kidneys,andgastrointestinaltract.Itfunctionsasaglucose-sensingreceptorintheliverandpancreas
butplayslessofaroleofgucosetransportinthenephron.
EducationalObjective:Filteredglucoseisreclaimedbythenephronthroughtheactionofsodium-glucosecotransporterproteins.Whenserumglucoseconcentrationsexceedapproximately130rrig/dL
;thesetransportersbegintosaturateandsomefilteredglucose
cannotberecoveredfromthetubularfiltrate,resultinginglycosuria.
© © ©
Previous Next ScoreReport LabValues Calculator Help Pause

ExamSection4:Item6of50 NationalBoardofMedicalExaminers^
ComprehensiveBasicScienceSelf-Assessment
Y
6.A5-year-oldboyhasseverecholestaticfiverdiseaseandpoorbilesecretion.ThispatientisalsolikelytohaveadeficiencyofvitaminEbecauseitsabsorptionrequireswhichofthefollowing?
A)Bindingtocholesterol
B)Conversiontoanactiveformbytheliver
C)Formationofmicelles
D)Intrinsicfactor
E)Trypsin
CorrectAnswer:C.
Theformationofmiceilesisanecessarystepintheabsorptionoffat-solublevitaminssuchasvitaminEAmicelleisaspherecontainingbiteacids,whichareamphipathic.Thehydrophilicheadsorientattheperipheryofthesphere,whilethehydrophobiccontents
remainontheinside.Wheningested,vitaminEisincorporatedintomicellesviaaffinitywithbileacidssecretedfromtheliver.Theiramphipathicnatureallowsthemtoremainsolubleinaqueoussolution,whiletransportinglipidsandfat-solublevitaminsintheir
hydrophobiccores.Bileacidsandmicellesthusarenecessaryfortransportinghydrophobicmoleculesintoenterocytes.Oncewithinanenterocyte,vitaminEisincorporatedintochylomicronsandtransportedtotheliverforstoragewhereitcanbesecretedinVLDL
particles.AdeficiencyinbileproductionorsecretionsuchasinthispatientwithcholestaticliverdiseasewouldgreatlyaltertheabsorptionofvitaminEasaresultoffailuretoformmicelles.DeficiencyofvitaminEcancausemyopathy,anemia,andneurological
dysfunction.
IncorrectAnswers:A,B,D,andE.
Bindingtocholesterol(ChoiceA)isnotthemechanismbywhichvitaminEisabsorbed.Cholesterolisanecessarycomponentofbileacids,whichareneededtosolubilizevitaminEinmicelles,butcholesteroldoesn
'tdirectlybindvitaminE.
Conversiontoanactiveformbytheliver(ChoiceB)isnotimpairedinpatientswhocannotadequatelysecretebile..VitaminEisprimarilystoredintheliverandsecretedinVLDLItdoesnotrequiremodificationbythelivertobecomeactive.
Intrinsicfactor(ChoiceD)isaproteinfoundinthestomachthatisrequiredfortheabsorptionofvitamin notvitaminE.Antibodiesdirectedagainstthisproteinresultinperniciousanemia.
Trypsin(ChoiceE)isadigestiveenzymethatdegradesproteins.Trypsinogeniscreatedinthepancreasandsecretedintotheduodenumafteramealwhereitisactivatedtotrypsin.ItdoesnotplayaroleintheabsorptionofvitaminE.
EducationalObjective:VitaminEisafat-sofubievitaminthatmustbeincorporatedintomicellesinthepresenceofbileacidsbeforeitcanbeabsorbedbyenterocytesandtransportedtotheliverAnabsenceofbileacids,asmightoccurinconditionswherebile
secretionisimpaired,wouldresultinvitaminEmalabsorptionduetothefailuretoformmicelles.
00#
Previous Next ScoreReport LabValues Calculator Help Pause

ExamSection4:Item7of50 NationalBoardofMetlicaiExaminers^
ComprehensiveBasicScienceSelf-Assessment
y
7.A23-year-oldwomancomestothephysicianbecauseofa1-weekhistoryoffatigue,musclepain,andgeneralizedweakness.Sheisunabletoclimbstairsbecauseofthesymptoms.Shealsohasa1-monthhistoryofarashonherface
:elbows
;and
knuckles.Hertemperatureis38.3°C(1G0.9T),pulseis100/min.respirationsare20/miandbloodpressureis115/65mmHgPhysicalexaminationshowsthefindingsinthephotographs.Thereisalsoproximalmuscleweaknessofthelowerextremities
bilaterally.Whichofthefollowingisthemostlikelydiagnosis?
A)Dermatomyositis
B)Gui
:ain-Barresyndrome
G)Nephroticsyndrome
D)Psoriasis
E)Systemiclupuserythematosus
CorrectAnswer:A.
Dermatomyositisisanautoimmuneconditionaffectingtheskinandtheproximalmuscles.Cutaneousmanifestationsincludepinkpapulesoverthedorsalfingers(Gottronpapules),pinktolightpurplepatchesovertheuppereyelids(heliotropesign),tightpinkto
lilaccoloredpatcheswithtetangiectasiasontheupperbackandsun-exposedchestanddilatedcapillarieswithdropoutontheproximalnailfoldsAmyopathicdermatomyositisisarareforminwhichonlycutaneousmanifestationsarepresent,butaconcomitant
inflammatorymyopathywithproximalmuscleweaknessismoretypical.Aninflammatoryinfiltrateofthemuscleoccurscausingdamagetothemyocytes,leadingtoreleaseofcreatinekinase,anintracellularmolecule.Thiscanbeusedtomonitordiseaseprogress
indermatomyositis.Dermatomyositisisalsocharacterizedbyseveralautoantibodies.Itdemonstratesapositiveantinuclearantibody(ANA)andanti-Jo-1.anti-SRP,andanti-Mi-2antibodiesarecommonlydetected.Dermatomyositisisalsoassociatedwithoccult
malignancyandpatientswithanewdiagnosisofdermatomyositisshouldundergoage-appropriatescreeningformalignancy.Treatmentofdermatomyositisinvolvessystemicsteroidsfollowedbylong-termimmunosuppressivetherapywithmedicationssuchas
methotrexateorazathioprine.
IncorrectAnswers:B.C,D,andE.
Guilaln-Barresyndrome(ChoiceB)istypicallyaself-limiteddemyelinatingpolyneuropathythatstartswiththedistalextremitiesandprogressesproximally.Itmayaffectthelungsthroughdiaphragmaticparalysis,whichisthemostseriouscomplicationofthe
disease.Whileproximalmuscleweaknesscanensue,thiswouldonlyoccurafterinvolvementofthedistalextremitiesintheclassicform.
Nephroticsyndrome(ChoiceC)ischaracterizedbyproteinuria,edema,hypoalbuminemia,andhypercholesterolemia.Duetodamagetotheglomerularfiltrationbarrier,largeproteinssuchasalbuminarefilteredintotheurinecausinghypoalbuminemia.
Membranousnephropathyisthemostcommonnephroticsyndromeinwhiteadultsandcanbeprimary(idiopathic)orsecondarytoconditionssuchassystemiclupuserythematosusautoimmunediseaseagainstphospholipaseA
2
-receptors.chronichepatitis
infection(especiallyhepatitisBorCvirus),orpharmaceuticalssuchasnonsteroidalanti-inflammatorydrugsorpenicillamine.Itisnotafeatureofdermatomyositis.
Psoriasis(ChoiceD)ischaracterizedbythick,salmon-coloredplaqueswithsilverywhitescaleclassicallypresentontheextensorextremities.Itcanbeaccompaniedbypsoriaticarthritis,aninflammatoryjointdisease,whichpresentswithmorningstiffnessand
swellingofthesmalljointsofthefingersandtoes.Psoriasisdoesnottypicallycausemusclepain,generalizedweakness,orfever.
Systemiclupuserythematosus(ChoiceE)isanautoimmunedisorderwithawiderangeofpresentation.Cutaneousmanifestationsincludeanerythematousrashoverthemalarcheeksthatsparesthenasolabiaifoldsaftersunexposureordiscoidlesionswith
hyperpigmentededgesandpink,atrophiccenters.Painlessoralulcersandhairlossmayoccur.Fatigue,jointpain,andfeverarealsocommonpresentingsigns.However,proximalmuscleweaknessismorespecificforaninflammatorymyositis.Systemiclupus
erythematosusdoesnottypicallypresentwithGottronpapulesonthedorsalhandsortheheliotroperashontheeyelids.
EducationalObjective:Dermatomyositisisanautoimmunediseaseaffectingtheskinandmuscles,whichdemonstratesapositiveANA,anti-Jo-1.anti-SRP.andanti-Mi-2antibodies.ClassiccutaneousfindingsincludeGottronpapules,heliotroperashonthe
eyelids,andalightpinkrashwithtelangiectasiasontheupperbackandchest.Thesefindingsareusuallyaccompaniedbyproximalmuscleweaknessandincreasedlevelsofcreatinekinase.
© © m
Previous Next SconeReport LabValues Calculator Help Pause

ExamSection4:ItemBof50 NationalBoardofMedicalExaminers^
ComprehensiveBasicScienceSelf-Assessment
Y
8.A58-year-oldwomanisbroughttotheemergencydepartment6hoursafterthesuddenonsetofleft-sidedfacial,arm,andlegweakness.Shehashypertensiontreatedwithadiuretic.Shereportsthesensationofaflutteringheartfor1week.Herpulseis
125/minandirregular,andbloodpressureis135/80mmHg.Physicalexaminationshowsaleft-sidedfacialdroop.Thereisflaccidweaknessoftheleftupperandlowerextremities.AnECGshowsarapid,irregularventricularratewithnodiscernableP
waves.Whichofthefollowingtherapieswouldbestimprovefunctionoftheweakenedmusclesoverthenext3hours?
A)Acetylcholineagonist
B)Acetylcholineantagonist
C)Antifrbrinolytic
D)Fibrinolytic
E)Procoagulant
CorrectAnswer:D.
Thispatientwithalikelythromboemboliccerebralinfarction(stroke)fromatrialfibrillationwouldbenefitfromfibrinolytictherapy.Strokesoccurduetoischemicorhemorrhagiclossofbloodsupplytothebrainandmanifestasfocalneurologicaldeficitsrelatedtothe
dysfunctionoftheaffectedbrainregion.Approximately80to35%ofstrokesareischemic,commonlyarisingfromthromboembolicdisease,forwhichatrialfibrillationisamajorriskfactorAtrialfibrillationtypicallypresentswithpalpitations,fatigue,lightheadedness,
andmilddyspneaifsymptomatic.OnECG,atrialfibrillationdemonstratesirregularlyirregularRRintervalswithoutdiscerniblePwaves.Prolongedatrialfibrillationleadstoleftatrialhemostasisandincreasestheriskofthrombosis.Athrombusfromthispatient'sleft
atriumlikelyembolizedtotherightinternalcarotidarterysupplyingtherightprecentralgyrus,resultinginleft-sidedhemiparesis.Alteplase(tissueplasminogenactivator)isafibrinolyticmedicationutilizedinischemicstrokesthatbindstofibrinclotsandconverts
plasminogentoplasmin,whichlysesclots.Ifgivenearly(generallywithin3to4.5hoursafteronsetoftheevent),alteplasemaypromoteneurologicalrecovery.
IncorrectAnswers:A.B,C,andE.
Acetylcholineagonists(ChoiceA)andantagonists(ChoiceB)areunlikelytobehelpfulinlysingthispatient'sthromboembolicclot,astheinteractionofacetylcholinewiththecoagulationcascadeispoorlydefined.Acetylcholineagonists(eg,bethanechol)may
improvecognitioninpatientswithdementia,decreaseheartrate,improvegutperistalsis,increasebladdercontraction,andincreaseexocrineglandsecretions.Acetylcholineantagonists(eg,benztropine)typicallyincreaseheartrate,decreasegutandbladder
activity,andworsencognitivefunction.Theyalsoactonmuscleatthemotorend-plate:however,thispatient'sweaknessresultsfromcentralnervoussystemdysfunctionwithoutpathofogyatthemusclefiberitself.
Antifibrinolytic(ChoiceC)therapy(eg
;tranexamicacid)displacesplasminogenfromfibrinclotstopromotehemostasisduringintraoperativebleeding,heavymenstrualbleeding,andtraumatichemorrhage.Procoagulant(ChoiceE)therapy(eg.protamine,
coagulationfactors)Increasesactivationofthecoagulationcascade.Bothantifibrinolyticandprocoagulanttherapywouldnotlysethispatient
'sclotandmayleadtofurtherthrombosis.
EducationalObjective:Patientswithatrialfibrillationareatriskforthromboembolismduetoleftatria1hemostasis,whichmayresultinischemicstrokes.Strokesmanifestasfocalneurologicaldeficitsrelatedtothedysfunctionoftheaffectedbrainregion.Ischemic
strokesaretreatedwithfibrinolytictherapy,whichpromotesneurofogicairecovery.
© © © oo
Previous Next ScoreReport LabValues Calculator Help Pause

ExamSection4:Item9of50 NationalBoardofMedicalExaminers^
ComprehensiveBasicScienceSelf-Assessment
Y
9.AninvestigatorisplanningtocreategenetherapyforLeighsyndrome,whichiscausedbyanA^GmutationinthemitochondrialtRNALeugene.WhichofthefollowingisthemostlikelyreasonwhymitochondriaencodetheirowntRNA?
A)MitochondriacannotimportproteinsorRNA
B)Mitochondriaproducelargeamountsofreactiveoxygenspecies
C)Mitochondriauseanon-standardgeneticcode
D)TheunusuallyhighmitochondrialpHdenaturesnuclear-encodedtRNA
E)TheunusuallylowmitochondrialpHhydrolyzesnuclear-encodedtRNA
CorrectAnswer:C.
Themitochondrialgenomediffersfromthenucleargenomeinthatitissmall,circularandpresentinnumerouscopieswithineachmitochondrion.Additionally,themitochondrialgenomereliesondifferentcodingsequencesforstopcodonsandforsomeamino
acids,suchastryptophan.TheuseofdifferentcodingsequencesnecessitatestheuseofaseparatesystemoftRNA.Themitochondrialgenomeencodesseveralproteinsfortheoxidativephosphorylationelectrontransportchainproteins(eg,cytachromes)
:but
mitochondrialfunctionremainsheavilydependentonimportingproteinscodedbythecellnucleus.
IncorrectAnswers:A,B.DaandE.
ThemitochondriadorelyonimportingproteinsandRNA(ChoiceA)fromthecellcytoplasm.Themitochondrialgenomeisnotcapableofproducingalloftherequiredproteinsformitochondrialfunction.MitochondrialcodingoftRNAdoesnotimpactthe
mitochondrialabilitytoimportproteins.
Mitochondriaproducelargeamountsofreactiveoxygenspecies{ChoiceB)butthisdoesnotcontributetotheneedforaseparatesystemoftRNA.Manyimportantmitochondrialantioxidants,suchasglutathione,aresynthesizedintheendoplasmicreticulumusing
standardcellulartRNA.
ThemitochondrialmatrixpHisnotunusuallyloworhigh,noristheinnermitochondrialmembranespace,despitethepresenceofaprotongradient(ChoicesDandE).Theinnermembraneisslightlyacidic,andthematrixisslightlyalkalinecomparedtothe
cytoplasmicpH.Nuclear-encodedtRNAisrelativelystableandisneitherhydrotyzednordenaturedunderslightlyacidicoralkalineconditions.
EducationalObjective:Themitochondrialgenomereliesondifferentcodingsequencesforstopcodonsandforsomeaminoacids
:suchastryptophan.TheuseofdifferentcodingsequencesnecessitatestheuseofaseparatesystemoftRNA.
O0
It
Previous Next ScoreReport LabValues Calculator Help Pause

ExamSection4:Item10ol50 NationalBoardofMedicalExaminers^
ComprehensiveBasicScienceSelf-Assessment
y
10A47-year-oldmanhasmicroscopicbloodonurinalysis.ACTscanoftheabdomenshowsa5-cmmassintheleftkidneyAphotomicrographoftissueobtainedonrenalbiopsyisshown.Whichofthefollowingis
themostlikelydiagnosis? -
-S
i
a.
V
A}Angiomyolipoma
B)Nephroblastoma
C)Oncocytoma
D)Renalcellcarcinoma
E}Transitionalcellcarcinoma
S
'J?v
EE
*
*

--V.A
r K
l i‘r
*
/
4f
,
V-*
-
CorrectAnswer:D.
Renalcellcarcinoma(RCC)isanadenocarcinomaoftubularepithelialcells.RCCisacommonprimarymalignancyofthekidneyandgenerallyoccursinoldermalesmokers.Itcanpresentwithgrossormicroscopichematuiia
:flankpain
;weightloss,orfever.
Laboratoryanalysismayrevealpolycythemiaorhypercalcemiaduetoassociatedparaneoplasticsyndromeproductionoferythropoietinorparathyroidhormone-relatedpeptide.Hypercalcemiacanresultfrombonymetastasis.DiagnosisofRCCtypicallyoccurs
withcontrast-enhancedCTscanorMRIandisconfirmedbybiopsyatthetimeofnephrectomy.ThehistologyofRCCisuniquelycharacterizedbythepresenceofpolygonalclearcells(seeninthephotomicrograph)duetotheaccumulationoflipidand
carbohydratewithintheceils.Itspreadshematogenously,andcommonlypresentsasametastaticneoplasm.Thebrainisafrequentsiteofmetastasis.
IncorrectAnswers:A,B.C.andE.
Angiomyolipomas(ChoiceA)aretumorsderivedfromperivascularepithelioidcells.Whiletypicallybenign,theycanbeassociatedwithtuberoussclerosis.Theypresentasamass
;whichcanbecomposedofsmoothmuscle,adipocyte
;andepithelioidcells.
Nephroblastoma(ChoiceB)isthemostcommonrenalmalignancyinchildhoodduetomutationsintumorsuppressorgenesWT1orWT2.Itischaracterizedbyalarge,oftenpalpable,unilateralflankmassandhematuria.Itisnotassociatedwithclearcellson
biopsy.
Oncocytomas(ChoiceC)areuncommonrenalmalignanciesandonhistology,appearaslargeneoplasticcellswithaneosinophilicgranuarcytoplasmduetopresenceofnumerousmitochondria.
Thetransitionalepitheliumofthecollectingsystemandbladderisspecializedstratifiedepitheliumthatisabletoexpandandcontractdependingonthebladdervolume.Transitionalcellcarcinoma(ChoiceE)occurswithinthecollectingsystemand/orbladderand
themalignantcellsdonotappearclear.
EducationalObjective:RCCoftenpresentsinoldermalesmokerswithgrossormicroscopichematuria,flankpain,weightloss,orfever.ThehistologyofRCC;suniquelycharacterizedbypolygonalclearceltsduetotheaccumulationoflipidandcarbohydrate
contentwithinthecells.
on ©QXV
Previous Next ScoreReport LabValues Calculator Help Pause

ExamSection4:Item11of50 NationalBoardofMedicalExaminers^
ComprehensiveBasicScienceSelf-Assessment
Y
11A27-year-oldmancomestothephysicianbecauseoffeverandcoughfor1month.Anx-rayofthechestshowsdiffuseinfiltrates.Tinyintracellularyeastformsareseenondirectsmears,andtubercuiatemacroconidiaareseenaftercultureatroom
temperature.Whichofthefollowingexposuresismostconsistentwiththispatient'sillness?
A)Birddroppings
B)Desertsandstorm
C)Insectbite
D)Moldyhay
E)Rosethornpuncture
CorrectAnswer:A.
HistoplasmosisisanendemicmycosissecondarytoHistoplasmacapsulatum.Pulmonaryhistoplasmosiscanpresentinacute
;subacute,orchronicphases,generallycharacterizedbyfevermalaise,myalgia,cough
:andhemoptysiswiththedevelopmentof
calcifiednodulesandmediastinalorhilarlymphadenopathy.Transmissionoccursthroughinhalationoffungalspores.ItisendemictotheMississippiandOhioRivervalleysintheUnitedStates,aswellaspailsofCentralandSouthAmerica.sub-SaharanAfrica,and
SoutheastAsia.ThevirulenceofH.capsulatumisdependentonitsgrowthinsidenonactivatedmacrophages,whichprotectthefungalcellsfromdefectionandelimination.Riskfactorsforhistoplasmosisincludeexposuretobirdorbatdroppings.Diagnosisis
typicallymadeviathedetectionofhistoplasmaantigenintheserumand/orurine.Histologywillrevealovalyeastcellswithinmacrophages.Histoplasmaspeciesonculturegrowswithseptatemyceliaandcharacteristictubercuiatemacroconidia.
IncorrectAnswers:B,C
:D
:andE.
Desertsandstorms(ChoiceB)areavectorbywhichmicroorganismsmaybedisseminatedtodistantenvironments.CoccidioidomycosisisanendemicmycosisprevalentintheSouthwesternUnitedStatesassociatedwithdustexposurethatmaybetransmittedby
sandstorms.HistologyrevealsspherulesfilledwithCocciciioldesspeciesendospores.
Insectbites(ChoiceC)areacommonvectorforinfectiousdiseasetransmission.H.capsulatumistransmittedviainhaledsporesfromtheenvironmentnotinsectbites.
Moldyhay(ChoiceD)isassociatedwithhypersensitivitypneumonitissecondarytotheinhalationofdustandmoldspares.H.capsulatumistransmittedviainhaledsporesfromsourcessuchasbirdorbatdroppings,notfromhay.
Rosethornpuncture(ChoiceE)isassociatedwithsporotrichosis,whichiscausedbythedimorphicfungusSporothrixschenckii.Transmissionisviatraumaticinoculationintotheskin.
EducationalObjective:HistoplasmosisisanendemicmycosiscausedbyH.capsulatumthatcausespulmonaryillnesswhensporesareinhaledfromtheenvironment.Histoplasmosisismostassociatedwithexposuretobirdorbatdroppings.
0'0
4
s
Previous Next ScoreReport LabValues Calculator Help Pause

ExamSection4:Item12of50 NationalBoardofMedicalExaminers^
ComprehensiveBasicScienceSelf-Assessment
Y
12.A68-year-oldmanwithtype2diabetesmelFitus
;hypertension.,andhyperlipidemiacomestothephysicianbecauseofpainwithurination
;bloodinhisurine,andfatigueduringthepastweek.Hehasa6-monthhistoryofincreasedurinaryfrequencywith
smallvolumeandnighttimeurination.Hehashadtwoepisodesofacutecystitisduringthepast8months.Histemperatureis38.3CC(100.9ClF),pulseis92/min,respirationsare2G/min,andbloodpressureis108/72mmHg.Physicalexaminationshows
suprapubictendernessandanenlarged,softprostate.Ultrasonographyshowsabladderdiverticulumandenlargedkidneyswithdilationofrenalpelvesandcalyces.AurineculturegrowsEscherichiacots.Whichofthefollowingisthemostlikelyunderlying
causeofthispatient'scondition?
A)Benignprostatichyperplasia
B)Hypertension
C)Nephrolithiasis
D)Type2diabetesmellitus
E)Xanthogranulomatouscystitis
CorrectAnswer:A.
Benignprostatichyperplasia(BPH)affectsmiddle-agedandelderlymalepatientsandoccursduetoexcessgrowthoftissueinthecentra!zoneoftheprostatecausinglowerurinarytractobstruction.BPHpresentswithurinarydribblingandincontinence,difficulty
startingormaintainingaurinarystream,nocturia,theneedtofrequentlyurinate,andincompletevoiding.Thesesymptomsaretypicallyinsidiousinonset,chronic,andprogressive.Chronicoutletobstructionleadstourinaryretentionandmaycausetheformationof
bladderdiverticulaorleadtohydronephrosis.Patientswithurinaryretentionmaydevelopurinarytractinfectionsduetothelossofhydrodynamicflushingofbacteriaoutoftheurethra.Diagnosisismadebyhistory,physical,andurodynamictesting,alongwithruling
outalternatediagnoses(eg,prostaticmalignancy).Treatmentiswithsupportivecare,a-bfockers(eg,tamsulosin),5-a-reductaseinhibitors(eg,finasteride),orsurgery.
IncorrectAnswers:B.C,D,andE.
Hypertension(ChoiceB)isacommoncauseofchronickidneydiseaseandend-stagekidneydisease,whichmaypresentwitholiguria.However,thiswouldnotproducelowerurinarytractobstructionwithurinaryretentionandhydronephrosis.
Nephrolithiasis(ChoiceC)ismorelikelytocauseurinarytractobstructionproximaltothebladder,ieadingtounilateralhydronephrosis.Bladderoutletobstructionismorefrequentlyduetoprostatichyperplasia,urethraEstricture,orbladderoutletmalignancy.
Type2diabetesmeIJitus(ChoiceD)cancauseneurogenicbladderthatpresentswithoverflowincontinence.Thispatient'senlargedprostatesuggestsprostatichyperplasiaasamorelikelycauseofhissymptoms.
Xanthogranulomatouscystitis(ChoiceE)isanexceedinglyrareentitythatmaypresentwithsymptomsofcystitisandischaracterizedbythepresenceofipid-ladenmacrophagesandTeutongiantcells.Thisrarediagnosisismuchlesslikelythanbenignprostatic
hyperplasia.
EducationalObjective:Benignprostatichyperplasiapresentswithurinarydribblingandincontinence,difficultystartingormaintainingaurinarystream,nocturia,frequenturination,andincompletevoiding.Chronicoutletobstruction:eadstourinaryretentionandmay
causetheformationofbladderdiverticulaorleadtohydronephrosis.Patientswithurinaryretentionmaydevelopfrequenturinarytractinfectionsduetothelimitedclearanceofurethra!bacteria.
© © ©
Previous Next ScoreReport LabValues Calculator Help Pause

ExamSection4:Item13of50 NationalBoardofMedicalExaminers^
ComprehensiveBasicScienceSelf-Assessment
Y
13.Aspartofanannualphysicalexamination,ahealthy50-year-oldmanhaslaboratorystudiesdone8hoursafteranovernightfast.Hisserumglucoseconcentrationis75mg/dLActivationofwhichofthefollowingreceptorsinlivercellsmostlikelymaintained
theserumglucoseconcentrationinthispatient?
A)Cytokine
B)Gprotein-coupled
C)Nuclear
D)Serinekinase
E)Tyrosinekinase
CorrectAnswer:B.
Thefastingstateisdependentonhepaticandmuscleglycogenolysis,hepaticgluconeogenesis,andlipolysis,whichareallmediatedbyglucagonandepinephrine.Glucagonisproducedbypancreaticisleta-celsandprimarilyworkstoincreaseserumglucose
throughglycogenolysisandgluconeogenesis.GlucagonactsonhepatocytesviaaGprotein-coupledreceptor,resultinginactivationofacyclicadenosinemonophosphatepathwaythatendsintheactivationofglycogenphosphorylase.nturn,glycogen
phosphorylasedegradesglycogenpolymerswiththereleaseofglucosemonomersintoserumfollowingdephosphorylationbyglucose-G-phosphatase.Glycogenphosphorylaseisupregulatedinstatesofstarvation,andresultsinthedegradationofglycogenstores.
Thismetabolicpathwayresultsinthereleaseofglucosemonomersforthemaintenanceofserumbloodsugarduringperiodsoffasting,asillustratedbythispatient'snormalbloodglucoseconcentrationfollowinganovernightfast.
IncorrectAnswers:A.C
;D.andE.
Cytokine(ChoiceA)receptorsareinvolvedintheimmuneresponse.CytokinessuchasinterferonsandintereukinsactoncytokinereceptorsandmediateintracellularresponsesthroughaJAK-STATpathway.
Nuclear(ChoiceC)receptorsarefoundintracellularly.Duetotheirintracellularlocation,theyareacteduponbyIrpid-solubleligands,suchasvitaminsAandD,steroidhormones{eg,estrogen,progesterone),andthyroidhormone.Nuclearreceptorsdirectlybind
DNAandregulateexpressionofgenes.Glucagonandepinephrinearehydrophobicpeptidemoleculesthatactatthecellsurface,nevercrossingtheplasmamembrane.
Serinekinase(ChoiceD)isanenzymeclassthatfunctionsinproteinphosphorylationandregulationofcellproliferation,celldifferentiation,andapoptosis.Kinasesareubiquitousincellsignalingpathways,howevertheinitialreceptortargetedbyglucagonor
epinephrineisaGprotein-coupledreceptoratthecellsurface.
Insulinactsonatyrosinekinase(ChoiceE)receptor.Insulinalsofunctionsinglucosehomeostasis,butgenerallyhastheoppositeeffectofglucagon.Insulinactstoincreaseperipheraltissueuptakeofglucoseandpromoteshepaticglucosestorageand
glycogenosis,notglycogenolysis.
EducationalObjective:GlucagonactsonhepatocytesviaaGprotein-coupledreceptorresultinginactivationofacyclicadenosinemonophosphatepathwaythatendsintheactivationofglycogenphosphorylase.Thispathwayresultsinthereleaseofglucose
monomersintotheserumtostabilizeserumbloodsugarduringperiodsoffasting.
00 & 00
Previous Next ScoreReport LabValues Calculator Help Pause

ExamSection4:Item14ol50 NationalBoardofMedicalExaminers^
ComprehensiveBasicScienceSelf-Assessment
Y
14.A73-year-oldmanwithstageIVcoloncancerbeginschemotherapywith5-fluorouracil,irinotecarr,andleucovorin.Thispatientisatgreatestriskfordevelopingsignsofdrugtoxicityinwhichofthefollowingtissues?
A)Bonemarrow,gutmucosa,andhairfollicles
B)Liver,kidneyandendocrineorgans
C)Nerves,bone
:andheartmuscle
D}Skeletalmuscle,heartandbone
E)Trachea,bronchialepithelium,andliver
CorrectAnswer:A.
Cytotoxicchemotherapyiscommonlyemployedinthetreatmentofmalignancy.Thesemedicationsareeffectivebytargetingactivelydividingcells.Theseincludemalignantneoplasmsaswellasnormalhostcellsthatdemonstratehighratesofreplicationsuchas
thebonemarrow,gutmucosa,andhairfollicles.Thesetissueshaveahighrateofcellturnover,referredtoasahighgrowthfraction.5-fluorouracilisanantimetabolitethatimpairsDNAsynthesisintheSphaseofthecellcyclebydecreasingthymidinesynthesis.
IrinotecanisaninhibitoroftopoisomeraseI.Leucovorin(folinicacid}enhancestheactivityandbindingof5-ftuorouraciICommonsideeffectsofcytotoxicchemotherapyincludefatigue,alopecia,gastrointestinaldistress,nausea,vomiting,diarrhea,rashes,and
pancytopenia.Neoplasmsmaydevelopresistancetochemotherapeuticagents.Mechanismsofresistanceincludedecreasedbindingaffinitythroughbindingsitemutations,expressionofeffluxpumps,enzymaticdegradationofthedrug,andinhibitorbindingtothe
drug.
IncorrectAnswers:B.C,D,andE.
Theotheranswerchoices(ChoicesB,C,D,andE)refertotissuetypesthathavealowgrowthfractionandarenotaffectedtothesamedegreebygeneralcytotoxicchemotherapy.Certainchemotherapeuticagentsareassociatedwithspecificorgansystem
toxicities.DoxorubicinisaDNA-intercaiatingagentassociatedwithcardiotoxicity.BleomycincausesoxidativedamagetoDNAandisassociatedwithpulmonaryfibrosis.Cisplatinandcarboplatinresultincross-linkingofDNA,whichdisruptsDNArepairandmay
causeototoxicityandnephrotoxicity.Vincristineinhibitsmicrotubuleformationandisassociatedwithperipheralneuropathy.Methotrexate.5-fluorouracil.andmercaptopurineinhibitnucleotidesynthesisandresultinbonemarrowsuppression.Cyclophosphamideis
analkylatingagentassociatedwithhemorrhagiccystitis.
EducationalObjective:Cytotoxicchemotherapytargetscellswithahighgrowthfraction,whichincluderapidlydividingmalignantneoplasmsaswellasnormalcellsofthebonemarrow,gastrointestinaltract,andhairfollicles.
00#
Previous Next ScoreReport LabValues Calculator Help Pause

ExamSection4:Item15of50 NationalBoardofMedicalExaminers^
ComprehensiveBasicScienceSelf-Assessment
Y
15.Aninvestigatorisconductingastudyofthefipopolysaccharidesynthesispathway.Duringthestudy,apolymerasechainreactiontestisdevelopedthatdetectsageneinvolvedinlipidAbiosynthesis.Whichofthefollowingorganismsismostlikelytoyielda
positiveresultusingthispolymerasechainreactiontest?
A)Candidaalbicans
B)Clostridiumdifficile
C)Mycoplasmapneumonias
D)Pasteurellamultocida
E)Staphylococcusaureus
CorrectAnswer:D.
PasteurellamuftocidaIsthemostlikelyorganismtodemonstratepresenceofthegenecodingforlipidAbiosynthesis,apartoftheoutermembraneinGram-negativebacteria.IncomparisontoGram-positiveorganisms,whichpossessacytoplasmicmembraneand
alargeouterpeptidoglycanwallGram-negativebacteriahaveaninnercytoplasmicmembrane,asmallpeptidoglycanwall,andanoutermembrane.LipidAconstitutespartofthelipopolysaccharide(LPS)moietythatlieswithintheoutermembraneofGram-
negativebacteria.LPSisthoughttoaccountformanyofthemanifestationsofGram-negativesepsis,includingsevererigorsandhighfevers,asitishighlyimmunogenic.LipidAismadeoftwoglucosamineunitslinkedtofattyacids.Immuneactivationisthoughtto
occurviasignalingthroughtoll-likereceptors.P.multocida.aGram-negativeorganism,islikelytoexhibitthepresenceofthelipidAgene.
IncorrectAnswers:A.B,C.andE.
Candidaalbicans(ChoiceA)isafungusthatcausesavarietyofinfectionsinbothimmunocompetentandimmunocompromisedpatients.C.albicansoftencausesskininfectionsinmoistintertriginousareassuchasthegroinandinframammaryfoldsbutcanalso
causeinvasiveinfectionswithhighmorbidityandmortality,includingbloodstreamandurinarytractinfections.Asitisafungus,itdoesnotpossesslipidA.
Clostridiumdifficile(ChoiceB)isaGram-positiveanaerobethatcausescolitis.ItdoesnotpossessiipidA,butdoessecretetoxinAandElwhicharelargelyresponsibleforitspathogenicity.
Mycoplasmapneumoniae(ChoiceC)isarod-shaped,intracellularGram-indeterminatebacteriathatmostcommonlycausespneumonia.ItdoesnotpossessacellwalloripidA.
Staphylococcusaureus(ChoiceE)isaGram-positivebacteriumthatcausesmultipletypesofinfectionsincludingskinandsofttissueinfections,pneumonia,endocarditis,andbacteremia.ItisaGram-positiveorganismthatdoesnotpossesslipidAinitsceilwall
EducationalObjective:Gram-negativebacteriapossesslipidAasapartofL.PS,ahighlyimmunogeniccomponentoftheGram-negativeoutermembranethatdoesnotexistinGram-positivebacteria.
00# 00
Previous Next ScoreReport LabValues Calculator Help Pause

ExamSection4:Item16o!50 NationalBoardofMedicalExaminers^
ComprehensiveBasicScienceSelf-Assessment
Y
16.A25-year-oldwomancomestothephysicianbecauseofgeneralmalaiseandafacialrashfor1week.Shehasa10-yearhistoryofepisodesofpleurisyandarthriticpaininperipheraljoints.Physicalexaminationshowsanerythematousmalarrashthat
doesnotinvolvethenasolabialfolds.Resultsofcardioiipinantibody
;anti-dsDNA.andanti-Smantibodyassaysarepositive.Whichofthefollowinghematologicabnormalitiesismostlikelyinthispatient?
A)Hemolyticuremicsyndrome
B)Macrocyticanemia
C)Multiplenucleatederythrocytes
D)Rouleauxformations
E)Thrombocytopenia
CorrectAnswer:E
Thrombocytopeniaisacommonfindinginpatientswithsystemiclupuserythematosus(SLE)andisoftenduetoimmunethrombocytopenicpurpura(ITP)
:aconditiondefinedbytheautoantibody-mediateddestructionofcirculatingplatelets.Bonemarrowbiopsy
:if
obtained,willclassicallyshowanincreasednumberofmegakaryocytesindicatingadequateplateletproductionbutincreasedperipheraldestructionofplatelets.Itusuallypresentswithpetechiaeandpurpura,andlaboratorystudieswillshowprolongedbleeding
time.SevereITRmayleadtouncontrolledhemorrhage.Giventhispatientspositiveanticardiolipinantibody,thrombocytopeniamayalsobesecondarytoantiphospholipidantibodysyndrome,butthisislesscommonthanITP.Othercausesofthrombocytopeniain
patientswithSEEincludesplenomegalyanddrug-inducedthrombocytopenia.TherearecriteriatodiagnoseSLE,withcriteriondividedintoclinicalandimmunologiccategories.Thispatientmeetscriteriagiventhepresenceofarthritis
;pfeuritis
:malarrash,
anticardiolipinantibodies,anti-dsDNAantibodies,andanti-Smantibodies.OtherpotentialfindingsinpatientswithSLEincludephotosensitivity,discoidrash,oralulcers,neurologicphenomena,andleukopenia.GiventheclinicaldiversityofpatientswithSEEitis
commonforpatientstogomanyyearswithoutmeetingenoughcriteriatoreceivethediagnosis,andITPcanoftenprecedeaformaldiagnosisofSLEbyseveralyears.
IncorrectAnswers:A.B,C
;and0.
Hemolyticuremicsyndrome(ChoiceA)isclassicallyassociatedwithinfectionfromEscherichiacoli0157:H7.aGram-negativerod.Itoftenpresentsinchildrenwithbloodydiarrhea,microangiopathichemolyticanemia,thrombocytopenia,andrenalfailure.
Macrocyticanemia(ChoiceB)describesthepresenceoflargeerythrocytesandismostcommonlyaresultoffolateorvitaminB
12
deficiency.Itisoftenseeninpatientswrithmalnutrition,alcoholism,perniciousanemia,orCrohndisease.WhilepatientswithSLEmay
deveiopmacrocyticanemiafromnutritionaldeficiency,itisnotaclassicallyassociatedcondition.
Multiplenucleatederythrocytes(ChoiceC)ontheperipheralbloodsmearFSseeninconditionssuchasmyelofibrosis,whereerythroidprecursorsstillpossessinganucleusareprematurelyreleasedfromthebonemarrow.Itcanalsobeseeninseverehemolytic
anemiaasaresultofincreasedreleaseofimmatureerythrocytesfromthebonemarrowinresponsetoanemia.ItisnotclassicallyseeninSLE.
Rouleauxformation(ChoiceD)isaphenomenonseeninconditionssuchasmultiplemyelomaandWaldenstrommacroglobulinemia,wherethereisanincreaseinserumimmunoglobulins.Itisoccasionallyseenininflammatorystateswherethereareincreased
levelsofacutephasereactantssuchasfibrinogen.Thesemoleculescaninteractwithsialicacidonthesurfaceoferythrocytesandcausethecellstoaggregateinrouleauxformation.ItisnotclassicallyassociatedwithSLE.
EducationalObjective:ThrombocytopeniaisacommonpresentingfeatureofSLEandmayprecedethediagnosisbyseveralyearsItismostoftencausedbyITPaconditiondefinedbytheautoantibody-mediateddestructionofcirculatingplatelets.
O0
It
Previous Next ScoreReport LabValues Calculator Help Pause

ExamSection4:Item17of50 NationalBoardofMedicalExaminers^
ComprehensiveBasicScienceSelf-Assessment
Y
17.Whichofthefollowingcharacteristicsoftheglucocorticoidreceptoristhemostlikelycauseofthepersistenceofthepharmacologiceffectsofdexamethasoneafterthedrugiseliminatedfromthebody?
A)Catalyzesformationofstabletyrosine-phosphatebonds
B)Competeswithp-arrestinforbindingtoadrenergicreceptors
C)CouplestoGproteinswithlowintrinsicGTPaseactivity
D)Depletescellularstoresofglutathione
E)Inducesproteinsthatremainfunctionalafterthedrugiseliminated
CorrectAnswer:E
Steroidhormones,suchasdexamethasone.bindandactivateintracellularreceptorsandisabletotraversethecellmembraneduetotheirlipophilicnature.Othersteroidhormonesincludeestrogen,progesterone,testosterone,andaldosterone.Allofthese
hormonesarederivedfromcholesterolasacommonprecursor.Steroidhormonereceptorsareintracellularandresideeitherinthecytoplasmorthenucleusandregulategeneexpression.Becauseoftheeffectsmediatedthroughgeneregulation,theonsetof
actionofsteroidhormonesoccursoverthecourseofhourstodays.Intracellularsteroidreceptorshaveanumberofdifferentproteindomains.TheDNAbindingdomainclassicallyhasazincfingermotifasatertiarystructure.Thisdomainhasaffinitywiththe
regulatory(eg.promotersilencer)regionsofDNA.Thealteredproteinexpressioninducedbysteroidhormonesmaypersistevenafterthehormoneitselfiseliminatedbecausethesynthesizedpeptidesmaynaveahalf-lifedistinctlylongerthanthatofthedrugitself.
IncorrectAnswers:A.B.C.andD.
Formationofstabletyrosine-phosphatebonds(ChoiceA)isafeatureoftyrosinekinasereceptors,suchastheinsulinreceptor.Onceactivatedbyhomodimerization,thecytoplasmicportionsofthereceptorengageinautophosphorylationofspecifictyrosine
residues,whichcreatesbindingsitesfordownstreamsignalingeffectors.
Competitionwithp-arrestinforbindingtoadrenergicreceptors(ChoiceEl)isafeatureofGproteins.AdrenergicreceptorsareGprotein-coupledreceptorsandinteractwithGproteinsonceactivated.P
-arrestinscompetetobindwithandinactivatetheadrenergic
receptorinordertoattenuatethesignalingresponse.CouplingtoGproteinswithlowintrinsicGTPaseactivity(ChoiceC)isalsoafeatureofGprotein-coupledreceptors.
Depletionofcellularstoresofglutathione(ChoiceD)ischaracteristicofacetaminophentoxicityandisnotaresultoftreatmentwithdexamethasone.
EducationalObjective:Steroidhormones,suchasdexamethasone,bindandactivateintracellularreceptorsthatfunctionastranscriptionfactors.Activationofthesteroidhormonereceptorinducesalterationsinproteinexpressionthatpersistafterthehormoneitself
iseliminated.
© © ©
Previous Next ScoreReport LabValues Calculator Help Pause

ExamSection4:Item13o!50 NationalBoardofMedicalExaminers^
ComprehensiveBasicScienceSelf-Assessment
Y
18.A28-year-oldwomancomestothephysicianforadviceonhowtoloseweight.Shetellsthephysicianthatshebingesonhigh-carbohydratefoodstwotothreetimesweekly,usuallyforcingherselftovomitafterabinge.Sheis168cm(5ft6in)talland
weighs63kg(140lb);BMIis23kg/m
2Whichofthefollowingadditionalphysicalfindingsismostlikelyinthispatient?
A)Bradycardia
B)Decreasedvibratorysensationattheankles
C)Parotidglandenlargement
D)Sluggishdeeptendonreflexes
E)Sparseaxillaryandpubichair
CorrectAnswer:C.
Patientswitheatingdisorderssuchasanorexianervosaandbulimianervosawhopurgebyvomitingmaydemonstrateparotidglandenlargementonphysicalexamination.Thispatientlikelyhasbulimianervosa(purgingtype).Bulimianetvosa(purgingtype)
involvescyclesofuncontrollableeatingandcompensatorybehaviorssuchasvomiting
;iaxative
;ordiureticoverusethatoccuratleastonceaweekoverthreemonthsormore.Unlikepatientswithanorexianervosa,patientswithbulimianervosatypicallyhavea
normalBMI.Patientswiththebinging/purgingtypeofanorexiaorbulimianervosacandemonstrateparotidglandenlargementdentalcariesfromgastrichydrochloricaciderosionofenamelandscarsontheknucklessecondarytoabrasionsfromtheincisorswhen
inducingvomiting.Parotidglandenlargementtypicallymanifestsaspainlessbilateralenlargement.Thelassofgastrichydrochloricacidleadstohypochloremiaandmetabolicalkalosis.Inseverecases,signsofhypovolemiasuchastachycardiaandhypotension
maybepresent.Treatmentofbulimianervosaisthroughacombinedmedicalandpsychiatricapproachandinvolvescorrectingfluidandelectrolytederangementsalongsidebehavioralandpharmacologictherapy.
IncorrectAnswers:A.R,D.andE.
Bradycardia(ChoiceA)wouldbeatypicalforbulimianervosa.Tachycardiamayoccurinseverecasesofthepurgingtypeofanorexiaorbulimianervosaduetohypovolemiaortorsadesdepointesfrommetabolicalkalosisandhypokalemia.Severeanorexianervosa
canbeassociatedwithbradycardiaasaphysiologicaladaptationtodecreasedmetabolismandincreasedvagaltone.
Decreasedvibratorysensationattheankles(ChoiceB)andsluggishdeeptendonreflexes(ChoiceD)mayresultfromseveremalnutritioninanorexianervosa,asvitamindeficiencycanleadtosubacutecombineddegeneration.Bulimianervosamorecommonly
presentswithphysicalexamsignsofvomitingsuchasparotidglandenlargementthansequelaeofseveremalnutrition.
Sparseaxillaryandpubichair(ChoiceE)maybedemonstratedinseverelymalnourishedpatientswithanorexianervosaduetodecreasedadrenocorticotropichormoneproduction.Bulimianervosamorecommonlypresentswithphysicalexamsignsofvomiting
suchasparotidgfandswellingthansequelaeofseveremalnutrition.
EducationaObjective:Bulimianervosa(purgingtype)involvescyclesofuncontrollableeafingandcompensatorybehaviorssuchasvomiting,axative,ordiureticoveruse.Patientswhopurgebyvomitingcommonlydemonstrateparotidglandswelling.
Previous Next ScoreReport LabValues Calculator Help Pause

ExamSection4:Item19of50 NationalBoardofMedicalExaminers^
ComprehensiveBasicScienceSelf-Assessment
Y
19.Stripsoftrachealmusclecontractafteradministrationofcarbacholthenrelaxafteradministrationofepinephrine.Thegraphshowsmuscletensionasafunctionofthelogdoseofepinephrineunder
controlconditions(solidcurve)andinthepresenceofequalconcentrationsofantagonistsX(dashedcurve)andY(dottedcurve).XandYaremostlikelytobewhichofdiefollowingdrugs?
cn
£=DrugY
yohimbine
Metoprolol propranolol
Phentolamineprazosin
phentolamine
Propranolol metoprolol
Yohimbine clonidine
DrugX
Clonidine
O
CO
A)
£Z
OJ
OJ
B)
o
cr.
i
=
3
C)
D)Prazosin
Log[epinephrine]
E)
F)
CorrectAnswer:B.
DrugsXandYaremostlikelymetoprololandpropranolol.Metoprololisaselectivep1-adrenergicblockingagentwhilepropranololisanonselectivep-adrenergicblockingagentwithactivityatp
1
andp
2
receptors.Trachealandbronchialsmoothmuscletone,and
thusairwaydiameter,isprimarilyregulatedbyparasympatheticinnervationandsignalingthroughmuscarinicreceptors.However,theairwaysmoothmuscledoesexpressp
2
receptors,andactivationresultsinbronchodilationthroughGsprotein-coupledreceptor
signaltransduction.Thismechanismistherationalefortheuseofp-adrenergicagonistsinthetreatmentofasthmaexacerbations.DrugXonthegraphislikelymetoprolol.whichisselectiveforprreceptorswithminimalp
2
activity
;resultinginaslightincreasein
theconcentrationofepinephrineneededtooutcompeteforthebindingsite.Propranololisactiveatbothandp
2
receptors,resultingiinagreatlyincreasedconcentrationofepinephrineneededtostimulatesmoothmusclecontractionasrepresentedbycurveY
onthegraph.
IncorrectAnswers:A,C,D,E.andF.
ChoicesAandFareincorrect,asclonidineisanazadrenergicagonistandyohimbineisana
^
adrenergicblockingagent.azadrenergicreceptorsareprimarilylocatedinthecentralnervoussystem,andactivationresultsindecreasedsympatheticoutflow.They
wouldnotbeexpectedtoaffecttheisolatedtrachealsmoothmuscleresponsetoepinephrine.
ChoicesCandDareincorrect,asphentolamineisanonselectivea-adrenergicblockingagentandprazosinisaselectivearadrenergicblockingagent.Activationofa
^
adrenergicreceptorsonsmoothmusclecellsresuitsinincreasedmuscletensionthroughGq
protein-coupledreceptorsignaltransduction.Thesereceptorsareprimarilylocatedonsmoothmusclecellsofbloodvesselsandthegenitourinarytract,notthetrachea.
ChoiceEisincorrectasmetoprololhasdecreasedinhibitoryactivityatp
2
receptorscomparedtopropranololandthushasadecreasedeffectontrachealsmoothmuscletension.
EducationalObjective:Stimulationofpradrenergicreceptorsresultsinairwaysmoothmusclerelaxationandbronchodilation.Useofnonselectivep-adrenergicblockingagentsmayworsenbronchoconstrictioninthesettingofasthmaorchronicobstructive
pulmonarydisease.
© © © ft*
Previous Next ScoreReport LabValues Calculator Help Pause

ExamSection4:Item20ol50 NationalBoardofMedicalExaminers^
ComprehensiveBasicScienceSelf-Assessment
20.A19-year-oldwomancomestothephysician6hoursafterthesuddenonsetofleft-sided,crampingabdominalpain.Shedoesnothavenausea.Shealsohasa3-weekhistoryofpainwithsexualintercourse,andsheissexuallyactivewiththreemale
partners.Herlastmensesstarted5daysago.Currentmedicationisanoralcontraceptive.Hertemperatureis38CC(100.4*F).Physicalexaminationshowsanormalcervixwithnodischarge.Bimanualexaminationshowstendernessandfullnessontheleft
sidecomparedwiththeright,andcervicalmotiontenderness.Aurinepregnancytestresultisnegative.Whichofthefollowingstructuresismostikeiyaffectedbythispatient'scurrentcondition?
A)Appendix
B)Bladder
C)Endometrium
D)Kidney
E)Peritoneum
F)Uterine(fallopian)tube
CorrectAnswer:F
NeisseriagonorrhoeaeandChlamydiatrachomatiscanbothcausevaginitis,cervicitis,andpelvicinflammatorydisease(PJD).Vaginitistypicallypresentswithmucopurulentdischargeandpruritus.Cervicitispresentswithsimilardischarge,butalsowithan
erythematous,friablecervixandcervicalmotiontendernessonexamination.Wheninfectionspreadstothefallopiantubesandovaries,PDcanresult,whichpresentswithcervicalmotiontenderness,purulentcervicaldischarge,uterineandadnexaltenderness
andsystemicsignsandsymptomssuchasfever,fatigue,nausea,andmyalgias.Ifleftuntreated,PIDcanbecomecomplicated,withtubo-ovarianorintra-abdominalabscesspresentingwithfocaltendernessandafluctuantmass.TreatmentofPIDinvolves
antibioticssuchascephalosporinsandtetracyclines.RiskfactorsforPIDandsexuallytransmittedinfectionsincludeadolescence,multiplesexualpartners,andnotusingbarriercontraceptionsuchascondoms.
IncorrectAnswers:A.Ei.C.D,andE
Acuteappendicitis(ChoiceA)presentswithrightlowerquadrantormigratoryperiumbilicalabdominalpain,nausea,anorexia,fever,leukocytosis,andtendernesstopalpation.Ruptureorperforationofappendicitiscanleadtoinflammationoftheperitoneum(Choice
E),whichpresentswithabdominalguarding,reboundtenderness,andrigidity.CervicalmotiontendernessandleftsidedabdominalpainismorelikelyduetoPIDinvolvingthefallopiantube.
Bladder(ChoiceB)andurinarytractinfectionscanpresentwithurinaryfrequency,urgency,anddysuria.Urinalysisfortypicaurinarytractinfectionsreveals>10leukocytesperhighpowerfield,mayshowredcellsasevidenceofmucosalinflammation,andmay
showwhitebloodcellcastsifinfectionhasascendedtopyelonephritis.Bacteriaaregenerallyseenonmicroscopy.Ascendingurinarytractinfectionscaninvolvethekidney(ChoiceD)resultinginpyelonephritis.Pyelonephritistypicallypresentswithfevers,nausea,
vomiting,andflankpainwithassociatedcostovertebralangletendernessonphysicalexamination.
Endometritisisanacute,typicallypolymicrobialinfectionoftheuterineendometrium(ChoiceC)involvingamixtureofaerobesandanaerobesfromthegenitaltract.Patientstypicallypresentwithfever,severetendernessoftheuterinefundus,andmucopurulent
vaginaldischarge.
EducationsObjective:PIDsecondarytoN.gonorrhoeaeorC.trachomatisinvolvesthefallopiantubesandcanpresentwithcervicalmotiontenderness,purulentcervicaldischarge,uterineandadnexaltenderness,andsystemicsymptomssuchasfever.
© © ©
Previous Next ScoreReport LabValues Calculator Help Pause

ExamSection4:Item21ol50 NationalBoardofMedicalExaminers^
ComprehensiveBasicScienceSelf-Assessment
21A21-year-oldwomanwithprimarypulmonaryhypertensionbeginstreatmentwithbosentan.Asaresult,blockadeofwhichofthefollowingismostlikelytooccur?
A)AngiotensinIIreceptors
B)Calciumchannels
C)Endothelinreceptors
D)Productionofphosphodiesterase5
E)Voltage-gatedsodiumchannels
CorrectAnswerC.
Bosentanisacompetitiveendothelin-1receptorantagonistwhichisusedforthetreatmentofprimarypulmonaryarterialhypertension(PAH).TwoclinicallyrelevantsubtypesofendothelinreceptorsaretheendothelinA(ETA)andtheendothelinB(ETB)receptors.
Bindingofendothelin-1toETAresultsinvasoconstriction,whereasbindingofendothelin-1toETBresultsinproductionofnitricoxideandvasodilation.JnthecontextofPAH,theaggregateeffectofendothefin-1resultsinvasoconstrictionandanincreasein
pulmonaryvascularresistance.Bosentancompetitivelyantagonizesthebindingofendotheiin-1tobothETAandETBreceptors.Theneteffectofblockadeisvasodilationwithreducedpulmonaryvascularresistance,limitingpulmonaryhypertensionandrelieving
strainontherightsideoftheheart.Bosentanishepatotoxicandpatientstreatedwithbosentanshouldbemonitoredforelevationsofserumtransaminases.
IncorrectAnswers:A.B.D,andE.
BlockadeofangiotensinIIreceptors(ChoiceA)isthemechanismofactionofdrugssuchaslosartan,valsartan,andcandesartan.AngiotensinreceptorblockersarenotusefulforthetreatmentofPAH.
Blockadeofcalciumchannels(ChoiceB)isthemechanismofactionofdrugssuchasamlodipine,nifedipine,verapamil,anddiltiazem.Whilethesedrugsareusefulforthetreatmentofsystemichypertensionandofsupraventriculardysrhythmias(eg,atrial
fibrillation),theyarenotusefulfortreatmentofPAH.
Blockingproductionofphosphodiesterase5(ChoiceD)isthemechanismofactionofsildenafilandvardenafii.WhilesildenafilisusedforthetreatmentofPAH
:blockadeofphosphodiesterase5isnotthemechanismofactionofbosentan.
Blockadeofvoltage-gatedsodiumchannels(ChoiceE)isthemechanismofactionofantiarrhythmicagentssuchaslidocaine.Sodiumchannelblockersarealsousefulforthetreatmentofepilepsyandforlocalanesthesia.
EducationalObjective:Bosentanisacompetitiveendothelin-1receptorantagonistandcausesreducedpulmonaryvascularresistance.Itiseffectiveinthetreatmentofpulmonaryarterialhypertension.
00 & 0
f*
Previous Next ScoreReport LabValues Calculator Help Pause

ExamSection4:Item22ol50 NationalBoardofMedicalExaminers^
ComprehensiveBasicScienceSelf-Assessment
22.AGG-year-oldwomancomestothephysicianbecauseofa1-monthhistoryofbleedingfromalesiononhernose.Shehasnohistoryofmajormedicaiillnessandtakesnomedications.Physicalexaminationshowsa1-cmlesionontherightnaris.
Microscopicexaminationofabiopsyspecimenofthemassshowsneoplasticcellsthatexhibitdensepigmentgranules.Whichofthefollowingisthemostlikelydiagnosis?
A)Actinickeratosis
B)Basalcellcarcinoma
C)Melanoma
D)Rhabdomyosarcoma
E)Squamousceilcarcinoma
CorrectAnswer:C.
Malignantmelanomaislikelytobepresentwhenalesiondemonstratesasymmetry,irregularappearingborders,variablecoloration,adiametergreaterthan6mm,andrapidevolutionincharacteristics,itisaneoplasmofmelanocytes,whichcontainpigmentin
melanosomes.Histologically,itmayappearasdendriticcellscontainingpigmentgranuleswithhighnucleartocytoplasmicratiosmitoticfigures,andincreasedsizecomparedtonormalmelanocytes.Malignantmelanomacouldrapidlyinvadeandmetastasize,
whichcarriesapoorprognosiswhendiagnosedlate.Subtypesincludesuperficialspreading,nodular,lentigomaligna,andacrallentiginous.Anylesionwithfeaturessuggestiveofmalignantmelanomashouldbesurgicallyexcisedwithnegativemarginsand
pathologicallyexaminedforthedepthofdermalinvasion.
IncorrectAnswers:A.B,D.andE.
Actinickeratosis(ChoiceA)isapremafignantlesionthatmayprogresstosquamouscellcarcinoma..Theypresentaschronicrough,scalypatchesofskininareasofprolongedsunexposure(eg.face,ears,hands).
BasalcelEcarcinoma(ChoiceB)isthemostcommonformofskincancerandisderivedfromthebasalcellsoftheepidermisduetomutationsinthesonichedgehogpathwayorp53tumorsuppressor.Theprimaryriskfactorfordevelopingbasalcellcarcinomais
ultravioletlightexposure,thoughage.fairskin,andfamilyhistoryalsocontribute.Basalcellcarcinomastypicallypresentaspink,pearlypapulesornoduleswithrolledbordersandcentralulcerationmostcommonlyonthesunexposedareasoftheheadandneck.
Rhabdomyosarcoma(ChoiceD)isaneoplasmofmesenchymaltissueincludingsmoothmuscle.Sarcomabotryoidesisavariantofembryonalrhabdomyosarcomaandpresentsinyounggirlswithagrape-like,clear,polypoidmassprotrudingfromthevagina.If
rhabdomyosarcomaoccurswithintheskin,itappearsasasubcutaneousnoduleratherthanapigmentedpapule.
Squamousceilcarcinoma(ChoiceE)oftheskintypicallypresentswithanon-healingulcerativelesionwithascale.Itcommonlyoccurssecondarytosunexposure,chronicdrainingsinustracts,orimmunosuppression.
EducationalObjective:Malignantmelanomaisaneoplasmofmelanocytes,whichmicroscopicallymayappearasdendriticceilscontainingpigmentgranuleswithhighnucleartocytoplasmicratios,mitoticfigures,andincreasedsizecomparedtonormal
melanocytes.
O f? &
Previous Next ScoreReport LabValues Calculator Help Pause

ExamSection4:Item23of50 NationalBoardofMedicalExaminers^
ComprehensiveBasicScienceSelf-Assessment
Y
23.A13-year-oldgirlisbroughttothephysicianbecauseofamassinthethoracicregionthathasbeenenlargingduringthepastmonth.Shewasplacedinafosterhome1yearago.Sheisatthe70thpercentileforheightandweight.Breastandpubichair
developmentisTannerstage2.Physicalexaminationshowsa3-mm
;mildlypigmentedmasslocatedinthemidclavicular^neatthe10thrib.Themasshasasmoothsurface,uniformouterborderandcentralpapule.Therearenootherpigmentedlesions.
Whichofthefollowingisthemostlikelydiagnosis?
A)Accessorynipple
B)Benignpigmentednevus
C)Melanoma
D)Neurofibroma
E)Seborrheickeratosis
CorrectAnswer:A.
Anaccessorynippleisanuncommonfindingduetoanaberranceintheembryologicmigrationofmammarytissue.Cfinically
;itwillbecharacterizedbyasmallslightlyhyperpigmentedpapulesurroundedbyanareola-like,roundpatchlocatedatthemidclavicular
fine,asthislinemarksthenorma!migrationofmammarytissue.Thepapulemaygrowinsizeduringpubertyandthuscometoclinicalattention.Onbiopsyhistopathologicevaluationwiishowtypicalfindingsofmammarytissueincludingapapillomatousepidermis,
smoothmusclebundlesinthedermis
;mammaryglandsinthedermis,andacentralductleadingtoanopeningintheepidermis.Anaccessorynippleistypicallyanisolatedfindingandisnotcommonlyseeninconjunctionwithotherembryologicalabnormalities.
Treatmentissimpleexcisionofthelesion.
incorrectAnswers:B.C
:D
:andE.
Benignpigmentednevus(ChoiceB)
ralsoknownasacompoundnevus,isabenignproliferationofmelanocyteslocatedinboththeepidermisanddermis.Theyareverycommon.Howevertheywifinottypicallypresentasahyperpigmentedmasswithacentral
papule.Compoundnevialsoshouldnotdisplayasymmetry,borderirregularity,ormultiplecolors,asthiswouldbemoresuggestiveofmelanoma.
Melanoma(ChoiceC)islikelytobepresentwhenalesiondemonstratesasymmetryirregularappearingborders
:variablecolorationa.diametergreaterthan6mm,andrapidevofutionincharacteristics.Whilethislesionhasbeenenlarging,thisisduetoits
responsetothehormonesofpuberty,notmalignantinvasion.
Neurofibroma(ChoiceD)ischaracterizedbyafleshy,skincoloredpapuleornodulethatisabletobepushedbackintothedermisonpalpation.ItisafindingofneurofibromatosistypeIandtypeII.itisnotusuallypigmented.
Seborrheickeratosis(ChoiceE)isabenignproliferationoftheepidermis:lesionsexhibitagreasy,adherentappearance.Whileseborrheickeratosesareoftenbrown,thisisduetothekeratinproducedbytheepidermisratherthanmelaninTheyalsotendtooccur
laterinlifeandwouldbeuncommoninanadolescent.
EducationalObjective:Anaccessorynippleischaracterizedbyaroundhyperpigmentedpatchwithacentralpapule,whichtendstogrowaroundthetimeofpubertysecondarytohormonalinfluences.Itisusuallylocatedalongthemammarymigrationline
(midclavicularJine)asitisaremnantofamammaryglandthatfailedtofullymigrate.
O0 0f*
Previous Next ScoreReport LabValues Calculator Help Pause

ExamSection4:Item24ol50 NationalBoardofMedicalExaminers^
ComprehensiveBasicScienceSelf-Assessment
Y
24.A52-year-oldwomancomestotheemergencydepartmentbecauseofa2-weekhistoryofprogressiveshortnessofbreathandfatigue.Herpulseis132/min.respirationsare22/mln,andbloodpressureis100/83mmHg.Physicalexaminationshows
muffledbreathsounds.Echocardiographyshowsalargepericardialeffusion.Pericardiocentesisyieldscfoudy.serosanguineousfluid.Analysisofthefluidshowsanincreasedproteinconcentration
:numerousRBCs
:andasmallnumberofWBCs
;indicative
ofmalignancy.Metastasisfromwhichofthefollowingsitesisthemostlikelycauseofthefindingsinthispatient?
A)Bladder
B)Breast
C)Colon
D)Ovaries
E)Stomach
CorrectAnswer:B.
Breastcancerencompassesmanytypesofbreastcarcinoma,includinginvasiveductalandlobularcarcinoma,medullarycarcinoma,andinflammatorybreastcarcinoma.Themostcommontypeisinvasiveductalcarcinoma,whichtypicallypresentsasapalpable,
immobilebreastmass,whichmaydemonstratefeaturessuchasspiculatedmarginsandmicrocalcificationsormammography.Metastasesoccurwhenthebreastcarcinomainvadesbeyondtheductalstructuresofthebreast,intothelymphatics,andtodistant
anatomicalsites.Metastaticbreastcancermostcommonlyspreadstothebone
;lung
:andbrain.Itmayresultinhypercalcemiasecondarytometastaticosteolyticlesionscausingdestructionofbone.Inthiscase,breastcancerinvolvementofthemediastinumand
pericardialspace,whetherbydirectinvasionormetastasis
:hascausedapericardialeffusion.Whilelungcanceristhemostcommoncauseofmalignancy-inducedpericardialeffusion,breastcancerFSalsofrequentlyimplicated,asinthisfemalepatient.The
Treatmentsofmalignancy(eg
;chemotherapyradiation)mayalsopotentiallycausepericardialeffusions,thoughnotinthecaseofthispatientwhosepericardia!effusionwastheinitialpresentationofmalignancy.
IncorrectAnswers:A.C,D,andE.
Bladdercancer(ChoiceA)commonlymetastasizestothelung,liver,andbone.Pericardialinvolvementisuncommon.
Colon(ChoiceC)andstomach(ChoiceE)cancercommonlymetastasrzetotheliver,Eung,lymphnodes,andperitoneum.Involvementofthesesiteswouldnotbeikelytocauseapericardialeffusion.
Ovariancancer(ChoiceD)mostcommonlymetastasizestothelung,brain,liver,arrddistantlymphnodes.Italsohasthepossibilityofinvolvingtheperitoneum,causingperitonealcarcinomatosis.Metastasestothelungsmaycausepleuraleffusions,butpericardia!
involvementisuncommon.
EducationalObjective:Pericardialeffusionssecondarytomalignancymayoccurfromdirectinvasionofthetumorormetastaticspread.
© © ©
Previous Next ScoreReport LabValues Calculator Help Pause

ExamSection4:Item25ol50 NationalBoardofMetlicaiExaminers^
ComprehensiveBasicScienceSelf-Assessment
y
25.A5-month-oldboyisbroughttotheemergencydepartmentbecauseofa3-dayhistoryoffeverandseverewetcough.Histemperatureis40.5°G(1Q4.9CF),andrespirationsare65/min.Cracklesare
heardoverallJungfields.Aphotomicrographofasilver-stainedspecimenobtainedviabronchoalveolarlavageisshown.Inadditiontoalackofexpressionofhumanleukocyteantigen-DR
moleculesbylymphocytes,flowcytometryofaperipheralbloodspecimenwiltmostlikelyshowmarkedlydecreasedpopulationsofwhichofthefollowingcelttypes?
A)CD4+-Tlymphocytes
B)Dendriticcelts
C)Monocytes
D)Naturalkillercells
E)Segmentedneutrophils
CorrectAnswer:A.
Pneumocystisjiroveciiisanopportunistic,yeast-likefungalorganismthatcancausepneumoniainimmunocompromisedpatients.Chestimagingtypicallyrevealsdiffuse,bilateralinfiltratesoftenprominentlyaboutthehila.Methenaminesilver,shownhere,or
toluidinebluestains,selectivelystainthecystwallsandareusedtoconfirmthediagnosis.ThepresenceofapneumoniacausedbyP.jiroveciiindicatesthatthispatientisirrimunodeficient.Immunodeficienciescanbedividedbasedanthecelltypeaffected:T
lymphocytes,Blymphocytes,bothTandBlymphocytes,andphagocytes.Inaddition,theorganismscausingtheinfectionmayalsoprovideadueastowhattypeofcellisdeficient.Forexample,fungalinfectionsduetoCandidaspecies,P.jiroveciitand
CryptococcusspeciesarecharacteristicofT-iymphocytedeficiency.Inthiscase,theabsenceofthehumanleukocyteantigen-DR(HLA-DR)moleculesonthesurfaceoflymphocytessuggestsbarelymphocytesyndrome,aformofseverecombined
immunodeficiency(SCID).SCIDisacombinedT-lymphocyteandB-lymphocyteimmunodeficiency.FindingsincludelowTlymphocytesonflowcytometry,lackofgerminalcentersonlymphnodebiopsy,andtheabsenceofathymicshadowonchestradiograph.
HIV-positivepatientswithCD4+countsbelow200celIs/mm-arealsoatriskfordevelopingpneumoniafromPjirovecii.However,ithasbecomelesscommonfollowingthewidespreaduseofprophylaxiswithtrimethoprim-sulfamethoxazoleinthesevulnerable
patients.
IncorrectAnswers:B,C,D,andE.
Decreasedpopulationsofdendriticcells{ChoiceB)wouldleadtodecreasedantigenpresentingtoTlymphocytesandsecondarilycausealackofTlymphocyteactivation.Thisisaesslikelyexplanationofthischild
'sP.jiroveciipneumonia.
Monocytes(ChoiceC),ormacrophagesthathavenotyetleftthebloodstream,aredeficientordysfunctionalinChediak-HigasITsyndrome,chronicgranulomatousdisease,andleukocyteadhesiondeficiency.Theseimmunodeficienciesaremorelikelytopresent
withrecurrentbacteriaiinfectionsthaninfectionwithPjirovecii.
Naturalkillercells(ChoiceD)aredeficientintherarecondition,naturalkillerdeficiency(NKD).Itiscommonlyseeninchildrenwithrecurrentviralinfectionsorcancer.Rjiroveciipneumoniaisnotacommoncomplication.
Segmentedneutrophils(ChoiceE)aredeficientinextravasculartissuesinleukocyteadhesiondeficiencyduetoaninabilitytoextravasatefromtheperipheralbloodstream.LFA-1integrinisthecharacteristicdefectiveprotein,whichimpairsintravascularneutrophil
adhesion.Recurrentskinbacterialabscesseswithoutpusanddelayedseparationoftheumbilicalcordatbirtharetypicalfindings.
EducationalObjective:P.jiroveciiisanopportunistic,yeast-likefungalorganismthatcancausepneumoniainimmunocompromisedpatients,particularlythosewithT-lymphocytedysfunctionordeficiency.
r© m 0
*
ft
Previous Next ScoreReport LabValues Calculator Help Pause

ExamSection4:Item26ol50 NationalBoardofMedicalExaminers^
ComprehensiveBasicScienceSelf-Assessment
26.A29-year-oldwomancomestothephysicianbecauseofirregularmenstrualperiodssincemenarcheattheageof12years.Sheis1GOcm(5ft3in)tallandweighs86kg(190lb);BMIis34kg/m
2Sheisevaluated,andthediagnosisofpolycysticovarian
syndromeismade.Afterexplainingthediagnosis,thephysiciandiscussesbehavioralchanges,includingdietarymodificationandexerciseaspartofhertreatmentplan.Whichofthefollowingismostlikelytoresultinpatientadherencetothisplan?
A)Ascertainthepatient'seducationallevelandprovideappropriatepublications
B)Askthepatienttobringafamilymemberorfriendtothenextappointment
G}Informthepatientofthehealthconsequencesofnottreatinghercondition
D)Providefollow-upappointmentstoassessthepatient’sprogressinattaininghergoals
E)Referthepatienttoasupportgroup
CorrectAnswer:D.
Makinggoalsthatarespecific,measurable,attainable,relevant,andtime-boundcanincreasepatients'motivationtomakebehavioralchangessuchasdietarymodificationsandexercise.Providingfollow-upappointmentstotrackpatients'incrementalprogressis
thebestwaytoincreasepatients
'
intrinsicmotivationtoattainthesegoalsFurther,theformationofatrustingpartnershipbetweenthephysicianandpatientthroughregularfollow-upappointmentshasbeenshowntoimproveadherence.Duringtheseappointments,
physicianscanalsoimprovepatients
'readinesstochangehealthbehaviorwithaninterviewtechniquecalledmotivationalinterviewing.Motivationalinterviewingutilizesopen-ended,nan-judgmentalquestionstohelpthepatientexploretheirreasonsforwantingto
changeormaintainthehabit.
IncorrectAnswers:A.B,C.andE.
Educatingthepatient(ChoicesAandC)maymodestlyimprovethepatient
'sextrinsicmotivationtochangeherbehavior.However,improvingthetherapeuticallianceandtrackingprogresstowardgoalscanincreaseapatient
'sintrinsicmotivation,whichismore
likelytoresultinbehaviorchange.
Improvingapatient'ssupportsystem(ChoicesBandE)mayincreasethepatient'sadherencetotheplan.However,fosteringthephysician-patientrelationshipandtrackingprogresstowardagoalaremorelikelytoimproveadherence.
EducationalObjective:Makingregularfollow-upappointmentstotrackpatients
'
incrementalprogresstowardtheirgoalscanimprovepatients'intrinsicmotivationtoadheretotreatmentrecommendations.Thesefollow-upappointmentscanfostertrustand
collaborationwithinthephysician-patientrelationship,whichalsoimprovestreatmentadherence.
© © © ©
f*
Previous Next ScoreReport LabValues Calculator Help Pause

ExamSection4:Item27of50 NationalBoardofMedicalExaminers^
ComprehensiveBasicScienceSelf-Assessment
Y
27.Astudyisconductedtodetermineifconcomitantoraladministrationofciprofloxacinandsucralfateresultsinadecreasedplasmaciprofloxacinconcentration.Twelvesubjectsareenrolledinthestudy:sixreceiveciprofloxacinonly,andsixreceiveboth
treatments.Thehalf-lifeisdetermined
;anda1-weekwashoutperiodfollows.Eachsubjectthenreceivestheoihertreatment
;andhalf-lifeismeasuredagain.Whichofthefollowingbestdescribesthedesignofthisstudy?
A)Case-control
B)Casesenes
C)Crossover
D)Historicalcohort
E)Prospectivecohort
CorrectAnswer:C.
Inacrossoverstudy
;patientswillbeswitchedbetweentreatmentcohortsduringthestudyduration.Inthismethod
;eachpatientcanserveastheirowncontrol.Asdescribedinthestudyhere,awashoutperiodisusedtoallowforresidualeffectsoftheprior
treatmenttobeadequatelyremovedfromthebody.Becauseeachpatientactsastheirownindependentcontrol
;thisstudydesignisefficient,asitrequiresasmallernumberofpatientsthanwouldarandomizedcontrolledtrialtoachievethesamepower.
IncorrectAnswers:A,B.D,andE.
Acase-controfstudy(ChoiceA)investigatesanassociationbetweenanexposureandanoutcome.Inthisstudydesign
:agroupofpatientswiththedisease(cases)areidentified.Agroupofpatientswithoutthedisease(controls)arematchedonbaseline
characteristicstothecases.Exposuredataforthetwogroupsiscollected
;andthesedataarecomparedtodetermineassociationwiththeoutcome(disease)inquestion.Anoddsratiomaybecalculatedtocompareexposuresbetweengroups.
Acaseseries(ChoiceB)isadescriptivestudydesigninwhichanumberofconsecutiveornonconsecutivecasesofadiseaseand/ortreatmentaredescribedindetail,withinformationaboutexposure
:demographics,andcomorbidities.Caseseriesdonotimplya
cause-and-effecfrelationship.TheydonottestahypothesisnoraretheyrandomizedTheyareusefulincharacterizingthenaturalhistoryofadiseaseorresponsetotreatment.Theyarealsousefulindescribingrarediseases,asthesmallpopulationsizemaynot
permitconductionoflargercohortorrandomizedtrialswithsufficientpower.
Ahistoricalcohortstudy(ChoiceD)(retrospectivecohortstudy)identifiesagroupofpatientsandlooksbackovertimetoidentitywhetheranexposureisassociatedwithanoutcomeofinterest.Inthisretrospectivedesign,thehypothesisorquestionisdesigned
afterthestudytimeperiodhaspassed.Forexample,anorthopedicsurgeonseekstoidentifytheriskfactorsassociatedwithprostheticjointinfection.Inordertoaccomplishthis,theinvestigatorsreviewtherecordsoftotaljointarthroplastypatientsfromtheprevious
10yearsandidentifydiabetesmellitusandobesityasriskfactors.
Aprospectivecohortstudy(ChoiceE)identifiesagroupofpatientsandfollowsthemovertimetoidentifywhetheranexposureisassociatedwithanoutcomeofinterest.Inaprospectivedesign,thehypothesisandanalysisprotocolsareestablishedpriortothestart
ofthestudyperiod.Anexampleofaprospectivecohortstudywouldbefollowingagroupof1000smokersforatimeperiodof10yearsandidentifyingtheproportionofthesepatientswhodeveloppancreaticcancertoidentifytheriskofpancreaticcancerin
smokersascomparedtoacontrolgroupofnonsmokers.
EducationalObjective:AcrossoverstudyisanefficientmethodologyaseachpatientundergoesmultipletreatmentstrategiesandservesastheirowncontrolThisstudydesignoftenusesawashoutperiodbetweentreatmentarmsinordertoavoidanoverlapof
treatmenteffects.
0
Previous Next ScoreReport LabValues Calculator Help Pause

ExamSection4:Item28ol50 NationalBoardofMedicalExaminers^
ComprehensiveBasicScienceSelf-Assessment
Y
28.A32-year-oldwomanisfoundtohavepanicdisorderwithagoraphobia.Adrugisprescribedthatactivatesbenzodiazepinebindingsitesonthey-aminobutyricacidA
(GABAJreceptor.Thisdrugismostlikelywhichofthefollowing?
A)Alprazolam
B)Buspirone
C)Flumazenil
D)Hydroxyzine
E)Rameiteon
CorrectAnswer:A.
Alprazolamisabenzodiazepinemedicationthatbindstobenzodiazepinesitesonthey-aminobutyricacidA
(GABAa
)receptor.BenzodiazepinesincreasethefrequencyoftheopeningoftheionotropicGABAA
receptorincreasingtheinfluxofchlorideionsand
decreasingthelikelihoodofneuronalactionpotentialsbyhyperpolarizingtheneuron.Anxietydisordersarerelatedtoincreasedneuronalactivityinthelimbicsystembrainareassuchastheamygdala
;whichmediatesemotionalresponsessuchasfear.
BenzodiazepinemedicationsdecreasefearresponsesbyincreasingGABAsignalingintheamygdala.Panicdisorderischaracterizedbyrecurrentpanicattacksthatareunexpectedandassociatedwithworryaboutfuturepanicattacksoravoidanceofpanicattack
triggers.Agoraphobia,orthefearofleavingone'shome,frequentlyaccompaniespanicdisorder.Treatmenttypicallyincludeslong-termselectiveserotoninreuptakeinhibitorsandshort-termbenzodiazepines.Long-termbenzodiazepineuseisassociatedwith
addiction.
IncorrectAnswers:B,C
:D
;andE.
Buspirone(ChoiceB)isaserotoninmodulatorthatdoesnotinteractwithGABAAreceptors.Buspironeistypicallyutilizedtotreatgeneralizedanxietydisorderratherthanpanicdisorderwithagoraphobia
Flumazenil(ChoiceC)isacompetitiveantagonistatthebenzodiazepinesiteontheGABA
^
receptor.Flumazenilisutilizedtoreverseaccidentalbenzodiazepineoverdoseinpatientswhoarenotchronicallydependentandbenzodiazepinesedationaftersurgical
proceduresinselectpatients.Itsuseislimitedasitcanprecipitateseizures.
Hydroxyzine(ChoiceD)isanantihistaminemedicationwithcentralnervoussystemactivity.Hydroxyzineisusedoff-labelforacuteanxietybutdoesnotbindtheGABAAreceptor
Rameiteon(ChoiceE)isaselectiveagonistofthemefatoninreceptors.Rameiteonisusedforsteep-onsetinsomnia,notanxietyorpanicattacks.
EducationalObjective:BenzodiazepinesbindtoandactivatetheGABAAreceptori
;
ntheamygdala,decreasingneuronalactionpotentialsandthereforeattenuatingthefearresponse.Benzodiazepinesareutilizedinconjunctionwithselectiveserotoninreuptake
inhibitorstotreatpanicdisorder.
00 & £ 0
f*
Previous Next ScoreReport LabValues Calculator Help Pause

ExamSection4:Item29of50 NationalBoardofMedicalExaminers^
ComprehensiveBasicScienceSelf-Assessment
Y
29.A1-year-oldAfricanAmericangirlisbroughttothephysicianbecauseofaslowgrowthrate.Atbirthshewasatthe25thpercentileforlengthand30thpercentileforweight.Shehasbeenexclusivelybreast-fedsincebirth.Herlengthandweightarenowat
the5thpercentile.Physicalexaminationshowsbowedlegs
;swellingofthewrists,andabulgingofthecostochondraljunctions.Herserumcalciumconcentrationiswithinthereferencerange
:andherserumphosphorusconcentrationisdecreased.Whichof
thefollowingsetsofserumfindingsismostlikelyinthispatient?
ParathyroidHormone 25-HydroxyvitaminD
A) T T
B) T i
C) Normal I
D) normall
E) I T
F) 1 i
CorrectAnswer:B
VitaminDplaysaroteinserumcalciumandphosphatehomeostasisbypromotingintestinalabsorptionofcalciumandphosphate,increasingbonemineralizationatlowlevels,andboneresorptionathigherlevels.Parathyroidhormone(PTH}alsohasarolein
calciumandphosphateregulationbystimulatingosteoclasticbonereabsorptionanddistalconvolutedtubularcalciumreabsorptionandphosphateexcretioninthekidney.VitaminDdeficiencycanbecausedbymalabsorptionintheintestinaltract,malnutritionor
insufficientdietaryintake,anddecreasedsunexposure.DecreasedlevelsofvitaminDresultindecreasedintestinalcalciumabsorptionandhypocalcemia,sensedbytheparathyroidglandviacalcium-sensingreceptors,leadingtoanincreaseinsecretionofPTH
tonormalizeserumcalciumlevels.IncreasedPTHresultsinincreasedserumcalciumfromincreasedosteoclastactivity,intestinalabsorption,andrenaltubularabsorption..Serumphosphorusconcentrationisdecreasedsecondarytoincreasedexcretionbythe
renaltubules.Thisleadstoricketsinchildren,characterizedbybonepainanddeformitiessuchasbowingofthelegs,bulgingofcostochondraljunctions,andgrowthdelay.BecauseoftheriskforvitaminDdeficiency,breast-fedinfantsarerecommendedtohave
oralvitaminDsupplementation.
IncorrectAnswers:A,C,D,E,andF.
VitaminDdeficiencyandricketsarecharacterizedbylowserumconcentrationofvitaminD,notnormalorincreased(ChoicesA.D,andE).
DecreasedlevelsofvitaminDresultindecreasedintestinalcalciumabsorptionandhypocalcemia,sensedbytheparathyroidglandviacaicium-sensingreceptors,leadingtoanincreaseinthesecretionofPTHtonormalizeserumcalciumconcentration,nota
decreaseinPTHornormalPTHconcentration(ChoicesC,D,E,andF).
EducationalObjective:VitaminDdeficiencyinbreast-fedinfantscanleadtorickets,characterizedbybonepainanddeformitiessuchasbowingofthelegs,bulgingofcostochondraljunctions,andgrowthdelay.DecreasedconcentrationofvitaminDresultin
decreasedintestinalcalciumabsorptionandhypocalcemia,sensedbytheparathyroidglandviacalcium-sensingreceptors,leadingtoanincreaseinthesecretionofPTHtonormalizeserumcalciumconcentration.
© © ©
Previous Next ScoreReport LabValues Calculator Help Pause

ExamSection4:Item30ol50 NationalBoardofMedicalExaminers^
ComprehensiveBasicScienceSelf-Assessment
Y
30.Apremenopausal49-year-olc3womanasksherphysicianaboutherriskforosteoporosis.Hermotherhadosteoporosisanddisablingbonefracturesaftermenopause.Thepatientworksasagardenerforalandscapeserviceandplaystennis3timesaweek.
Shedoesnotsmoke,drinks1glassofwineaweek,andtakesnomedications.Herweightis55kg(121lb).Whichofthefollowingaddifionalinformationisneededtoevaluateherriskforosteoporosis?
A)Ageatfirstsexualintercourse
B)Dietaryhistory
C)Exposuretoionizingradiation
D}Exposuretoorganophosphate
E)Familyhistoryofrheumaticdiseases
F)Socioeconomichistory
CorrectAnswer:B.
Therearenumerousfactorsthatcontributetothedevelopmentofosteoporosis,whichisdefinedasadecreaseinbonemineraldensity2.5standarddeviationsbelowthenormalmean.Ittypicallyoccursinthefifththroughseventhdecadesoflifeandis
approximatelyfourtimesmorecommoninfemalesthaninmales.ItisasomorecommoninindividualsofAsianornorthernEuropeandescent,inadditiontogenderandgeneticfactors,therearemanyenvironmentalfactorsthatalsocontribute.Long-termtherapy
withcorticosteroids
;anticonvulsants,andprotonpumpinhibitorshavebeenshowntoincreasetheriskofosteoporosis.Sedentarylifestyleorimmobilizationhasalsobeenimplicatedinthedevelopmentofosteoporosis.Additionalenvironmentalfactorsinclude
malnutritionormalabsorptionsyndromes(specificallythosethatleadtolowcalciumandvitaminDlevels)
:lowbodyweightchronicalcoholuse,cigarettesmoking,anddecreasedestrogenortestosteroneproduction.Thepatientsdietaryhistoryshouldbe
investigatedtounderstandanyriskfactorsshemayhaveformalnutritionormalabsorption,whichwouldplaceheratagreaterriskfordeficienciesthatpredisposetoosteoporosis.
IncorrectAnswers:A.C,D.E,andF.
Ageoffirstsexualintercourse(ChoiceA)doesnotcontributetoosteoporosis.Anoveralllongerdurationofexposuretoestrogenandtestosteronecancontributetoincreasedbonemineraldensityanddecreasetheriskofosteoporosisasthesesexhormoneslead
toincreasedboneproduction.
Exposuretoionizingradiation(ChoiceC)doesnotincreasetheriskofosteoporosis.Exposuretosignificantdosesofionizingradiationdoesincreasetheriskofcancer,particularlyinthethyroid,bonemarrow,andcolor
Exposuretoorganophosphate(ChoiceD)leadstoorganophosphatepoisoning,whichiscausedbyincreasedlevelsofacetylcholineduetotheinhibitionofacetylcholinesterase.Symptomsoforganophosphatepoisoningincludediarrhea,vomiting,urination,
sweating,salivation,bronchospasm,bradycardia,fasciculations,lethargy,andpossiblecoma.
Familyhistoryofrheumaticdiseases(ChoiceE)isariskfactorforautoimmunediseases,butnotnecessarilyosteoporosis.Theincreasedinflammatorystateassociatedwithrheumaticdiseasesmayfeadtodecreasedbonemineraldensityrelatedtointerieukin-1
activationofosteoclasts,howeverafamilyhistoryofrheumaticdiseasedoesnotdirectlyincreasetheriskofosteoporosis.
Socioeconomichistory(ChoiceF)mayindirectlyleadtoincreasedriskofosteoporosis.Forexample,personsoflowsocioeconomicstatusmayhavedecreasedaccesstoappropriatenutritionthatmayleadtodecreasedlevelsofcalciumandvitaminD.However,
thisisnotadirectcauseofosteoporosis.
EducationalObjective:Osteoporosisisaconditionofsignificantlydecreasedbonemineraldensity,whichincreasestheriskforfractures,commonlyofthedistalradius,hip,andvertebrae.Therearenumerousriskfactorsforthisdisease.Decreasedintakeor
absorptionofvitaminDandcalciumsignificantlyincreasestheriskforlowbonemineraidensity.
© © ©
A
Previous Next ScoreReport LabValues Calculator Help Pause

ExamSection4:Item31ol50 NationalBoardofMetlicaiExaminers^
ComprehensiveBasicScienceSelf-Assessment
y
31.A5-year-oldboyisbroughttothephysicianbyhisparentsbecauseoflethargyfor1week.Hehasahistoryofoccasionalcoldsandearinfections.Physicalexaminationshowsmildpittingedemaofthelowerextremities.Auscultationofthechestand
palpationoftheabdomenshownoabnormalities.Laboratorystudiesshow:
Hemoglobin
Hematocrit
Leukocytecount
Serumcholesterol
Urine
Blood
Protein
15g/dl
45%
7300/mm
3
330mg/dL
absent
3+
Ifakidneybiopsyspecimenwereobtained
:lightmicroscopicexaminationwouldmostlikelyshowwhichofthefollowing?
A)Crescentswithintheglomerularspace
B)Focalsegmentalglomerulosclerosis
C)Neutrophilicinfiltrationoftheglomerulartuft
D}Thickenedglomerularbasementmembranes
E)Nopathologicabnormality
CorrectAnswer:E
Nephroticsyndromeischaracterizedbyedemasecondarytoexcessiveproteinuriaresultinginhypoalbuminemiaanddiminishedintravascularoncoticpressure.Thisoccursbecauseoflossofnormalsizeandchargefiltrationoftheglomerularcapillaryand
Bowmancapsuleinterface.Etiologiesofnephroticsyndromeincludeminimalchangedisease,membranousnephropathy,andfocalsegmentalglomerulosclerosis.Minimalchangediseaseisthemostcommoncauseofnephroticsyndromeinchildrenandcanbe
idiopathicortriggeredbyarecentinfectionorimmunestimulus.Patientsclassicallypresentwithdependentedemainthebuttocks,lowerback,andlegs,foamyordark-coloredurine,hypoalbuminemia,andproteinuria.Minimalchangediseaseappearsnormalon
lightmicroscopy.
IncorrectAnswers:A,B,C,andD.
Crescentswithintheglomerularspace(ChoiceA)istypicalofcrescenticglomerulonephritis,whichischaracterizedbyacrescentofeosinophilicfibrinandplasmaproteinsonbiopsyoftheglomerulus,adjacenttoglomerularparietalcellsmonocytes,and
macrophages.Itisgenerallyseeninnephriticsyndrome,whichischaracterizedbyhematuriaandredbloodcellcastsinurine,oliguria,andhypertension.Itcanbeassociatedwithproteinuria,buttoalesserextentthannephroticsyndrome.
Focalsegmentalglomerulosclerosis(FSGS)(ChoiceB)isacauseofnephroticsyndromethatcanresultinproteinuriaandpittingedema.FSGSismostcommonlyassociatedwithsicklecelldisease,opioidabuse,andHIVinfection,andischaracterizedby
segmentalsclerosisoftheglomeruli,visibleonlightmicroscopy.
Neutrophilicinfiltrationoftheglomerulartuft(ChoiceC)isseeninpoststreptococcalglomerulonephritis,whichisanephriticsyndromecharacterizedbydarkurine,hypertension,andmildproteinuriafollowinganinfectionwithstreptococci,groupA.
Diabeticglomerulosclerosiscancausethickenedglomerularbasementmembranes(ChoiceD).Itoccursfollowingnonenzymaticglycosylationoftheglomerularbasementmembraneandefferentarterioles,characteristicallypresentingasKimmelstiel-Wilson
lesionsonlightmicroscopy.Diabeticnephropathyprogressesovertime,initiallybeginningasmicroalbuminuria,whichleadstomacroalbuminuriaandend-stagekidneydisease.
EducationalObjective:Minimalchangediseaseisthemostcommoncauseofnephroticsyndromeinchildrenandcanbeidiopathicortriggeredbyarecentinfectionorimmunestimulus.Patientsclassicallypresentwithdependentedemainthebuttocks,lower
back,andlegs,foamyordark-coloredurine,hypoalbuminemia,andproteinuria.Minimalchangediseaseappearsnormalonlightmicroscopy.
r© m
<e
Previous Next ScoreReport LabValues Calculator Help Pause

ExamSection4:Item32ol50 NationalBoardofMedicalExaminers^
ComprehensiveBasicScienceSelf-Assessment
Y
32Feedbackinhibitionoftheactivityoftheenzymethatcatalyzesthisreactioniscriticalinthecontrolofthedenovosynthesisofwhichofthefollowing?
®
-G-CH?
H
A}Complexcarbohydrates
B)Folates
C)Pentosephosphates
D)Purinenucleotides
E)Pyrimidinenucleotides
-©-©H
H\H
OH OH
Glutamine
®:
^
Glutamate
SPFj
T
®-0-CH?A3
NH,
H
H\H H
OH OH
CorrectAnswerD.
Thischemicalreactiondemonstratesdenovosynthesisofpurinenucleotides.PurineandpyrimidinenucleotidesarethebuildingblocksofDNA.PurineandpyrimidinenucleotidescanbesynthesizeddenovoorsalvagedfromintermediatebyproductsofDNAor
RNAdegradation,nthedenovosynthesisofbothpurinesandpyrimidines,ribose5-phosphate(providedbythepentosephosphatepathway)isinitiallyconvertedto5-phospho-a-D-ribosyl1-pyrophosphate(PRPP)byPRPPsynthetase.Inthefirststepthat
commitsthenucleotideprecursortothepurinesynthesispathway,PRPPisthenconvertedtoS
'
-phosphoribosylamine(PRA)byamidophosphoribosyltransferase(alsocalledPRPPamidatransfera.se).Inthisreaction,glutamineservesasanitrogendonorandis
convertedtoglutamate.PRAisultimatelyconvertedtoinosinemonophosphate(IMP),whichcanthenbeconvertedtoadenineorguaninemonophosphate(AMPorGMP).AMPandGMPcanthenformphosphodiesterbondswithothernucleotidestoformDNA.
IMP.AMP,andGMPinhibitPRPPamidotransferaseactivity.Immunemodulatingandcancermedicationsinhibitenzymesinthepurineandpyrimidinesynthesispathways.
IncorrectAnswers:A,B
:C,andE.
Complexcarbohydrates(ChoiceA)areoligosaccharidesinwhichmonosaccharidesarelinkedbygtycosidicbondsratherthanthephosphodiesterbondsofnucleotidechains.Complexcarbohydratesynthesisisnotregulatedbyfeedbackinhibition.
Thoughfolates(ChoiceB)areinvolvedintheenzymaticreactionsthatsynthesizepyrimidineandpurinebases,humanscannotsynthesizefolates.Humansinsteadrelyondietarysourcesoffolates.
Thepentosephosphatepathway(ChoiceC)ultimatelyconvertsglucose-6-phosphatetopentoses,NADPH,andribose-5-phosphate.Ribose-5-phosphateisanucleotideprecursorthatisusedtosynthesizepurinesandpyrimidines.However,glutamineisnot
involvedinthesynthesisofpentoses.Additionally,pentosesdonotcontainnitrogenasthepurineprecursorinthisreactiondoes.
Inthesynthesisofpyrimidinenucleotides(ChoiceE),PRPP(thereactant)isusedonlytodonatearibosegrouptoapreformedpyrimidineringtoformanucleotide.Inthepurinesynthesisreactionabove,atomsandmoleculesareincrementallyaddedtoPRPP
(eg,nitrogenfromglutamine),whichultimatelyresultsinnucleotideformation.
EducationalObjective:ThefirstcommittedstepofthedenovosynthesisofpurinenucleotidesinvolvestheenzymePRPPamidotransferase.PRPPamidotransferaseconvertsPRPPtoamoleculecalledPRAusingglutamineasanitrogendonorandthe
nucleotidesIMPAMRandGMPareultimatelyproduced.ThesenucleotidestheninhibitfurtherPRPPamidotransferaseactivity.
O0
It
0f*
Previous Next ScoreReport LabValues Calculator Help Pause

ExamSection4:Item33of50 NationalBoardofMedicalExaminers^
ComprehensiveBasicScienceSelf-Assessment
33.A72-year-oldwomanwithcoronaryarterydiseasecomestothephysicianbecauseofa2-monthhistoryofprogressiveanginasymptomswithexertion.Fivemonthsago,thepatientunderwentstentplacementforsignificantstenosesoftheproximal
anteriorinterventricular(leftanteriordescending)andrightcoronaryarteries.Sheremainedsymptom-freefor3months.Herpulseis76/minandregular,andbloodpressureis135/85mmHg.AnEGGatrestshowsnoabnormalities;anexercisestresstest
showsST-segmentchangesinleadsII
:IIIandaVF.Whichofthefollowingisthemostlikelycauseofthispatient'srecurrentanginasymptoms?
A)Aneurysmoftherightcoronaryartery
6)Oissectionoftherightcoronaryartery
C)Neointimaformationintherightcoronarystent
D)Thrombosisoftherightcoronarystent
E)Vasospasminthestentedrightcoronaryartery
CorrectAnswer:C.
Neointimaformationintherightcoronarystentisthemostlikelycauseofthepatient'srecurrentsymptoms.ThelocationofST-segmentchangesintheexercisestresstestsuggestsimpairmentofbloodflowtotherightcoronaryarteryterritory.Neointimaformationis
acommoncauseofvascularrestenosisafterpercutaneouscoronaryintervention(PCI).Itisprimarilycomposedofvascularsmoothmuscleceilsinarichextracellularmatrixthathavemigratedintothestenttoformalayerthatsharessomefeatureswiththetunica
intima.Migrationispromotedbymacrophagesandinflammatorycellsinthesettingofaforeignbodyreaction.Theresultingneointimaformationdecreasesthelumendiametermaycauseinadequatemyocardialperfusionduringexertionwithrecurrenceofangina.
Drug-elutingstents(DES)attempttoreducetheriskofneointimaformationbythecontinuous,slowreleaseofdrugsthatinhibitceilproliferation.
IncorrectAnswers:A.B,D,andE.
Aneurysmoftherightcoronaryartery(ChoiceA)is'esslikelythanneointimalhyperplasia.AnaneurysmisadilationofabloodvesselinvolvingallthreelaysofthevesselwallCoronaryarteryaneurysmsareassociatedwithatherosclerosisandcertainformsof
vasculitis,suchasmucocutaneouslymphnodesyndrome(Kawasakidisease).Theyareusuallyasymptomatic,butmaybesusceptibletorupture,dissection,orthrombusformation.
Dissectionoftherightcoronaryartery(ChoiceB)isanuncommonoccurrence,buttypicallypresentswithsymptomsofacutecoronarysyndrome,dysrhythmias,orsuddencardiacdeath.CoronaryarterydissectionisapotentialcomplicationofPCIbutwouldpresent
intheimmediateperiproceduralsettingratherthanfivemonthslater
Thrombosisoftherightcoronarystent(ChoiceD)isapotentialcomplicationofPCI,andtheindicationforuseofantiplateletagentsfollowingstentplacement,n-stentthrombosisislesscommonthanneointimaformation.
Vasospasminthestentedrightcoronaryartery(ChoiceE)isanotherpotentialcomplicationofstentplacement.Vasospasmsymptomsarenottypicallyreservedtoexertionandmayoccurwhiiethepatientisatrestormaywakethepatientfromsleep.
tducationaObjective:Neointimaformationisacommoncomplicationofcoronarystentingandthemastcommoncauseofrecurrentangina.DESattempttoreducetheriskbyreleasingantiproliferativeagents.
0
Previous Next ScoreReport LabValues Calculator Help Pause

ExamSection4:Item34ol50 NationalBoardofMedicalExaminers^
ComprehensiveBasicScienceSelf-Assessment
Y
34.A55-year-oldmancomestothephysicianbecauseofa3-weekhistoryofnightsweats,weakness,fatigue,lossofappetite
;andlumpsi
'
nhisneck.Heownsaprintingbusinessandworkswithpaintsandsolvents.Physicalexaminationshowsedematous
;
tenderaxillaryandinguinailymphnodes.Hisleukocytecountis45
;570/mm
3(30%large
:pleomorphicblastswithAuerrods).Occupationalexposuretowhichofthefollowingmostlikelycontributedtothedevelopmentofthispatient'sdisease?
A)Benzene
B)Cadmium
C)Ghlordane
D)Naphthylamine
E)Vinylchloride
CorrectAnswer:A.
Benzeneisknowntoincreasetheriskfordevelopingacutemyelogenousleukemia(AML)
;which;sthemostlikelydiagnosisinthispatient.Theriskisthoughttobedose-dependentandindividualsinindustriessuchasplasticmanufacturingandprintingareata
higherriskofexposure.BenzeneismetabolizedprimarilyinthelungandliverviatheactionofcytochromeP450(CYP)enzymestoformbenzeneoxide
:whichspontaneouslyconvertstophenolandisfollowedbyconversiontohydroquinonebyCYP2E1
Hydroquinoneisconvertedinthebonemarrowbymyeloperoxidasetoreactivebenzoquinones,whicharetoxictohematopoieticcells.Commonmutationsthatarisefrombenzeneexposureinclude5qdeletionsandt(B;21)translocations.AMLisconsideredan
oncologicemergency.Treatmentwithcytarabineandidarubicinisstartedimmediatelyfollowingdiagnosisunlessthepatientisfoundtohaveacutepromyelocyticleukemia(APML)inwhichcasetreatmentiswithall-trans-retinoicacidtoinducedifferentiationof
malignantblasts.AuerrodssuggestadiagnosisofAPMLbutcanbeseeninotherformsofAML.Cytogeneticstudiesdeterminethelikelihoodthatthepatientwillrequiresubsequentstemcelltransplant,buttherapyisrarelydelayedwhileawaitingthesestudies.
IncorrectAnswers:B,C
:D
;andE.
Cadmiumexposure(Choice6)isassociatedwithlung
:renal,andprostatecancer.ItdoesnotpredisposetoAML.Occupationalexposureiscommoninpatientswhoworkmakingbatteries,plastics,andpigments.
Chlordaneexposure(ChoiceC)isassociatedwithcentralnervoussystemdiseaseincludingseizures,headaches,andvisionchanges.Wheningested,nauseaandvomitingarecommon.Itwaspreviouslyusedasapesticideandisnotfoundnaturallyinthe
environment,itisnotassociatedwithanymalignancy.
Naphthylamineexposure(Choice0)canoccurinthedyeindustry.OccupationalexposureisassociatedwithbladdercancerbutnotAML.
Vinylchlorideexposure(Choicet)predisposestohepaticangiosarcomaandisoftenseeninpatientswhoworkwithpolyvinylchloridepipes.
EducationalObjective:AMLisamalignancyofthemyeloidstemcellandisassociatedwithoccupationalexposuretobenzene.Industrialexposuresoftenoccurinthedye,printing,orplasticindustry.
00#
Previous Next ScoreReport LabValues Calculator Help Pause

ExamSection4:Item35ol50 NationalBoardofMedicalExaminers^
ComprehensiveBasicScienceSelf-Assessment
Y
35.Amatchedcase-controlstudyisconductedtoassesstheassociationbetweenthefrequentuseoftanningbedsduringteenageyearsandthedevelopmentofmelanoma.Agroupofwomenwithmelanomaarematchedwithagroupofcancer-freewomenof
thesameageandethnicity.Anoddsratio(OR)of1.4isobtained.Whichofthefollowing95%confidenceintervalsfortheORisthemostprecisestatisticallysignificantinterval?
A)-0.9to0.9
B)0.9to1.7
C)0.93to1.01
D)1.01to1.73
E)1.2to1.7
CorrectAnswer:E
Aconfidenceintervalproposesarangeofvalueswithinwhichatruevalueislikelytofall.Formally,a95%confidenceintervalistherangeofvalueswithinwhichthecalculatedstatisticwouldfallbetweenin95%ofcasesifthesampleandcalculationwererepeated,
reflectingareasonablemarginoferror(5%).Inanormaldistribution,the95%confidenceintervalextendsfromtwostandarddeviationsbelowthemeantotwostandarddeviationsabovethemean.Thus,anarrowerconfidenceintervalindicatesasmallerstandard
deviation,andincreasedprecisionofthedata.Inaddition,whenevaluatingoddsratiosorrelativerisk,avalueof1.0indicatesthatthetwogroupsaresimilarwithregardtotheoutcomeofinterest.Thus,aconfidenceintervalthatdoesnotinclude1.0(aboveor
below),wouldindicateastatisticallysignificantdifferencebetweenthetwogroups.Oftheprovidedconfidenceintervals,1.2to1.7isthenarrowest,ormostprecise,intervalthatdoesnotinclude1.0,indicatingstatisticalsignificance.
IncorrectAnswers:A.B,C.andD.
-0.9to0.9(ChoiceA)isnotavalid:theconfidenceintervalforanoddsratiocannotbeanegativenumber
0.9to1.7(ChoiceB)isaconfidenceintervalthatincludes1.0.Thiswouldindicatethedifferencebetweenthetwogroupsisnotstatisticallysignificant.
0.93to1.01(ChoiceC)describesanarrowconfidenceinterval,howevertheconfidenceintervalincludes1.0andthereforeindicatesthedifferencebetweenthegroupsisnotstatisticallysignificant.
1.01to1.73(ChoiceD)describesarelativelywideconfidenceinterval,indicatingdecreasedprecision,itdoesindicateastatisticallysignificantdifferencebetweenthegroups,asitdoesnotcontain1.0.
EducationalObjective:Confidenceintervalsareameasureoftheprecisionofastatisticalestimate.Smallerconfidenceintervalsindicategreaterprecision.Wheninterpretinganoddsratioorarelativerisk,aconfidenceintervalthatdoesnotinclude1.0indicatesa
statisticallysignificantdifferencebetweenthegroups.
00# 0f*
Previous Next ScoreReport LabValues Calculator Help Pause

ExamSection4:Item36ol50 NationalBoardofMedicalExaminers^
ComprehensiveBasicScienceSelf-Assessment
36.A6-week-oldmaleinfanthasapersistentdischargefromaswollenumbilicalstump.Ifthedischargecontainsintestinalfluid,towhichofthefollowingportionsofthegastrointestinaltractisthefistulamostlikelyconnected?
A)Cecum
B)Duodenum
C)Ileum
D)Jejunum
E)Sigmoidcolon
CorrectAnswer:C.
Meckeldiverticulumresultsfrompersistenceofthevitellineduct.Insomecases,Meckeldiverticulumisassociatedwithumbilicalanomalies,includingformationofafistuloustractanddischargeofintestinalfluidincaseswhentheduddidnotcompletelyobliterate
duringdevelopment.IVteckeldiverticulumiscommonlylocatedintheileum
:twofeetproximaltotheileocecalvalve.Thediverticulumisapproximatelytwoinchesinlength.Inchildren,itmayserveasapotentiallead-pointforintussusception.Atechnetium99m
pertechnetatescandemonstratesuptakeintherightlowerquadrantcorrespondingwithedopicgastricmucosawithintheMeckeldiverticulum.Treatmentofsymptomaticcasesisthroughsurgicalresection.
IncorrectAnswers:A,B.DaandE.
Thececum(ChoiceA)isthesiteoftheileocecalvalveandtheappendix.Itislocatedintherightlowerquadrantoftheabdomen.TheileocecalvalveisausefullandmarkforidentifyingMeckeldiverticulum,whichoccursintheadjacentileum.
Theduodenum(ChoiceB)isthefirstsegmentofthesmallintestineandisthesiteofthepyloricvalveandthesphincterofOddiMeckeldiverticulumoccursintheileum
:ratherthantheduodenum.
Thejejunum(ChoiceD)maybeaffectedincasesofpediatricinidgutmatrotationorvolvulus,itcanbedistinguishedfromothersegmentsofthesmallintestinebyitsprominentplicaecirculares,largercaliberthickermuscularwalls,andlongvasareda.Mecke:
diverticulumoccursintheileum
;ratherthanthejejunum.
Thesigmoidcolon(ChoiceE)isadistalsegmentofthelargeintestineandisfoundproximaltotheredum.Itisacommonsiteofpolyps
:coloncarcinoma,andnon-congenitaldiverticulosis,butisnotthesiteofMeckeldiverticulum.
EducationalObjective:Meckeldiverticulumresultsfrompersistenceofthevitellinedud.Insomecases
:Meckeldiverticulumisassociatedwithumbilicalanomalies,includingformationofafistuloustractanddischargeofintestinalfluidwhentheductpersists.
00# 00
Previous Next ScoreReport LabValues Calculator Help Pause

ExamSection4:Item37of50 NationalBoardofMedicalExaminers^
ComprehensiveBasicScienceSelf-Assessment
Y
37.A32-year-oldmancomestothephysicianbecauseofa6-monthhistoryofprogressivedroopingoftheeyelidsandfacia!weakness.Muscleweaknessisexacerbatedbyuseandrelievedbyrestifaneoplasmisthecauseofthesefindings,whichofthe
followingisthemostlikelylocationinthispatient?
A)Femur
B)Liver
C)Lymphnodes
D)Meninges
E)Thymus
CorrectAnswer:E
MyastheniagravisIsassociatedwiththymomas.MyastheniagravisIsanautoimmunedisorderofneuromusculartransmissionthathasabimodalageofonset,peakinginbothyoungadults(teensto20s)andolderaduEts(50sto60s}.Patientswithmyasthenia
gravispossessantibodiesthatblockordestroynicotinicacetylchotinereceptors(nAChRs),whichinterfereswithneuromuscularjunctionsignaling.Manypatientswithmyastheniagravisdemonstratethymichyperplasiaorathymoma.Inmyastheniagravis
;the
thymus,whichisthesiteofT-lymphocytematuration,maydemonstrateabnormalitiesinthepresentationofthenAChRasanantigentohelperTlymphocytes.ThisabnormalantigenpresentationispostulatedtoresultintheautoimmuneattacktargetingnAChRson
skeletalmuscleceEJs.Patientspresentwithprogressivelyworseningskeletalmuscleweaknessatrestandfatigabilityonactivationinfocalorgeneralizedmusclegroups.Theocular,bulbar,andneckmusclesarecommonlyaffected.Mostcancernmgly,respiratory
muscleweaknesscanteadtorespiratoryinsufficiency,andinanacuteflare,thesepatientsmayrequiremechanicalventiation.Managementincludessymptomatictreatmentssuchasacetylcholinesteraseinhibitorsalongwithimmune-moduEatingtherapies,in
patientswithorwithoutathymoma,completethymectomymaybecurative.
incorrectAnswers:A.B,C
;and0.
Neoplasmsofthefemur,liver,lymphnodes,andmeninges(ChoicesA,B,C,andD)havenotbeenimplicatedinthepathogenesisofmyastheniagravisThoughTlymphocytesareinvolvedintheautoimmuneetiologyofmyastheniagravis,thymomasratherthan
lymphomascontributetoabnormalantigenpresentation.
EducationalObjective:Inpatientswithmyastheniagravis,athymomapromotestheautoimmuneattackofnAChRsonskeletalmusclecells,leadingtofocalorgeneralizedmuscleweaknessandfatigability.Completethymectomymaybecurative.
00# 0f*
Previous Next ScoreReport LabValues Calculator Help Pause

ExamSection4:Item33ol50 NationalBoardofMedicalExaminers^
ComprehensiveBasicScienceSelf-Assessment
Y
38.Whencontrollymphocytesaretreatedwithcorticosteroids,amajorityofcellsshrinkinsizeanddevelopperipheralchromatincondensation;cytoplasmicorganellesareintact.CNAisolatedfromthecontrollymphocytes,electrophoresedonanagarosegef
andstainedwithethidiumbromide,showsaladderofregularlyspacedbands.WhenlymphocytesfromthesameculturearetransfectedsothattheyoverexpressGeneX,whichencodesanormalproteinproduct,thecellscontinuetoproliferateanddonot
undergoanyofthemorphologicchangesseeninthecontrolcells.GeneXismostlikelytoencodewhichofthefollowing?
A)BCL2protein
B)ERB62protein
C)p53protein
D)Platelet-derivedgrowthfactor
E)Rbprotein
CorrectAnswer:A.
BCL2isthemostlikelyproteinencodedbyGeneX.BCL2isanormalcellularproteinthatplaysaroleintheregulationofapoptosis,itisfoundintheoutermitochondrialmembrane,nuclearenvelope,andendoplasmicreticulum.BCL2functionsbypreventingthe
oligomerizationofBAX/BAKintheoutermitochondrialmembrane,whichwouldnormallycausereleaseofcytochromecandactivationofcaspasesinthecytoplasmviainteractionwithapoptoticproteaseactivatingfactors.Therearetwoprimarymorphologicstages
ofapoptosisthatcanberecognizedatthecellularlevel.Inthefirststage,nudeappearwrinkledanddevelopperipheralchromatincondensationasaresultoffragmentationofhigh-molecular-weightDNA.Thisisfollowedbyincreasedchromatincondensationand
thedevelopmentofnuclearbodies,classifiedasstage(JInhibitionofcaspases,asoccurswithupregulaticnofBCL2,inhibitsapoptosis,socellswithincreasedBCL2expressionwillnotdemonstratethesecharacteristicmorphologicchanges.Upregulationor
constitutiveexpressionofBCL2leadstoprolongedcellularsurvival.BCL2overexpressionisseeninseveraltypesofBcellnon-Hodgkinlymphoma.
IncorrectAnswers:B,C,D,andE.
ERBB2protein(ChoiceB)isagrowthfactorreceptorwithintracellulartyrosinekinaseactivitythatsignalsthroughthePI3K/Akt/mTORpathwaytoupregulategenesinvolvedincellularproliferation.Constitutiveexpressiondoesincreasecellularproliferationbutdoes
notdirectlyaffectapoptosis.
p53protein(ChoiceC)isapotenttumorsuppressorgenethatiscommonlyinactivatedinavarietyofmalignancies.Increasedorconstitutiveactivationofp53wouldbemorelikelytoinduceapoptosis,notinhibitit.
Platelet-derivedgrowthfactor(PDGF)(ChoiceD)isasubstancesecretedbyplateletsthatstimulatescellularproliferationandangiogenesis.ThePDGFreceptors,notthegrowthfactoritself,areoftenmutatedinvariousmalignanciesincludingcolonandbreast
cancer.
Rbprotein{ChoiceE)isatumorsuppressorproteinthatblockscellularentryintoS-phaseandtherebypreventingcellulardivision.Increasedexpressionwouldleadtoreducedcellulargrowth.
EducationalObjective:BCL2isanantiapaptoticproteinthatblockstheoligomerizationofBAX/BAKintheoutermitochondrialmembraneandpreventsreleaseofcytochromec,whichnormallyactivatescaspasesandpromotesapoptosis.OverexpressionofBCL2
preventsapoptosis.Thisisseenvisuallyinlymphocytesasanabsenceofperipheralchromatincondensation,anindicatorofapoptosis,Whenexposedtocorticosteroids.
Previous Next ScoreReport LabValues Calculator Help Pause

ExamSection4:Item39of50 NationalBoardofMedicalExaminers^
ComprehensiveBasicScienceSelf-Assessment
Y
39.AG5-year-oldmancomestothephysicianforafollow-upexamination.Hehasa15-yearhistoryofpoorlycontrolledtype2diabetesmellitus,resultinginmultipleperipheralneuropathies.Neurologicexaminationshowswastingoftheinterosseousmuscles
ofthelefthandandinabilitytoabductthefingersofthishand:plantarflexionoftherightfootisabsent.Thefunctionofwhichofthefollowingpairsofnervesismostlikelyimpairedinthispatient?
A)Medianandcommonfibular(peroneal)
B)Medianandtibial
C)Radialandcommonfibular(peroneal)
D)Radialandtibial
E)Ulnarandcommonfibular(peroneal)
F)Ulnarandtibial
CorrectAnswer:F.
Long-sfandingdiabetesmellituscanleadtothedevelopmentofdiabeticneuropathy.Chronichyperglycemialeadstoglycpsylationofaxonalproteinsalongwithsorbitol-relatedosmoticdamagetotheneuron
:withresultingprogressivesensoryandmotor
neuropathy.Initially,theconditionmostlyaffectsdistalnerves,thatis.thesmallnervesofthefeetandfingers.Overtime,theneuropathyprogressesprgorimally,leadingtopainanddecreasedsensationintheextremitiesTheassociateddecreasedmotorinnervation
canleadtomusclewasting,whiledecreasedsensationmayleadtoskinandjointdamagethatfailstohealasthepatientfailstodecreasepressureontheinjuredregionasprotectivesensationislimited.Thispatientpresentswiththeinabilitytoabductthefingersof
hishandandplantarflexhisrightfoot.Theinterosseousmusclesofthehandabductandadductthefingersandareinnervatedbytheulnarnerve.Plantarflexionofthefootiscontrolledbythemusclesoftheposteriorcompartmentsuchasthegastrocnemiusand
soEeus,whichareinnervatedbythetibialnerve.
IncorrectAnswers:A.B.CaD,andE
Themediannerve(ChoiceAandB)providesmotorinnervationtothemajorityoftheflexorsoftheforearmandthethenareminence.Intheanteriorforearm,theonlymusclesthatarenotinnervatedbythemediannervearetheulnarhalfofflexordigitorum
profundusandflexorcarpiulnaris,whichareinnervatedbytheulnarnerve.Themediannervealsosuppliessensoryinnervationtothepalmarradialsideofthehandincludingthethumb,index,middle,andhalfoftheringfinger.
Thetibialnerve(ChoicesBandD)providesinnervationtotheposteriorcompartmentofthelowerlegandtheintrinsicmusclesofthefoot.Itprovidessensoryinnervationtothebottomofthefootviathemedialandlateralplantarnerves.Howeverthemediannerve
andradialnervedonotcorrelatewiththispatientslesion.
Theradialnerve(ChoicesCandD)providessensoryinnervationtotheposteriordistalpartoftheforearm.Italsoinnervatesalloftheextensorsoftheelbow,wrist,andhand.
Theulnarnerve(ChoiceE)innervatesmostoftheintrinsicmusclesofthehandexceptforthelateraltwolumbricalmusclesandthoseofthethenareminence.Italsoprovidessensoryinnervationtothemedialaspectoftheforearm,smallfinger,andulnaraspectof
theringfinger.
Thecommonfibularnerve(peroneal)(ChoicesAandC)providesmotorinnervationtotheanteriorandlateralcompartmentsofthelowerlegandprovidessensationtothelaterallowerleganddorsumofthefoot.
EducationalObjective:Long-standingdiabetesmellituscanleadtothedevelopmentofneuropathy.Musclewasting,sensoryloss,anddysfunctioninspecificdermatomalandmyotomaldistributionscanallowforidentificationofspecificnerveinvolvement.
© © ©
Previous Next ScoreReport LabValues Calculator Help Pause

ExamSection4:Item40ol50 NationalBoardofMetlicaiExaminers^
ComprehensiveBasicScienceSelf-Assessment
40.A12-year-oldgirlisbroughttothephysicianbecauseofarashonherleftbuttockforthepast2days.Therashdevelopedafterthefamilyreturnedfroma2-week-longearlysummervacationinMaine.Hervitalsignsarewithinnormallimits.Aphotograph
ofthelesionisshown.Thelikelycauseofthispatient'sinfectionistaxonomicailyandmorphologicallymostsimilartotheinfectiousagentofwhichofthefollowingconditions?
A)Bacillaryangiomatosis
B)Chancroid
C)Leptospirosis
D}Lymphogranulomavenereum
E)Qfever
CorrectAnswer:C.
Erythemachrontcummigrans,whichcanbeidentifiedbyitsclassictargetoidpatches,isoftenthefirstsignofLymedisease.LymediseaseiscausedbythespirocheteBorreliaburgdorferi,whosevectoristheIxodestick.Thetickandorganismarecommoninthe
northeasternUnitedStatesbutcanbeseeninotherpartsofthecountryaswell.EarlylocalizedLymediseasepresentswitherythemachronicummigransandflu-likesymptoms.Earlydisseminatedinfectionpresentswithcarditis,heartblock,facialnervepalsy,
andtransientarthritis.LatedisseminatedLymepresentswithencephalopathyandarthritis.Treatmentofthisinfectioniswithdoxycycline.Ceftriaxoneisanalternativeinpregnantpatientsorchildrenlessthan8yearsoldandisgenerallythefirstlineincentral
nervoussystemdisease.OtherspirochetesincludeLeptospiraspeciesandTreponemaspeciesInfectionwithL.interrogans,foundinwatercontaminatedwithanimalurine,causesleptospirosisLeptospirosisischaracterizedbyflu-likesymptoms,myalgias,
jaundice,andphotophobiawithconjunctiva!injection.T.pallidumistheorganismresponsibleforcausingsyphilis.
IncorrectAnswers:A.B.D,andE.
Bartonellahenselaeisthecauseofbacillaryangiomatosis(ChoiceA).Thisinfectionismostcommonlyseeninimmunocompromisedindividualsandpresentswithred,vascularpapules.ItmaybemistakenforKaposisarcomaclinicallybutdemonstrates
neutrophilsonhistologicexamination.
Chancroid(ChoiceB)presentswithanexudative,painfulgenitalulcerwithinguinallymphadenopathy.11iscausedbyinfectionwiththeorganismHaemophilusducreyi.T.pallidum,thespirochetethatcausessyphilis,resultsinapainlesschancreinstead.
Lymphogranulomavenereum(ChoiceD)iscausedbyinfectionwithChlamydiatrachomatissubtypesL1,L2
;andL3.Itpresentsassmallpainlessgenitalulcerswithtenderinguinallymphadenopathy.C.trachomatisisnotaspirochete.
Qfever(ChoiceE)isanothervector-borneillnesscausedbyinfectionwithGoxieHaburnetii
^
whichisnotaspirochete.C.burnetiiisacquiredthroughtheinhalationofsporesfromcattleorsheepamnioticfluid.Qfevercommonlypresentsaspneumoniaor
endocarditis.
EducationalObjective:Borrelia.Leptospira,andTreponemaarespirochetespeciesthatcauseadiversegroupofinfectionsincludingLymedisease,leptospirosis,andsyphilis,respectively.
C O ®
r-tS
Previous Next ScoreReport LabValues Calculator Help Pause

ExamSection4:Item41ol50 NationalBoardofMedicalExaminers^
ComprehensiveBasicScienceSelf-Assessment
Y
41.Asingle,oral.6-mgdoseofashort-acting(3
^
agonistisstudiedtodetermineitspharmacokineticsandpharmacodynamicsinyoungsubjectswithstablemildasthma.Inthisgroup,theCma,.-AIJGfollowingoraldosingvariesby:essthan8%.However,in30%
ofsubjects
:themaximalFEV,increaseis25%lowerthanthatoftheothersubjects.Whichofthefallowingmechanismsbestexplainstheseobservationsinthissubgroup?
A)DecreasedcatecholO-methyltransferaseactivityinbronchialsmoothmuscle
B)DecreasedphosphodiesteraseIVinbronchialsmoothmuscle
C)IncreasedgastrointestinalP-glycoprotein
D)Receptorpolymorphismwithreducedsignaltransduction
E)Receptorupregulation
CorrectAnswer:D.
Airwaysmoothmusclecellsexpressp
2
receptors,andactivationresultsinbranchediiationthroughGE
protein-coupledreceptorsignaltransduction.Thismechanismistherationalefortheuseofp-adrenergicagonistsinthetreatmentofasthmaThedifferencein
responsebetweenthesubgroupsisbestexplainedbyreceptorpolymorphismwithreducedsignaltransductioninthelowermaximalFEV
^
group.TheminimalvariationinCm
^
/AUCfollowingoraldosing
:ndicatesthatsimilarabsorptionoccurredbetweenthe
groups,anditcanbeextrapolatedthattheairwaysmoothmusclecellswereexposedtosimilarconcentrationsofthedrug.Receptorpolymorphismreferstominorvariationsinthegenesencodingreceptors(mostoftensinglenucleotidepolymorphisms)thatmay
resuitinfunctionalproteinswithdifferingamplitudesofdownstreamresponseuponactivation.
IncorrectAnswers:A,B
(C.andE.
DecreasedcatecholO-methyltransferase(COIVTT)activityinbronchialsmoothmuscle(ChoiceA)doesnotadequatelyexplainthefindingsinthisexperiment.COMTisresponsiblefortheenzymaticdegradationofcatecholamines,anddecreasedactivitymayresult
inareducedsusceptibilitytobronchoconstrictionbutwouldnotbeexpectedtoaltertheresponsetoa-adrenergicagonist.
DecreasedphosphodiesteraseIVinbronchialsmoothmuscle(ChoiceB)resultsinreduceddegradationofcAIVfPandincreasedbronchodilation.ApolymorphisminphosphodiesteraseIVthatresultsinreducedlevelswouldbeexpectedtoincreasethemaximal
FEVi
andwouldnotcauseavariableresponsetothep
2
-adrengericagonist.
IncreasedgastrointestinalP-glycoprotein(ChoiceC)decreasesserumconcentrationsoforallyadministereddrugsthroughincreasedeffluxoutofenterocytes.ThesimilarC
wasnotthecauseofthedifferentresults,anditcanbeextrapolatedthattheairwaysmoothmusclecellswereexposedtosimilarconcentrationsofthedrug.
/AUClevelsinthesubgroupssuggeststhatdrugabsorptioninthegastrointestinaltract
max
Receptorupregulation(ChoiceE)islesslikelythanreceptorpolymorphismintheexperimentsubgroupsassumingsimilarexposuresandcontrolsforconfounders.
EducationalObjective:Minorvariationsinnucleotidesequencescanresultinfunctionalproteinsthatrespondtoactivationwithdifferentamplitudesofsignalingeffects.Thisiscalledreceptorpolymorphism.
O0
It A
Previous Next ScoreReport LabValues Calculator Help Pause

ExamSection4:Item42ol50 NationalBoardofMedicalExaminers^
ComprehensiveBasicScienceSelf-Assessment
Y
42A19-year-oldmancomestotheemergencydepartmentbecauseofincreasinglysevereshoulderandabdominalpainfor3days.Histemperatureis39X(102.2T)Physicalexaminationshows
signsofacuteperitonitis.Anabdominalx-rayisshown.Whichofthefollowingisthemostlikelycauseofthispatient'scurrentcondition?
A}Ileus
B}Intraperitonealabscess
C)Nephrolithiasis
D)Perforatedviscus
E}Retainedforeignbody
CorrectAnswer:D.
Feverabdominalpain
;peritonitis,andfreeairseenunderthediaphragmonuprightchestx-rayraisessuspicionforaperforatedviscus.Patientswithperforatedviscuscandemonstrateintraabdominalfreeairandleakageofentericcontents,resultingin
inflammation,irritation,andinfectionoftheperitoneum.InflammationinvolvingthediaphragmcanleadtoreferredpaintothescapulaorshoulderduetosharedinnervationbycervicalspinalnervesC3-5.Perforatedviscuscanoccursecondarytopepticulcer
disease,diverticulitis,appendicitis,orbluntorpenetratingabdominaltraumaamongotheretiologies.Resultingperitonitisisasurgicalemergency,andpatientsaretreatedwithbroad-spectrumantibioticsandsurgerytorepairtheperforatedviscusandlavagethe
peritonealcavity.
IncorrectAnswers:A,6,C,andE.
Ileus(ChoiceA)referstointestinalhypomotilitywithoutamechanicalobstruction,mostcommonlyoccurringfollowingsurgeryorinthesettingofelectrolyteabnormalities,andtypicallypresentswithdiffusesmaIandlargeboweldilatation.
Intraperitonealabscess(ChoiceB)canalsocauseabdominalpainandfeverandcanresultfromareasofinflammationandinfectionsuchasappendicitisordiverticulitis.Abscessesarecontainedstructuresanddonotresultinintraabdominalfreeair.
Nephrolithiasis(ChoiceC)typicallypresentswithunilateralflankpainthatiscolickyandsometimesradiatestothegroinwithassociatedhematuria.Thecommontypesofnephrolithsarecalciumoxalateorphosphate,ammoniummagnesiumphosphate,uricacid,
andcystine.Itwouldnotcausefeverunlesscomplicatedbyaurinarytractinfection.
Retainedforeignbody(ChoiceE)cancauseabdominalpaindependingontheforeignbody,buttypicallydoesnotcausefeverorfreeairunderthediaphragmun
:
essithasperforatedtheintestinecausingleakageofentericcontents.
EducationalObjective:Aperforatedviscusresultsintheleakageofentericcontents,fever,abdominalpain,peritonitis,andpneumoperitoneum.Diaphragmaticirritationcancausereferredshoulderorscapularpain.
00# 0f*
Previous Next ScoreReport LabValues Calculator Help Pause

ExamSection4:Item43of50 NationalBoardofMedicalExaminers^
ComprehensiveBasicScienceSelf-Assessment
Y
43.A28-year-oldmanisparticipatinginastudyoftheeffectofaltitudeandconditioningoncombatreadiness.HeisaSpecialForcessergeantinthearmy.WhileheexercisesinanenvironmentalchambertheambientPo
2
isdecreasedfrom160mmHgto60
mmHg.ThedecreaseinambientPo
2willmostlikelycauseanincreaseinwhichofthefollowinginthispatient?
A)Arterialdiastolicpressure
B)Arterialpulsepressure
C)Arterialsystolicpressure
D)Meanarterialpressure
E)Meanpulmonaryarterypressure
CorrectAnswer:E
Adecreaseintheambientpartialpressureofoxygen(Po
2
)resultsinhypoxicpulmonaryvasoconstrictionwitharesultantincreaseinthemeanpulmonaryarterypressure.Acutehypoxiastimulatescontractionofsmoothmusclecellsinthepulmonaryvasculature.
Thismechanismdivertsbloodflowawayfromregionsofthetungthathavelowalveolaroxygencontentinwhatiscalledventilation-perfusionmatching.Ventilation-perfusionmatchingservestomaximizethefunctionalsurfaceareaofthealveolar-capillaryinterface
thatisparticipatingingasexchange.Chronichypoxicpulmonaryvasoconstrictionmayoccurfromiivingataltitudeorinthesettingofchroniclungdisease(eg
;chronicobstructivepulmonarydisease).Complicationsincluderightventricularhypertrophyandtricuspid
regurgitationthatmayprogresstoright-sidedheartfaiure.
IncorrectAnswers:A.B.C.andD.
Anacutechangeinambientoxygencontentisnotexpectedtocauseasignificantchangeinthearterialdiastolicpressure(ChoiceA)
;arterialpulsepressure(ChoiceB)
;arterialsystolicpressure(ChoiceC).orthemeanarterialpressure(ChoiceD).Ahealthy
patientisabletocompensateforthereducedoxygencontentinthealveolibyhyperventilatingandincreasingcardiacoutputthroughincreasedheartratetomaintainadequateoxygendeliverytotheorgans.Localizedhypoxemiainducesvasodilationinsystemic
arterialbloodvessels,whichpromotesincreasedbloodflowandoxygendeliverytothearea.
EducationalObjective:Lowoxygencontentinthebloodinducesvasoconstrictioninthepulmonaryvasculatureinordertodirectbloodflowtoregionswithahigheroxygencontent.Theoppositeeffectoccursinthesystemicvasculature,withvasodilationinresponse
tohypoxemia.Thispromotesincreaseddeliveryofoxygentohypoxictissue.
00#
Previous Next ScoreReport LabValues Calculator Help Pause

ExamSection4:Item44ol50 NationalBoardofMetlicaiExaminers^
ComprehensiveBasicScienceSelf-Assessment
44.A52-year-oldwomanisadmittedtothehospitalbecauseofchestpainandshortnessofbreathfor2hours.Herpulseis1D2/miinrrespirationsare35/min,andbloodpressureis110/65mmHg.Cardiacexaminationshowsalaterallydisplacedapex,a
systolicthrill,andagrade4/6.crescendo-decrescendosystolicejectionmurmur.Cardiaccatheterizationshowsapulmonarycapillarywedgepressureof40mmHgfN=5—16).Whichofthefollowingsetsofchangesinthispatient'sleftventricleismost
likely?
Stroke
Work
Increased
MyocardialOxygen
Consumption
increased
increased
decreased
decreased
MyocardialOxygen
Tension
increased
decreased
Adenosine
Concentration
decreasedA)
B) Increased increased
C) Increased increased increased
0) Increased decreased increased
E) Decreased increased increased increased
F) Decreased increased increased decreased
G) Decreased decreased increased increased
H) Decreased decreased decreased decreased
CorrectAnswer:B.
Aorticstenosis(AS)isacommondisorderoftheaorticvalvethatresultsfromcalcification.Calcificationoftheaorticvalveisapathologicconsequenceofmechanicalstressesonheartvalves,andresultsfromrepetitivemicrotraumafromtheopeninganddosing
ofvalveleafletswithassociatedchronicinflammation.ManypeoplewilldevelopsomedegreeofASovertime,butstructuralabnormalitiesofthevalve,suchasabicuspidaorticvalve
:alterthebiomechanicsofvalveopeningandclosingandincreasethe
likelihoodofearliercalcificationandresultantstenosis.Whilemanypatientsmaybeasymptomatic,thosewithsevereASmaycomplainoffatigue
:shortnessofbreath,cough,diminishedexercisetolerance
:angina,orsyncopewithexertion.Examfindingsinclude
acrescendo-decrescendosystolicmurmurbestheardattheupperrightsterna!border,andpulsusparvusettardus(weakanddelayed)maybenotedonexaminationofperipheralpulses.Duetothechronicincreasedafterloadfromafixedobstructionbythe
valve,leftventricularhypertrophyandresultantdiastolicdysfunctioncanoccurwithevidenceofincreasedpulmonarycapillarywedgepressureoncardiaccatheterization.Contractingagainstastenosedvalverequiresincreasedstrokework,resultinginincreased
myocardialoxygenconsumption,decreasedmyocardialoxygentension,andincreasedadenosineconcentration.Increasedadenosineconcentration,whichoccursasadenosinetriphosphate(ATP)isdephosphorylatedforcontraction,anddecreasedoxygen
tensionpromotecoronaryarteryvasodilationtoincreaseperfusiontoregionsofhighcardiamyocytedemand.
IncorrectAnswers:A,C,D
;E,F
;G,andH.
ChoicesA.F.andIHareincorrectastheadenosineconcentrationincreasesduetothebreakdownofadenosine-phosphatecomplexesforenergyutilization.
ChoicesC,D,G,andHareincorrectbecausemyocardialoxygenconsumptionisincreasedinordertogeneratetherequiredcontractileforce.Myocardialoxygenconsumptionfallsasmyocardialworkloadfalls.Similarly,choicesA.C,E,F,andGareincorrectas
themyocardialoxygentensiondecreasesduetoincreasedmetabolicdemandforenergyintheformofATPandincreasedoxygenutilizationbymitochondria.
ChoicesE,F,G,andHareincorrectasthestrokeworkoftheleftventricleincreasestogenerateapressuregradientacrossastenosedaorticvalvetodrivebloodflowforward.Surgerytoreplacethestenoticvalvewouldreducethemyocardialstrokework
necessarytopropelbloodforwardbydecreasingafterloadontheventricle.
EducationalObjective:ASmayleadtoleftventricularhypertrophyduetochronicallyincreasedafterloadIncreasedafterloadresultsinincreasedventricularstrokeworkandmyocardialoxygenconsumption.
m r
Previous Next SconeReport LabValues Calculator Help Pause

ExamSection4:Item45ol50 NationalBoardofMedicalExaminers^
ComprehensiveBasicScienceSelf-Assessment
Y
45.AG5-year-oldwomanwithseveresensorineuralhearinglossundergoessurgicalplacementofacochlearimplant.Thisneuralprosthesisconvertssoundenergytoelectricalsignals,whichresultsinstimulationofwhichofthefollowingstructures?
A)Auditorynerveendingsinthecochlea
B)Cochlearnerveasitentersthepons
C)Innerhaircellsofthecochlea
D}Olivocochlearefferentaxonsthatinnervateouterhaircells
E)Ovalwindowofthecochlea
CorrectAnswer:A.
Cochlearimplantsdirectlystimulatetheauditorynerveendingsinthecochlea.Sensorineuralhearinglosstypicallyresultsfromdamageofthehaircellsinthecochlea.Cochlearimplantsbypassthedamagedcochlea.Theexternalmicrophoneoftheimplant
convertsmechanicalsoundwavestoanelectricalsignalthatistransmittedtoanelectrodewithinthecochlealocatedadjacenttotheauditorynerveendings.Normally,soundtransmissioninthecochleabeginswhenthestapesfootplateofthemiddleearvibrates
againsttheovalwindowofthecochlea(innerear),whichcreatesvibrationsintheperilymphofthecochlea.ThesevibrationsareultimatelytransmittedtotheorganofCorti
;whichcontainshaircellsthatgenerateelectricalimpulsesandsendtheimpulsestothe
spiralganglionofthecochlea(auditorynerveendings).Axonsofthespiralganglionunitetoformtheauditorynerve,andultimatelythesignalistransmittedtotheauditorycortex.Insensorineuralhearingloss
;thedamagedhaircellscannolongerconvert
mechanicalvibrationstoelectricalimpulses.
IncorrectAnswers:B.C,D
:andE.
Stimulatingthecochlearnerveasitentersthepons(Choice8)isnotthelocationofacochlearimplant,asdistalplacementismoreinvasiveandlesseffective.
Stimulatingtheinnerhaircellsofthecochlea(ChoiceC)ortheolivocochlearefferentaxonsthatinnervateouterhaircells(ChoiceD}wouldnotbeeffective,asthehaircellsaredefectiveinsensorineuralhearinglossandwoudbeunabletotransmitelectrical
signals.Theolivocochlearefferentaxonsprovideconstantstimulationtothehaircellsandarethoughttoprotectthecochleafromloudsounds.
Stimulatingtheovalwindowofthecochlea(ChoiceE)wouldnotbeeffective.Theovalwindownormallytransmitsmechanicalvibrationstotheperilymphandultimatelytothehaircells.Ifthehaircellsaredefective,ovalwindowstimulationwouldnotsolvethe
problem.
EducationalObjective:Insensorineuralhearingloss
:thehaircellswithinthecochleaaredamaged.Cochlearimplantsutilizeanexternalmicrophonethattransmitselectricalimpulsestoanelectrodethatdirectlystimulatestheauditorynerveendings.Inthisway,
cochlearimplantsbypassthedamagedcochlea.
O f? & oo
Previous Next ScoreReport LabValues Calculator Help Pause

ExamSection4:Item46ol50 NationalBoardofMedicalExaminers^
ComprehensiveBasicScienceSelf-Assessment
Y
46.A4S-year-oldmanbeginsfurosemidetherapyforpedaledemaassociatedwithbiventricularfailureandhypertension.Fivedayslater,theedemaisnotfullyresolved,andhisserumpotassiumconcentrationhasdecreasedfrom4.2mEq/Lto3mEq/L.Adrug
withwhichofthefollowingactionsshouldbeaddedtothispatient'smedicationregimen?
A)BlocksbasolateralK
+channelsinthecollectingduct
B)DecreasestheluminalpermeabilitytoNa
+inthecollectingduct
C)IncreasesthedeliveryofNa
+andK
+tothecollectingduct
D}increasesthenegativechargeoftheluminaltubulefluid
E)StimulatesNa
+
-K
+
ATPaseinthecollectingduct
CorrectAnswer:B.
Thepatientshouidbestartedonapotassium-sparingdiureticgivenhispersistentedemaandhypokalemiatnthesettingoffurosemidetherapy.Potassium-sparingdiureticsworkbydecreasingtheluminalpermeabilitytoNa
+inthecollectingduct.Triamtereneand
amiforideachievethisbydirectinhibitionoftheepithelialsodiumchannel(ENaC)ontheluminalsurface.ThisreducestheamountofNa
^
thatcanpassthroughthemembraneandresultsinnatriuresisanddiuresis.Spironolactoneandeplerenoneare
mineralocorticoidreceptorantagonists(MRAs)thatblocktheactionofaldosterone.AldosteronenormallyactsonthecellsofthecollectingductbyactivationoftranscriptionfactorsthatncreasetheexpressionofENaContheluminalsurfaceandtheNaVK
_
ATPase
onthebasolateralsurface.TheNa
-;
K
+ATPasemaintainsanelectrochemicalgradientthatpromotesNa
^
diffusionintothecell.InhibitionbyMRAsresultsindecreasedNa
+andK>exchangeinthecollectingduelwiththeoveralleffectofeliminatingmoreNa
+inthe
urineandretainingK
+
.Themajoradverseeffectofpotassium-sparingdiureticsishyperkalemia.MRAsmayalsoresultinanti-androgensideeffectssuchasgynecomastia.
IncorrectAnswers:A,C,D,andE.
BlocksbasolateralK
+channelsinthecollectingduct(ChoiceA)isincorrectaspassiveHDchannelsarelocatedontheluminalsurfaceofcollectingductcells.ModulationofK
+diffusionisachievedbydecreasingNa
^entryintothecell,whichresultsinadecreased
electrochemicalgradientforK
+
movementintothelumen.
LoopdiureticsincreasethedeliveryofNa+andK+tothecollectingduct(ChoiceC)byblockingtheNa+/KV2CI
-
cotransporterinthethickascending:oopofHenle.Furosemideisaloopdiuretic.Theyareassociatedwithhypokalemia,hypocalcemia,and
hypomagnesemia.
Increasesthenegativechargeoftheluminaltubulefluid(ChoiceD)isincorrectasthedecreasedmovementofNa
+outofthelumendecreasesthenegativechargeoftheluminaltubulefluid,resultinginareducedelectrochemicalgradientforthediffusionofK
+into
theurine.
Aldosteronestimu.atesNa
+
-K
+
ATPaseinthecollectingduct(ChoiceE)resultinginincreasedNa
+diffusionthroughENaCchannels.MRAsblockthisactivity.
EducationalObjective:Potassium-sparingdiureticsmaybeaddedtoadiureticregimeninthesettingofpersistentedemawithhypokalemia.TheyactbyreducingtheexchangeofK
+
forNa
+inthecollectingducts.
© © © OO
Previous Next ScoreReport LabValues Calculator Help Pause

ExamSection4:Item47ol50 NationalBoardofMetlicaiExaminers^
ComprehensiveBasicScienceSelf-Assessment
y
47.A70-year-oldmancomestothephysicianbecauseofa4-monthhistoryoffatigueandcoughproductiveofblood-tingedsputum.Hehasahistoryofhyperlipidemia.Hehadsmoked1packofcigarettesdailyfor
30yearsuntilhequit5yearsago.Histemperatureiis37.2CC(993F),pulseis120/min,respirationsare20/min.andbloodpressureis130/85mmHg.Diffusecracklesandwheezesareheardoninspiration.A
chestx-rayisshown.Serumstudiesshow:
Na
+ 133mEq/L
50mEq/L
102mEq/L
24mEq/L
15mg/dL
K
+
ct
-
HCO
3
-
Ca
2+
Inadditiontointravenoushydrationandinductionofdiuresistotreathishypercalcemia,itismostappropriateforthispatienttobegintherapywithadrugthatinhibitstheactivityofwhichofthefollowingcells?
A}Chondroblasts
B)Chondrocytes
C)Osteoblasts
D)Osteoclasts
E}Parathyroidcells
CorrectAnswer:D.
Squamouscellcarcinomaofthelungcommonlyoccursinsmokersandpresentswithsymptomsofweightloss,malaise,cough,blood-tingedsputum,andpossiblerecurrentpneumoniaduetoblockageofbronchibythemalignantgrowthTumorsthatoccurinthe
lungapicesandimpingeonthethoracicoutletarereferredtoasPancoasttumors.Acommonparaneoplasticsyndromethatoccursinsquamouscellcarcinomaofthelungishypercalcemiaduetosecretionofparathyroidhormone-relatedpeptide(PTHrP).This
proteinactssimilarlytoparathyroidhormone(PTH)
;causinganincreaseinosteoclastactivitywithresultantosteolysis,liberatingfreecalciumandincreasingtheserumcalciumconcentration.PTHrPalsoincreasestheresorptionofcalciumandexcretionof
phosphatebythekidney.LaboratoryvaluestypicallydemonstrateahypercalcemiaandhypophosphatemiawithalowPTHlevef.TreatmentofhypercalcemiarelatedtoPTHrPrequirestheadministrationofbisphosphonates,whichinhibittheactivityofosteoclasts.
Bisphosphonatesaresimilarinstructuretopyrophosphateandthusincorporateintothebonemineralmatrix.Whenthisbonemineralmatrixisbrokendownbyosteoclasts,thebisphosphonatesentertheosteoclastsandEeadtotheinitiationofosteoclast
apoptosis.Definitivemanagementrequirestreatmentoftheprimarysquamouscellcarcinomaofthelungwithacombinationofeithersurgery,chemotherapy,and/orradiationdependingonthestage.
IncorrectAnswers:A,B
(C,andE.
Inhibitionofchondroblasts(ChoiceA)wouldnotdecreaseserumcalciumaschondroblastsdonotregulatecalciumhomeostasis.Inhibitionofchondroblastswouldleadtothedecreasedabilityofcartilagetoremodelandhea:
Inhibitionofchondrocytes(ChoiceB)wouldleadtothedecreasedproductionofcartilagematrixsuchastypeIIcollagenandaggrecan,leadingtoadecreasedvolumeofcartilage.
Inhibitionofosteoblasts(ChoiceC)wouldleadtodecreasedboneformationduringthenaturalprocessofboneturnover.Decreasedosteoblastactivitynaturallyoccurswithagingandleadstoanimbalanceofrelativelyincreasedosteoclastactivityresultingin
decreasedbonemineraldensitywithage.
Inhibitionofparathyroidceils{ChoiceE)wouldleadtoadecreasedproductionofparathyroidhormone.AlthoughthiswouldleadtodecreasedPTH-dependentosteoclastactivityandrenaltubularcalciumabsorption,thiswouldnotinhibittheproductionandactivity
ofPTHrPbythemalignantsquamouscells.Inthispatient,thereleaseofPTHbyparathyroidcellswouldlikelyalreadybedownregulatedduetothehighcalciumlevelsdrivenbyPTHrP.Directinhibitionofosteoclastsisthereforerequired.
EducationalObjective:ParaneoplasticproductionofPTHrPiscommonlycausedbysquamouscellcarcinomaofthelung.Osteolyticmetastasesofmultiplemyelomaandbreastcancercanalsoleadtohypercalcemia.Hypercalcemiaassociatedwithmalignancyis
oftentreatedwithbisphosphonatestoinduceosteoclastinhibition.
ro m
ii?
Previous Next ScoreReport LabValues Calculator Help Pause

ExamSection4:Item43ol50 NationalBoardofMetlicaiExaminers^
ComprehensiveBasicScienceSelf-Assessment
48.A22-year-oldmancomestothephysicianbecauseofa2-monthhistoryoffoul-smelling.waterydiarrheawithsignificantflatulence:healsohashada4.5-kg(10-lb)weightlossduringthisperiod.Herecentlyreturned
fromatriptoruralIndonesia,wherehedidnotalwayshaveaccesstocleanwater.Heappearsthin.Physicalexaminationshowsasoft,mildlydistendedabdomenwithactivebowelsounds.Aphotomicrographofa
stoolspecimenisshown.Themostappropriatepharmacotherapyhaswhichofthefollowingmechanismsofaction?
A)Enhancementofcellmembranepermeabilitytochlorideions
6)Formationofdestructivefreeradicals
C)InhibitionofDNApolymerase
D)Inhibitionofproteinsynthesis
E)Preventionofmicrotubuleassembly
10Mtn
CorrectAnswer:B.
Thephotomicrographdisplaysaflagellated,pear-shapedtrophozoitewithtwolargecentralnucleiandamedianaxostyle,consistentwiththeappearanceofGiarclialambliaG.lambitaisaflagellated,parasiticorganismthatcausesgiardiasis.Giardiasisisusually
acquiredthroughtheingestionofcystsincontaminatedwater,whichthenmatureintotrophozoitesinthesmallbowel.Parasiticovergrowthofthesmallbowelleadstointestinalmalabsorption.Giardiasispresentswithpersistentdiarrheathatisfoul-smeltingand
fatty,aswet!asabdominalpain,bloating,cramping,andweightloss.Diagnosisismadewithstoolculture,microscopy,orstoo!antigentesting.Metronidazoleisthefirstlinetreatmentandassertsitsantimicrobialeffectthroughtheformationofdestructivefree
radicals.
IncorrectAnswers:A.CTD.andE.
Enhancementofceflmembranepermeabilitytochlorideions(ChoiceA)isthemechanismofactionoftheanthelminthicagentivermectin,ivermectinalterscellularmembranepotentials,leadingtoparalysisanddeathofthetargetorganism.Ivermectinisnot
usefulfortreatmentofgiardiasis.
InhibitionofDNApolymerase(ChoiceC)isthemechanismofactionofseveralimportantantivirals,includingacyclovircidofovir,andfoscarnet.Whileseveralacuteviralsyndromescanpresentwithdiarrhea,includingcytomegalovirusandHIV,the
photomicrographidentifiesG.lambliaasfhecausativeorganismofthispatient'sdiarrhea.
Inhibitionofproteinsynthesis(ChoiceD)isthemechanismofactionofseveralimportantclassesofantibiotics,includingmacrolides.aminoglycosides,andtetracyclines.Noneofthesedrugsarefirstlineforthetreatmentofgiardiasis.
Preventionofmicrotubuleassembly(ChoiceE)isthemechanismofactionoftheanthelminthicagentsalbendazoleandmebendazole.Whilealbendazoleissometimesusedforthetreatmentofgiardiasis,metronidazoleisfirstline.
EducationalObjective:Metronidazoleisthefirstlinetreatmentofgiardiasisandenactsitsantimicrobialeffectthroughtheformationofdestructivefreeradicals.
£ %
iS
Previous Next ScoreReport LabValues Calculator Help Pause

ExamSection4:Item49of50 NationalBoardofMedicalExaminers^
ComprehensiveBasicScienceSelf-Assessment
y
49.A70-year-oldwomanisbroughttotheemergencydepartment30minutesaftershewasfoundunresponsiveathome.Sheappearsstuporous.Physicalexaminationshows2-mm-diameterpupilsthatare
reactivetolight.Eyemovementsarefullwithteewatercaloricstimulation.Shewithdrawstheextremitiessymmetricallytopainfulstimuli.AnMRIofthebrainisshown;thearrowheadsindicate
abnormalities.Whichofthefollowingismostlikelytobepresentinthispatient2monthslater?
A)Difficultyrecognizingfamiliarfaces
8}Hesitanttelegraphiclanguageoutput
C)Impairedabstractionandproblemsolving
D)Inabilitytolearnandrecallnewfacts
E)Inappropriate
;disinbibitedbehavior
CorrectAnswer:D.
Thehippocampus,locatedinthemesialtemporallobe.mediateslearningandmemory .TheCA1regionofthehippocampalcodexisfrequentlydamagedinanoxicbraininjuryduetothehighmetabolicdemandofthisregion.Thehippocampusisveryvulnerableto
ischemicinjury.Thepatient'sbrainstemandspinalcordarefunctioningonphysicalexamination,butshestilldemonstrateshippocampaldamageonMRIofthebrain.Thehippocampusformsanterograde,declarativememories(eg.memoriesaboutfacts)and
consolidatesthesememoriesintolong-termmemories.Thehippocampusalsoassistswithspatialnavigation.Additionally,thehippocampusispartofthelimbicsystemandformsstrongconnectionswiththebrainregionsthatgenerateemotionssuchasthe
amygdala.Assuch,thehippocampusmediatestherecallofemotionalmemories.Bilaterallesionsofthehippocampusclassicallyleadtoartamnesicsyndromeinwhichnewfactscannotbelearnedorrecalled.
IncorrectAnswers:A.8,C.andE.
Difficultyrecognizingfamiliarfaces(ChoiceA),orprosopagnosia,resultsfromdamagetothenon-dominantfusiformgyrus,whichmediatesfacialrecognition.Thefusiformgyrusislocatedontheinferioraspectofthebrainbetweenthetemporalandoccipitallobes,
lateraltotheparahippocampalgyrusandhippocampus.Thefusiformgyrusislessvulnerablethanthehippocampustoischemicdamagedueitslowermetabolicneeds.
Hesitant,telegraphiclanguageoutput(Choice8)wouldresultfromalesionofBrocaarea.Brocaareaislocatedwithintheleftposterior,inferiorfrontallobeofthedominanthemisphere.Thispatienthasbilateral:esionsofthemesialtemporallobe.
Impairedabstractionandproblemsolving(ChoiceC)wouldresultfromalesionoftheprefrontalcortexoranotherhigher-orderassociationareasuchastheangulargyrus.Thesebrainareasarelocatedoutsideofthemesialtemporalobe.
Inappropriate,disinhibitedbehavior(ChoiceE)wouldresultfrombilaterallesionsoftheamygdala,whichcausesKluver-Bucysyndrome.Kluver-Bucysyndromefeaturesnyperorality,hyperphagia,andhypersexuality.Theamygdalaislessvulnerablethanthe
hippocampustoischemicdamageduringanoxicbraininjuryandwouldappearsmallerincoronalcross-sectionthanthehippocampus.
EducationalObjective:Thehippocampalcortexisvulnerabtetoischemicdamageinanoxicbraininjury.Bilateralhippocampaldamageleadstoanamnesicsyndromeinwhichpatientscannotformorconsolidateanterograde,declarativememories.
c o ® * 0
Previous Next ScoreReport LabValues Calculator Help Pause

ExamSection4:Item50ol50 NationalBoardofMedicalExaminers^
ComprehensiveBasicScienceSelf-Assessment
Y
50.A34-year-oldwomancomestothephysicianbecauseshehasbeenfeelingweepyandoverwhelmedsincedeliveringahealthymalenewbornGweeksago.Shehashadfatigueandirritabilityduringthisperiod
:andshehashadnointerestinengagingin
activitiessheusedtoenjoypriortothebirthofthebaby.Shealsohashaddifficultysleepingbecausesheisalwayslisteningforsoundsindicatingthatherbabyisawake.Shesays.
"Myhusbanddoesn'thelpmeatallwiththebaby.Weargueallthetime
now."Shethensays
:"IfeelguiltybecauseI’mnotenjoyingmybabymore.!don'tknowifIwillbeagoodmotherornot.
'
1
Physicalexaminationshowsnoabnormalities.Whichofthefollowingstatementsbythephysicianismostappropriate?
A)"Hasitcomeasasurprisetoyouhowhardparentingis?Manypeoplefeelthatwray."
B)"I'mconcernedabouthowbadyou'vebeenfeelinglately.Haveyouhadanythoughtsaboutdeathorwantingtobedead?"
C)"I'msurethatyou'llfeelbettersoon.Afteralllookatyourbeautifulbaby."
D)"Mostnewmothersfeelthiswayandsometimesithelpstoseeatherapist.Wouldyoulikemetogiveyouareferral?"
E)"Yourrelationshipwithyourhusbandsoundsstrained.1don'tblameyouforbeingangrywithhim."
CorrectAnswer:B.
Thispatientmeetscriteriaformajordepressivedisorder,postpartumonsetandshouldbeimmediatelyevaluatedforsuicidalideation.Majordepressivedisorder,postpartumonsetpresentswiththetypicalsymptomsofmajordepressivedisorder(depressedmood,
decreasedenergy,anhedonia.sleepdisturbances,appetitedisturbances,guilt,poorconcentration)andmaybedelayedbyasmuchas12monthsafterdelivery.Riskfactorsforpostpartumdepressionincludesleepdisturbance,childcarestressdecreasedestrogen
levels,breastfeedingdifficulty,bodyimagedissatisfaction,andpoorpostpartumsocialsupport.Consequencesofpostpartumdepressioncanbesevereandincludeimpairedbondingwiththechild,childneglect,suicide,andinfanticide.Patientswithpostpartum
depressionshouldbeproactivelyandexpeditiouslymonitoredforthesesequelaeTreatmentofmoderatetoseverepostpartumdepressiontypicallyincludesantidepressantmedication,whichposesminimalrisktobreastfeedinginfants,andinpatientadmissionin
high-risksituations.
IncorrectAnswers:A.C,D,andE.
Normalizingthepatient'sexperience(ChoicesAandD)mayhelpthepatientfeelemotionallyvalidatedsnthemoment,andreferraltoatherapist(ChoiceD)mayassistinlong-termimprovement.However,thepatientandinfant
'ssafetyarethemostacuteissuesand
shouldbeimmediatelyassessed.
Tellingthepatientthatshewillfeelbettersoon(ChoiceC)isalesseffectivewayofvalidatingapatient'semotionsthannormalizationsorreflectivestatements.Additionally,thepatientandinfant'ssafetyarethemostacuteissuesandshouldbeimmediately
assessed.
Exploringthepatient'srelationshipwithhernusband(ChoiceE)wouldnotaddressthesafetyofthispatientandherinfantinanexpeditiousmanner.Further,interpersonaldynamicsappeartoplayonlyasmallroleinthispatient'sdepression.
EducationalObjective:Majordepressivedisorder,postpartumonsetpresentswithsymptomsofmajordepressivedisorderinthe12monthsfollowingdeliveryandcanbeassociatedwithseriousconsequencessuchassuicideandinfanticide.Allpatientswithmajor
depressivedisorder,postpartumonsetshouldbeassessedexpeditiouslyforsuicidalandhomicidalideation.
© © © oo
Previous Next ScoreReport LabValues Calculator Help Pause
Tags